Vous êtes sur la page 1sur 390

Physics

BOARD QUESTION PAPER : MARCH 2016


PHYSICS
Time: 3 Hours Total Marks: 70

Note:
i. All questions are compulsory.
ii. Neat diagrams must be drawn wherever necessary.
iii. Figures to the right indicate full marks.
iv. Use of only logarithmic table is allowed.
v. All symbols have their usual meaning unless otherwise stated.
vi. Answers to both sections must be written in the same answerbook.
vii. Answer to every question must be written on a new page.

SECTION – I

Q.1. Attempt any SIX: [12]


  
i. In U.C.M. (Uniform Circular Motion), prove the relation v   r , where symbols have their
usual meanings.
ii. Derive an expression for critical velocity of a satellite revolving around the earth in a circular
orbit.
1  K2 
iii. Obtain an expression for total kinetic energy of a rolling body in the form MV2 1  2  .
2  R 
iv. Define ‘emissive power’ and ‘coefficient of emission of a body’.
v. A coin kept at a distance of 5 cm from the centre of a turntable of radius 1.5 m just begins to
slip when the turnable rotates at a speed of 90 r.p.m. Calculate the coefficient of static friction
between the coin and the turntable. (g = 9.8 m/s2).
vi. The fundamental frequency of an air column in a pipe closed at one end is in unison with the
third overtone of an open pipe. Calculate the ratio of lengths of their air columns.
vii. A particle performing linear S.H.M. has a period of 6.28 seconds and a path length of 20 cm.
What is the velocity when its displacement is 6 cm from mean position?
viii. The energy of the free surface of a liquid drop is 5 times the surface tension of the liquid.
Find the diameter of the drop in C.G.S. system.

Q.2. Select and write the most appropriate answer from the given alternatives for each
sub-question: [7]
i. A particle rotates in U.C.M. with tangential velocity ‘v’ along a horizontal circle of diameter
‘D’. Total angular displacement of the particle in time ‘t’ is _______.
v
(A) vt (B)  t
D
vt 2vt
(C) (D)
2D D

ii. Two springs of force constants K1 and K2 (K1 > K2) are stretched by same force. If W1 and
W2 be the work done stretching the springs then _______.
(A) W1 = W2 (B) W1 < W2
(C) W1 > W2 (D) W1 = W2 = 0

iii. A and B are two steel wires and the radius of A is twice that of B. If they are stretched by the
same load, then the stress on B is _______.
(A) four times that of A (B) two times that of A
(C) three times that of A (D) same as that of A
1
Board Question Paper : March 2016
iv. If sound waves are reflected from surface of denser medium, there is phase change of
_______.

(A) 0 rad (B) rad
4

(C) rad (D)  rad
2

v. A sonometer wire vibrates with frequency n1 in air under suitable load of specific gravity ‘’.
When the load is immersed in water, the frequency of vibration of wire n2 will be _______.
 1  1
(A) n1 (B) n1
 
 
(C) n1 (D) n1
 1  1

CP
vi. For polyatomic molecules having ‘f ’ vibrational modes, the ratio of two specific heats, is
CV
_______.
1 f 2f
(A) (B)
2f 3f
4f 5f
(C) (D)
3f 4f
vii. A body of moment of inertia 5 kg m2 rotating with an angular velocity 6 rad/s has the same
kinetic energy as a mass of 20 kg moving with a velocity of _______.
(A) 5 m/s (B) 4 m/s
(C) 3 m/s (D) 2 m/s
Q.3. A. Define linear S.H.M. Show that S.H.M. is a projection of U.C.M. on any diameter.
B. A metal sphere cools at the rate of 4 C/min. when its temperature is 50 C. Find its rate of
cooling at 45 C if the temperature of surroundings is 25 C. [7]
OR
A. Explain analytically how the stationary waves are formed. Hence, show that the distance

between node and adjacent antinode is .
4
B. A set of 48 tuning forks is arranged in a series of descending frequencies such that each fork
gives 4 beats per second with preceding one. The frequency of first fork is 1.5 times the
frequency of the last fork, find the frequency of the first and 42nd tuning fork. [7]
Q.4. Attempt any THREE: [9]
i. What is the decrease in weight of a body of mass 600 kg when it is taken in a mine of depth
5000 m? (Radius of earth = 6400 km, g = 9.8 m/s2)
ii. State and prove theorem of parallel axes about moment of inertia.
iii. Derive Laplace’s law for spherical membrane of bubble due to surface tension.
iv. A steel wire having cross sectional area 1.5 mm2 when stretched by a load produces a lateral
strain 1.5  105. Calculate the mass attached to the wire.
(Ysteel = 2  1011 N/m2, Poisson’s ratio  = 0.291, g = 9.8 m/s2)

SECTION – II

Q.5. Attempt any SIX: [12]


i. What is ‘diffraction of light’? Explain its two types.
ii. Draw a neat labelled diagram for the construction of ‘cyclotron’.
iii. Distinguish between ‘paramagnetic’ and ‘ferromagnetic’ substances.
iv. Write a short note on surface wave propagation of electromagnetic waves.
v. The combined resistance of a galvanometer of resistance 500  and its shunt is 21 .
Calculate the value of shunt.
22
Physics
vi. The susceptibility of magnesium at 200 K is 1.8  10–5. At what temperature will the
susceptibility decrease by 6  10–6 ?
vii. The co-efficient of mutual induction between primary and secondary coil is 2 H. Calculate
induced e.m.f. if current of 4 A is cut off in 2.5  10–4 seconds.
viii. The decay constant of radioactive substance is 4.33  10–4 per year. Calculate its half life
period.
Q.6. Select and write the most appropriate answer from the given alternatives for each
sub-question: [7]
i. If the polarising angle for a given medium is 60, then the refractive index of the medium is
_______
1 3
(A) (B)
3 2
(C) 1 (D) 3
ii. The resolving power of a telescope depends upon the _______.
(A) length of the telescope (B) focal length of an objective
(C) diameter of an objective (D) focal length of an eyepiece
iii. Electric intensity due to a charged sphere at a point outside the sphere decreases with
_______.
(A) increase in charge on sphere
(B) increase in dielectric constant
(C) decrease in the distance from the centre of sphere
(D) decrease in square of distance from the centre of sphere
iv. In potentiometer experiment, if l1 is the balancing length for e.m.f. of cell of internal
resistance r and l2 is the balancing length for its terminal potential difference when shunted
with resistance R then:
Rr
(A) l1 = l2   (B) l1 = l2  R 
 R   Rr
R r
(C) l1 = l2  R  (D) l1 = l2  
R r  R 

v. The energy of photon of wavelength  is _______.


(h = Planck’s constant, c = speed of light in vacuum)
h
(A) hc (B)
c
 hc
(C) (D)
hc 

vi. Which logic gate corresponds to the truth table given below?
A B Y
0 0 1
0 1 0
1 0 0
1 1 0
(A) AND (B) NOR
(C) OR (D) NAND
vii. The process of superimposing a low frequency signal on a high frequency wave is _______.
(A) detection (B) mixing
(C) modulation (D) attenuation
3
Board Question Paper : March 2016
Q.7. A. State the principle on which transformer works. Explain its working with construction. Derive
an expression for ratio of e.m.f.s and currents in terms of number of turns in primary and
secondary coil.
B. A conductor of any shape, having area 40 cm2 placed in air is uniformly charged with a
charge 0.2 C. Determine the electric intensity at a point just outside its surface. Also, find
the mechanical force per unit area of the charged conductor.
(0 = 8.85  10–12 S.I. units) [7]
OR
A. With the help of a neat labelled diagram, describe the Geiger-Marsden experiment. What is
mass defect?
B. The photoelectric work function for a metal surface is 2.3 eV. If the light of wavelength
6800 Å is incident on the surface of metal, find threshold frequency and incident frequency.
Will there be an emission of photoelectrons or not?
(Velocity of light c = 3  108 m/s, Planck’s constant, h = 6.63  10–34 Js) [7]

Q.8. Attempt any THREE: [9]


i. Determine the change in wavelength of light during its passage from air to glass. If the
refractive index of glass with respect to air is 1.5 and the frequency of light is 3.5  1014 Hz,
find the wave number of light in glass. (Velocity of light in air c = 3  108 m/s)
ii. In biprism experiment, 10th dark band is observed at 2.09 mm from the central bright point on
the screen with red light of wavelength 6400 Å. By how much will fringe width change if
blue light of wavelength 4800 Å is used with the same setting?
iii. Describe Kelvin’s method to determine the resistance of galvanometer by using metre bridge.
iv. Explain the elementary idea of an oscillator with the help of block diagram.

44
Chemistry

BOARD QUESTION PAPER : MARCH 2016


CHEMISTRY
Time: 3 Hours Total Marks: 70
Note:
i. All questions are compulsory.
ii. Answers to the two sections are to be written in the same answer book.
iii. Figures to the right hand side indicate full marks.
iv. Write balanced chemical equations and draw neat and labelled diagrams, wherever necessary.
v. Use of logarithmic table is allowed.
vi. Answer to every question must be started on a new page.

SECTION – I

Q.1. Answer any SIX of the following: [12]


i. What is ferromagnetism? Iron (Z = 26) is strongly ferromagnetic. Explain.
ii. Define boiling point. Write the formula to determine molar mass of a solute using freezing
point depression method.
iii. Write mathematical equations of first law of thermodynamics for the following processes:
a. Adiabatic process b. Isochoric process
iv. Explain graphical method to determine activation energy of a reaction.
v. Write the names and chemical formulae of any one ore of iron and zinc each.
vi. What is the action of
a. Sodium on arsenic? b. Magnesium on bismuth?
vii. Define enthalpy of sublimation. How is it related to enthalpy of fusion and enthalpy of
vaporization?
viii. What are Ellingham diagrams? Write any two features of it.

Q.2. Answer any THREE of the following: [9]


i. Silver crystallises in fcc structure. If density of silver is 10.51 g cm3, calculate the volume of
unit cell.
[Atomic mass of silver (Ag) = 108 g mol1]
ii. The vapour pressure of pure benzene is 640 mm of Hg. 2.175  103 kg of non-volatile solute
is added to 39 g of benzene, the vapour pressure of solution is 600 mm of Hg. Calculate molar
mass of solute (C = 12, H = 1).
iii. Calculate CCl bond enthalpy from the following reaction:
CH3Cl(g) + Cl2(g)  CH2Cl2(g) + HCl(g) ; H =  104 kJ
If CH, ClCl and HCl bond enthalpies are 414, 243 and 431 kJ mol1 respectively.
iv. Define cell constant. Draw a neat and well labelled diagram of primary reference electrode.

Q.3. Answer any ONE of the following: [7]


i. Write four points of differences between properties of nitrogen and other elements of
group 15.
Explain the structure of ClF3.
Conductivity of a solution is 6.23  105 1cm1 and its resistance is 13710 . If the
electrodes are 0.7 cm apart, calculate the cross-sectional area of the electrode.
Why is molality of a solution independent of the temperature?
ii. What are neutral oxides? Explain the nature of zinc oxide with the help of the reactions.
Define ‘molar conductivity’ and ‘zero order reaction’.
In a first order reaction x  y, 40% of the given sample of compound remains unreacted in
45 minutes. Calculate rate constant of the reaction.
5
Board Question Paper : March 2016
Q.4. Select and write the most appropriate answer from the given alternatives for each
sub-question: [7]
i. The molecular formula H2S2O2 represents which oxoacid among the following?
(A) Hydrosulphurous acid (B) Thiosulphurous acid
(C) Sulphuric acid (D) Pyrosulphurous acid
ii. Iodine exists as _______.
(A) polar molecular solid (B) ionic solid
(C) non-polar molecular solid (D) hydrogen bonded molecular solid
iii. Absolute entropies of solids, liquids and gases can be determined by _______.
(A) measuring heat capacity of substance at various temperatures
(B) subtracting standard entropy of reactants from products
(C) measuring vibrational motion of molecules
(D) using formula S = ST  S0

iv. The determination of molar mass from elevation in boiling point is called as _______.
(A) cryoscopy (B) colorimetry
(C) ebullioscopy (D) spectroscopy
v. The process of leaching alumina, using sodium carbonate is called _______.
(A) Baeyer’s process (B) decomposition
(C) cyanide process (D) Hall’s process
vi. On calculating the strength of current in amperes if a charge of 840 C (coulomb) passes through
an electrolyte in 7 minutes, it will be _______.
(A) 1 (B) 2
(C) 3 (D) 4

vii. A  B is a first order reaction with rate 6.6  105 M s1. When [A] is 0.6 M, rate constant of
the reaction is _______.
(A) 1.1  105 s1 (B) 1.1  104 s1
5 1
(C) 9  10 s (D) 9  104 s1

SECTION – II

Q.5. Answer any SIX of the following: [12]


i. Why is Sc3+ colourless while Ti3+ coloured? (Atomic number Sc = 21, Ti = 22)
ii. Illustrate with example, the difference between a double salt and a coordination compound.
iii. How is chlorobenzene prepared from aniline? How is chlorobenzene converted into diphenyl?
iv. What is metamerism? Explain metamerism with suitable examples of ethers.
v. What are ketones? How are ketones classified?
vi. How are
a. 1-nitropropane and b. 2-nitropropane
prepared from suitable oxime?
vii. Define antioxidants. Draw structure of BHT.
viii. What are carbohydrates? Write the reaction for the preparation of nylon-6.
Q.6. Answer any THREE of the following: [9]
i. What are f-block elements? Distinguish between lanthanoids and actinoids.
ii. Explain the terms
a. Optical activity
b. Ligand
c. Interstitial compounds
66
Chemistry
iii. Write the formula of Tetraamminedichloroplatinum(IV) chloride. How is propene converted
into 1-bromopropane and 2-bromopropane?
iv. What are broad-spectrum antibiotics?
How are polythene and neoprene prepared?
Q.7. Answer any ONE of the following: [7]
i. Explain the mechanism of esterification. Write the reactions involved in dehydration of 1, 2
and 3 alcohols.
ii. What are vitamins? Name any two diseases caused by deficiency of vitamin A. Write the
structures of:
a. nucleoside b. nucleotide
How are 1-nitropropane, 2-nitropropane and 2-methyl-2-nitropropane are distinguished from
each other using nitrous acid?
Q.8. Select and write the most appropriate answers from the given alternatives: [7]
i. The preparation of alkyl fluoride from alkyl chloride, in presence of metallic fluorides is
known as _______.
(A) Williamson’s reaction (B) Finkelstein reaction
(C) Swarts reaction (D) Wurtz reaction
ii. Identify the weakest acidic compound amongst the following:
(A) p-Nitrophenol (B) p-Chlorophenol
(C) p-Cresol (D) p-Aminophenol
iii. On acid hydrolysis, propanenitrile gives _______.
(A) propanal (B) acetic acid
(C) propionamide (D) propanoic acid
iv. Which of the following amines yield foul smelling product with haloform and alcoholic
KOH?
(A) Ethylamine (B) Diethylamine
(C) Triethylamine (D) Ethylmethylamine
v. Which of the following is NOT present in DNA?
(A) Adenine (B) Guanine
(C) Thymine (D) Uracil
vi. Amongst the following, identify a copolymer.
(A) Orlon (B) PVC
(C) PHBV (D) Teflon
vii. Phenelzine is used as an _______.
(A) analgesic (B) antiseptic
(C) antipyretic (D) antidepressant

7
Board Question Paper : March 2016

BOARD QUESTION PAPER : MARCH 2016


MATHEMATICS AND STATISTICS
Time: 3 Hours Total Marks: 80
Note:
i. All questions are compulsory.
ii. Figures to the right indicate full marks.
iii. Graph of L.P.P. should be drawn on graph paper only.
iv. Answer to every new question must be written on a new page.
v. Answers to both sections should be written in the same answer book.
vi. Use of logarithmic table is allowed.

SECTION – I

Q.1. (A) Select and write the most appropriate answer from the given alternatives in each of the
following sub-questions: (6) [12]
i. The negation of p  (q  r) is
(A) p  (~ q  r) (B) ~ p  (q  r)
(C) ~ p  (~ q  ~ r) (D) ~ p  (q  ~ r)

ii. If sin1(1  x)  2 sin1x = then x is
2
1
(A)  (B) 1
2
1
(C) 0 (D)
2
iii. The joint equation of the pair of lines passing through (2, 3) and parallel to the coordinate axes is
(A) xy  3x  2y + 6 = 0 (B) xy + 3x + 2y + 6 = 0
(C) xy = 0 (D) xy  3x  2y  6 = 0
(B) Attempt any THREE of the following: (6)
1 1
1 1 2 3   
i. Find (AB) if A =   B = 1 2 
1 2 3 1 2 
ii. Find the vector equation of the plane passing through a point having position vector 3iˆ  2ˆj  kˆ
and perpendicular to the vector 4iˆ  3jˆ  2kˆ .
iii. If p  ˆi  2ˆj  kˆ and q  ˆi  4ˆj  2kˆ are position vector (P.V.) of points P and Q, find the position
vector of the point R which divides segment PQ internally in the ratio 2:1.
iv. Find k, if one of the lines given by 6x2 + kxy + y2 = 0 is 2x + y = 0.
x 1 y  2 z  3 x 1 y  5 z  6
v. If the lines   and   are at right angle then find the value
3 2k 2 3k 1 5
of k.
Q.2. (A) Attempt any TWO of the following: (6)[14]
i. Examine whether the following logical statement pattern is tautology, contradiction or contingency.
[(p  q)  q]  p
ii. By vector method prove that the medians of a triangle are concurrent.
iii. Find the shortest distance between the lines r  (4iˆ  ˆj)   (iˆ  2ˆj  3k)
ˆ and
r  (iˆ  ˆj  2k)
ˆ (iˆ  4ˆj  5k)
ˆ where  and  are parameters.
(B) Attempt any TWO of the following: (8)
i. In ABC with the usual notations prove that
C C
(a  b)2 cos2   + (a + b)2 sin2   = c2.
2  2 

88
Mathematics and Statistics
ii. Minimize z = 4x + 5y subject to 2x + y ≥ 7, 2x + 3y ≤ 15, x ≤ 3, x ≥ 0, y ≥ 0. Solve using
graphical method.
iii. The cost of 4 dozen pencils, 3 dozen pens and 2 dozen erasers is ` 60. The cost of 2 dozen
pencils, 4 dozen pens and 6 dozen erasers is ` 90 whereas the cost of 6 dozen pencils, 2 dozen
pens and 3 dozen erasers is ` 70. Find the cost of each item per dozen by using matrices.

Q.3. (A) Attempt any TWO of the following: (6)[14]


i. Find the volume of tetrahedron whose coterminus edges are 7iˆ  k,ˆ 2iˆ  5jˆ  3kˆ and 4iˆ  3jˆ  kˆ .
ii. Without using truth table show that
~ (p  q)  (~p  q)  ~ p
iii. Show that every homogeneous equation of degree two in x and y, i.e., ax2 + 2hxy + by2 = 0
represents a pair of lines passing through origin if h2  ab ≥ 0.
(B) Attempt any TWO of the following: (8)
i. If a line drawn from the point A(1, 2, 1) is perpendicular to the line joining P(1, 4, 6) and
Q(5, 4, 4) then find the co-ordinates of the foot of the perpendicular.
ii. Find the vector equation of the plane passing through the points ˆi  ˆj  2kˆ , ˆi  2ˆj  kˆ ,
2iˆ  ˆj  kˆ . Hence find the cartesian equation of the plane.
iii. Find the general solution of sin x + sin 3x + sin 5x = 0.

SECTION – II

Q.4. (A) Select and write the most appropriate answer from the given alternatives in each of the
following sub-questions: (6)[12]
i. If the function
f(x) = k + x, for x < 1
= 4x + 3, for x  1
is continuous at x = 1 then k =
(A) 7 (B) 8 (C) 6 (D) –6
ii. The equation of tangent to the curve y = x2 + 4x + 1 at (1, –2) is
(A) 2x – y = 0 (B) 2x + y  5 = 0
(C) 2x – y – 1 = 0 (D) x + y  1 = 0
iii. Given that X ~ B(n = 10, p). If E(X) = 8 then the value of p is
(A) 0.6 (B) 0.7 (C) 0.8 (D) 0.4
(B) Attempt any THREE of the following: (6)
x dy
i. If y = x , find .
dx
ii. The displacement ‘s’ of a moving particle at time ‘t’ is given by s = 5 + 20t – 2t2. Find its
acceleration when the velocity is zero.
iii. Find the area bounded by the curve y2 = 4ax, X-axis and the lines x = 0 and x = a.
iv. The probability distribution of a discrete random variable X is:
X=x 1 2 3 4 5
P(X = x) k 2k 3k 4k 5k
Find P(X  4).
sin x
v. Evaluate:  36  cos 2 x
dx

Q.5. (A) Attempt any TWO of the following: (6)[14]


i. If y = f(u) is a differentiable function of u and u = g(x) is a differentiable function of x then
dy dy du
prove that y = f(g(x)) is a differentiable function of x and   .
dx du dx
9
Board Question Paper : March 2016
ii. The probability that a person who undergoes kidney operation will recover is 0.5. Find the
probability that of the six patients who undergo similar operations.
a. None will recover.
b. Half of them will recover.

x
iii. Evaluate: a
0
2
cos 2 x  b 2 sin 2 x
dx

(B) Attempt any TWO of the following: (8)


i. Discuss the continuity of the following functions. If the function have a removable
discontinuity, redefine the function so as to remove the discontinuity.
4x  ex
f(x) = , for x  0
6x  1
2 at x = 0
= log   , for x = 0
3
ii. Prove that:
x 2 a2 1  x 
 a 2  x 2 dx =
2
a  x 2  sin   + c
2 a
iii. A body is heated at 110C and placed in air at 10C. After 1 hour its temperature is 60C.
How much additional time is required for it to cool to 35C?
Q.6. (A) Attempt any TWO of the following: (6)[14]
2a a a
i. Prove that:  f ( x)dx   f ( x)dx   f (2a  x)dx
0 0 0

1  log x
ii. Evaluate:  dx
x(2  log x)(3  log x)

iii. 
If y = cos1 2 x 1 x 2 , find  dy
dx
(B) Attempt any TWO of the following: (8)
i. Solve the differential equation cos(x + y)dy = dx
Hence find the particular solution for x = 0 and y = 0.
ii. A wire of length l is cut into two parts. One part is bent into a circle and other into a square.
Show that the sum of areas of the circle and square is the least, if the radius of circle is half
the side of the square.
iii. The following is the p.d.f. (Probability Density Function) of a continuous random variable X:
x
f(x) = , 0<x<8
32
=0 otherwise
a. Find the expression for c.d.f. (Cumulative Distribution Function) of X.
b. Also find its value at x = 0.5 and 9.

10
10
Biology

BOARD QUESTION PAPER : MARCH 2016


BIOLOGY
Time: 3 Hours Total Marks: 70
Note:
i. All questions are compulsory.
ii. Answers to Section-I and Section-II should be written in Two Separate answer books.
iii. Questions from Section-I attempted in the answer book of Section-II and vice-versa will not be assessed /
not be given any credit.
iv. Draw neat and labelled diagrams wherever necessary.
v. Figures to the right indicate full marks.
vi. Answer to every new question must begin on a new page.

SECTION – I
[BOTANY]

Q.1. Select and write the most appropriate answer from the given alternatives for each sub-
question: [7]
i. The phenotypic ratio of incomplete dominance is _______.
(A) 2 : 1 (B) 1 : 2 : 1
(C) 1 : 1 : 1 (D) 1 : 1 : 2
ii. The number of purines in a segment of DNA molecule is 68. What will be the number of
pyrimidines in this segment?
(A) 34 (B) 43
(C) 68 (D) 86
iii. Alcoholic fermentation is brought about by _______.
(A) Lactobacillus (B) Saccharomyces
(C) Trichoderma (D) Streptomyces
iv. Which of the following is not a photosynthetic pigment?
(A) Carotene (B) Xanthophyll
(C) Phycobillins (D) Anthocyanin
v. Which one of the following is a stop codon?
(A) UAG (B) UAC
(C) AUG (D) UCA
vi. Pyruvate undergoes oxidative decarboxylation to produce _______.
(A) 2-PGA (B) -Ketoglutarate
(C) Succinyl - Co- A (D) Acetyl - Co – A
vii. Which day is observed as ‘World Environment Day’?
(A) 21st May (B) 5th June
th
(C) 25 September (D) 13th December
Q.2. (A) Answer in ‘One’ sentence only : (6)[12]
i. What is test cross?
ii. What is mycoherbicide?
iii. What is Anticodon?
iv. What is Humification?
v. How CO2 makes idlies puffy?
vi. What is ecological succession?
(B) Sketch and label ‘Ultrastructure of Chloroplast’. (2)
11
Board Question Paper : March 2016
(C) Answer the following (Any TWO): (4)
i. Write a short note on ‘Mutational breeding’.
ii. Enlist the advantages of Biogas.
iii. Explain ‘Carbon cycle’.
iv. Give the floral adaptations for chiropterophily.
Q.3. (A) Answer the following (Any TWO): (6)[9]
i. Why the ratio in pleiotropy is 2 : 1? Explain it with example.
ii. Give the schematic representation of TCA cycle.
iii. Differentiate between cyclic and non-cyclic photophosphorylation.
(B) Give diagrammatic representation to show a perfect pairing and any ‘two’ Wobble pairings. (3)
Q.4. What is double fertilization? Describe the process in brief. [7]
OR
Define r-DNA technology. Give the basic steps in r-DNA technology and give any ‘three’ examples
of the therapeutic products produced by r-DNA technology.

SECTION – II
[ZOOLOGY]

Q.5. Select and write the most appropriate answer from the given alternatives for each sub-
question: [7]
i. If centromere is situated in the middle of the chromosome, it is called _______.
(A) metacentric (B) acrocentric
(C) submetacentric (D) telocentric
ii. Which one of the following is useful in treatment of burns and wound healing?
(A) Tissue plasminogen activator.
(B) Tissue Growth Factor.
(C) DNAse
(D) Bovine growth hormone.
iii. Erythroblastosis foetalis is caused when mother is
(A) Rh+ve (B) with antibody ‘a’
(C) Rh–ve (D) with antibody ‘b’
iv. Deposition of fatty substances in the lining of arteries results in _______.
(A) arteriosclerosis (B) atherosclerosis
(C) hyperglycemia (D) hypotension
v. Struggle between cow and cow for getting grass is called _______.
(A) inter-specific struggle (B) environmental struggle
(C) struggle against natural calamities (D) intra-specific struggle
vi. In which type of adaptation, forelimbs are modified into wings?
(A) Aquatic adaptation (B) Volant adaptation
(C) Arboreal adaptation (D) Cursorial adaptation
vii. Mating of two closely related individuals within the same breed is called _______.
(A) in-breeding (B) out-breeding
(C) out-crossing (D) cross-breeding
Q.6. (A) Answer in ‘One’ sentence each: (6) [12]
i. Visit of a veterinary doctor to dairy farm is mandatory. Give reasons.
ii. Why aquatic animals can afford to be ammonotelic?
iii. Why PUC is mandatory for all vehicles?
iv. Define organic evolution.
v. Give the genotype of Turner’s syndrome.
vi. Write down the full form of R.F.L.P.
(B) Sketch and label T.S. of vein. (2)

12
12
Biology
(C) Attempt any TWO of the following: (4)
i. Give any ‘two’ factors responsible for genetic variation.
ii. Give the name and functions of ‘A’ and ‘B’ from the diagram given below:
A

iii. Write a note on ‘artificial acquired active immunity’.


iv. Give the economic importance of ‘fisheries’.
Q.7. (A) Attempt any TWO of the following: (6) [9]
i. Explain sex determination in human beings.
ii. Describe steady population with the help of a pyramid.
iii. Illustrate any ‘three’ genes which can be used in gene therapy.
(B) Sketch and label ‘L.S. of human kidney’. (3)
Q.8. With the help of diagrammatic representation, explain the process of gametogenesis. [7]
OR
Describe the structure of cerebrum. Add a note on its functions.

13
Board Question Paper : July 2016

BOARD QUESTION PAPER : JULY 2016


PHYSICS
Time: 3 Hours Total Marks: 70

Note:
i. All questions are compulsory.
ii. Neat diagrams must be drawn wherever necessary.
iii. Figures to the right indicate full marks.
iv. Use of only logarithmic table is allowed.
v. All symbols have their usual meaning unless otherwise stated.
vi. Answers to both sections must be written in the same answerbook.
vii. Answer to every question must be written on a new page.

SECTION – I

Q.1. Select and write the most appropriate answer from the given alternatives for each
sub-question: [7]
i. The difference in tensions in the string at lowest and highest points in the path of the particle
of mass ‘m’ performing vertical circular motion is:
(A) 2 mg (B) 4 mg
(C) 6 mg (D) 8 mg
ii. The body is rotating with uniform angular velocity () having rotational kinetic energy (E).
Its angular momentum (L) is:
2E E2
(A) (B)
ω ω
E E
(C) (D)
ω2 2ω

iii. The S.I. unit of compressibility is ______.


m2
(A) (B) Nm2
N
N kg
(C) (D)
m2 m3

iv. The working of RADAR is based on ______.


(A) resonance (B) speed of a star
(C) Doppler effect (D) speed of rotation of sun
v. If two capillary tubes of different diameters are partially dipped in the same liquid vertically,
then the rise of liquid ______.
(A) is same in both the tubes (B) is more in the tube of larger diameter
(C) will not be in smaller diameter tube (D) is more in the tube of smaller diameter
vi. A sonometer wire vibrates with three nodes and two antinodes, the corresponding mode of
vibration is ______.
(A) first overtone (B) second overtone
(C) third overtone (D) fourth overtone
vii. Two gases exert pressure in the ratio 3:2 and their densities are in the ratio 2:3, then the ratio
of their R.M.S. velocities is ______.
(A) 2:3 (B) 3 : 2
(C) 2:1 (D) 1 : 2

14
14
Physics
Q.2. Attempt any SIX: [12]
i. Draw a neat labelled diagram showing the various forces and their components acting on a
vehicle moving along curved banked road.
ii. Obtain an expression for critical velocity of a satellite orbiting around the earth.
iii. Draw a neat labelled diagram of rise of liquid in capillary tube showing different components
of tension (force).
iv. State any four assumptions of kinetic theory of gases.
v. A tube open at both ends has length 47 cm. Calculate the fundamental frequency of air
column. (Neglect end correction. Speed of sound in air is 3.3  102 m/s).
vi. A uniform solid sphere has a radius 0.1 m and density 6  103 kg/m3. Find its moment of
inertia about a tangent to its surface.
vii. A particle executes S.H.M. with a period of 10 seconds. Find the time in which its potential
energy will be half of its total energy.
viii. A stone of mass 2 kg is whirled in a horizontal circle attached at the end of 1.5 m long string.
If the string makes an angle of 30o with vertical, compute its period. (g = 9.8 m/s2)
Q.3. Attempt any THREE: [9]
i. State Kepler’s laws of planetary motion.
ii. Obtain an expression for torque acting on a body rotating with uniform angular acceleration.
iii. A steel wire having cross-sectional area 2 mm2 is stretched by 10 N. Find the lateral strain
produced in the wire.
(Given : Y for steel = 2  1011 N/m2, Poisson’s ratio  = 0.29)
iv. A body cools from 62 oC to 54 oC in 10 minutes and to 48 oC in the next 10 minutes. Find the
temperature of the surroundings.
Q.4. A. Explain the formation of stationary wave by analytical method. Show that nodes and
antinodes are equally spaced in a stationary wave.
B. The speed limit for a vehicle on road is 120 km/hr. A policeman detects a drop of 10% in the
pitch of horn of a car as it passes him. Is the policeman justified in punishing the car driver
for crossing the speed limit?
(Given : Velocity of sound = 340 m/s) [7]
OR

A. Define practical simple pendulum.


Show that motion of bob of pendulum with small amplitude is linear S.H.M. Hence obtain an
expression for its period. What are the factors on which its period depends?
B. The total free surface energy of a liquid drop is π 2 times the surface tension of the liquid.
Calculate the diameter of the drop in S.I. unit. [7]

SECTION – II

Q.5. Select and write the most appropriate answer from the given alternatives for each
sub-question: [7]
i. A parallel beam of light travelling in water is incident obliquely on a glass surface. After
refraction its width ______.
(A) decreases (B) increases
(C) remains the same (D) becomes zero
ii. If ‘a’ is the aperture of telescope and ‘’ is the wavelength of light then resolving power of
telescope is ______.
λ 1.22a 1.22λ a
(A) (B) (C) (D)
1.22a λ a 1.22λ

iii. From earth’s surface, ionospheric layer of atmosphere lies between ______.
(A) 12 km to 50 km (B) 50 km to 80 km
(C) 80 km to 400 km (D) 400 km to 700 km

15
Board Question Paper : July 2016
iv. The kinetic energy of emitted photoelectrons is independent of ______.
(A) frequency of incident radiation (B) intensity of incident radiation
(C) wavelength of incident radiation (D) collector plate potential
v. In hydrogen atom Balmer series is obtained when the electron jumps from ______.
(A) higher orbit to first orbit (B) first orbit to a higher orbit
(C) higher orbit to the second orbit (D) second orbit to a higher orbit
vi. The fraction of the total current passing through the galvanometer is ______.
S G SG SG
(A) (B) (C) (D)
SG SG G S
vii. A metre gauge train is heading north with speed 54 km/hr in earth’s magnetic field
3  10–4 T. The e.m.f. induced across the axle joining the wheels is ______.
(A) 0.45 mV (B) 4.5 mV
(C) 45 mV (D) 450 mV
Q.6. Attempt any SIX: [12]
i. Distinguish between intrinsic and extrinsic semiconductor. (Give any two points).
ii. Draw the block diagram of a receiver in communication system.
iii. A point is situated at 6.5 cm and 6.65 cm from two coherent sources. Find the nature of
illumination at the point, if wavelength of light is 5000 Å.
iv. Draw the diagrams showing the dipole moments in paramagnetic substance when external
magnetic field is (a) absent (b) strong.
v. A voltmeter has a resistance of 100 . What will be its reading when it is connected across a
cell of e.m.f. 2 V and internal resistance 20 ?
vi. The susceptibility of magnesium at 300 K is 1.2  10–5. At what temperature will the
susceptibility increase to 1.8  10–5?
vii. What is de Broglie wavelength of an electron accelerated through 25000 volt?
viii. Draw the schematic symbols for AND, OR, NOT and NAND gate.
Q.7. Attempt any THREE: [9]
i. Using analytical method for interference bands, obtain an expression for path difference
between two light waves.
ii. State law of radioactive decay. Hence, derive the relation N = N0e–t. Represent it graphically.
iii. Determine the change in wavelength of light during its passage from air to glass, if the
refractive index of glass with respect to air is 1.5 and the frequency of light is 5  1014 Hz.
Find the wave number of light in glass (velocity of light in air c = 3  108 m/s).
iv. Light of wavelength 3000 Å falls on a metal surface having work function 2.3 eV. Calculate
the maximum velocity of ejected electrons.
(Planck’s constant h = 6.63  10–34 Js, Velocity of light c = 3  108 m/s,
mass of an electron = 9.1  10–31 kg)
Q.8. A. What is electromagnetic induction?
d
Prove theoretically e = –
dt
B. A potentiometer wire has length of 2 m and resistance 10 . It is connected in series with
resistance 990  and a cell of e.m.f. 2 V. Calculate the potential gradient along the wire. [7]
OR
A. With the help of a neat diagram, describe the construction and working of Van de Graaff
generator.
B. A moving coil galvanometer has a resistance of 25  and gives a full scale deflection for a
current of 10 mA. How will you convert it into a voltmeter having range 0 – 100 V? [7]

16
16
Chemistry

BOARD QUESTION PAPER : JULY 2016


CHEMISTRY
Time: 3 Hours Total Marks: 70
Note:
i. All questions are compulsory.
ii. Answers to the two sections are to be written in the same answer book.
iii. Figures to the right hand side indicate full marks.
iv. Write balanced chemical equations and draw neat and labelled diagrams, wherever necessary.
v. Use of logarithmic table is allowed.
vi. Answer to every question must be started on a new page.

SECTION – I

Q.1. Select and write the most appropriate answer from the given alternatives for each
sub-question: [7]
i. Schottky defects are observed in which solid among the following?
(A) Brass (B) Cesium Chloride
(C) Zinc sulphide (D) Stainless steel
ii. ‘No machine has an efficiency unity’, is stated in _______ .
(A) first law of thermodynamics (B) second law of thermodynamics
(C) third law of thermodynamics (D) Hess’s law of constant heat summation
iii. Which among the following reactions is an example of a zero order reaction?
(A) H2(g) + I2(g)  2HI(g)
(B) 2H2O2(l)  2H2O(l) + O2(g)
(C) C12H22O11(aq) + H2O(l)  C6H12O6(aq) + C6H12O6(aq)
(D) 2NH3(g)  Pt
 N2(g) + 3H2(g)
iv. Potential of saturated calomel electrode is _______ .
(A) 0.242 V (B) 1.1 V (C) 0.337 V (D) 0.28 V
v. Which of the following compounds is used as a semipermeable membrane?
(A) Potassium ferrocyanide (B) Potassium argentocyanide
(C) Sodium meta aluminate (D) Copper ferrocyanide
vi. Which among the following minerals does NOT contain aluminium?
(A) Cryolite (B) Siderite
(C) China clay (D) Corundum
vii. The group 15 element having inner electronic configuration as of argon is ______ .
(A) phosphorus (Z = 15) (B) antimony (Z = 51)
(C) arsenic (Z = 33) (D) nitrogen (Z = 7)
Q.2. Answer any SIX of the following: [12]
i. Write chemical reactions involved in Van Arkel method for refining titanium.
ii. Explain the relationship between Gibbs standard energy change of the reaction and
equilibrium constant.
iii. A face centred cube (fcc) consists of how many atoms? Explain.
iv. Describe isolation method in determination of rate law and order of reaction.
v. Explain the following methods to protect metals from corrosion:
a. Galvanization b. Passivation
vi. Write the Nernst equation and explain the terms involved.
vii. What happens when dilute sulphuric acid is treated with:
a. Fe? b. CaF2?
viii. Define:
a. Osmosis b. Freezing point
17
Board Question Paper : July 2016
Q.3. Answer any THREE of the following: [9]
i. The rate constant of a first order reaction are 0.58 s–1 at 313 K and 0.045 s–1 at 293 K. What is
the energy of activation for the reaction?
ii. Calculate the standard enthalpy of the reaction,
2C(graphite) + 3H2(g)  C2H6(g) ; H = ?
from the following H values:
7
a. C2H6(g) + O2(g)  2CO2(g) + 3H2O(l) ; H = –1560 kJ
2
1
b. H2(g) + O2(g)  H2O(l) ; H = –285.8 kJ
2
c. C(graphite) + O2(g)  CO2(g) ; H = –393.5 kJ.
iii. 3.795 g of sulphur is dissolved in 100 g of CS2. This solution boils at 319.81 K. What is
molecular formula of sulphur in solution? The boiling point of CS2 is 319.45 K.
(Given that Kb for CS2 = 2.42 K kg mol–1 and atomic mass of S = 32.)
iv. Write the reactions involved in large scale preparation of nitric acid.
Q.4. Answer any ONE of the following: [7]
i. State third law of thermodynamics. Write applications of standard molar entropy.
Draw neat labelled diagram of electrolytic refining of blister copper.
Determine the density of cesium chloride which crystallizes in bcc type structure with the
edge length 412.1 pm. The atomic masses of Cs and Cl are 133 and 35.5 respectively.
Predict the coordination number of Cs+ ion if rCs = 1.69 Å and r  = 1.81 Å.
Cl
ii. What happens when thin copper leaves are thrown in jar containing chlorine?
H2O is liquid while H2S is gas at room temperature. Explain.
The conductivity of 0.02 M AgNO3 at 25C is 2.428  10–3 –1 cm–1. What is its molar
conductivity? State Henry’s law.

SECTION – II

Q.5. Select and write the most appropriate answer from the given alternatives for each
sub-question: [7]
i. Sodium acetate reacts with ethanoyl chloride to form _______.
(A) acetic acid (B) acetone
(C) acetic anhydride (D) sodium formate
ii. Natalite is a mixture of _______.
(A) diethyl ether and methanol (B) diethyl ether and ethanol
(C) dimethyl ether and methanol (D) dimethyl ether and ethanol
iii. What is effective atomic number of Fe (Z = 26) in [Fe(CN)6]4–?
(A) 12 (B) 30
(C) 26 (D) 36
iv. Maltose is a ______ .
(A) polysaccharide (B) disaccharide
(C) trisaccharide (D) monosaccharide
v. Which one of the following oxidation state of manganese is unstable?
(A) +2 (B) +4 (C) +5 (D) +7
vi. IUPAC name of the following compound is _______ .

Br
(A) 3-Bromo-3,4-dimethylheptane (B) 3,4-Dimethyl-3-bromoheptane
(C) 5-Bromo-4,5-dimethylheptane (D) 4,5-Dimethyl-5-bromoheptane
18
18
Chemistry
vii. Which of the following compounds is NOT prepared by the action of alcoholic NH3 on alkyl
halide?
(A) CH3  NH2 (B) CH3 – CH2 – NH2
(C) CH3 – CH2 – CH2 – NH2 (D) (CH3)3C – NH2
Q.6. Answer any SIX of the following: [12]
i. Write IUPAC names of the following compounds:
a. O b.
O O
CH3 CH3 – C – C – CH3
ii. What are the sources of vitamin C and vitamin K?
iii. Write four points of distinction between lanthanoids and actinoids.
iv. How is benzonitrile converted to benzophenone?
v. Write the formulae of the raw materials used for preparation of:
a. Buna-S b. Dextron
vi. Write a note on Sandmeyer’s reaction.
vii. What is the action of benzene diazonium chloride on:
a. phenol in alkaline medium? b. aniline?
viii. Explain any two chemical methods of food preservation.
Q.7. Answer any THREE of the following: [9]

i. What is the action of following reagents on glucose?


a. Bromine water b. Dilute HNO3
c. Hydroxylamine
ii. Define ligand. Write four postulates of Werner’s theory.
iii. Write reactions involved in preparation of potassium dichromate from chrome iron ore.
iv. What is metamerism?
Write the structure and IUPAC name of methyl-n-propyl ether.
What is the action of hot HI on it?
Q.8. Answer any ONE of the following: [7]
i. a. How are the following polymers prepared?
1. Orlon 2. Teflon

b. Classify the following drugs into analgesics and antibiotics.


1. Ofloxacin 2. Morphine
3. Ampicillin 4. Chloramphenicol
c. Identify ‘A’ and ‘B’ and rewrite the reactions.

1. CH3
NH2

NaNO2
HBr
(A) 
Cu 2 Br2
 (B) + N2 
273K

2. H3C – CH2 – CH – CH3 


Alcoholic KOH
 (A) + (B) + 2KBr + 2H2O

Br
 
3. C2H5 – N  CH 3 3 I 
Ag 2 O/ H 2 O


 (A)   (B) + (CH3)3N + H2O

ii. How are the following conversions carried out?


a. 2-Methylbutan-l-ol into 2-methylbutanoic acid
b. Phenylethene into benzoic acid
c. Benzoic acid into meta-nitrobenzoic acid.
What is the action of benzene sulphonyl chloride on primary, secondary and tertiary amines?
Write two uses of formaldehyde.

19
Board Question Paper : July 2016

BOARD QUESTION PAPER : JULY 2016


MATHEMATICS AND STATISTICS
Time: 3 Hours Total Marks: 80
Note:
i. All questions are compulsory.
ii. Figures to the right indicate full marks.
iii. Graph of L.P.P. should be drawn on graph paper only.
iv. Answer to every new question must be written on a new page.
v. Answers to both sections should be written in the same answer book.
vi. Use of logarithmic table is allowed.

SECTION – I

Q.1. (A) Select and write the correct answer from the given alternatives in each of the following
sub-questions: (6)[12]
i. Inverse of the statement pattern (p  q)  (p  q) is
(A) (p  q)  (p  q) (B)  (p  q)  (p  q)
(C) ( p   q)  ( p   q) (D) ( p   q)  ( p   q)

ii. If the vectors 2iˆ  qjˆ  3kˆ and 4iˆ  5jˆ  6kˆ are collinear, then value of q is
5 5
(A) 5 (B) 10 (C) (D)
2 4

A
iii. If in ABC with usual notations a = 18, b = 24, c = 30 then sin is equal to
2
1 1 1 1
(A) (B) (C) (D)
5 10 15 2 5

(B) Attempt any THREE of the following: (6)


i. Find the angle between the lines
  
r  3iˆ  2ˆj  4kˆ   ˆi  2jˆ  2kˆ and r  5iˆ  2kˆ  3iˆ  2ˆj  6kˆ 
ii. If p, q, r are the statements with truth values T, F, T, respectively then find the truth value of
(r  q)   p

 2 3
iii. If A =  then find A-1 by adjoint method.
3 5 

iv. By vector method show that the quadrilateral with vertices A (1, 2, –1), B (8, –3, –4),
C (5, –4, 1), D (–2, 1, 4) is a parallelogram.

v. Find the general solution of the equation sin x = tan x.

Q.2. (A) Attempt any TWO of the following: (6)[14]


i. Find the joint equation of pair of lines passing through the origin and perpendicular to the
lines represented by ax2 + 2hxy + by2 = 0
 1 
ii. Find the principal value of sin–1  
 2
iii.     
Find the cartesian form of the equation of the plane r  ˆi  ˆj  s ˆi  ˆj  2kˆ  t ˆi  2ˆj  kˆ 
20
20
Mathematics and Statistics
(B) Attempt any TWO of the following: (8)
i. Simplify the following circuit so that new circuit has minimum number of switches. Also
draw simplified circuit.

S1 S2

S1 S2

S1 S2
L

ii. A line makes angles of measures 45 and 60 with positive direction of y and z axes
respectively. Find the d.c.s. of the line and also find the vector of magnitude 5 along the
direction of line.
iii. Maximize:
z = 3x + 5y
Subject to: x + 4y  24
3x + y  21
x+y9
x  0, y  0
Q.3. (A) Attempt any TWO of the following: (6)[14]
x +1 y +1 z +1 x 3 y 5 z 7
i. Find the shortest distance between the lines   and  
7 6 1 1 2 1
ii. Show that the points (1, –1, 3) and (3, 4, 3) are equidistant from the plane
5x + 2y – 7z + 8 = 0
iii. In any triangle ABC with usual notations prove c = a cos B + b cos A

(B) Attempt any TWO of the following: (8)


i. Find p and k if the equation px2 – 8xy + 3y2 +14x + 2y + k = 0 represents a pair of
perpendicular lines.
ii. The cost of 4 dozen pencils, 3 dozen pens and 2 dozen erasers is ` 60. The cost of 2 dozen
pencils, 4 dozen pens and 6 dozen erasers is ` 90 whereas the cost of 6 dozen pencils, 2 dozen
pens and 3 dozen erasers is ` 70. Find the cost of each item per dozen by using matrices.
iii. Prove that the volume of the parallelopiped with coterminus edges as a ,b,c is a ,b,c  and

hence find the volume of the parallelopiped with its coterminus edges 2iˆ  5jˆ  4k,
ˆ

5iˆ  7ˆj  5k,


ˆ and 4iˆ  5jˆ  2kˆ .

SECTION – II

Q.4. (A) Select and write the correct answer from the given alternatives in each of the following
sub-questions: (6)[12]
7
  dy  
3 3
 d2 y 
i. The order and degree of the differential equation 1     7  2  are respectively.
  dx    dx 
(A) 2, 3 (B) 3, 2
(C) 2, 2 (D) 3, 3
9
1
ii. 
4 x
dx = _______

(A) 1 (B) –2
(C) 2 (D) –1
21
Board Question Paper : July 2016
iii. If the p.d.f. of a continuous random variable X is given as
x2
f(x) = for –1  x  2
3
=0 otherwise.
then c.d.f. of X is
x3 1 x3 1
(A)  (B) 
9 9 9 9
x2 1 1 1
(C)  (D) 
4 4 9 x3 9

(B) Attempt any THREE of the following: (6)


dy
i. If y = sec x then find .
dx

ii. Evaluate : 
 x 1 dx
 2  x  3
x 
iii. Find the area of the region lying in the first quadrant bounded by the curve y2 = 4x, X axis and
the lines x = 1, x = 4.
iv. Solve the differential equation sec2 x tan y dx + sec2 y tan x dy = 0
v. Given X ~ B(n, p) if E(X) = 6, Var (X) = 42, find the value of n and p.
Q.5. (A) Attempt any TWO of the following: (6)[14]
4  1
sin x 2

i. If the function f (x) = , for x  0 is continuous at x = 0, find f (0).


x  log 1 2 x 
1
ii. Evaluate :  3  2 sin x  cos x dx
iii. If y = f(x) is differentiable function of x such that inverse function x = f–1 (y) exists then prove
dx 1 dy
that x is a differentiable function of y and  , where 0
dy   d y dx
 
 dx 

(B) Attempt any TWO of the following: (8)


i. A point source of light is hung 30 feet directly above a straight horizontal path on which a
man of 6 feet in height is walking. How fast is the man’s shadow lengthening and how fast
the tip of shadow is moving when he is walking away from the light at the rate of 100 ft/min?
ii. The p.m.f. for X = number of major defects in a randomly selected appliance of a certain type
is
X=x 0 1 2 3 4
P (x) 008 015 045 027 005

Find the expected value and variance of X.


a a
iii. Prove that  f  x  dx =  f  a – x  dx
0 0

2
sin x
hence evaluate  sin x  cos x  dx
0

Q.6. (A) Attempt any TWO of the following: (6)[14]


dy
i. If y = etan x + (log x)tan x then find
dx
ii. If the probability that a fluorescent light has a useful life of at least 800 hours is 09, find the
probabilities that among 20 such lights at least 2 will not have a useful life of at least 800
hours. [Given : (09)19 = 01348]
22
22
Mathematics and Statistics

iii. Find  and , so that the function f (x) defined by



f (x) = –2 sin x, for –  x  –
2
 
=  sin x + , for – x
2 2

= cos x, for  x   is continuous on [–, ]
2

(B) Attempt any TWO of the following: (8)


i. Find the equation of a curve passing through the point (0, 2), given that the sum of the
coordinates of any point on the curve exceeds the slope of the tangent to the curve at that
point by 5.
ii. If u and v are two functions of x then prove that :
 du 
 uv dx  u  v dx –   dx  vdx  dx

Hence evaluate  xe x dx
iii. Find the approximate value of log10 (1016) given that log10 e = 04343.

23
Board Question Paper : July 2016

BOARD QUESTION PAPER : JULY 2016


BIOLOGY
Time: 3 Hours Total Marks:70

Note:
i. All questions are compulsory.
ii. Answers to Section-I and Section-II should be written in Two Separate answer books.
iii. Questions from Section-I attempted in the answer book of Section-II and vice-versa will not be assessed /
not be given any credit.
iv. Draw neat and labelled diagrams wherever necessary.
v. Figures to the right indicate full marks.
vi. Answer to every new question must begin on a new page.

SECTION – I
[BOTANY]

Q.1. Select and write the most appropriate answer from the given alternatives for each sub-
question: [7]
i. Genotype of human blood group ‘O’ will be __________.
(A) IAIA (B) IAIB
(C) ii (D) IAi
ii. ‘Cry’ genes are present in __________.
(A) Agrobacterium tumefaciens (B) Bacillus thuringiensis
(C) Rhizobium species (D) Escherichia coli
iii. ‘Tropane’ is obtained from __________.
(A) Daucus carota (B) Catharanthus roseus
(C) Datura stramoneum (D) Mentha piperata
iv. Which of the following vitamins is not fat soluble?
(A) Vitamin A (B) Vitamin B
(C) Vitamin D (D) Vitamin K
v. How many meiotic divisions are required to produce 200 seeds?
(A) 50 (B) 100
(C) 200 (D) 250
vi. Energy flow in ecosystem is __________.
(A) reverse (B) unidirectional
(C) bidirectional (D) multidirectional
vii. Endosperm of angiosperm is __________.
(A) haploid (B) diploid
(C) triploid (D) tetraploid
Q.2. (A) Answer in ‘One’ sentence only : (6)[12]
i. What is Bacteriophage?
ii. What is emasculation?
iii. What is VAM?
iv. What is the role of decomposers in an ecosystem?
v. Name any two edible mushrooms.
vi. What is ecological succession?
(B) Sketch and label the ‘Ultrastructure of Mitochondrion’. (2)
24
24
Biology
(C) Answer the following (Any TWO): (4)
i. Give the floral adaptations for anemophily.
ii. Name any two antibiotics with their microbial source.
iii. Describe the formation of helobial endosperm.
iv. Give the hazardous effects of agrochemicals.
Q.3. (A) Answer the following (Any TWO): (6)[9]
i. Explain incomplete dominance with suitable example.
ii. What are transgenic plants? Explain with any two examples.
iii. Explain micropropagation and somatic hybridization.
(B) Sketch and label stages in development of angiospermic female gametophyte from functional
megaspore. (3)
Q.4. Give the central dogma of protein synthesis. Explain the process of translation. [7]
OR
What is dark reaction in photosynthesis? Describe C3 Pathway.

SECTION – II
[ZOOLOGY]

Q.5. Select and write the most appropriate answer from the given alternatives for each sub-
question: [7]
i. Vermiform appendix is an example of __________ organ.
(A) vestigeal (B) homologous
(C) sense (D) analogous
ii. In which of the following disorders number of chromosomes present is 47?
(A) Turner’s syndrome
(B) Cushing’s syndrome
(C) Acquired Immuno - Deficiency Syndrome
(D) Down’s syndrome
iii. Human blood clotting factor VIII is used to treat __________.
(A) pituitary dwarfism (B) diabetes mellitus
(C) haemophilliacs (D) cystic fibrosis
iv. A person suffering from diabetes insipidus shows __________.
(A) hyperglycemia (B) hypoglycemia
(C) polyuria (D) hypertension
v. Photoreceptor cells are present in __________.
(A) blind spot (B) retina
(C) cochlea (D) cornea
vi. ‘HAART’ is suggested for the treatment of __________
(A) malaria (B) cancer
(C) AIDS (D) high chloesterol level
vii. For DNA fingerprinting radioactive probe obtained from __________ is used in India.
(A) Banded Krait (B) King Cobra
(C) Viper (D) Rat snake
Q.6. (A) Answer in ‘One’ sentence each: (6)[9]
i. Give any two examples of commensalism.
ii. What is the common name of Apis mellifera?
iii. Which growth hormone is used to enhance dairy productivity?
iv. Define Linkage.
v. Name of two classes to which Archaeopteryx acts as connecting link.
vi. Define the term natality.
(B) Sketch and label Malpighian body. (2)

25
Board Question Paper : July 2016
(C) Attempt any TWO of the following: (4)
i. Enlist any four sequential evolutionary names of human ancestors.
ii. Give the significance of transgenic animals for betterment of life.
iii. Give the economic importance of lac.
iv. Give the fate of mesoderm.
Q.7. (A) Attempt any TWO of the following: (6)[9]
i. Explain the mechanism of sex determination in birds.
ii. Give any two unique features of acquired immunity.
iii. With the help of pyramid, describe the growing population.
(B) Sketch and label reflex arc. (3)
Q.8. Explain the process of early cleavage till the formation of morula. [7]
OR
Describe five types of leucocytes, with the help of diagrams. Add a note on their functions.

26
26
BOARD ANSWER
PAPERS

27
Board Answer Paper : March 2016

BOARD ANSWER PAPER : MARCH 2016


PHYSICS
Note: Answer to every question must be written on a new page.

SECTION – I

Q.1. Attempt any SIX:


i. a. Consider a particle performing circular motion in anticlockwise sense with centre
O and radius r as shown in the figure.

b. Let,  = angular velocity of the particle

v = linear velocity of the particle

r = radius vector of the particle



O  v
r
[½]
c. Linear displacement in vector form is given by,
  
s =   r [½]
Dividing both side by t,
 
s  
=  r ….(1)
t t
d. Taking limiting value in equation (1)
 
s  
lim = lim  r
t  0 t t  0 t
 
ds d 
 =  r [½]
dt dt

ds 
But, = v = linear velocity,
dt

d 
=  = angular velocity
dt
  
 v =   r [½]
ii. Expression for critical velocity:
m
vC
h
r

M
Earth R

28
28
Physics
a. Let, M = mass of the earth
R = radius of the earth
h = height of the satellite from the earth’s surface
m = mass of the satellite
vc = critical velocity of the satellite in the given orbit
r = (R + h) = radius of the circular orbit [1]
b. For the circular motion of the satellite, the necessary centripetal force is given as,
mvc2
FCP = ….(1)
r
c. The gravitational force of attraction between the earth and the satellite is given by,
GMm
FG = .…(2)
r2
d. Gravitational force provides the centripetal force necessary for the circular motion of
the satellite.
 FCP = FG
mv c2 GMm
 = ….[From equations (1) and (2)] [½]
r r2
GM
 vc2 =
r
GM
 vc = .…(3)
r
e. But, r = R + h
GM
 vc = ….(4) [½]
(R  h)
Also, GM = gh (R + h)2
where, gh is acceleration due to gravity at height ‘h’ above earth’s surface.
 vc = gh  R  h  ….(5)
Equations (4) and (5) represent critical velocities of satellite orbiting at a certain
height above the earth’s surface.
iii. a. Let, M = mass of the body
v = linear velocity of the body
 = angular velocity of the body
I = moment of inertia of the body
K= radius of gyration
b. Translational kinetic energy of body,
1
(K.E.)translational = Mv2 [½]
2
Similarly,
Rotational kinetic energy of rolling body is
1
(KE)rotational = I2 [½]
2
c. Total K.E of rolling body = (K.E) translational + (K.E) rotational
1 1 2
(K.E)rolling = Mv2 + I
2 2
But I = MK2 [½]
and v = r [½]
1 1  v2 
 (K.E)rolling = Mv2 + MK2  2 
2 2 r 
1  K2 
 (K.E)rolling = Mv2 1 + 2 
2  r 

29
Board Answer Paper : March 2016
iv. a. Emissive power: The emissive power of a body at a given temperature is defined as
the quantity of radiant energy emitted by the body per unit time per unit surface area
of the body at that temperature. [1]
b. Coefficient of emission of a body: The ratio of the emissive power of a body at a given
temperature to the emissive power of a perfectly black body at the same temperature is
called coefficient of emission (emissivity) of the body. [1]
v. Solution:
90
Given: r = 5 cm = 0.05 m, n = 90 r.p.m. = r.p.s., g = 9.8 m/s2
60
To find: Coefficient of static friction (s)
r2
Formula: s = [½]
g
Calculation: Since,
2    90
 = 2n = = 3 rad/s
60
0.05  (3) 2
 s = [½]
9.8
0.45  (3.14) 2
=
9.8
= {antilog [( 1 .6532) + 2(0.4969) – (0.9912)]}
= {antilog( 1 .6558)}
= 0.4527
Ans: The coefficient of static friction between the coin and the turntable is 0.4527. [1]
vi. Solution:
Given: Fundamental frequency for closed pipe = Third overtone of open pipe.
l 
To find: Ratio of lengths of air columns in both the pipes  c 
 lo 
Formulae: a. Fundamental frequency of pipe closed at one end:
v
nc = [½]
4lc
b. Fundamental frequency of pipe open at both ends:
v
no = [½]
2lo
c. Third overtone of open pipe:
n3 = 4n0
Calculation: Fundamental frequency of closed pipe (nc) is same as third overtone of
open pipe.
 nc = n3
 nc = 4n0 [½]
v  v 
 = 4 
4lc  2lo 

1 2
 =
4lc l0
lc 1
 =
lo 8
Ans: The ratio of lengths of air columns in closed pipe and open pipe is 1:8. [½]
vii. Solution:
Given: 2A = 20 cm, A = 10 cm, T = 6.28 s, x = 6 cm
To find: Velocity (v)
Formula: v =  A2  x 2 [½]

30
30
Physics
2 2  3.14
Calculation: Since,  = = = 1 rad/s
T 6.28
 v = (1) (10)2  (6) 2 [½]
= 100  36 = 8 cm/s
Ans: The velocity of the particle at x = 6 cm is 8 cm/s. [1]
viii. Solution:
Given: E = 5T
To find: Diameter of the drop (d) in C.G.S. system
Formula: E = TA
Calculation: A = 4r2
 E = 4r2T
 5T = 4r2T [½]
 4r2 = 5
5 5
 r= =
4 2
 d = 2r = 5 [½]
= 2.236 cm
Ans: Diameter of the drop is 2.236 cm. [1]
Q.2. Select and write the most appropriate answer from the given alternatives for each
sub-question:
i. (D)
s
=
r
D
s = vt and r =
2
2vt
 = [1]
D
ii. (B)
1
K and x  W
x
1
 W [1]
K
iii. (A)
1
Stress  [1]
r2
iv. (D) [1]
v. (B) [1]
vi. (C) [1]
vii. (C)
K.E1 = K.E2
1 2 1 2
 I11  mv
2 2
5  36 20v 2
 
2 2
 90 = 10 v2
 v = 3 m/s [1]
Q.3. A. Linear S.H.M is defined as the linear periodic motion of a body, in which the restoring
force (or acceleration) is always directed towards the mean position and its magnitude is
directly proportional to the displacement from the mean position. [1]
i. Consider a particle ‘P’ moving along the circumference of a circle of radius ‘A’ with
constant angular speed ‘’ in anticlockwise direction.
31
Board Answer Paper : March 2016
ii. At any instant t = 0, particle P has its projection at point M as shown in the
figure (a). Particle P is called as reference particle and the circle on which it moves is called
as reference circle.
iii. As reference particle P revolves, its projection moves back and forth about centre O along
the diameter BC. [½]
Y Y

P P 
A A
 t + 
X X X X
B O M C B Mx O A

t=0 t>0
Y Y
Figure (a) Figure (b)
[½]
iv. The x-component of the displacement, velocity and acceleration of P is always same as the
displacement, velocity and acceleration of M.
v. Suppose that the particle P starts from an initial position with initial phase ‘’. In time ‘t’,
the angle between OP and X-axis is (t + ) as shown in the figure (b).
x
vi. In figure (b), cos (t + ) =
A
where x = displacement from mean position.
 x = A cos (t + ) ….(1) [½]
Equation (1) represents displacement of projection of P at time ‘t’.
vii. The velocity of particle is the time rate of change of displacement.
dx d
v= = [A cos (t + )]
dt dt
 v =  Asin (t + ) ….(2) [½]
Equation (2) represents velocity of projection of P at time ‘t’.
viii. The acceleration of particle is the time rate of change of velocity.
dv d
a= = [A sin(t + )]
dt dt
 a = A2 cos (t + )
 a = 2x ….[ x = A cos (t + )]
2
 a =  x ….(3) [½]
Equation (3) represents acceleration of projection of P at time ‘t’.
From equation (3),
ax ….[ 2 = constant quantity]
ix. As acceleration of projection of P is directly proportional to its displacement and its
direction is opposite to that of displacement, thus projection of particle P performs simple
harmonic motion. But M is projection of particle P performing U.C.M. Hence, S.H.M is the
projection of U.C.M along a diameter of circle. [½]
B. Solution:
Given: 0 = 25 C, 1 = 50 C, 2 = 45 C,
 d 
 dt  = 4 C/min
 1
 d 
To find: Rate of cooling at 2  
 dt  2
d
Formula: = K(  0) [½]
dt

32
32
Physics
Calculation: Using formula, for 1 = 50 C
4 = K(50 – 25) [½]
4
 K= min1 [½]
25
 For 2 = 45 C
 d  4
  =  (45 – 25) [½]
 dt  2 25
4
=  (20)
25
16
=
5
= 3.2 C/min
Ans: The rate of cooling at 45 C is 3.2 C/min. [1]
OR
A. Formation of stationary waves by analytical method:
i. Consider two identical progressive waves of equal amplitude and frequency travelling
along X axis in opposite direction. They are given by,
2
y1 = A sin (vt  x) along positive X-axis .…(1) [½]

2
y2 = A sin (vt + x) along negative X-axis ….(2) [½]

ii. The resultant displacement ‘y’ is given by the principle of superposition of waves,
y = y1 + y2 ….(3)
2 2
y = A sin (vt  x) + A sin (vt + x)
 
iii. By using,
CD C D
sin C + sin D = 2sin   cos  2  ,
 2   
we get,
 2  vt  x  vt  x    2  vt  x  vt  x  
y = 2A sin    cos   
  2    2 
 2vt   2 
= 2Asin   cos  (  x) 
     
 2x   v 
 y = 2Asin 2πnt cos   ….  n  and cos ()  cos
     
 2x 
 y = 2Acos   sin 2πnt
  
 2x 
iv. Let R = 2Acos  
  
 y = Rsin (2πnt) ….(4)
But,  = 2n
 y = R sin t ….(5) [½]
Equation (5) represents the equation of S.H.M. Hence, the resultant wave is a S.H.M. of
amplitude R which varies with x.
v. The absence of x in equation (5) shows that the resultant wave is neither travelling forward
nor backward. Therefore it is called as stationary wave.
vi. Amplitude at node is minimum, i.e., 0.
 Rmin = 0
 2x 
Since R = 2A cos  ,
  

33
Board Answer Paper : March 2016
 2x 
 cos   =0 [½]
  
2x  3 5
 = , , , …….
 2 2 2
 3 5
 x= , , ,……. [½]
4 4 4
vii. At antinodes: R =  2A
 2x 
 cos   = 1 [½]
  
 2x 
   = 0, , 2, 3,…….n
  
 3
 x = 0, ,, ……… [½]
2 2
 Distance between first node and adjacent
 λ
antinode = xn  xan = 0= [½]
4 4
λ
Thus, the distance between a node and an adjacent antinode is .
4

B. Solution:
Given: N = 48, X = 4, nF = 1.5 nL, i.e. n1 = 1.5 n48 [½]
To find: a. Frequency of first tuning fork (n1)
b. Frequency of 42nd tuning fork (n42)
Formula: nL = nF  (N  1)X
Calculation: Using formula, for N = 42
n42 = n1  [(42 – 1)  4]
= n1  164 [½]
Also for N = 48,
n48 = n1  [(48 – 1)  4]
= n1  188 [½]
n
n48 = 1 ….(Given)
1.5
n1
 = n1 – 188
1.5
188  1.5
 n1 = = 564 Hz [1]
0.5
Substituting the value
 n42 = 564 – 164 = 400 Hz [½]
Ans: a. Frequency of first tuning fork is 564 Hz
b. Frequency of 42nd tuning fork is 400 Hz.
Q.4. Attempt any THREE:
i. Solution:
Given: m = 600 kg, g = 9.8 m/s2, d = 5000 m, R = 6400 km = 6.4  106 m
To find: Decrease in weight of body (dW)
 d
Formula: gd = g 1 
 R 
Calculation: Weight of body on the earth surface,
W = mg = 600  9.8 = 5880 N [½]
Since, Wd = mgd
34
34
Physics
From formula,
 d
Wd = mg 1  [½]
 R 
 5  103 
= 5880 1   [½]
 6.4  106 
 6.4  0.005 
= 5880  
 6.4 
= {antilog{log(5880) + [log(6.395)  log(6.4)]}}
= {antilog[3.7694 + (0.8058 – 0.8062)]}
= {antilog(3.7694 + 1 .9996)}
= {antilog(3.7690)}
 Wd = 5875 N [1]
 Decrease in weight = W  Wd = 5880 – 5875
dW = 5 N
Ans: Decrease in weight is 5 N. [½]
ii. Theorem of parallel axes:
The moment of inertia of a body about any axis (IO) is equal to the sum of its moment of
inertia about a parallel axis passing through its centre of mass (IC) and the product of its
mass (M) and the square of the perpendicular distance between the two parallel axes (h2).
Mathematically, IO = IC + Mh2 [1]
Proof:
a. Consider a rigid body of mass ‘M’ rotating about an axis passing through a point O
as shown in the figure.
Let C be the center of mass of the body, situated at distance h from the axis of
rotation.

P(dm)
M
r ro

O C D
h x
IO IC
[½]
b. Consider a small element of mass dm of the body, situated at a point P.
c. Join PO and PC and draw PD perpendicular to OC when produced.
d. M.I of the element dm about the axis through O is OP2 dm
 M.I of the body about the axis through O is given by
IO =  OP 2 dm =  r 2 dm ....(1)
e. M.I of the element dm about the axis through C is CP2 dm
 M.I of the body about the axis through C
IC =  CP 2 dm =  ro2 dm ....(2)
f. From the figure,
OP2 = OD2 + PD2
= (OC + CD)2 + PD2
= OC2 + 2 OC  CD + CD2 + PD2
 CP = CD2 + PD2
2

 OP2 = OC2 + 2 OC  CD + CP2


i.e. r2 = h2 + 2hx + ro2 ....(3) [½]

35
Board Answer Paper : March 2016
g. From equation (3) and (1),
IO =   h 2  2hx  ro2  dm

 h dm +  2 hx dm+  r dm
= 2 2
o

= h  dm + 2 h  x dm +  r dm
2 2
o

IO = h  dm + 2h  x dm + I
2
C ....[From equation (2)]
 IO = IC + h  dm + 2 h  x dm
2
....(4)
h. Since  dm = M and  x dm = 0.
algebraic sum of the moments of the masses of its individual particles about the
centre of mass is zero for body in equilibrium. [½]
 Equation (4) becomes,
IO = IC + Mh2
Hence proved. [½]
iii. Expression for excess pressure inside a bubble:
a. Free surface of drops or bubbles are spherical in shape. Po
Let, Pi = inside pressure of a drop or air bubble Pi
Po = outside pressure of bubble
r = radius of drop or bubble.
b. Let the radius of drop increases from r to r + r so that inside pressure remains
constant.
c. Initial area of drop A1 = 4r2, Final surface area of drop A2 = 4(r+r)2 [½]
Increase in surface area,
A = A2  A1 = 4[(r + r)2  r2]
= 4[r2 + 2rr + r2  r2]
= 8rr + 4r2
d. As r is very small, the term containing r2 can be neglected.
 A = 8rr [½]
e. Work done by force of surface tension,
dW = TA = (8rr)T ….(1) [½]
But, dF = (Pi  Po) A [½]
 dW = Fr = (Pi  Po) Ar [½]
From equation (1),
(Pi  Po) Ar = (8rr) T
8πrrT
 Pi  Po = ….[ A = 4r2]
4πr 2 r
2T
 Pi  Po = ….(2) [½]
r
In case of soap bubble, there are two free surfaces in contact with air.
4T
 Excess pressure, Pi  Po =
r
Equation (2) represents excess pressure inside a drop or air bubble. It is also called
Laplace’s law of spherical membrane.
iv. Solution:
Given: A = 1.5 mm2 = 1.5  10–6 m2, lateral strain = 1.5  10–5,
Ysteel = 2  1011 N/m2,  = 0.291,
g = 9.8 m/s2
To find: Mass attached (M)
lateral strain
Formulae: a. = [½]
longitudinal strain
longitudinal stress
b. Y= [½]
longitudinal strain

36
36
Physics
Calculation: From formula,
longitudinalstress
Ysteel =
 lateralstrain 
 
  
Mg
=
 lateralstrain 
A 
  
Ysteel  A  lateralstrain
 M= [½]
g
2  1011  1.5  106  1.5  105
= [½]
0.291  9.8
4.5
 M=
0.291  9.8
= {antilog{log(4.5)  [log(0.291) + log(9.8)]}}
= {antilog[0.6532  ( 1 .4639 + 0.9912)]}
= {antilog[0.6532 – 0.4551]}
= {antilog(0.1981)}
= 1.578 kg
Ans: Mass attached to the wire is 1.578 kg. [1]

SECTION – II

Q.5. Attempt any SIX:


i. The bending of light near the edge of an obstacle or slit and spreading into the region of
geometrical shadow is called diffraction of light. [1]
Two types of diffraction are:
a. Fresnel diffraction:
Diffraction pattern in which source of light and screen are kept at finite distance from the
slit is called fresnel diffraction.
eg: Diffraction of straight edge, small opaque disc, narrow rectangular slit, etc. [½]
b. Fraunhofer diffraction:
Diffraction pattern in which, the source of light and the screen are effectively at
infinite distances from the diffracting system is called Fraunhoffer diffraction.
In this diffraction pattern convex lens is used.
eg: Diffraction due to single slit, double slit, etc. [½]

ii. Construction of cyclotron:

Electromagnet
N Coils

dees

(Diagram and labelling) (1 + 1) [2]

37
Board Answer Paper : March 2016
iii.
No. Paramagnetic substance Ferromagnetic substance
i. It is weakly attracted by a magnet. It is strongly attracted by a magnet.
ii. When kept in a non-uniform magnetic When kept in a non-uniform magnetic
field, it shows moderate tendency to field, it shows strong tendency to move
move from weaker to the stronger part from weaker to the stronger part of the
of the field. field.
iii. When kept in an external magnetic field When kept in an external magnetic field
it becomes weakly magnetised, and the it becomes strongly magnetised, and the
direction of magnetic moment acquired direction of magnetic moment acquired
will be same as that of the field. will be same as that of the field.
iv. When the external magnetic field is When the external magnetic field is
removed, the paramagnetic substance removed, the ferromagnetic substance
loses its magnetism. retains magnetism permanently.
v. They cannot be converted into When heated above curie temperature,
ferromagnetic substances. they become paramagnetic substances.
vi. Every atom has some magnetic dipole The resultant magnetic dipole moment
moment but resultant dipole moment is is greater.
zero.
vii. They can be temporarily magnetised in They can be permanently magnetised.
external magnetic field.
(Any two points) [2]
iv. Surface wave propagation:
When the electromagnetic waves (radiowaves) from the transmitting antenna propagate
along the surface of the earth so as to reach the receiving antenna, the wave propagation
is called surface wave propagation (ground wave propagation).
Explanation:
a. Ground waves are the radiowaves which propagate along the surface of the earth.
b. The electromagnetic waves which are vertically polarised, can travel along the
surface of earth. The horizontal component of electric field in contact with earth is
short circuited. The electric field vector of the wave induces charge in the earth.
c. There is loss of power in a signal during its propagation on the surface of the earth
due to partial absorption of energy by ground. Loss of energy is also due to
diffraction effect. The absorption of energy is high for high frequency.
Hence ground wave propagation is suitable for low frequency and medium
frequency. It is used for local broadcasting For examples: ship, communication, radio
navigation.
d. Ground wave propagation is possible only when the transmitting and receiving antenna
are close to the earth’s surface. [2]
v. Solution:
Given: G = 500 , Req. = 21 
To find: Shunt Resistance (S)
1 1 1
Formula:   [½]
Req G S
Calculation: From formula,
1 1 1
= 
S Req G
G  Req
=
Req G
Req G
 S =
G  Req
21  500
= [½]
500  21

38
38
Physics
10500
=
479
= {antilog[log(10500)  log(479)]}
= {antilog[4.0212  2.6803]}
= {antilog(1.3409)}
 S = 21.93 
Ans: The value of shunt resistance is 21.93 . [1]
vi. Solution:
Given: 1 = 1.8  10–5, T1 = 200 K, 1  2 = 6  10–6
To find: Required temperature (T2)
Formula: T = constant
Calculation: Given 1  2 = 6  10–6
 2 = 1.8  10–5  0.6  10–5 = 1.2  10–5
From formula,
1T1 = 2T2
T
 T2 = 1 1 [½]
2
1.8  105  200
= [½]
1.2  105
= 300 K
Ans: The required temperature is 300 K. [1]
vii. Solution:
Given: M = 2 H, dI = 4 A, dt = 2.5  10–4 s
To find: The induced e.m.f.(e)
dI
Formula: e=M [½]
dt
Calculation: From formula,
4
e=2 [½]
2.5  104
= 2  4  0.4  104
= 3.2  104 V
Ans: The induced e.m.f. in the coil is 3.2  104 V. [1]
viii. Solution:
Given:  = 4.33  10–4 year–1
To find: Half life period (T)
0.693
Formula: T= [½]

Calculation: From formula,
0.693 6.93
T= =  103 [½]
4.33  104 4.33
3
= {antilog[log(6.93)  log(4.33)]}  10
= {antilog(0.8407 – 0.6365)}  103
= 1.601  103
= 1601 years
Ans: Half-life period of a radioactive element is 1601 years. [1]
Q.6. Select and write the most appropriate answer from the given alternatives for each
sub-question:
i. (D) [1]
ii. (C) [1]
iii. (B) [1]
iv. (A) [1]
39
Board Answer Paper : March 2016
v. (D) [1]
vi. (B) [1]
vii. (C) [1]
Q.7. A. Principle of transformer:
It is based on the principle of mutual induction i.e., whenever the magnetic flux linked with
a coil changes, an e.m.f. is induced in the neighbouring coil. [1]
Construction:
i. A transformer consists of two sets of coils P and S insulated from each other. The coil P is
called the primary coil and coil S is called the secondary coil.
ii. The two coils are wound separately on a laminated soft iron core.
iii. The a.c input voltage is applied across the primary and the induced output a.c voltage is
obtained across the secondary, which is used to drive current in the desired circuit.
iv. The two coils are electrically insulated from each other but they are magnetically linked.
v. To minimise eddy currents, the soft iron core is laminated. [½]
Soft iron core

Input a.c.
voltage  P S
Load

P S

core [½]

Working:
i. When an alternating voltage is applied to the primary coil the current through the coil goes
on changing. Hence, the magnetic flux through the core also changes.
ii. As this changing magnetic flux is linked with both the coils, an e.m.f is induced in each coil.
iii. The amount of the magnetic flux linked with the coil depends upon the number of turns of
the coil.
iv. Let, ‘’ be the magnetic flux linked per turn with both the coils at certain instant ‘t’.
v. Let ‘NP and ‘NS’ be the number of turns of primary and secondary coil,
NP = magnetic flux linked with the primary coil at certain instant ‘t’
NS = magnetic flux linked with the secondary coil at certain instant ‘t’
vi. Induced e.m.f produced in the primary and secondary coil is given by,
dP d
eP =  =  NP ….(1)
dt dt
d d
eS =  S =  NS ….(2)
dt dt
vii. Dividing equation (2) by (1),
eS N
 = S ….(3) [1]
eP NP
Equation (3) represents equation of transformer.
NS
The ratio is called turns ratio (transformer ratio) of the transformer.
NP
viii. For an ideal transformer, Input power = Output power
 ePIP = eSIS
eS I
 = P ….(4)
eP IS
ix. From equation (3) and (4),
eS N I
= S = P [1]
eP NP IS

40
40
Physics
B. Solution:
Given: Q = 0.2 C = 0.2  10–6 C, A = 40 cm2 = 40  10–4 m2
0 = 8.85  10–12 S.I.units
To find: a. Electric field intensity (E)
b. Mechanical force per unit area (f)

Formulae: a. E=
0 k
1
b. f= 0kE2 [½]
2
Q
c. =
A
Calculation: Using formula (a) and (c),
Q
E= [½]
A0 k
0.2  106
= ….(k = 1 for air)
40  104  8.85  1012
20  108
=
3.540  1014
= {antilog[log(20)  log(3.54)]}  106
= {antilog(1.3010 – 0.5490)}  106
= {antilog(0.7520)}  106
= 5.649  106 V/m
From formula (b)
1
f =  8.85  10–12  (5.649  106)2
2
= 4.425  10–12  (5.649)2  1012
= {antilog[log(4.425) + 2log(5.649)]}
= {antilog[0.6459 + (2  0.7520)]}
= {antilog[2.1499]}
 f = 141.2 N/m2
Ans: a. Electric intensity is 5.649  106 V/m [1]
b. Mechanical force per unit area is 141.2 N/m2 [1]
OR
A. Geiger-Marsden experiment:
a. The experimental arrangement is as shown in the figure.

Screen
Lead Gold
bricks foil


Source of
 particles

Detector
[1]
b. In this experiment, a narrow beam of -particles from radioactive source was
incident on a gold foil.
c. The scattered -particles were detected by the detector fixed on rotating stand.
Detector used had zinc sulphide screen and microscope.
d. -particles produced scintillations on screen which could be observed through
microscope.
e. The whole setup is enclosed in an evacuated chamber.
41
Board Answer Paper : March 2016
Observation:
a. Most of the -particles passed undeviated.
b. Only few -particles (about 0.14%) were scattered by more than 1.
c. Some -particles were deflected slightly and very few (1 in 8000) deflected by more
than 90.
d. Some -particles were bounced back with  = 180. [2]
Mass defect:
The difference between the actual mass of the nucleus and the sum of masses of constituent
nucleons is called mass defect.
Let, M = measured mass of nucleus
A = mass number
Z = atomic number
mp = mass of hydrogen atom
mn = mass of free neutron
(A  Z) = number of neutrons
 Mass defect,
m = [Zmp + (A Z)mn]  M [1]
B. Solution:
Given: 0 = 2.3 eV,  = 6800 Å = 6.8  10–7 m, c = 3  108 m/s, h = 6.63  10–34Js
To find: a. Threshold frequency (0)
b. Incident frequency ()
c. Whether photoemission is possible?
Formulae: a. 0 = h0
c
b. = [½]

Calculation: From formula (a),
0
0 = [½]
h
2.3  1.6  1019
= [½]
6.63  1034
.68  1015
=
6.63
36.8
=  1014
6.63
= {antilog[log(36.8) – log(6.63)]}  1014
= {antilog(1.5658 – 0.8215)}  1014
= {antilog(0.7443)}  1014
= 5.550  1014 Hz [½]
From formula (b)
3  108
=
6.8  107
30  107
=
6.8  107
= {antilog[log(30)  log(6.8)]}  1014
= {antilog(1.4771  0.8325)}  1014
= {antilog(0.6446)}  1014
= 4.412  1014 Hz [½]
As  < 0, emission of photoelectrons is not possible. [½]
Ans: a. Threshold frequency is 5.55  1014 Hz.
b. Frequency of incident light is 4.412  1014 Hz.
c. Emission of photoelectrons is not possible.
42
42
Physics
Q.8. Attempt any THREE:
i. Solution:
14 8
Given: ag = 1.5,  = 3.5  10 Hz, c = 3  10 m/s
To find: a. Change in wavelength of light ()
b. Wave number of light   
a
Formulae: a.  = 1

b. ag =
g
c. c =  [1]
Calculation: a. Using formula (c),
c 3  108 6
a =  =  106 = 0.8571  106 m
 3.5  10 14
7
 a = 8571 Å [½]
Using formula (b),
a 8571A
g = =
a g 1.5
 g = 5714 Å [½]
  = a  g = 8571  5714
  = 2857 Å
Now, using formula (a),    =
1
b.
g
1
 = = 1.75  106 m1
5.714  107
Ans: a. The change in wavelength of light is 2857 Å. [½]
b. The wave number of light is 1.75  106 m1. [½]
ii. Solution:
Given: x10 = 2.09 mm, R = 6400 Å, B = 4800 Å
To find: Change in fringe width (X)
D D
Formulae: a. X= b. x = (2m  1)
d 2d
Calculation: Using formula (b),
D
x10 = (2(10) – 1)
2d
19
2.09 =  XR
2
2.09  2
 XR = [½]
19
4.18
=
19
= 0.22 mm [½]
From formula (a) we can conclude, for same setting,
X
X1 X 2
 
1  2
B
i.e. XB =  XR [½]
R
4800Å
=  0.22 mm
6400Å
0.66
=
4
= 0.165 mm [½]
 change in fringe width (X) = XR  XB = 0.22 – 0.165 = 0.055 mm
Ans: The change in fringe width is 0.055 mm. [1]
43
Board Answer Paper : March 2016
iii. Kelvin’s method to determine the resistance of a galvanometer:
Construction:
a. In Kelvin’s method, the galvanometer (G) whose resistance is to be determined is
connected in the left gap of a metrebridge and a known resistance R is connected in
the right gap.
b. A jockey (J) is connected directly to the point B and it can slide along the wire.

G R
B
T1 T2
T3
lg D J
A lR
C
Scale
0 10 20 30 40 50 60 70 80 90 100

+  ( )
E K Rh
G : Galvanometer
R : Resistance from resistance box
AC : Metal wire one metre long
Rh : Rheostat
E : Cell
K : Plug key
J : Jockey
[1]
c. A cell of e.m.f ‘E’ is connected between points A and C of the wire in series with
a high resistance box.
d. The rheostat is used to adjust the deflection in the galvanometer to half of its maximum
value. Hence, this method is also called half current method or half scale method.
e. First the deflection in the galvanometer is adjusted at half of its original value and the
reading is noted. It acts as null position.
f. The value of R is adjusted, so that the galvanomter gives a fairly large deflection,
i.e., full scale deflection.
If the jockey is touched to different points on the wire then galvanometer shows
increase or decrease in the deflection.
g. A point D is located on the wire so that when the jockey is touched at that point,
galvanometer shows the same deflection as before. It means that point D and B are at
the same potential, i.e., bridge is balanced. [½]
Working:
a. Let, lg = length of wire corresponding to left gap
lR = length of wire corresponding to right gap
G = resistance of galvanometer
b. In the balanced condition,
G Resistance of wire of length lg
= [½]
R Resistance of wire of length lR
G l l
 = g = g
R  lR lR
where,  = resistance per unit length of wire
lg
 G = R [1]
lR
c. Since lg + lR = 100 cm
 lR = (100  lg)
 lg 
 G = R  
 100  lg 
Measuring lg and R, value of G can be determined.
44
44
Physics
iv. Principle of working of an oscillator:

+ 
d. c. supply

Input Transistor amplifier Output


Vi (A) Vo = AVi

Feedback Network
Vf = Vo
()

[1]
a. A simple oscillator consists of an amplifier and feedback network with frequency
determining components.
b. A frequency-determining network, (resonant tank circuit) which also works as
feedback network and transistor amplifier acts as element.
c. With enough feedback, the oscillations start as soon as the circuit is switched on.
d. With positive feedback, the output current of the amplifier will be in the right phase
to increase the alternating current in the resonant circuit.
e. The oscillations then built up in amplitude until the power losses in the circuit are
equal to the power that the amplifier can develop.
f. The natural frequency of the oscillator is close to the resonant frequency of the
resonant circuit. [1]
Vo
g. Suppose the voltage gain without feedback of the amplifier is A = .
Vi
h. The feedback factor  is the fraction of the output voltage fed back to the input,
Vi = Vf = Vo
Vo 1
 A= =
Vi 
 A = 1
i. The condition A = 1, is called Berkhausen’s criterion. It states that the phase shift
of the feedback voltage will be zero or integral multiple of 2 rad, i.e., there will be
positive feedback. [½]
A
j. The voltage gain of complete system is given by, Af = .
1  A
Thus, for the frequency for which A = 1, Af will be infinite, i.e., the circuit will
operate without any external signal voltage, which means the circuit will oscillate at
that frequency. [½]

45
Board Answer Paper : March 2016

BOARD ANSWER PAPER : MARCH 2016


CHEMISTRY

Note: Answer to every question must be written on a new page.

SECTION – I

Q.1. Answer any SIX of the following:


i. a. The substances which are strongly attracted by the magnetic field and show
permanent magnetism even when the magnetic field is removed are known as
ferromagnetic substances. The property thus exhibited is called ferromagnetism. [1]
b. Ferromagnetic substances contain a large number of unpaired electrons.
The electronic configuration of iron (atomic number 26) is [Ar] 3d6 4s2.
[Ar]      
3d6 4s2
Since, iron (Fe) contains four unpaired electrons, it is strongly ferromagnetic. [1]
ii. The boiling point of a liquid is defined as the temperature at which the vapour pressure of
the liquid becomes equal to the atmospheric pressure. [1]
The formula to determine molar mass of a solute using freezing point depression method is
K f W2
M2 = [1]
Tf W1
Where M2 = Molar mass of solute in kg mol–1
Kf = Cryoscopic in K kg mol–1 constant
Tf = Depression in freezing point in K
W2 = Mass of solute in kg
W1 = Mass of solvent in kg
iii. a. Adiabatic process: The mathematical expression of first law of thermodynamics is
U = q + W.
In adiabatic process, q = 0.
Hence, the mathematical expression of first law of thermodynamics for adiabatic process
is,
U = 0 + W
 U = +W
or  U = W (Explanation + Final equation) [1]
b. Isochoric process: The mathematical expression of first law of thermodynamics is
U = q + W (W =  pex V).
In isochoric process, V = 0, W = 0
Hence, the mathematical expression of first law of thermodynamics for isochoric process
is,
U = qv + 0
 U = qv (Explanation + Final equation) [1]
iv. Activation energy (Ea) can be calculated graphically (from the collected data) as follows:
log10 k =  
Ea  1
  + log10 A ....(Arrhenius equation) (Arrhenius equation) [½]
 2.303R  T
1
Thus, when the graph of log10k is plotted against , a straight line with negative slope is
T
obtained. (Description of graph) [½]
46
46
Chemistry
From the slope of the graph, activation energy can be calculated.
Ea
Slope =  (Equation of slope) [½]
2.303R
 Ea =  slope  2.303  R. (Final equation) [½]
v. a. Iron:
Name of the ore: Haematite [½]
Formula: Fe2O3 [½]
b. Zinc:
Name of the ore: Zinc blende [½]
Formula: ZnS [½]
vi. a. Arsenic forms sodium arsenide (Na3As2) with sodium.
2As + 3Na  Na3As2 (Explanation + Chemical reaction) [1]
Arsenic Sodium Sodium
arsenide
b. Bismuth forms magnesium bismuthide (Mg3Bi2) with magnesium.
2Bi + 3Mg  Mg3Bi2 (Explanation + Chemical reaction) [1]
Bismuth Magnesium Magnesium
bismuthide
vii. a. The enthalpy change that accompanies the conversion of one mole of solid directly
into its vapour at constant temperature and pressure is called its enthalpy of
sublimation (subH). [1]
b. Whether the conversion of solid to vapour takes place directly in one step or in two
steps (i.e., first melting of solid into liquid and then its vaporization), the enthalpy
change is the same because enthalpy is a state function.
Therefore,
ΔsubH = ΔfusH + ΔvapH [1]
Where fusH is enthalpy of fusion and vapH is enthalpy of vaporization.
viii. Ellingham diagram:
The graphical representation showing the variation of Gibbs energy with increase of
temperature for the formation of oxide (oxidation) is known as Ellingham diagram.
OR
The Ellingham diagram is the plot of free energy change G against temperature for the
reaction of a metal and other elements with one mole of gaseous oxygen at 1 atmosphere. [1]
Features of Ellingham diagram:
The Ellingham diagram for oxides shows the following important features:
a. The graph for the formation of a metal oxide is a straight line with an upward slope.
b. There is sudden change in the slopes for some metal oxides, like MgO, ZnO and HgO.
c. For a few metal oxides of mercury and silver, the graph is at the upper part in
Ellingham diagram (Ag2O and HgO).
d. The graph for the formation of CO is straight line with the negative slope. This line
intersects the lines of many metal oxides.
e. The graph for the formation of CO2 is straight line almost parallel to the temperature
axis.
(Any two features) [½  2] [1]
Q.2. Answer any THREE of the following:
i. Given: Density (d) = 10.51 g cm3.
Molar mass of Ag (M) = 108 g mol1
To find: Volume of the unit cell
Atomic mass
Formulae: a. Mass of one atom =
Avogadro number
Mass of unit cell
b. Density =
Volume of unit cell

47
Board Answer Paper : March 2016
1 1
Calculation: Unit cell of fcc type contain =  8 +  6 = 4 atoms
8 2
Atomic mass of Ag
Mass of one atom = [½]
Avogadro number

108 g mol1
= [½]
6.022 1023 mol1
 Mass of fcc unit cell of Ag = Number of atoms in the unit cell
 Mass of one Ag atom [½]
108
=4 g [½]
6.022  1023
Mass of unit cell
Density =
Volumeof unit cell

Mass of unit cell


 Volume of unit cell =
Density

108
4 g
= 6.022  10 23 [½]
10.51 g cm 3

= 6.825  1023 cm3 [½]


z.M
Note: The above problem can alternatively be solved by using the formula: d = .
a 3 .N A

ii. Given: Vapour pressure of pure benzene  p1o  = 640 mm Hg

Mass of solute (W2) = 2.175  103 kg


Mass of solvent (W1) = 39 g = 39  103 kg
Vapour pressure of solution (p) = 600 mm Hg
To find: Molar mass of solute (M2)
p1o  p W M
Formula: o
= 2 1 [½]
p1 W1 M 2
Calculation: Molar mass of solvent (benzene) (M1) = 78  103 kg mol1 [½]
From formula,
640  600 2.175  103 78  103
=  [1]
640 39  103 M2
640  2.175  103  78  103
 M2 = = 69.6  103 kg/mol = 69.6 g/mol [1]
39  103  40

iii. Given: H for the given reaction = 104 kJ


To find: H(C  Cl)
Formula: ΔH = ΔH (reactant bonds)  ΔH (product bonds)
Calculation: ΔH = ΔH (reactant bonds)  ΔH (product bonds) [½]
= [3ΔH (CH) + ΔH(CCl) + ΔH(ClCl)] [2ΔH(CH)
+ 2ΔH(CCl) + ΔH(HCl)] [1]
= ΔH(CH )  ΔH(CCl) + ΔH(ClCl)  ΔH(HCl)
 104 = 414  ΔH(CCl) + 243  431 [½]
1
 ΔH(C  Cl) = 414 + 104 + 243  431 = 330 kJ mol [1]
48
48
Chemistry
iv. Cell constant: The ratio of the distance between the electrodes divided by the area of
cross-section of the electrode is called as cell constant. [1]

Copper Pure and dry H2


wire gas at 1 atm
Glass jacket

Vessel
Mercury
Platinum
wire
Solution of Platinised
H+ ions (1M) platinum foil

Standard Hydrogen Electrode


(Diagram-1 mark, Labelling-1 mark) [2]
Q.3. Answer any ONE of the following:
i. a. Nitrogen differs from the other elements of group 15 in the following ways:
1. Nitrogen is a gas at room temperature while the rest of the members of the
family are solids.
2. Nitrogen exists as diatomic molecule (N2), whereas the other elements exist as
tetratomic molecules (As4, Sb4, P4, etc).
3. Nitrogen can form hydrogen bonds in its hydride compounds due to small size
and high electronegativity and rest of the members do not form hydrogen
bonds due to small electronegativities and larger size.
4. Nitrogen forms p-p multiple bonds. Other elements of the group form
multiple bonds through d-p overlapping.
5. Nitrogen shows all oxidation states from –3 to +5. The other group 15
elements show limited number of oxidation states.
6. The trihalides of nitrogen (except NF3) are unstable. Trihalides of other group
15 elements are stable.
7. The hydride of nitrogen (NH3) is more stable and more basic than the hydrides
of the other members of the family.
8. Nitrogen does not exhibit pentavalency in its compounds while other members
show pentavalency.
9. Nitrogen does not form coordination compounds due to absence of d-orbitals.
Other group 15 elements form coordination compounds.
(Any four points of difference) [½  4] [2]
b. Interhalogen compound, ClF3 possesses trigonal bipyramidal or T-shaped structure.
In ClF3, Cl is the central atom and it undergoes sp3d hybridization forming five
hybrid orbitals. (Type of hybridization) [½]
3s2 3p5 3d0
Ground state of Cl

3s2 3p4 3d1


First excited state

sp3d Hybridized state


sp3d hybrid orbitals 3d0


Hybridization of Cl in ClF3
(Hybridized state of Cl) [½]

49
Board Answer Paper : March 2016
Three half filled hybrid orbitals overlap with half filled p-orbitals of three F atoms
and form three ClF sigma bonds.
Two lone pairs occupy the equatorial positions to minimise the lone pair-lone pair
and the lone pair-bond pair repulsions which are greater than bond pair-bond pair
repulsions.
The two axial F atoms bend towards equatorial F atom to minimise the lone pair-lone
pair repulsion, giving T-shaped structure.

o
Cl

8729
F F
F
Molecular Shape of ClF3

[Diagram (geometry + bond angle)] [1]

c. Given: Conductivity (k) = 6.23 × 10–5 –1 cm–1


Resistance (R) = 13710 
Distance between electrodes (l) = 0.7 cm
To find: Area of cross-section of the electrode (a)
l
Formula: k×R= [½]
a
Calculation: From formula,
l
a=
kR
0.7
= [½]
6.23  105  13710
= 0.82 cm2 [1]

d. Molality is the best way of expressing concentration of solutions as it is a


temperature independent quantity. This is because molality involves measurement of
masses of solute and solvent, which are independent of temperature. [1]

ii. a. 1. Neutral oxides: Oxides which are neither acidic nor basic are called as
neutral oxides.
eg. CO, NO and N2O [1]
2. ZnO is an amphoteric oxide (show both acidic and basic characterstics). This
oxide react with acids as well as bases to form salts.
Basic character (Reaction with acid)
ZnO(s) + 2HCl(aq)  ZnCl2(aq) + H2O(l)
(Basic) Zinc chloride [1]

Acidic character (Reaction with base)


ZnO(aq) + 2NaOH  Na2ZnO2(aq) + H2O(l) [1]
(Acidic) (excess)

b. 1. Molar conductivity is defined as the conductance of a volume of solution


containing 1 mole of dissolved electrolyte when placed between two parallel
electrodes 1 cm apart and large enough to contain between them all the
solution. [1]
2. Zero order reaction is the reaction whose rate is independent of the reactant
concentration and remains constant throughout the course of the reaction. [1]
50
50
Chemistry
c. Given: For a first order reaction, amount of reactant that remains after
45 minutes = 40%
To find: Rate constant (k)
Formula: k=
2.303  A 0
log10
t  A t
Calculation: [A]0 = 100, [A]t = 40, t = 45 min [½]
From formula,
2.303  A 0
k= log10 [½]
t  A t
2.303 100
 k= log10 [½]
45 40
 k = 0.02036 min–1 [½]
Q.4. Select and write the most appropriate answer from the given alternatives for each
sub-question:
i. (B) Thiosulphurous acid [1]
ii. (C) non-polar molecular solid [1]
iii. (A) measuring heat capacity of substance at various temperatures [1]
iv. (C) ebullioscopy [1]
v. (D) Hall’s process [1]
vi. (B) 2 amperes [1]
Q = It
Q 840
 I= =
t 7  60
= 2 amperes
vii. (B) 1.1 × 10–4 s–1 [1]
Rate = k[A]
Rate 6.6  105
 k= =
A 0.6
= 1.1 × 10–4 s–1

SECTION – II

Q.5. Answer any SIX of the following:


i. a. Electronic configuration of:
Sc3+ : [Ar] 3d0 [½]
Ti3+: [Ar] 3d1 [½]
b. Sc3+ ion has completely empty 3d-orbital, i.e., no unpaired electrons are present. As
d-d transitions are not possible, Sc3+ ion is colourless. [½]
Ti3+ ion has one unpaired electron in 3d-orbital. As d-d transitions are possible, Ti3+
ion is coloured. [½]
ii. a. Double salts are the molecular or addition compounds that exist only in solid state
and dissociate into ions in aqueous solution or in any other solvent. They lose their
identity in aqueous solution. Fe(NH4)2(SO4)2(H2O)6 or Mohr’s salt is a double salt,
which dissolves in water and gives the characteristics of Fe2+, NH +4 and SO24 ions. [1]
b. Coordination compounds are the molecular or addition compounds that exist in the
solid state as well as when dissolved in water or any other solvent. They do not
completely lose their identity in aqueous solution. K4[Fe(CN)6] is a coordination
compound and when dissolved in water its constituent complex ion [Fe(CN)6]4 does
not dissociate into Fe2+ and CN ions. [1]
51
Board Answer Paper : March 2016
iii. Preparation of chlorobenzene from aniline:
+ 
NH2 N2Cl Cl
NaNO2  HCl

273K
 
Cu 2 Cl2
+ N2
Benzene diazonium Chlorobenzene
Aniline
chloride
(Reaction + names of reactants, products and reagents) [1]
Conversion of chlorobenzene to diphenyl:
Cl

+ 2Na  
dry ether
2 + 2NaCl

Chlorobenzene Diphenyl
(Reaction + names of reactants, products and reagents) [1]

iv. Metamerism (positional isomerism): Ethers having same molecular formula but different
alkyl groups attached on either side of the oxygen atom are called metamers of each other.
This phenomenon is called metamerism (positional isomerism). [1]
eg. Ethers with molecular formula C4H10O have three metamers having different alkyl
groups attached on either side of oxygen atom.
a. CH3  CH2  O  CH2  CH3 Diethyl ether
b. H3  O  CH2  CH2  CH3 Methyl n-propyl ether
c. CH3  O  CH  CH3 Isopropyl methyl ether

CH3
(Example of any two ethers) [1]

v. Ketones are the first oxidation products of secondary alcohols. The functional group in
ketones is keto or oxo group ( C = O). The keto group is not located at the terminal
carbon of the chain. [1]

Classification of ketones: Depending upon the type of alkyl group (R) attached to the carbonyl
carbon, ketones are classified into simple ketones (symmetrical ketones) and mixed ketones
(unsymmetrical ketones).
a. Simple ketones (Symmetrical ketones): When both the alkyl groups present in a
ketone are same (identical), then the ketone is called a simple or symmetrical
ketone.
O

General formula: R  C  R, where R may be alkyl or aryl group.


eg. CH3  CO  CH3 H5C6  CO  C6H5
Acetone Diphenyl ketone
(Dimethyl ketone)

(Explanation + One example) [½]

b. Mixed Ketones (Unsymmetrical ketones): When both the alkyl groups present in a
ketone are different, then the ketone is called mixed or unsymmetrical ketone.
O
R  C  R, where R and R may be alkyl or aryl group (R  R).
General formula:
eg. CH3  CO  C2H5 C2H5  CO  C6H5
Ethyl methyl ketone Ethyl phenyl ketone

(Explanation + One example) [½]


52
52
Chemistry
vi. a.
H H O O

C2H5  C = N  OH + [O] 


trifluoroperoxyacetic  C H  CH  N+ = O
 C2H5  C = N+  OH 
acid 2 5 2
Propionaldoxime 1-Nitropropane

(Reaction + names of reactant, product and reagent) [1]


b. CH3 CH3 O CH3 O

CH3  C = N  OH + [O] 


trifluoroperoxyacetic  CH  CH  N+ = O
 CH3  C = N+  OH 
acid 3
Acetoxime 2-Nitropropane

(Reaction + names of reactant, product and reagent) [1]

vii. Antioxidant is a substance which when added to food, retards or prevents oxidative
deterioration of food. [1]
Structure of Butylated hydroxy toluene (BHT):
OH
(CH3)3C C(CH3)3

CH3
Butylated hydroxy toluene (BHT)
[1]
viii. Carbohydrates are optically active polyhydroxy aldehydes or polyhydroxy ketones or the
compounds that can be hydrolysed to polyhydroxy aldehydes or polyhydroxy ketones. [1]
Preparation of nylon-6:
O H
C N
O H O H
CH2 CH2
533K
n 
N2
  C (CH2)5 N    C (CH2)5 Nn
CH2 CH2 Repeating unit Nylon-6 [1]
CH2
-Caprolactum

Q.6. Answer any THREE of the following:


i. a. The elements in which the last electron enters into (n2) f-orbital of the atoms are
called f-block elements. [1]
b.

Lanthanoids Actinoids
1. In lanthanoids, last differentiating In actinoids, last differentiating electron
electron occupies 4f orbital. occupies 5f orbital.
2. They are the elements of first inner They are the elements of second inner
transition series. transition series.
3. They are present in period 6. They are present in period 7.
4. Most of lanthanoids (with exception of Most of actinoids (with exception of uranium
promethium) occur in nature. and thorium) are prepared in laboratory.
5. Most of lanthanoids (with exception of All the actinoids are radioactive.
promethium) are non-radioactive.

53
Board Answer Paper : March 2016

6. Lanthanoids do not form oxo cation. Actinoids form oxo cation such as UO22  ,
PuO2+, UO+.
7. 4f orbitals in lanthanoids have higher 5f orbitals in actinoids have lower binding
binding energy. energy.
8. Contraction in atomic and ionic radii is Contraction in atomic and ionic radii is
relatively less in lanthanoids. relatively greater in actinoids due to poor
shielding of 5f electrons.
9. Lanthanoids show +2, +3 and +4 Actinoids show +3, +4, +5, +6, +7 oxidation
oxidation states. states.
10. Lanthanoids have less tendency to form Actinoids have greater tendency to form
complexes. complexes.
11. Some ions of lanthanoids are coloured. Most of the ions of actinoids are deeply
coloured.
12. Lanthanoid hydroxides are less basic in Actinoid hydroxides are more basic in
nature. nature.
(Any four distinguishing points) [½  4] [2]

ii. a. Optical activity (Chirality): The property of a substance to rotate the plane of a
plane polarised light towards the right (clockwise) or towards the left (anticlockwise)
is called optical activity.
eg. Lactic acid, glucose, cane sugar, 2-chlorobutane, etc., shows optical activity. [1]

b. The molecules or ions which are coordinated to the central atom or ion in the
coordination compound are called ligands or donor groups.
eg. 1. In [Ni(NH3)6]2+, central metal ion is Ni2+ and ligands are NH3 molecules.
2. In [Cr(H2O)6]3+, the central metal ion is Cr3+ and the ligands are H2O
molecules. [1]
c. 1. Interstitial compounds are those which are formed when small atoms like H,
C, N, B, etc., are trapped inside the crystal lattice of metals.
2. Interstitial compounds have variable composition and are non-stoichiometric
in nature.
eg. TiC, TiH1.73, Mn4N, Fe3H, VH0.56, ZrH1.92, etc. [1]

iii. Formula: [Pt(NH3)4Cl2]Cl2 [1]


Propene on reaction with hydrogen bromide in the presence of peroxide follows anti-
Markownikoff’s rule and forms n-propyl bromide as a major product.

peroxide
CH3  CH = CH2 + HBr  CH3  CH2  CH2Br
Propene n-Propyl bromide
(1-Bromopropane)
(Major product)

(Explanation + Reaction) [1]


Addition of hydrogen bromide to propene gives isopropyl bromide as a major product
according to Markownikoff’s rule.
Markownikoff's rule
CH3  CH = CH2 + H  Br  CH3  CH  CH3
Propene
Br
Isopropyl bromide
(2-Bromopropane)
(Major product)

(Explanation + Reaction) [1]

54
54
Chemistry
iv. Broad spectrum antibiotics:
Antibiotics which are effective against a wide range of gram positive and gram negative
bacteria are known as broad spectrum antibiotics.
eg. Tetracycline, chloramphenicol, ampicillin, amoxicillin, etc. [1]
Preparation of polythene:
CH2
n 
Polymerization
 , pressure,catalyst
  CH2  CH2   CH2  CH2
CH2 n
Ethene
(Monomer) Repeating unit Polythene
(Reaction + names of reactants, products and reagents + Reaction condition) [1]
Preparation of neoprene:

nCH2 = C  CH = CH2    CH2 C = CH  CH2 


Polymerization
catalyst CH2 C = CH  CH2 

Cl Cl Cl
n
Chloroprene Repeating unit Neoprene

(Reaction + names of reactants, products and reagents + Reaction condition) [1]


Q.7. Answer any ONE of the following:
i. a. Mechanism of esterification reaction:
The mechanism involves two steps:
1. The mineral acid protonates the carbonyl oxygen of the carboxylic acid. This
increases the positive character of carbonyl carbon. The nucleophile (alcohol)
attacks the carbonyl carbon. This is followed by the loss of proton to obtain
ester hydrate.
+ OH OH
O OH
 R 'O  H
 R  C  OH +
 R  C  OH + ROH
H
  
R  C  OH  R  C  OH R
2

+OH O O  R
H
R
Tetrahedral intermediate Ester hydrate

(Explanation + chemical reaction for acid catalysed addition) [2]


2. In the ester hydrate, one of the OH group is protonated and lost as water.
Then another OH group loses a proton to obtain ester.

OH OH H +O  H O
H 
+  R  C + H3O+

R  C  O  H  R C  O H
 RC + H2O 
OR OR OR OR

(Explanation + chemical reaction for acid catalysed dehydration) [2]


b. Dehydration of alcohols:
1. Primary alcohol is dehydrated by heating with 95% H2SO4 at 443 K.
eg. H H
H H
H  C  C  H  95% H 2SO4
443K
 C=C + H2O

H OH H H
Ethanol Ethene
(Reaction + names of reactants, products and reagents) [1]

55
Board Answer Paper : March 2016
2. Secondary alcohol is dehydrated by heating with 60% H2SO4 at 373 K.
eg. H H H
 

H  C  C  C  H 
60% H2SO4
373K
 CH3  CH = CH2 + H2O
Propene
H OH H
Propan-2-ol
(Reaction + names of reactants, products and reagents) [1]
3. Tertiary alcohol is easily dehydrated by heating with 20% H2SO4 at 363 K.
eg. 
H CH3 H CH3
  
H  C  C  C  H 
20% H2SO4
363K
 CH3  C = CH2 + H2O
2-Methylpropene
H OH H
2-Methylpropan-2-ol

(Reaction + names of reactants, products and reagents) [1]


ii. a. Vitamins: Vitamins are the organic substances that must be supplied to permit
proportionate growth in living beings (humans) or for the maintenance of the
structure. [1]
b. Diseases caused due to deficiency of vitamin A:
1. Night blindness
2. Dryness of skin and hair
3. Retardation of growth
(Any two diseases) [½ × 2] [1]
c. Structure of nucleoside and nucleotide:
O
O  O
HO H2C Base O P O H2C Base
Sugar Sugar
H H O H H
H H H H
OH OH OH OH
(I) (II)
Nucleoside Nucleotide
[1  2] [2]
d. Reaction with nitrous acid:
The reaction is used to distinguish between primary, secondary or tertiary
nitroalkanes. With nitrous acid, -hydrogen is replaced by nitroso group.
1. 1-Nitropropane (Primary nitroalkane) reacts with nitrous acid to form blue
coloured nitroso-nitroalkane (aci form) which on dissolution in sodium
hydroxide gives red solution.
H H
O O
+
C2H5  C – N + HO  N = O 
NaNO2 /HCl
 C2H5  C N+ + H2O
O O
H NO
1-Nitropropane (nitroform)

OH ONa
C2H5  C = N + NaOH  C2H5  C = N+ + H2O
+

O O
NO NO
(aci form) Red solution
Blue solution
[1]

56
56
Chemistry
2. 2-Nitropropane (Secondary nitroalkane) reacts with nitrous acid to form
corresponding blue coloured nitroso-nitroalkane, which is insoluble in sodium
hydroxide as it does not contain replaceable -hydrogen atom.
CH3 O CH3
O
+
H3C  C – N + HO  N = O 
NaNO 2 / HCl
 H3C  C N+ + H2O
O O [1]
H NO
2-Nitropropane (blue solution)

3. 2-Methyl-2-nitropropane (Tertiary nitroalkane) does not react with nitrous acid


because there is no replaceable -hydrogen atom on the carbon atom carrying
the  NO2 group. [1]
Q.8. Answer any ONE of the following:
i. (C) Swarts reaction [1]
ii. (D) p-Aminophenol [1]
Phenols with electron donating substituents like alkyl (R) and amine ( NH2) groups are
less acidic because these substituents do not delocalize the negative charge.
p-aminophenol (pKa  10.5) is weaker acid than p-cresol (pKa  10.2)
iii. (D) propanoic acid [1]

CH3  CH2  C  N + 2H2O + HCl 
 CH3  CH2  COOH + NH4Cl
Propanenitrile Propanoic acid
iv. (A) Ethylamine [1]
Primary amines undergo this reaction.
v. (D) Uracil [1]
vi. (C) PHBV [1]
Others are homopolymers.
vii. (D) antidepressant [1]

57
Board Answer Paper : March 2016

BOARD ANSWER PAPER : MARCH 2016


MATHEMATICS AND STATISTICS

Note: Answer to every question must be written on a new page.

SECTION – I

Q.1. (A) Select and write the most appropriate answer from the given alternatives in each of
the following sub-questions:
i. (D)
~[p  (q  r)]
≡ ~p  ~(q  r) ….(Negation of conjunction)
≡ ~p  (q  ~r) .…(Negation of implication) [2]
ii. (C)

sin1 (1  x)  2 sin1 x =
2
Let x = sin 

 sin1 (1  sin )  2 sin1 (sin ) =
2

 sin1 (1  sin )  2 =
2

 sin1 (1  sin ) = + 2
2
 
 1  sin  = sin   2  
2 
  
 1  sin  = cos 2 …  sin  
    cos  
  2  
 1  sin  = 1  2sin2 
 2sin2   sin  = 0
 sin  (2 sin   1) = 0
1
 sin  = 0 or sin  =
2
1
 x = 0 or x =
2

, sin1 (1  x)  2sin1 x = sin1  1    2 sin1  


1 1 1
For x =
2  2 2
 
= sin1    2 sin1   =  sin1   =  
1 1 1
2 2 2 6 2
1
 x
2
 x=0 [2]
iii. (A)
Equations of the co-ordinate axes are x = 0 and y = 0.
Equations of the lines passing through the point (2, 3) and parallel to the co-ordinate axes
are x = 2 and y = 3
i.e., x  2 = 0 and y  3 = 0.
 the joint equation of these lines is
(x  2) (y  3) = 0
 xy – 3x – 2y + 6 = 0 [2]
58
58
Mathematics and Statistics
(B) Attempt any THREE of the following:
1 1
1 2 3   6 3
i. AB =   1 2  =   [1]
1  2  3 1 2   4 1 

6 3
Consider, AB = = (6  1) – (–4  –3) = 6 – 12 =  6 ≠ 0
4 1
 AB is a non-singular matrix
1 1 1 3
 (AB)1 = (Adj AB) = [1]
AB 6  4 6 

ii. Let a = 3iˆ  2jˆ + kˆ , n = 4iˆ + 3jˆ + 2kˆ


Vector equation of the plane passing through the point A  a  and perpendicular to the
vector n is r . n = a . n [1]
     
r . 4iˆ  3jˆ  2kˆ = 3iˆ  2ˆj  kˆ . 4iˆ  3jˆ  2kˆ 
= 3(4)  2(3) + 1(2)
= 12  6 + 2
  
r . 4iˆ  3jˆ  2kˆ = 8, [1]
which is the vector equation of the plane

iii. Given, p = ˆi  2ˆj  kˆ and q = ˆi  4ˆj  2kˆ


Since R  r  , divides the line segment PQ internally in the ratio 2 : 1,

 r =
 
2 q 1 p
2 1

=
  
2 ˆi  4ˆj  2kˆ  ˆi  2ˆj  kˆ  [1]
3
2iˆ  8ˆj  4kˆ  ˆi  2ˆj  kˆ
=
3
1

= 3iˆ  6ˆj  3kˆ
3

 ˆ ˆ
r = i  2 j k ˆ [1]

iv. Given equation is 6x2 + kxy + y2 = 0.


Its auxiliary equation is m2 + km + 6 = 0
Since, 2x + y = 0 is one of the lines given by 6x2 + kxy + y2 = 0
Slope of the line 2x + y = 0 is 2.
 2 is one of the roots of the auxiliary equation m2 + km + 6 = 0 [1]
 (2)2 + k(2) + 6 = 0
 4  2k + 6 = 0
 2k = 10
 k=5 [1]
v. Direction ratios of the given lines
are 3, 2k, 2 and 3k, 1, 5
Since, the lines are at right angles
 a1a2 + b1b2 + c1c2 = 0
 (3)(3k) + (2k)(1) + (2) (5) = 0 [1]
 9k + 2k  10 = 0
10
 k= [1]
7

59
Board Answer Paper : March 2016
Q.2. (A) Attempt any TWO of the following:
i.
1 2 3 4 5
p q pq (p  q)  q [(p  q)  q]  p
T T T T T
T F F F T
F T T T F
F F T F T
In the above truth table, the entries in the last column are a combination of T and F.
 [(p  q)  q]  p is contingency [1]
[one mark each for column 4 and column 5]
ii. Consider  ABC.
Let P, Q, R be the midpoints of the sides BC, CA, AB respectively.
Let a , b , c , p , q , r , g be the position vectors of the points A, B, C, P, Q, R, G
respectively.
Since P, Q, R are the mid-points of the sides BC, CA, AB respectively
A

R Q
G

B C
P
 By midpoint formula, we get
bc
p= ….(i)
2
ca
q= ….(ii) [1]
2
ab
r= ….(iii)
2
From (i), (ii) and (iii), we get
2p = b + c  2p + a = a + b + c
2q = c + a  2q + b = a + b + c
2r = a + b  2r + c = a + b + c
2p  a 2q  b 2r  c a bc
 = = =
3 3 3 3
2p  a 2q  b 2r  c a bc
 = = = = g (say) [1]
2 1 2 1 2 1 3
This shows that the point G whose position vector is g lies on the three medians AP, BQ,
CR dividing them internally in the ratio 2:1.
Hence, the three medians are concurrent. [1]
iii. The shortest distance between the lines
r  a1   b1 and r  a 2   b2 is

d
a 2 
 a1  b1  b2 
b1  b 2

 
Given lines are r  4 ˆi  ˆj   (iˆ  2ˆj  3k)
ˆ

and r  (iˆ  ˆj  2k)


ˆ  (iˆ  4 ˆj  5k)
ˆ

60
60
Mathematics and Statistics
Here, a1  4iˆ  ˆj , b1  ˆi  2ˆj  3k,
ˆ

a 2  ˆi  ˆj  2kˆ and b2  ˆi  4ˆj  5kˆ

Now, a 2  a1  (iˆ  ˆj  2 k)
ˆ  (4 ˆi  ˆj)

  3iˆ  2 kˆ
ˆi ˆj kˆ
and b1  b2  1 2 3
1 4 5

 ˆi ( 10  12)  ˆj(  5  3)  kˆ (4  2)

 2 ˆi  2 ˆj  2 kˆ

 b1  b 2 = 22  22  22

= 444
= 12
= 2 3 [1]

 
Consider,  a 2  a1    b1  b 2   3iˆ  2kˆ  2iˆ  2ˆj  2kˆ 
=32+02+22
=6+0+4
=2 [1]
 shortest distance between the given lines
2 1
  units [1]
2 3 3

(B) Attempt any TWO of the following:


C C
i. Consider L.H.S. = (a  b)2 cos2 + (a + b)2 sin2
2 2
C C
= (a2 + b2  2ab) cos2 + (a2 + b2 + 2ab) sin2 [1]
2 2
C C C C
= (a2 + b2) cos2  2ab cos2 + (a2 + b2) sin2 + 2ab sin2
2 2 2 2

= (a2 + b2)  cos2  sin 2   2ab  cos2  sin 2 


C C C C
[1]
 2 2  2 2
= (a2 + b2)(1)  2ab cos C ….[ cos2   sin2  = cos 2] [1]
= a2 + b2  2ab cos C
= c2 ….[By cosine rule]
= R.H.S. [1]
C C
 (a  b)2 cos2 + (a + b)2 sin2 = c2
2 2

ii. To draw the feasible region, construct table as follows:

Inequality 2x + y  7 2x + 3y  15 y3
Corresponding equation (of line) 2x + y = 7 2x + 3y = 15 y=3
Intersection of line with X-axis 7   15 
 ,0   ,0  
2  2 
Intersection of line with Y-axis (0, 7) (0, 5) (0, 3)
Region Non-origin side Origin side Origin side

61
Board Answer Paper : March 2016
Y

5
x=3
4 C(3/2,4) [1]

3 B(3, 3)

1 A(3, 1)

X
O 1 2 3 4 5 6 7 8 9

2x + y = 7 2x + 3y = 15
Y
Shaded portion ABC is the feasible region whose vertices are A, B and C.
A is the point of intersection of the lines x = 3 and 2x + y = 7.
Putting x = 3 in 2x + y = 7, we get
2(3) + y = 7
 y=1
 A  (3, 1)
B is the point of intersection of the lines x = 3 and 2x + 3y = 15.
Putting x = 3 in 2x + 3y = 15, we get
2(3) + 3y = 15
 x=3
 B  (3, 3)
C is the point of intersection of the lines 2x + y = 7 and 2x + 3y = 15.
Solving both equations ,we get
C(3/2 ,4)
 A  (3, 1), B  (3, 3) and C (3/2 ,4) [1]
Here, the objective function is
Z = 4x + 5y,
 Z at A(3, 1) = 4(3) + 5(1) = 17
Z at B (3, 3) = = 4(3) + 5(3) = 27 [1]
Z at C(3/2 ,4) = 4(3/2) + 5(4) = 26
 Z has minimum value 17 at A(3 ,1)
 Z is minimum, when x = 3 and y = 1. [1]

iii. Let the cost of 1 dozen pencils, 1 dozen pens and 1 dozen erasers be ` x, ` y and ` z
respectively.
According to the given conditions,
4x + 3y + 2z = 60
2x + 4y + 6z = 90 i.e. x + 2y + 3z = 45
6x + 2y + 3z = 70 [1]
62
62
Mathematics and Statistics
Matrix form of the given system of equations is,
é 4 3 2ù é xù é60ù
ê ú ê ú ê ú
ê 1 2 3ú ê y ú = ê 45ú [1]
ê ú ê ú ê ú
ê 6 2 3ú êzú ê70ú
ë û ë û ë û
Applying R1  R2,
é 1 2 3ù é x ù é 45ù
ê ú ê ú ê ú
ê 4 3 2ú ê y ú = ê60ú
ê ú ê ú ê ú
ê 6 2 3ú ê z ú ê70ú
ë û ë û ë û
Applying R2  R2  4R1, R3  R3  6R1,
é1 2 3ù é xù é 45ù
ê ú ê ú ê ú
ê0 - 5 - 10ú ê y ú = ê-120ú
ê ú ê ú ê ú
ê0 - 10 -15 ú êzú ê-200ú
ë û ë û ë û
Applying R3  R3  2R2,
é1 2 3ù é xù é 45ù
ê ú ê ú ê ú
ê0 - 5 - 10ú ê y ú = ê-120ú
ê ú ê ú ê ú
ê0 0 5 úû êzú ê 40ú
ë ë û ë û
Hence, the original matrix is reduced to an upper triangular matrix.
 By equality of matrices, we get
x + 2y + 3z = 45 ....(i)
5y  10z = 120
i.e. y + 2z = 24 ....(ii) [1]
5z = 40 ....(iii)
i.e. z = 8
Substituting z = 8 in equation (ii), we get
y + 2(8) = 24
 y=8
Substituting z = 8 and y = 8 in equation (i), we get
x + 2(8) + 3(8) = 45
 x + 16 + 24 = 45
 x=5
 x = 5, y = 8, z = 8
Thus, the cost of pencils is ` 5 per dozen, that of pens is ` 8 per dozen and that of erasers is
` 8 per dozen. [1]
Q.3. (A) Attempt any TWO of the following:
i. Let a = 7iˆ  kˆ , b  2iˆ  5jˆ  3kˆ , c  4iˆ  3jˆ  kˆ be the coterminus edges of a tetrahedron.
1
Volume of the tetrahedron =  a b c  [1]
6
7 0 1
1
= 2 5 3 [1]
6
4 3 1
1
= 7(5  9)  0 1(6  20)
6
1
= (98  14)
6
1
= (84) = 14
6
 Volume of the tetrahedron is 14 cubic units. [1]
ii. Consider, LHS = (p  q)  (p  q)
 (p  q)  (p  q) …. (Negation of disjunction) [1]
 p  (q  q) ….(Distributive law) [1]
63
Board Answer Paper : March 2016
 p  T ....(Complement law)
p ....(Identity law) [1]
= RHS.
 (p  q)  (p  q)  p
iii. Let ax2 + 2hxy + by2 = 0 ….(i)
be a homogeneous equation of degree 2 in x and y.
Case I:
If b = 0 (i.e., a  0, h  0), then the equation (i) reduces to ax2 + 2hxy = 0
i.e., x(ax + 2hy) = 0
This represents two lines, x = 0 and ax + 2hy = 0, both passing through the origin.
Case II:
If a = 0 and b = 0 (i.e., h  0), then the equation (i) reduces to 2hxy = 0, i.e., xy = 0 which
represents the coordinate axes and they pass through the origin. [1]
Case III:
If b  0,
Multiplying both sides of equation (i) by b, we get
abx2 + 2hbxy + b2y2 = 0
 b2y2 + 2hbxy = abx2
To make L.H.S. a complete square, we add h2x2 on both the sides.
 b2y2 + 2hbxy + h2x2 = abx2 + h2x2
 (by + hx)2 = (h2  ab)x2

 
2
 (by + hx)2 =  h 2  ab x 
 

(by + hx)   h  ab  x  = 0


2
2
 2
 

    
 by  hx   h  ab x   (by  hx ) 
2
 
h 2  ab x  = 0

[1]
This is the joint equation of two lines
(by + hx) +  
h 2  ab x = 0

and (by + hx)   h  ab  x = 0


2


i.e., h  h 2  ab  x + by = 0

and  h  h2  ab  x + by = 0

These lines pass through the origin.


From the above three cases, we conclude that the equation ax2 + 2hxy + by2 = 0 represents a
pair of lines passing through the origin, if h2  ab  0. [1]
(B) Attempt any TWO of the following:
i. Let AB be a line drawn from the point A(1, 2, 1), which is perpendicular to the line joining
P(1, 4, 6) and Q(5, 4, 4).
Let B divides PQ internally in the ratio  : 1.
5  1 4  4 4  6
 B   , ,

 ….(i) [1]
  1  1  1 
Direction ratios of AB are
5  1 4  4 4  6
 1,  2, 1
 1  1  1
4 2  2 3  5
i.e., , ,
 1  1  1
Now, direction ratios of PQ are 5  1, 4  4, 4  6 i.e., 4, 0, –2.
Since AB is perpendicular to PQ

64
64
Mathematics and Statistics
 4   2  2   3  5 
 4   0   2  0 [1]
   1     1    1 
16  6  10
 0
 1
 10  = 10
 =1 [1]
Putting  = 1 in (i), we get
B   , , 
6 8 10
2 2 2 
 B  (3, 4, 5)
 Coordinates of the required foot of the perpendicular are (3, 4, 5). [1]

ii. Let a, b, c be the position vectors of the points A, B, C respectively.


 a = î + ĵ  2 k̂ , b = î + 2 ĵ + k̂ , c = 2 î  ĵ + k̂

B C

Now, AB = b  a = ĵ + 3 k̂
AC = c  a = î  2 ĵ + 3 k̂
n is perpendicular to AB and AC both.
 n = AB  AC
ˆi ˆj kˆ
= 0 1 3
1 -2 3

= î (3 + 6)  ĵ (0  3) + k̂ (0  1)
= 9 î + 3 ĵ  k̂ [1]
Vector equation of the plane is r.n  a.n
 r   9iˆ + 3jˆ  kˆ  =  ˆi + ˆj  kˆ    9iˆ + 3jˆ  kˆ  [1]
= 1(9) + 1(3)  2(1)
=9+3+2
 r   9i + 3j  k  = 14
ˆ ˆ ˆ [1]
for cartesian form,
putting r = xˆi + yˆj+ zkˆ in (i), we get
 xˆi + yˆj+ zkˆ  9iˆ  3jˆ  kˆ  = 14
9x + 3y  z = 14, [1]
which is the cartesian form of equation
iii. sin x + sin 3x + sin 5x = 0
 (sin 5x + sin x) + sin 3x = 0
5x  x   5x  x 
 2sin   cos   + sin 3x = 0 [1]
 2   2 
 2 sin 3x cos 2x + sin 3x = 0
 sin 3x (2cos 2x + 1) = 0
 sin 3x = 0 or 2cos 2x + 1 = 0
65
Board Answer Paper : March 2016
1
 sin 3x = 0 or 2cos 2x =  [1]
2
  
 sin 3x = 0 or cos 2x =  cos = cos  
3  3
2
 sin 3x = 0 or cos 2x = cos [1]
3
Since, sin  = 0 implies  = n and cos  = cos  implies  = 2n   , n  Z.
2
 3x = n or 2x = 2m 
3
n 
 the required general solution is x = or x = m  , where n, m  Z. [1]
3 3

SECTION – II

Q.4. (A) Select and write the most appropriate answer from the given alternatives in each of
the following sub-questions:
i. (C)
f(x) is continuous at x = 1. ….(given)
 lim f  x   lim f(x)
x 1 x 1 

 lim  k  x   lim  4 x + 3 
x 1 x 1

 k +1 = 4(1)+3
 k=6 [2]
ii. (A)
Equation of the curve is y = x2 + 4x + 1
Differentiating w.r.t. x, we get
dy
= 2x + 4
dx
 Slope of tangent at (1, 2) is
 dy 
  = 2(1) + 4 = 2 + 4 = 2
 dx ( 1,  2)
 dy 
Equation of tangent is y  y1 =   (x  x1)
 dx ( x1 , y1 )
Here, (x1, y1)  (1, 2)
 [ y  (2)] = 2[x  (1)]
 y + 2 = 2(x + 1) = 2x + 2
 2x  y = 0 [2]
iii. (C)
n = 10, E(X) = 8 ....(given)
But, E(X) = np
 8 = 10 (p)
8 4
 p=   0.8 [2]
10 5
(B) Attempt any THREE of the following:
i. y = xx
Taking logarithm on both sides, we get
log y = x log x
Differentiating w.r.t. x, we get
1 dy 1
 = x + log x1 [1]
y dx x
dy
 = y (1 + log x) = xx (1 + log x) [1]
dx

66
66
Mathematics and Statistics
ii. Let s = 5 + 20t  2t2
Differentiating w.r.t. t, we get
ds
= 20  4t
dt
ds
v= = 20  4t ....(i) [1]
dt
Given, v = 0
 20  4t = 0
t=5
Differentiate equation (i) by t, we get
d 2s dv
a =  =04
dt 2 dt
=4
 d 2s  2
  2  =  4 unit  sec [1]
 dt  t 5
 The acceleration is  4 unit  sec2
iii. Given equation of parabola is y2 = 4ax.
 y=2 a x

Y
y2 = 4ax
A

X X
O S(a, 0)

Y
 required area = area of the region OBSAO
= 2(area of the region OSAO)
a


= 2 y dx
0
a


= 2 2 a x dx
0
[1]

=4 a 
0
x dx

é2 3 a
ù
= 4 a êê x 2 úú
ë 3 û0
8 é 3 ù 8 2
= a êa 2 - 0ú = a sq. units [1]
3 ë ê ú 3
û
n
iv. Given,  Pi = 1
i 1
 k + 2k + 3k + 4k + 5k = 1
 15k = 1
1
 k= [1]
15

67
Board Answer Paper : March 2016
X=x 1 2 3 4 5
P(X = x) 1 2 3 4 5
15 15 15 15 15

 P(x ≤ 4) = P(x = 1) + P(x = 2) + P(x = 3) + P(x = 4)


1 2 3 4
=   
15 15 15 15
10
=
15
2
= [1]
3

sin x
v. Let I =  36  cos2 x
dx

Put cos x = t
Differentiating w.r.t.x, we get
sin x dx =  dt [1]
dt
 I=  62  t 2
 t 
=  sin 1    + c
 6 

I =  sin1 
cos x 
  +c [1]
 6 

Q.5. (A) Attempt any TWO of the following:


i. Let x be a small increment in the value of x.
Since u is a function of x, there should be a corresponding increment u in the value of u.
Also y is a function of u.
 there should be a corresponding increment y in the value of y.
y u
Consider, y = 
x u x
Taking lim on both sides, we get
x  0

y y u
lim = lim  lim ….[ x  0, u  0] [1]
x 0 x x 0 u x 0 x

 y = lim y  lim u ….(i)


lim
x  0 x u  0 u x  0 x

du
But, lim u = exists and is finite.
x 0 x dx
dy
Also, lim y = exists and is finite.
u  0 u du
 limits on R.H.S. of (i) exist and are finite. [1]
Hence, limits on L.H.S. should also exist and be finite.
y dy
 lim = exists and is finite.
x  0 x dx
dy dy du
 =  [1]
dx du dx
ii. Since, there are six patients
n=6
p = P(success) = 0.5 and q = 1  p = 0.5
X  B(n = 6, p = 0.5)
The p.m.f. of x is given as P(X = x) = nCx(p)x (q)nx [1]
P(X = x) = P(x) = 6Cx (0.5)x (0.5)6x,
x = 0, 1, 2, 3, 4, 5, 6
68
68
Mathematics and Statistics
a. P(none will recover)
P(X = 0) = 6C0 (0.5)0 (0.5)60
= (1) (1) (0.5)6 [1]
= 0.015625
b. P(half of them will recover)
P(X = 3) = 6C3 (0.5)3 (0.5)63
6!
= (0.5)3 (0.5)3
3!.3!
6  5  4  3!
= (0.125) (0.125)
3  2 1 3!
= 20  0.015625
= 0.3125 [1]

x
iii. Let I = a 0
2
cos x  b2 sin 2 x
2
dx ....(i)


 x  a a

 I=  dx ....   f  x  dx   f  a  x  dx  [1]
0
a 2 cos2    x   b 2 sin 2    x   0 0 

 x
 I= a
0
2
cos x  b 2 sin 2 x
2
dx ....(ii)

Adding (i) and (ii), we get



x x
2I = a
0
2
cos2 x  b2 sin 2 x
dx

1
= a
0
2
cos2 x  b2 sin 2 x
dx

Dividing Nr and Dr by cos2x, we get



sec2 x
2I =   dx
0
a 2  b 2 tan 2 x

2
 sec2 x sec2 (   x )   2a a

=   2  dx ….   f ( x )dx   f ( x )  f (2a  x ) dx 
0
a  b 2 2
tan 2
x a  b 2
tan 2
(   x )   0 0 

2
 sec 2 x sec2 x 
=    2 2 2  2 2 2  dx
0
a  b tan x a  b tan x 

2
sec 2 x
= 2 dx
0
a  b 2 tan 2 x
2


2
sec 2 x
 I =  dx [1]
0
a 2  b2 tan 2 x
Put tan x = t
 sec2 x dx = dt

When x = 0, t = 0 and when x = ,t=
2
 
dt  dt
 I =  a 2
 b 2 2
t
= 2
b a 2

  t
0 0 2

b

  
 1  1  t  
= 2   tan  a  
b a   
  
b   b   0

69
Board Answer Paper : March 2016

  1  bt  
=  tan  a  
ab  0

= (tan1   tan 1 0)
ab
  
=   0
ab 2 
2
 I = [1]
2ab

(B) Attempt any TWO of the following:


f(0) = log  
2
i. ….(given)
 3
(4 x  1)  (e x  1)
x
4 e 4 1 e 1
x x
x
x
lim f(x) = lim = lim = lim
x 0 x 0 6x  1 x 0 6x  1 x 0 6x  1
x
4x  1 ex  1

= lim x x x [1]
x 0 6 1
x
4x  1 ex  1
lim  lim
=
x0 x x 0 x
6x  1
lim
x 0 x
log 4  log e
= [1]
log 6
4
log  
= e
log 6
 lim f(x)  f(0)
x 0

 f is discontinuous at x = 0. [1]
The discontinuity of f is removable and it can be made continuous by redefining the function
as
x x
4 e
f(x) = x
, for x  0
6 1
4 [1]
log   at x = 0
= e , for x = 0
log 6
ii. Let I =  a 2  x 2  1dx

= d
a 2  x 2  1dx   
 dx
 
a 2  x 2   1dx  dx
 [1]
2 x
= x  a2  x2    x dx
2 a2  x2
(a 2  x 2 )  a 2
= x  a2  x2   dx
a2  x2
 a 2  x2
 a2
= x  a2  x2     dx 
 a x a x  2 2 2 2

1
= x  a 2  x 2   a 2  x 2 dx + a 2  2 2 dx
a x [1]
70
70
Mathematics and Statistics

I = x  a 2  x 2  I + a 2 sin 1   + c1
x

a [1]
2I = x  a  x + a sin   + c1
x
 2 2 2 1

 
a
x 2 a2 x c
 I= a  x 2 + sin 1   + 1
2 2 a 2
x 2 a2 x
  a 2  x 2 dx =
2
a  x 2  sin 1    c ,
2 a
[1]

c1
where c =
2

iii. Let  be the temperature of the body at time t. Temperature of air is 10C.
According to the Newton’s law of cooling we have.
d
 (  0)
dt
d
= k (  0), k > 0 [1]
dt
 = 0 + c ekt
Using initial condition
 = 10 + 100 ekt [1]
But  = 60C, t = 1 hour
1
ek = [1]
2
we have to find t when  = 35C
t
1
35 = 10 + 100  
2  
t

=  
1 1
4 2
2 t
1 1
  =  
 
2 2
t = 2 hours
Hence additional time required to cool the body form 60C to 35C is equal to
= Time required to cool the body to 35 C  Time required to cool body to 60 C
= 2  1 = 1 hour [1]
Q.6. (A) Attempt any TWO of the following:
a a
i. Let I =  f ( x)dx  f (2a  x)dx
0 0

= I1 + I2
For I2, put 2a – x = t
 dx = dt
 dx = dt
When x = 0, t = 2a and when x = a, t = a [1]
a
 I2 =   f  t  dt
2a

 I = I1 + I2
a a
=  f ( x )dx   f (t) dt
0 2a
a 2a
 b a

=  f ( x )dx  f (t)dt ....   f ( x )dx   f ( x )dx 
0 a  a b 
[1]
71
Board Answer Paper : March 2016
a 2a
 b b

=  f ( x )dx   f ( x )dx ….   f ( x )dx f (t)dt 
0 a  a a 
2a
 b c b

=  f ( x )dx ….   f  x  dx   f  x  dx   f  x  dx;a  c  b  [1]
0  a a c 
2a a a
  f ( x )dx f ( x )dx   f (2a  x )dx
0 0 0

1  log x
ii. Let I =  x  2  log x 3  log x  dx
1  log x
=  x 1  1  log x  2  1  log x  dx
Put 1 + log x = t
1
 dx = dt
x
t dt
 I= 
1  t  2  t 
[1]

t A B
Let = +
1  t  2  t  1 t 2t
 t = A(2 + t) + B(1 + t) ….(i)
Putting t = 1 in (i), we get
 1 = A(2  1)
 A = 1
Putting t = 2 in (i), we get
 2 = B(1  2)
 B=2 [1]
t 1 2
 = +
1  t  2  t  1 t 2t
1 2
 I = 1 t dt + 2t dt

= log |1 + t| + 2log |2 + t| + c
= log |1 + 1 + log x| + 2log |2 + 1 + log x| + c
= log |2 + log x| + 2log |3 + log x| + c
 I = log |(3 + log x)2 |  log | (2 + log x) | + c [1]

iii. Let y = cos1 2 x 1  x 2  


Put x = sin 
  = sin1 x
y = cos1  2sinθ 1 sin 2θ  [1]
 
= cos1 [2 sin  cos ]
= cos1 [sin 2 ]
 
= cos1 cos   2θ  
π
 2  

y =  2
2

y=  2 sin1 x [1]
2
Differentiating w.r.t.x, we get
dy 1 2
= 0 2 = [1]
dx 1  x2 1 x 2

72
72
Mathematics and Statistics
(B) Attempt any TWO of the following:
dy
i. cos (x + y) =1 ....(i)
dx
Put x + y = v
Differentiating, we get
dy dv
1+ = [1]
dx dx
 equation (i) becomes
cos v 
dv 
1 = 1
 dx 
dv
 cos v  cos v = 1
dx
dv
 cos v = 1 + cos v
dx
cos v
 dv = dx [1]
1  cos v
Integrating both sides, we get
cos v
 1 cos v dv =  dx
1 cos v 1
  1 cos v
dv   dx

 1 
  1 1 cos v  dv =  dx
1
 1.dv   1 cos v dv   dx
1
 1.dv   v
dv   dx
2cos 2
2

v  tan   = x + c
v
 [1]
2  

x + y  tan 
x+ y 
  =x+c
 2 

y = tan 
x+ y 
  +c
 2 
This is general solution of different equation.
Now x = 0, y = 0 then c = 0
y = tan 
x+ y 
  is particular solution. [1]
 2 
ii. Let r be the radius of the circle and x be the length of the side of the square.
Then,
circumference of the circle + perimeter of the square = total length of the wire
 2r + 4x = l
 2r = l  4x
l  4x
 r= ….(i)
2
Let A = area of the circle + area of the square
= r2 + x2
2
l  4x 
=    +x
2
….[From (i)] [1]
 2 
l 2  8lx  16 x 2
=  x2
4

73
Board Answer Paper : March 2016
4x 2  16 x 2  8lx  l 2
=
4
4(  4) x 2  8lx  l 2
=
4
dA 8(   4) x  8l
 =
dx 4
d2A 8(   4) 2(   4)
and = = [1]
dx 2
4 
dA
Now, A is minimum, if =0
dx
8(   4) x  8l
i.e., if =0
4
i.e., if 8( + 4) x  8l = 0
i.e., if 8( + 4) x = 8l
8l l
i.e., if x = = [1]
8(   4) 4
 d2A  2(   4) l
Also,  2  = > 0 and hence A is minimum when x = .
 dx  x  l  4
 4

From (i),
 l 
l  4 
r =    4  = l  4l  4l
2 2    4 
l
=
2    4 
l
=
2    4
x
=
2
Thus, the sum of the areas of the circle and the square is the least, when radius of the circle
is half the side of the square. [1]
iii. c.d.f. of a continuous random variable X is given by
x
F (x) =  f  y  dy

[1]
x
a. F(x) =  f  y  dy
0
x
y
=  32
0
dy
x
 y2  x2
F(x) =   = xR [1]
 64  0 64
b. At x = 0.5 we can write
 0.5
2
1
F(x) = F(0.5) = = [1]
64 256
At x = 9,
For any value of x ≥ 8
F(x) = 1 [1]

74
74
Biology

BOARD ANSWER PAPER : MARCH 2016


BIOLOGY

Note: Answer to every question must be written on a new page.

SECTION – I
[BOTANY]

Q.1. Select and write the most appropriate answer from the given alternatives for each
sub-question:
i. (B) 1 : 2 : 1 [1]
ii. (C) 68 [1]
iii. (B) Saccharomyces [1]
iv. (D) Anthocyanin [1]
v. (A) UAG [1]
vi. (D) Acetyl - Co – A [1]
vii. (B) 5th June [1]
Q.2. (A) Answer in ‘One’ sentence only :
i. The cross between F1 hybrid (or F2) and the recessive parent is called test cross. [1]
ii. The pathogenic fungi used to selectively kill weeds or inhibits the growth of weeds is
known as ‘mycoherbicide’. [1]
iii. Anticodon is a triplet of nucleotides present on the loop of middle arm of t–RNA and is
complementary with the codon. [1]
iv. The process of formation of dark coloured amorphous substance known as humus,
which is a partially decomposed organic matter, is known as humification. [1]
v. The bubbles of CO2 trapped in glutein makes idlies puffy. [1]
vi. Ecological succession is the gradual (and predictable) change in the species
composition of a given area. [1]
(B)
Outer membrane
Inner membrane
DNA
Stroma
1 to 2 

Grana

Stroma lamellae
Ribosome

4 to 6
Ultrastructure of Chloroplast
[Proportionate diagram] [½]
[Any three labels: ½ mark each] [1½]
[Labels: Outer membrane, inner membrane, grana, stroma, stroma lamellae]
75
Board Answer Paper : March 2016
(C) Answer the following (Any TWO):
i. a. The process of induction and utilization of mutation for development of new crop
varieties having desirable traits is known as mutational breeding. [½]
b. Mutational breeding helps in producing disease resistant varieties. [½]
c. Plants are produced by inducing mutations. Chemicals or physical mutagens are used
for bringing about mutation. [½]
d. Varieties of moong beans resistant to yellow mosaic virus and powdery mildew have
been developed by the technique of mutational breeding. [½]

ii. Advantages of biogas:


a. Biogas is a cheap, safe and renewable source of energy.
b. It burns with a blue flame and without smoke.
c. It can be used for cooking, domestic lighting, street lighting and small scale
industries.
d. It is eco-friendly and does not cause pollution and imbalance of the environment.
e. It helps to improve sanitation of the surroundings.
f. It can be easily generated, stored and transported.
[Any four points: ½ mark each] [2]

iii. Carbon cycle:


a. The carbon enters the biotic components of ecosystem through green plants,
photosynthetic bacteria and cyanobacteria in terrestrial ecosystem and through
phytoplanktons and hydrophytes in the aquatic ecosystems.
These are called producers and carry out photosynthesis by taking in the atmospheric
CO2 and make carbohydrates and oxygen.
b. Carbohydrates are used as source of food by animals. Thus, carbon fixed by
producers enters the food chain and keeps moving through different living
organisms. They are returned to the soil in the form of detritus.
c. The decomposers breakdown the larger organic compounds into simpler ones and
then into inorganic molecules. In this process, small amount of CO2 is returned back
to atmosphere during enzyme catalyzed breakdown of dead matter by decomposers.
d. CO2 is released into atmosphere by respiration of producer and consumers.
e. Burning of fossil fuels in automobiles and machineries to produce energy, burning of
wood, organic debris also release CO2 in atmosphere.
f. Volcanic eruption and hot springs also release CO2 into atmosphere. [1]

CO2

Sugars,
Starches
Fossil
Fuels

Decomposition

Carbon Cycle [1]

76
76
Biology
iv. Floral adaptations for Chiropterophily:
a. Flowers are large, stout enough so that bats can hold on to the flowers.
b. Chiropterophilous plants are nocturnal, i.e. open their flowers during night time.
c. Flowers emit rotten fruits like fermenting fruity odour.
d. Flowers produce copious nectar.
e. Flowers have large number of stamens to produce a considerably large quantity of pollen
grains. [Any four points: ½ mark each] [2]

Q.3. (A) Answer the following (Any TWO):


i. a. When a single gene controls two (or more) different traits, it is called pleiotropic
gene and this phenomenon is called pleiotropy or pleiotropism.
The ratio is 2:1 instead of 3:1. [½]
b. According to Mendel’s principle of unit character, one gene (factor) controls one
character (trait), but sometimes single gene produces two related or unrelated
phenotypic expressions.
c. For example, the disease, sickle cell anaemia is caused by a gene Hbs. Normal or
healthy gene is HbA and is dominant. [½]
A S
d. The carriers (heterozygotes – Hb /Hb ) show signs of mild anaemia as their RBCs
become sickle-shaped (half-moon shaped) in oxygen deficiency. They are said to
have sickle-cell trait and are normal in normal conditions.
e. The homozygotes with recessive gene Hbs however, die of fatal anaemia.

Phenotype of parents Sickle-cell carrier  Sickle-cell carrier


Genotype HbA HbS HbA HbS

Gametes HbA HbS HbA HbS

Genotype of offsprings HbAHbA HbAHbS HbSHbA HbSHbS

Phenotype of offsprings Normal Sickle-cell carriers Sickle-cell Anaemics


1 2 1(dies)
Graphical representation of pleiotropy

[1]
f. Thus, the gene for sickle-cell anaemia is lethal in homozygous condition and
produces sickle cell trait in heterozygous carrier.
g. Two different expressions are produced by a single gene.
h. A marriage between two carriers will produce normal, carriers and sickle-cell
anaemic children in 1:2:1 ratio. But, sickle-cell anaemics who are homozygous for
gene HbS will die, as HbS is a lethal gene causing death of the bearer. [1]
Thus the sickle cell anemics die leaving carriers and normals in the ratio 2:1.

77
Board Answer Paper : March 2016
ii.
Glucose (6C)
Glycolysis
Pyruvic acid (3C)
NAD
CO2 NADH2
Acetylation Acetyl (2C)

Co-A

Acetyl Co-A (2C)


H2O
Co-A

Citric acid (6C)


H2O
Oxaloacetic acid (4C)
NADH2 Cis-aconitic acid (6C)
H 2O
NAD

Malic acid (4C)


H2O Isocitric acid (6C)
NAD

NADH2
Fumaric acid (4C)

FADH2 Oxalosuccinic acid (6C)

FAD
Succinic acid (4C) CO2
Co-A
H2O -ketoglutaric acid (5C)
Co-A
GTP GDP
Succinyl Co-A CO2
(4C)
ADP ATP
NAD
NADH2
Schematic representation of TCA Cycle/ Krebs Cycle

[Correct schematic representation] [3]


iii.
No. Cyclic Photophosphorylation Non-cyclic Photophosphorylation
a. Electrons emitted by chlorophyll The electrons emitted by chlorophyll do not
return back to the same chlorophyll. return back to the same chlorophyll.
b. First electron acceptor is FRS. First electron acceptor is CO - Q
c. It forms only ATP. It forms NADPH2 and ATP.
d. It does not involve photolysis of It involves photolysis of H2O.
H2O.

78
78
Biology
e. No evolution of O2. There is evolution of O2.
f. It is less efficient and less significant It is more efficient and significant process.
process.
g. Only Photosystem-I (P700) is Both Photosystem PS-I (P700) as well as PS-
involved in this cycle. II (P680) are involved.
h. It operates under low light intensity, It takes place under optimum light, aerobic
anaerobic conditions and poor conditions and in the presence of sufficient
availability of CO2. CO2.
i. It is found in photosynthetic bacteria It is absent in photosynthetic bacteria.
also.
j. Plastoquinone is not involved in Plastoquinone carries electrons from Co-Q to
cyclic photophosphorylation. the cytochrome complex during non-cyclic
photophosphorylation.
[Any six points:½ mark each] [3]
(B)
Perfect pairing
m-RNA
G U U
C A A

t-RNA
[1]
5 G C
C
A

Wobble pairing

G U C G U A G U G
m-RNA
C A A C A A C A A

t-RNA
5 G C 5 G C 5 G C
C C C
A A A

Wobble Hypothesis [2]

Q.4. Double Fertilization:


The fusion of one male gamete with egg and that of another male gamete with
secondary nucleus is called as double fertilization. [1]
It is the characteristic feature of angiosperms. It was discovered by Nawaschin (1898)
in Lilium and Fritillaria species.
It consists of two processes:
a. Syngamy: [½]
It is the fusion of first male gamete with egg. It results in the formation of diploid
zygote which develops to form embryo. It is also called generative fertilization.
b. Triple Fusion: [½]
It is the fusion of second male gamete with secondary nucleus. It results in the
formation of triploid PEN (Primary Endosperm Nucleus) which develops to form
endosperm.
Since both male gametes participate in fertilization, it is called double
fertilization.
79
Board Answer Paper : March 2016
Process of double fertilization is described as follows:
i. After pollination, the intine of the pollen grain forms pollen tube and passes through the germ
pore.
ii. The growth of pollen tube is stimulated by the sugary substance produced on the
stigma.
iii. The pollen tube with two male gametes and tube nucleus runs through the style and
finally turns towards the micropylar end of the ovule in the cavity of the ovary.
iv. The length of the pollen tube depends on the length of style.
v. When the pollen tube enters through the micropylar end of the ovule for fertilization, it
is called porogamy (sometimes it may enter through integuments and called as
mesogamy or sometimes through chalaza and called as chalazogamy). [½]
vi. Filiform apparatus of synergids attract the pollen tube towards egg apparatus.
vii. As the pollen tube elongates, it carries with it two haploid, non-motile male gametes
and hence, the fertilization is also called siphonogamy (siphon = tube). [½]
viii. On piercing the nucellus, the pollen tube penetrates the embryo sac. Its tip penetrates
the embryo sac and reaches the egg apparatus passing either between the egg and
synergids or between one synergid and wall of embryo sac.
ix. Ultimately, the tip of the pollen tube bursts and two male gametes are released.
x. The tube nucleus degenerates before bursting of the pollen tube.
xi. One of these male gametes fuses with the egg cell or oosphere causing fertilization, as a
result of which diploid oospore or zygote is formed. This is called first fertilization or
syngamy. [½]
xii. The other male gamete fuses with the secondary nucleus forming the triploid
endosperm nucleus which later on gives rise to endosperm. This is called as triple
fusion or second fertilization. [½]
xiii. Thus, this process of fertilization which occurs twice in the same embryo sac at a time
by two male gametes (syngamy and triple fusion) is called double fertilization.

Pollen grain (Germinating)

Stigma

Pollen tube Chalaza

Style
Antipodals
Triple fusion
Ovary Second male gamete
Secondary nucleus
Antipodal cells Male gamete
Secondary nucleus Egg Syngamy
Egg Damaged synergid
Synergid Pollen tube
Male gametes Micropyle

Double fertilization

[Proportionate diagram: 1 mark]


[Any four labels: ½ mark each] [3]
[Labels: Pollen tube, micropyle, male
gamete, secondary nucleus, egg
apparatus, antipodal cells, chalaza]
OR
i. r-DNA technology is defined as the technique of manipulating the genome of a cell or
organism so as to change the phenotype desirably. [1]
80
80
Biology
ii. Manipulation of the genome involves addition, removal, replacement or repair of a part of
genetic material that results into the desirable phenotype change.
iii. The following steps are involved in genetic engineering or r-DNA technology:
a. Isolation of desired gene:
The cell or organism from which the gene is taken is called ‘donor’. [½]
b. Fragmentation of desired gene:
From the DNA of this donor, desired gene is selected and isolated with the help of
restriction endonuclease enzyme. The donor DNA containing the desired gene is
called passenger DNA. [½]
c. Selection of vector:
A vector DNA (usually plasmid DNA or Phage DNA) is selected. [½]
d. Formation of recombinant DNA:
The vector DNA is cleaved at a specific point using restriction endonuclease enzyme.
The cut ends of vector DNA are sticky, i.e. cohesive.
The desired gene is now ligated with the vector DNA using ligase enzyme.
The vector DNA containing a new introduced gene is called recombinant DNA
(r-DNA) or chimeric DNA (Chimeric vector or chimeric plasmid). [½]
e. Gene transfer to the host:
This chimeric plasmid is introduced into bacterial cell (Host cell). e.g. E.coli for
cloning. Bacillus subtilis and Saccharomyces cerevisiae can also be used as host.
Such a bacterial cell with chimeric or r-DNA is called transformed host.
The incorporation of r-DNA into bacterial cell is generally done by electroporation. [½]
f. Cloning:
The transformed bacterial cell is now allowed to grow on the nutrient medium where it
multiplies rapidly.
It results in the formation of a large number of transformed bacterial cells.
All these cells have a copy of recombinant or chimeric DNA.
Generally, after introduction of r-DNA or chimeric plasmid in the host cell, amplification
is done.
It is a process in which the number of r-DNA in a bacterial cell is increased. [½]
iv. Therapeutic products made by recombinant DNA technique:
No. Therapeutic product Examples
a. Blood proteins Erythropoietin; Factors VII, VIII, IX, Tissue
plasminogen activator, Urokinase
b. Human hormones Epidermal growth factor, Follicle stimulating hormone,
Insulin, Nerve growth factor, Relaxin, Somatotropin
c. Immune modulators Interferon, interferon, Colony stimulating factor,
Lysozyme, Tumor necrosis factor
d. Vaccines Cytomegalovirus, Hepatitis B, Measles, Rabies.
[Any three points: 1mark each] [3]
SECTION – II
[ZOOLOGY]

Q.5. Select and write the most appropriate answer from the given alternatives for each
sub-question:
i. (A) Metacentric [1]
ii. (B) Tissue Growth Factor [1]
–ve
iii. (C) Rh [1]
iv. (B) atherosclerosis [1]
v. (D) intra-specific struggle [1]
vi. (B) Volant adaptation [1]
vii. (A) in-breeding [1]
81
Board Answer Paper : March 2016
Q.6. (A) Answer in ‘One’ sentence each:
i. Visit of veterinary doctor is mandatory for checking health problems of cattles, diseases of
cattles and their rectification.
[Any two points: ½ mark each] [1]
ii. Aquatic animals can afford to be ammonotelic because large quantity of water is required to
eliminate ammonia and water is available in excess to them. [1]
iii. PUC is mandatory for all vehicles in order to check air pollution. [1]
iv. Organic evolution is a slow, gradual, continuous and irreversible changes through which
the present day complex forms have descended from their simple, pre-existing forms of
the past. [1]
v. The genotype of Turner’s syndrome is 44 + XO where 44 is the autosome number. [1]
vi. R.F.L.P. stands for restriction fragment length polymorphism. [1]
(B)
Tunica interna
(endothelium)

Lumen
Elastic membrane

Tunica media
Tunica externa

T.S. of Vein

[Neat and Proportionate diagram: ½ mark]


[Any three labels: ½ mark each] [2]
(C) Attempt any TWO of the following:
i. Following factors are responsible for genetic variation:
a. Gene mutation:
Changes in the chemical make up of a gene is called gene mutation or point
mutation.
These mutations are the cause for new alleles that introduce variations in the gene
pool and it accounts for the change in gene frequency.
b. Gene flow:
Transfer of genes between populations that differ genetically from one another is
called gene flow. e.g. Migration of animals from one area to another, alters the gene
pool of the population. They contribute their genes to the gene pool of that area
which results in a change in gene frequency.
c. Genetic recombination:
In sexually reproducing organisms, during gametogenesis, the homologous
chromosomes exchange genetic material by the process of crossing over.
This produces new combinations and the phenomenon is called genetic
recombination.
It adds variability to individuals.
d. Genetic drift:
Any alteration in allele frequency, in the natural population by pure chance is called
genetic drift. For e.g. elimination of a particular allele from a population due to
events like accidental death prior to mating of an organism, i.e. the sole possessor of
particular allele. Smaller populations have greater chances for genetic drift. It is also
called Sewall Wright effect.
82
82
Biology
e. Chromosomal aberrations:
It refers to the structural alterations in a chromosome causing changes in the gene
arrangement. e.g., deletion, duplication, translocation and inversion.
They also cause variations in the Mendelian populations.
[Any two points:1 mark each] [2]
ii. A: Acrosome
[½]
Function: It secrets an enzyme hyaluronidase which helps on the penetration of egg during
fertilization. [½]
B: Tail piece or flagellum [½]
Function: It provides motility to the sperm (helps its movements in the female genital tract). [½]
iii. Artificial acquired active immunity:
a. Artificial acquired active immunity is acquired artificially by vaccination.
b. Vaccines contain dead or live the but attenuated pathogens or toxoids.
c. Vaccines are introduced into the body to stimulate the formation of antibodies by the
immune system.
d. Example: polio vaccine, BCG vaccine.
[Any two points:1 mark each] [2]
iv. Economic importance of fisheries:
a. Fishes are caught, processed, raised and marketed under fisheries. It provides good
job opportunities and self employment.
b. Culturing of fishes on a large scale in ponds, lakes and reservoirs boost the
productivity and economy of the nation.
c. Prawns and lobsters have market value all over the world.
d. They also yield a number of byproducts that hold commercial value.
e. The byproducts obtained from fishes include fish oil, fish meal, fertilizers, fish
guano, fish glue and Isinglass, which are widely used in paints, soaps, oils and
medicines.
[Any four points: ½ mark each] [2]
Q.7. (A) Attempt any TWO of the following: (6)[9]
i. Sex determination in human being:

Parents : Father Mother


XY XX
Sperms Ova
Gametes: X Y X X

Progeny : XX XX XY XY

[Chart: ½ mark]
Daughters 50% Sons 50%
[Labels: 1 mark] [11/2]

Human diploid cell has 46 chromosomes, i.e. 23 pairs in each cell.


Out of these, 22 pairs of chromosomes are called autosomes.
Autosomes determine all body characters like colour of hair, skin, colour of eyes, height, etc.
Out of 23 pairs, one pair of chromosomes is called sex chromosomes.
They are X and Y chromosomes. [½]
83
Board Answer Paper : March 2016
Every individual gets one set of chromosomes from his mother and one from his father.
A human male thus has 44 +XY chromosomes, whereas a female has 44 + XX chromosomes.
During gamete formation; meiosis or reductional division takes place and a gamete gets
only one set of chromosomes and thus it is haploid.
e.g. Female gamete (ovum) 22 + X, Male gamete (sperm) 22 + X or 22 + Y. [½]
When the male and female gametes unite to form a zygote, the chromosomes again become diploid.
Thus, the offspring gets the same number of chromosomes as his parents. Sex of the baby is
determined by the nature of sperm (X or Y) that fertilizes the ovum. Thus, in human being, it is the
male which determines the sex of the baby. [½]

ii. Steady population:


The relative proportion of various age groups in population is called the age structure. It
determine the reproductive status of a population. [½]
The three major age groups are:-
a. Pre  reproductive group ( 0 to 14 years)
b. Reproductive group (15 to 59 years)
c. Post  reproductive group ( age 60 years and above) [½]
The age structure determines the trend of the population. A population is said to be steady,
when it has same number of pre  reproductive and post  reproductive age groups. It is a
stable population. [½]

Post
reproductive

Reproductive

Pre 
reproductive

[Pyramid :1½ mark]


Pyramid of steady population

iii. a. Tissue plasminogen activator (TPA) is used to prevent or reverse blood clots.
b. Human Growth Hormone producer gene helps treat pituitary dwarfism.
c. Human blood clotting factor VIII is used to treat hemophiliacs.
d. Human insulin (HUMULIN) to treat diabetes (insulin dependent diabetes)
e. Tissue growth factor  Beta promotes epidermal growth and formation of new blood
vessels. Used in treatment of burns and for wound healing.
f. DNase is used to treat cystic fibrosis.
g. Bovine growth hormone helps to increase cattle and dairy yield.
h. Recombinant vaccines for prophylaxis of human and animal viral diseases (hepatitis
B).
i. Genetically engineered bacteria helps in enhanced production of industrial enzymes,
citric acid and ethanol.
[Any three points: 1 mark each] [3]
84
84
Biology
(B) L.S. of human Kidney:

Cortex

Capsule

Medullary
pyramid

Columns of
Bertini
Major
calyx
Pelvis

Minor calyx

Papilla of
pyramid
Ureter

V.S. /L.S of human kidney


[3]
[Proportionate diagram: 1 mark]
[Any four labels: ½ mark each] [3]
Q.8. Gametogenesis is the process of formation of gametes in sexually reproducing animals. [1]
It includes two process: spermatogenesis and oogenesis.
i. Spermatogenesis:
Spermatogenesis is the process of formation of haploid, microscopic and motile male
gametes, called spermatozoa, from the diploid spermatogonia of the testis of male
organism.
Primordial cell

Mitosis

Spermatogonium

Growth phase Primay spermatocyte

Secondary spermatocyte
Meiotic division
Spermatid

4 Spermatozoa
Spermatogenesis

[Diagrammatic representation] [1½]


85
Board Answer Paper : March 2016
Each testis has seminiferous tubules which are lined by cuboidal epithelium called germinal
epithelium.
Germinal cells in testes are known as primary germinal cells. Primordial cells passes through
three phases, namely:
a. Multiplication phase: Primordial cells undergo repeated mitotic divisions to
produce large number of spermatogonia.
Each spermatogonium is diploid (2n). [½]
b. The Growth phase: Spermatogonium cell accumulates food and grows in size.
Now, it is called primary spermatocyte. [½]
c. The Maturation phase: The primary spermatocyte undergoes first meiotic or
maturation division. The homologous chromosomes start pairing.
Each homologous chromosome splits longitudinally. Chiasma formation results in
exchange of genetic material.
At the end of 1st meiotic division, two haploid, secondary spermatocytes are formed. Each
secondary spermatocyte undergoes 2nd meiotic division and produces spermatids. So, at the
end of maturation phase, each spermatogonium produces four haploid spermatids.
Spermatid is non-motile, so it has to undergo spermiogenesis to become functional, motile
male gamete, i.e. spermatozoan. [½]
ii. Oogenesis is the process of formation of haploid female gametes, called ova, from the
diploid oogonia of the ovary of female organism. It is completed in three stages:

Primordial cell

Mitosis

Oogonium

Primary oocyte Growth


phase

Secondary
oocyte
Meiotic Division

1st polar
body

Ovum 2nd polar


body

3 Polar Bodies
Oogenesis
[Diagrammatic representation] [1½]
86
86
Biology
a. Multiplication phase: Germinal cells undergo mitosis to form large number of oogonia.
Oogonia in human beings are formed in the ovary of a female baby even before her birth. [½]
b. Phase of Growth: Just before puberty, under the influence of follicle stimulating hormone,
one of the oogonium grows in size. Growth in size of oogonium is larger than that seen in
spermatogenesis. This grown up cell is called primary oocyte. [½]
c. Maturation phase: Primary oocyte undergoes maturation or meiotic division.
Meiotic I division of primary oocyte shows equal nuclear division but unequal cytoplasmic
division.
So, at the end of meiosis I division, large sized haploid secondary oocyte and haploid small
sized polar body are formed.
Unequal division is meant for sufficient supply of food for developing embryo.
Secondary oocyte and polar bodies undergo 2nd meiotic division.
2nd meiotic division is arrested at metaphase stage and secondary oocyte is released from
ovary. This division is also unequal. Remaining part of division is completed at the time of
fertilization.
This ovum is ready for fertilization. [½]
OR

Frontal lobe
Central sulcus
Cerebrum Parietal lobe
Lateral sulcus
Parieto occipital sulcus

Occipital lobe
Temporal lobe
Pons varolii
Medulla oblongata
Cerebellum

Spinal cord

Lateral view of human brain

[Neat and Proportionate diagram: 1 mark]


[Any four labels: ½ mark each] [3]
Cerebrum is the largest part of the brain accounting for 80-85% of its weight.
It is divided into cerebral hemispheres (right and left) by a median longitudinal cerebral
fissure.
Two cerebral hemispheres are connected by a single thick bundle of nerve fibres called
corpus callosum.
It is the largest commissure in human brain.
It connects both the hemispheres and helps in co-ordination.
The outer part of cerebrum is called cortex, while the inner part is called medulla. [1]
Cerebral cortex is formed by neurons cell bodies which appear grey and hence are called
grey matter.
Cerebral medulla is mainly formed of white matter (axons of nerve cells).
Deep within the white matter, certain masses of grey matter are located which are called
basal nuclei.
Thick dorsal wall (roof) of cerebrum is called pallium and the ventrolateral wall is known
as corpora striata.
Cerebral cortex shows number of ridges called gyri and depressions called sulci.
The gyri increase the surface area of cerebral cortex.
There are three deep sulci on the cerebrum, namely Central sulcus, Lateral sulcus and
Parieto-occipital sulcus. [1]

87
Board Answer Paper : March 2016
These sulci divide each cerebral hemisphere into four lobes, viz. anterior frontal lobe,
middle parietal lobe, posterior occipital lobe, lateral temporal lobe.
The frontal and parietal lobes are separated by central sulcus, the parietal and temporal
lobes are separated by lateral sulcus, while the parieto-occipital sulcus separates parietal
lobe from occipital lobe. [1]
Functions of Cerebrum:
i. It controls all voluntary activities.
ii. It perceives sensory stimuli through vision, taste, smell, sound, touch and speech.
iii. It is a centre of memory, will power, intelligence, reasoning and learning.
iv. It is a centre for micturition, defaecation, weeping and laughing.
v. It is also a centre for emotions, thoughts and feelings like pain, pleasure, fear,
fatigue, pressure, temperature, etc.
[Any two functions : ½mark each] [1]

88
88
Physics

BOARD ANSWER PAPER : JULY 2016


PHYSICS

Note: Answer to every question must be written on a new page.

SECTION – I

Q.1. Select and write the most appropriate answer from the given alternatives for each
sub-question:
i. (C) [1]
ii. (A)
1
E= I2
2
 2E = (I)
2E
 =L [1]

iii. (A) [1]
iv. (C) [1]
v. (D)
2T cos 
h=
rg
1
 h [1]
r
vi. (A) [1]
vii. (B)
1 2
P= c
3
c1 P1 2 3
 =  = [1]
c2 P2 1 2

Q.2. Attempt any SIX:


i.
N N cos 


N sin 
G C
F cos  

F h
F sin 

A
B
W = mg
AC : inclined road surface
AB : horizontal surface
BC : height of road surface
G : centre of gravity of vehicle
W : (mg) weight of vehicle
N : normal reaction exerted on vehicle
 : angle of banking
(Diagram and labelling) (1+1) [2]
89
Board Answer Paper : July 2016
ii. Expression for critical velocity:

vC m
h
r

M
Earth R

[½]
a. Let, M = mass of the earth
R = radius of the earth
h = height of the satellite from the earth’s surface
m = mass of the satellite
vc = critical velocity of the satellite in the given orbit
r = (R + h) = radius of the circular orbit [½]
b. For the circular motion of the satellite, the necessary centripetal force is given as,
mvc2
FCP = ….(1)
r
c. The gravitational force of attraction between the earth and the satellite is given by,
GMm
FG = .…(2)
r2
d. Gravitational force provides the centripetal force necessary for the circular motion of
the satellite.
 FCP = FG
mv c2 GMm
 = ….[From equations (1) and (2)] [½]
r r2
GM
 vc2 =
r
GM
 vc = .…(3)
r
e. But, r = R + h
GM
 vc = ….(4) [½]
(R  h)
Equation (4) represents the expression for critical velocity.
iii.
T Tcos  T
 
r T cos   2r
Tsin  r T sin 
  h

Water

Rise of liquid in capillary tube


r = radius of capillary tube h = height of liquid level in the tube
T = surface tension of liquid  = density of liquid
g = acceleration due to gravity

(Diagram and labelling) (1+1) [2]


90
90
Physics
iv. a. A gas consists of very large number of extremely small particles known as
molecules. [½]
b. The intermolecular force of attraction between gas molecules are negligible. [½]
c. Molecules are always in the state of random motion, i.e., they are moving in all possible
directions with all possible velocities. This state is called molecular chaos. [½]
d. Between any two successive collisions, a molecule travels in a straight line with
constant velocity. It is called free path. [½]

v. Solution:
Given: L = 47 cm = 47  10–2 m, v = 3.3  102 m/s
To find: Fundamental frequency of air column (n)
v
Formula: n= [½]
2L
Calculation: From formula,
3.3  102
n= [½]
2  47  102
330
=  102
94
= antilog [log(330) – log(94)]  102
= antilog[2.5185 – 1.9731]  102
= antilog[0.5454]  102
= 3.512  102
 n = 351.2 Hz
Ans: The fundamental frequency of air column is 351.2 Hz. [1]

vi. Solution:
Given: R = 0.1 m,  = 6  103 kg/m3
To find: M.I of sphere about a tangent to its surface (It)
7
Formula: It = MR2 [½]
5
7
Calculations: It = MR2
5
4 3
But M = V = R 
3
7 4 3  2
 It =  R   R
5 3 
28 5
= R  [½]
15
28
=  3.14  (0.1)5  6  103
15
28  3.14  6
=  102
15
56  3.14
=  102
5
= antilog[log(56) + log(3.14) – log(5)]  10–2
= antilog[1.7482 + 0.4969 – 0.6990]  10–2
= antilog[1.5461]  10–2
= 35.17  10–2
 It = 0.3517 kg m2
Ans: Moment of inertia of solid sphere about a tangent to its surface is 0.3517 kg m2. [1]
91
Board Answer Paper : July 2016
vii. Solution:
1
Given: T = 10 s, P.E = (T.E) [½]
2
To find: Time (t)
1 1
Formulae: a. P.E = m2x2 b. T.E = m2A2
2 2
c. x = Asint
Calculation: From formula (a) and (b),
1 11 
m2 x 2 =  m2 A 2  ….(From given condition)
2 2 2 
A
 x=
2
Using formula (c), x = Asint [½]
A
Asint =
2
1
 sint =
2
 1 
 t = sin–1  
 2
 2  
  t = [½]
 
T 4
T 10
 t= =
8 8
 t = 1.25 s
Ans: The time in which the potential energy will be half of total energy is 1.25 s. [½]

viii. Solution:
Given: m = 2 kg, l = 1.5 m,  = 30, g = 9.8 m/s2
To find: Period (T)
l cos 
Formula: T = 2 [½]
g
Calculation: From formula,
1.5  cos30
T = 2  3.14 [½]
9.8
1.5  0.8660
= 6.28 
9.8
 1 
= antilog log  6.28    log1.5  log 0.8660  log 9.8  
 2 

= antilog 0.7980   0.1761  1.9375  0.9912  


 1 
 2 
 1 
= antilog 0.7980   0.1136  0.9912  
 2 
 1 
= antilog 0.7980   0.8776  
 2 
= antilog[0.3592]
 T = 2.2875 s
Ans: Period of revolution is 2.287 s. [1]

92
92
Physics
Q.3. Attempt any THREE:
i. Kepler’s first law (Law of orbit):
Every planet revolves around the sun in an elliptical orbit with the sun situated at one of the
focii of the ellipse. [1]
Kepler’s second law (Law of equal areas):
The radius vector drawn from the sun to any planet sweeps out equal areas in equal
intervals of time, i.e., areal velocity of the radius vector is constant. [1]
Kepler’s third law (Law of periods):
The square of the period of revolution of the planet round the sun is directly proportional to
the cube of the semi-major axis of the elliptical orbit. [1]
ii. Expression for torque acting on a rotating body:

m3 


F3 F2
r3 O r m2
2
mn 
rn r1 F1

Fn m1

a. Suppose a rigid body consists of n particles of masses m1, m2, m3, ......, mn which are
situated at distances r1, r2, r3, …, rn respectively, from the axis of rotation as shown in
figure.
b. Each particle revolves with angular acceleration .
c. Let F1, F2, F3, …., Fn be the tangential force acting on particles of masses,
m1, m2, m3, …, mn respectively. [1]
d. Linear acceleration of particles of masses m1, m2,…, mn are given by,
a1 = r1, a2 = r2, a3 = r3, …, an = rn
e. Magnitude of force acting on particle of mass m1 is given by,
F1 = m1a1 = m1r1 ….[ a = r] [½]
Magnitude of torque on particle of mass m1 is given by,
1 = F1 r1 sin 
But,  = 90 ….[ Radius vector is ar to tangential force]
 1 = F1 r1 sin 90
= F1r1
= m1a1 r1
 1 = m1r12  ….[ a1 = r1 ] [½]
Similarly,
2 = m 2 r22 
3 = m3 r32 ,
.…

.…

n = mn rn2 
f. Total torque acting on the body,
 = 1 + 2 + 3 + …. + n
  = m1r12  + m 2 r22 + m3 r32  + … + m n rn2  [½]
2 2 2 2
  = (m1r1 + m2r2 + m3r3 +.....+ m r ) n n

93
Board Answer Paper : July 2016
 n

  =  mi ri 2  
 i 1 
n
But, m r
i 1
i i
2
=I

  = I [½]
g. If  = 1 rad/s2 then  = I.
Thus, when a torque rotates the body with uniform angular acceleration of 1 rad/s2
then M.I of the body about a given axis of rotation becomes equal to torque acting
on it.
iii. Solution:
Given: A = 2 mm2 = 2  10–6 m2, F = 10 N
Ysteel = 2  1011 N/m2,  = 0.29
To find: Lateral strain
longitudinalstress lateralstrain
Formulae: a. Y= b. =
longitudinalstrain longitudinalstrain
F
Calculation: Longitudinal stress =
A
10
= = 5  106 N/m2 [½]
2  106
Using formula (a),
longitudinalstress
longitudinal strain = [½]
Y
5  106
= = 2.5  10–5 [½]
2  1011

Using formula (b),


lateral strain =   longitudinal strain [½]
= 0.29  2.5  10–5
 lateral strain = 7.25  10–6
Ans: Lateral strain produced in the wire is 7.25  10–6. [1]
iv. Solution:
62  54
Given: 1 = = 58 C in 10 minutes
2
54  48
2 = = 51 C in next 10 minutes
2
 d  62  54 8
  = = = 0.8 C/min
 dt 1 10 10
 d  54  48 6
  = = = 0.6 C/min
 dt  2 2 10
To find: Temperature of surroundings (0)
d
Formula: = K( – 0) [½]
dt
Calculation: From formula,
 d 
  = K(1 – 0)
 dt 1
 0.8 = K(58 – 0) ….(1) [½]
and
 d 
  = K(2 - 0)
 dt  2
 0.6 = K(51 – 0) ….(2) [½]
Dividing equation (1) by (2)
0.8 58  0
 =
0.6 51  0

94
94
Physics
8 58  0
 = [½]
6 51  0
 8(51 – 0) = 6(58 – 0)
408 – 80 = 348 – 60
 20 = 408 – 348
 20 = 60
 0 = 30 C
Ans: Temperature of surroundings is 30 C. [1]
Q.4. A. Formation of stationary waves by analytical method:
i. Consider two identical progressive waves of equal amplitude and frequency travelling
along X axis in opposite direction. They are given by,
2
y1 = A sin (vt  x) along positive X-axis .…(1)

2
y2 = A sin (vt + x) along negative X-axis ….(2) [½]

ii. The resultant displacement ‘y’ is given by the principle of superposition of waves,
y = y1 + y2 ….(3)
2 2
y = A sin (vt  x) + A sin (vt + x) [½]
 
iii. By using,
CD C D
sin C + sin D = 2sin   cos  2  ,
 2   
we get,
 2  vt  x  vt  x    2  vt  x  vt  x  
y = 2A sin    cos   
  2    2 
 2vt   2 
= 2Asin   cos  (  x) 
     
 2x   v
 y = 2Asin 2πnt cos    n   ….[cos ( ) = cosθ]
    
 2x 
 y = 2Acos   sin 2πnt [½]
  
 2x 
iv. Let R = 2Acos  
  
 y = Rsin (2πnt) ….(4) [½]
But,  = 2n
 y = R sin t ….(5)
Equation (5) represents the equation of S.H.M. Hence, the resultant wave is a S.H.M. of
amplitude R which varies with x.
v. The absence of x in equation (5) shows that the resultant wave is neither travelling forward
nor backward. Therefore it is called as stationary wave.
Amplitude at node is minimum, i.e., 0.
 Rmin = 0
 2x 
Since R = 2A cos  ,
  
 2x 
 cos   =0 [½]
  
2x  3 5
 = , , , …….
 2 2 2
 3 5
 x= , , ,…….
4 4 4
95
Board Answer Paper : July 2016
Distance between two consecutive nodes,
3  
x1  x0 =  = ,
4 4 2

x2  x1 = 5  3 = and so on.
4 4 2

Thus, distance between two successive nodes is . [½]
2
 2x 
Since R = 2A cos  ,
  
At antinodes: R =  2A
 2x 
 cos   =1 [½]
  
 2x 
   = 0, , 2, 3,…….n
  
 3
 x = 0, ,, ………
2 2
 Distance between two consecutive
  
antinodes = x1  x0 = , x2  x1 =  = and so on.
2 2 2
Thus, distance between two successive antinodes is /2.
Hence, nodes and antinodes are equispaced. The distance between a node and an adjacent
λ
antinode is . [½]
4

B. Solution:
Given: Speed limit, vL = 120 km/hr,
10
nA = nA – nA = 0.9 nA
100
Velocity of sound, v = 340 m/s [½]
To find: Velocity of source (vS)
 v 
Formulae: nA =  n .…(1) [½]
 v  vS 
 v 
nA =  n ….(2) [½]
 v  vS 
Calculation: Dividing formula (1) by (2),
nA v  vS
 = [½]
nA v  vS
1 340  vS
 = [½]
0.9 340  vS
 340 – vS = 0.9(340 + vS)
 340 – vS = 306 + 0.9 vS
 1.9 vS = 34
34 340
 vS = = = 17.89 m/s
1.9 19
 vS = 64.40 km/hr [½]
 vS < vL
Ans: The policeman is not justified in punishing the car driver.

96
96
Physics
OR
A. Practical simple pendulum:
Practical simple pendulum is defined as a small heavy sphere (bob), suspended by a light
and inextensible string from a rigid support. [½]
To show motion of the bob of simple pendulum is S.H.M:

l
T T
mg sin 
B
A x 
mg cos 
mg mg

i. Consider a simple pendulum of mass ‘m’ and length ‘L’.


L = l + r,
where, l = length of string
r = radius of bob
ii. Let OA be the initial position of pendulum and OB, its instantaneous position when the
string makes an angle  with the vertical.
In displaced position, two forces are acting on the bob:
a. Gravitational force (weight) ‘mg’ in downward direction.
b. Tension T in the string.
iii. Weight ‘mg’ can be resolved into two rectangular components:
a. Radial component mg cos  along OB and
b. Tangential component mg sin  perpendicular to OB and directed towards mean
position. [1]
iv. mg cos  is balanced by tension T in the string, while mg sin  provides restoring force.
 F =  mg sin  [½]
where, negative sign shows that force and angular displacement are oppositely directed.
Hence, restoring force is proportional to sin  instead of . So, the resulting motion is not
S.H.M.
v. If  is very small then,
x
sin    =
L
x
 F =  mg [½]
L
F x
 =g
m L
ma x
 =g
m L
g
 a= x .…(1)
L
 g 
 ax .…    constant  [½]
 L 
Hence, motion of the bob of simple pendulum is simple harmonic.
Expression for time period:
In S.H.M,
a =  2 x ….(2)
97
Board Answer Paper : July 2016
Comparing equations (1) and (2),
g
2 =
L
2
But,  = [½]
T
2
 2  g
   =
 T  L
2 g
 =
T L
L
 T = 2 ….(3) [½]
g
Equation (3) represents time period of simple pendulum.
Thus period of simple pendulum depends on the length of the pendulum and acceleration
due to gravity. [1]
B. Solution:
Given: E= 2T
To find: Diameter of drop (d)
Formula: E = TA
Calculation: A = 4r2
From formula,
E= 4r2T
  2 T = 4r2T [½]
 4r2 = 2
2 1.414
 r2 = =
4 4
r2 = 0.3535
 r = 0.3535
r = 0.5946 m [1]
 d = 2r
= 2(0.5946)
 d = 1.1892 m
Ans: Diameter of the drop is 1.1892 m. [½]

SECTION – II

Q.5. Select and write the most appropriate answer from the given alternatives for each
sub-question:
i. (B) [1]
ii. (D) [1]
iii. (C) [1]
iv. (B) [1]
v. (C) [1]
vi. (A) [1]
vii. (B)
e = vlb
5
= 54   1  3  104
18
= 45  10–4 = 4.5 mV [1]
98
98
Physics
Q.6. Attempt any SIX:
i.

No. Intrinsic semiconductors Extrinsic semiconductors


The semiconductor, resulting from
Semiconductor in the pure form is
i. mixing of impurity in it, is known as
known as intrinsic semiconductor.
extrinsic semiconductor.
ii. Their conductivity is low. Their conductivity is high.
Its electrical conductivity depends
Its electrical conductivity is a function upon the temperature as well as on the
iii.
of temperature alone. quantity of impurity atoms doped in
the structure.
The number of free electrons in In these semiconductors, number of
iv. conduction band is equal to the free electrons and number of holes are
number of holes in valence band. unequal.
v. These are not practically used. These are practically used.
In these, the Fermi energy level lies in In these, the Fermi energy level shifts
vi. the middle of valence band and towards valence or conduction energy
conduction band. band.

(Any two points) [2]


ii.
Receiving
Antenna

Output
Amplifier IF Stage Detector Amplifier
Received
signal
Block diagram of a receiver

(Diagram and labelling) (1+1) [2]

iii. Solution:
Given: x1 = 6.5 cm = 6.5  102 m,
x2 = 6.65 cm = 6.65  102 m,
 = 5000 Å = 5  107 m
To find: Nature of illumination at the point
Formula: x=n
Calculation: Path difference is given by,
∆x = x2  x1 = 6.65  102  6.5  102 = 0.15  102 m
From formula,
x
n= [½]
λ
0.15×10 –2
n= [½]
5×10 –7
= 3000 [½]
λ
 ∆x = 3000 = 6000 
2
λ
As the path difference is even multiple of , the point is bright.
2
Ans: The point is bright. [½]

99
Board Answer Paper : July 2016
iv. Magnetic dipole moments in paramagnetic substance:

B

S N

(a): Absence of external field (b): Strong external magnetic field


[2]
v. Solution:
Given: R = 100 , r = 20 , E = 2 V
To find: Reading of voltmeter (V)
Formula: V = E  Ir [½]
Calculation: Current through the circuit is given by
E 2 2
I= = = [½]
R  r 100 + 20 120
1
 I= A
60
From formula,
 1 
V = 2    20  = 2  0.3333
 60 
 V = 1.667 V
Ans: The reading on the voltmeter is 1.667 V. [1]
vi. Solution:
5 5
Given: 1 = 1.2  10 , T1 = 300 K,  2 = 1.8  10
To find: Required temperature (T2)
Formula: χT = constant
Calculation: From formula,
1T1 =  2T2 [½]
1 T1 5
1.2  10  300
 T2 = = [½]
2 1.8 105
 T2 = 200 K
Ans: The required temperature is 200 K. [1]
vii. Solution:
Given: V = 25000 volt
To find: De-broglie wavelength ()
12.27
Formula: = Å [½]
V
Calculation: Using formula,
12.27
=  10–10 [½]
25000
12.27  1010 12.27
= = 1
 1012
2.5  104  2.5 2

  1 
= antilog log 12.27   log  2.5     1012
  2  

  1 
= antilog 1.0888   0.3979     1012
  2 
= antilog  0.8899   1012 = 7.761  10–12 m = 0.07761  10–10 m
Ans: The de-broglie wavelength of electron is 0.0776  10–10 m. [1]

100
Physics
viii.
A
Input Output A Output
B Inputs
B
AND gate
OR gate

Input Output Output


A
Inputs
A
B
NOT gate NAND gate

(½ mark for each symbol) [2]


Q.7. Attempt any THREE:
i. a. Let S1 and S2 be the two coherent monochromatic sources which are separated by
short distance d. They emit light waves of wavelength .
b. Let D = horizontal distance between screen and source.
c. Draw S1M and S2N  AB
OP = perpendicular bisector of slit.
Since S1P = S2P, the path difference between waves reaching P from S1 and
S2 is zero, therefore there is a bright point at P.
d. Consider a point Q on the screen which is at a distance x from the central point P on
the screen. Light waves from S1 and S2 reach at Q simultaneously by covering path
S1Q and S2Q, where they superimpose. [½]

A
Q

x
S1 M
d/2
d O P
x d/2
K
S2 N
D

B
[½]
e. In  S1MQ,
(S1Q)2 = (S1M)2 + (MQ)2
2
 d
(S1Q)2 = D2 +  x   ….(1) [½]
 2
f. In  S2NQ,
(S2Q)2 = (S2N)2 + (NQ)2
2
 d
 (S2Q)2 = D2 +  x   ….(2) [½]
 2
g. Subtract equation (1) from (2),
 d  
2
d 
2

(S2Q)2  (S1Q)2 =  D2   x      D2   x   
  2    2 
   
2 2
 d  d
= D2   x    D2   x  
 2  2  
2 2
 d  d
= x   x  
 2  2

101
Board Answer Paper : July 2016
 d2   d2 
=  x2   xd    x 2   xd 
 4   4 
d2 d2
= x2   xd  x 2   xd
4 4
(S2Q)2  (S1Q)2 = 2xd [½]
 (S2Q + S1Q) (S2Q  S1Q) = 2xd
2xd
 S2Q  S1Q = ….(3)
S2Q  S1Q
h. If x << D and d << D then,
S1Q  S2Q  D
S2Q + S1Q = 2D
 Equation (3) becomes,
2xd
S2Q  S1Q =
2D
xd
 S2Q  S1Q =
D
xd
 x = ….(4) [½]
D
Equation (4) gives the path difference of two interfering light waves.
ii. Law of radioactive decay:
The number of nuclei undergoing the decay per unit time is proportional to the number of
unchanged nuclei present at that instant. [1]
If ‘N’ is the number of nuclei present at any instant ‘t’, ‘dN’ is the number of nuclei that
disintegrated in short interval of time ‘dt’, then according to decay law,
dN
 N
dt
dN
 =  N [½]
dt
where,  is known as decay constant or disintegration constant. The negative sign indicates
disintegration of atoms.
Derivation of relation N = N0 et :
a. Let, N = number of nuclei present at any instant t.
dN = number of nuclei disintegrated in short interval dt.
b. According to decay law,
dN
=  N ….(1)
dt
c. Integrating both sides of equation (1),
dN
 N =   dt [½]
 loge N = t + c .…(2)
where, c is constant of integration whose value depends on initial conditions.
d. At t = 0, N = N0
 loge N0 = 0 + c ….[From equation (1)]
e. Substituting the value of c in equation (1), we get,
loge N = t + loge N0
 loge N  loge N0 = t
 N
 loge   = t
 N0 
N
 = et
N0
 N = N0 e t [½]

102
Physics
Decay curve:

N0

No. of unchanged
nuclei N

(NdN)

O t t + dt time
[½]
iii. Solution:
Given: ag= 1.5,  = 5  1014 Hz
To find: a. Change in wavelength ()
b. Wave number of light in glass  νg 
λa 1 c
Formulae: a. ag = b. ν = c. =
λg λ ν
Calculation: Using formula (c),
c 3  108
a = = = 0.6  10–6 m
ν 5  1014
 a = 6000 Å [½]
From formula (a),
g = λ a [½]
a μg
6000
= = 4000 Å [½]
1.5
 Change in wavelength  = a  g
= 6000 – 4000
= 2000 Å [½]
1
Wave number of light in glass = νg = [½]
λg
1
=
4000Å
= 2.5  106 m–1 [½]
Ans: a. Change in wavelength is 2000 Å
b. Wave number of light in glass is 2.5  106 m–1
iv. Solution:
Given:  = 3000 Å = 3  10–7 m, me = 9.1  10–31 kg,
o = 2.3 eV, h = 6.63  10–34 Js, c = 3  108 m/s
To find: Maximum velocity (vmax)
hc
Formula: (K.E)max =  o [½]
λ
Calculation: From formula,
 6.63  1034  3  108 
(K.E.)max =  7
  2.3  1.6  1019   J [½]
 3  10 
= (6.63  10–19)  (3.68  10–19 ) J
= 2.95  10–19 J [½]
Also,
1
m e v 2max = (K.E.)max
2

103
Board Answer Paper : July 2016
2(K.E.) max
 vmax = [½]
me
2  2.95  1019
= [½]
9.1  1031
5.90  1012 590
= =  105
9.1 9.1
 1 
= antilog  log  590   log  9.1    105
 2  
 1 
= antilog   2.7709  0.9590     105
 2 
 1 
= antilog  1.8119     105
 2 
= antilog  0.9059   10 5
= 8.052  105 m/s
Ans: The maximum velocity of electron is 8052  105 m/s [½]
Q.8. A. Electromagnetic induction:
The phenomenon of producing an induced e.m.f in a conductor or conducting coil due to
changing magnetic flux is called electromagnetic induction. [1]
i. Consider a rectangular loop of conducting wire ‘PQRS’ partly placed in uniform magnetic
field of induction ‘B’ as shown in figure.

     
 B  PP 
F1    I Q
      
F       v
l
      
      
x
 S      R
dx
   F2   
[½]
ii. Let ‘l’ be the length of the side PS and ‘x’ be the length of the loop within the field.
 A = lx = area of the loop, which lies inside the field. [1]
iii. The magnetic flux () through the area A at certain time ‘t’ is  = BA = Blx [½]
iv. The loop is pulled out of the magnetic field of induction ‘B’ to the right with a uniform
velocity ‘v’.
d d
v. The rate of change of magnetic flux is given by, = (Blx)
dt dt
d
= Bl  
dx

dt  dt 

But,   = v
dx
 dt 
d
 = Blv .…(1) [½]
dt
vi. Due to change in magnetic flux, induced current is set up in the coil. The direction of this
current is clockwise according to Lenz’s law. Due to this, the sides of the coil experiences
the forces, F1, F2 and F as shown in figure. The directions of these forces is given by
Flemings left hand rule.
vii. The magnitude of force ‘F’ acting on the side PS is given by, F = BIl.
 
viii. The force F1 and F2 are equal in magnitude and opposite in direction, therefore they cancel

out. The only unbalanced force which opposes the motion of the coil is F . Hence, work
must be done against this force in order to pull the coil.
104
Physics
ix. The work done in time ‘dt’ during the small displacement ‘dx’ is given by,
dW = Fdx
 ve sign shows that F and ‘dx’ are opposite to each other.
 dW =  (BIl) dx ….(2) [½]
x. This external work provides the energy needed to maintain the induced current I through
the loop (coil).
dW
xi. If ‘e’ is the e.m.f induced then, electric power = = eI
dt
 dW = eIdt .…(3) [½]
xii. From equations (2) and (3),
eIdt =  BIl dx
e =  Bl 
dx 
 
 dt 
 e =  Blv .…(4) [½]
xiii. From equation (1) and (4),
d
e= 
dt

B. Solution:
Given: L = 2 m, R = 10 , RE = 990 , E = 2 V
To find: Potential gradient of wire (K)
V
Formula: K= [½]
L
E
Calculation: Since, I = [½]
R  RE
ER
Also, V = IR =
R  RE
2  10 20
 V= = = 2  102 volt
10  990 1000
From formula,
2  102
K= = 102 V/m = 0.01 V/m
2
Ans: The potential gradient of wire is 0.01 V/m. [1]
OR
A. Construction:
Steel
chamber
Metallic S
comb

P2 Metallic
C2 sphere
D
I
Insulating
M1 M2 belt
C1
HT
P1 Motor
driven pulley

Target [1]

105
Board Answer Paper : July 2016
i. It consists of a large hollow metallic sphere S mounted on two insulating columns M1 and
M2.
ii. There are two pulleys P1 and P2. P2 is mounted at the centre of sphere S while P1 is
mounted near the bottom. A long narrow belt made of insulating material passes over the
pulleys. The belt is driven by an electric motor, connected to the lower pulley P1.
iii. C1 and C2 are two pointed metal combs, having a number of metallic needles, pointing
towards the belt. C1 is called the spray comb and C2 is called the collecting comb. C1 is kept
at a positive potential of about 10,000 V by using a high tension source (HT) while C2 is
connected to the sphere S.
iv. D is an evacuated accelerating tube having an electrode I at its upper end. This electrode is
connected to the sphere S. To prevent the leakage of charge from the spray, the generator is
enclosed in a steel chamber filled with nitrogen or methane at high pressure. [1½]

Working:
i. When the spray comb is maintained at high positive potential, it produces charges in its
vicinity. The positive charges get sprayed on the belt due to the repulsive action of comb
C1, which are carried upward by the moving belt. A comb C2, called collecting comb is
positioned near the upper end of the belt, such that the pointed ends touch the belt and the
other end is in contact with the inner surface of the metallic sphere S.
ii. The comb C2 collects the positive charge and transfers them to the metallic sphere. The
charge transferred by the comb C2 immediately moves on to the outer surface of the hollow
sphere.
iii. As the belt goes on moving, the accumulation of positive charge on the sphere also keeps
on taking place continuously and its potential rises considerably.
iv. With the increase of charge on the sphere, its leakage due to ionisation of surrounding air
also becomes faster. When the rate of loss of charge due to leakage becomes equal to the
rate at which the charge is transferred to the sphere, the potential of sphere becomes
maximum.
v. The projectiles such as protons, deuterons etc. are introduced in the upper part of the
evacuated accelerator tube. They get accelerated in the downward direction along the length
of the tube and acquire very high energy. At the other end, they come out and hit the target
with large kinetic energy and bring about nuclear disintegration. [1½]

B. Solution:
Given: G = 25 , Ig = 10 mA = 10  10–3 A, V = 100 V
To find: Resistance (Rs)
V
Formula: Rs = G [½]
Ig
Calculation: From formula,
100
Rs =  25 [½]
10  103
= 104 – 25 [½]
= 10000 – 25 [1]
= 9975 
Ans: A resistance of 9975  should be connected in series. [½]

106
Chemistry

BOARD ANSWER PAPER : JULY 2016


CHEMISTRY

Note: Answer to every question must be written on a new page.

SECTION – I

Q.1. Select and write the most appropriate answer from the given alternatives for each
sub-question:
i. (B) Cesium chloride [1]
ii. (B) Second law of thermodynamics [1]
iii. (D) 2NH3(g)  Pt
 N2(g) + 3H2(g) [1]
Rate of reaction is independent of the total concentration of NH3.
iv. (A) 0.242 V [1]
v. (D) Copper ferrocyanide [1]
vi. (B) Siderite [1]
Siderite is FeCO3.
vii. (C) Arsenic (Z = 33) [1]
[Ar]3d104s24p3
Q.2. Answer any SIX of the following:
i. Van Arkel method for refining titanium: The volatile iodide of titanium (formed after
treatment of impure titanium with iodine) is electrically heated using tungsten filament
above 1700 K. Pure titanium gets deposited on the filament and as the deposition goes on,
the current is steadily raised to maintain the temperature.
Ti(s) + 2I2(g)  523K
 TiI4(g)
Impure Volatile
iodide of
titanium [1]

TiI4(g) 
1700 K
 Ti(s) + 2I2(g)
Volatile Pure
iodide of
titanium [1]

ii. 
For the reaction, aA + bB 
 cC + dD [½]
before approaching equilibrium, the reaction quotient is given by,
C  D or Q =
c d
p cC  p dD
Qc = p [½]
 A   B p aA  p Bb
a b

Here, the values of concentrations or partial pressures are other than equilibrium values.
At equilibrium, Q = K called equilibrium constant and G = 0, because the reaction
mixture has no tendency to change in either direction. [½]
Therefore, equation G = G + RT ln Q becomes:
0 = G + RT ln K
 G =  RT ln K
 G =  2.303 RT log10K [½]
The above equation gives the relationship between standard Gibbs energy change of the
reaction and its equilibrium constant.

107
Board Answer Paper : July 2016
iii. Face-centred cubic lattice (fcc):
a. In face-centred cubic unit cell, eight constituent particles (spheres) are present at
eight corners of unit cell. Six constituent particles (spheres) are present at centres of
six faces.
1
atom
2
1
atom (Diagram) [1]
8

Actual portion of atoms


belonging to one fcc unit cell
b. A constituent particle present at a corner is shared by eight neighbouring unit cells.
Its contribution to a unit cell is only 1/8.
Thus, the number of atoms present at corners per unit cell
1
= 8 corner atoms  atom per unit cell = 1 [½]
8
c. A constituent particle present at the centre of a face is shared by two neighbouring
unit cells. Its contribution to a unit cell is only 1/2.
The number of atoms present at faces per unit cell
1
= 6 atoms at the faces  atom per unit cell = 3
2
d. The total number of atoms per unit cell = 1 + 3 = 4
Thus, a face-centred cubic unit cell has 4 atoms per unit cell. [½]
iv. Isolation method:
a. In the isolation method, the concentrations of all the reactants except one are made to
remain constant throughout the course of the reaction by taking them in large excess.
The dependence of rate on the concentration of the isolated species is experimentally
determined.
b. The experiment is repeated by isolating one of the reactants each time and
determining the rate law with respect to the isolated species.
c. Consider the reaction,
aA + bB  cC + dD [½]
The rate law has the form,
Rate = k[A]x[B]y [½]
d. In one of the experiment, A is isolated by making the initial concentration of B (i.e.,
[B]0) to remain constant throughout the course of the reaction (by taking it in large
excess).
Hence, the rate law is written as
rate = k[A]x  B0 = k[A]x (where, k = k  B0 )
y y

The dependence of rate on [A] that is ‘x’ is found out. [½]


e. In another experiment, B is isolated by keeping initial concentration of A (i.e., [A]0)
constant throughout the course of the reaction (i.e., by taking it in large excess).
The rate law then becomes, Rate = k [A]0x [B]y = k [B]y (where, k = k [A]0x ).
The dependence of rate on B that is y is found out.
f. Thus, by knowing the values of x and y, the order of the reaction (x + y) and the rate
law can be determined. [½]
v. a. Galvanization:
1. The rusting of iron can be prevented by coating the metal surface with another
metal such as zinc, that is oxidized in preference to iron. This process is called
galvanization.

108
Chemistry
2. If the zinc layer is scratched, even then it protects iron from corrosion as long
as Fe and Zn are in contact. If Fe is oxidized to Fe2+, then Zn immediately
reduces Fe2+ back to Fe. [1]
b. Passivation: The surface of metal is treated with strong oxidizing acid such as
concentrated nitric acid and a thin preventive layer of oxide is formed on the surface
of metal. Thus, the surface is made inactive. This is process is called as passivation. [1]
vi. Nernst equation can be given as,
2.303RT [Products]
E = E0  log10 [1]
nF [Reactants]
where,
E0 = Standard potential of electrode or cell,
n = Number of moles of electrons used in reaction,
F = Faraday = 96500 C/mol e, [Products] = Concentration of products,
[Reactants] = Concentration of reactants, T = temperature in K and
R = gas constant = 8.314 J K1 mol1 [1]
vii. a. Dilute H2SO4 reacts with Fe which is below hydrogen in the electrochemical series,
with the evolution of H2.
Fe + H2SO4  FeSO4 + H2(g)
Iron Sulphuric Ferrous Hydrogen [1]
acid sulphate

b. When H2SO4 is added to calcium fluoride, hydrogen fluoride is obtained.


CaF2 + H2SO4  CaSO4 + 2HF [1]
Calcium Sulphuric Calcium Hydrogen
fluoride acid sulphate fluoride

viii. a. Osmosis: The spontaneous and unidirectional flow of solvent molecules through a
semipermeable membrane, into the solution or flow of solvent from a solution of
lower concentration to the solution of higher concentration through a semipermeable
membrane is called osmosis. [1]
b. Freezing point: The freezing point of a substance may be defined as the temperature
at which the vapour pressure of solid is equal to the vapour pressure of liquid. [1]
Q.3. Answer any THREE of the following:
i. Given: Rate constant k1 = 0.58 s1; Rate constant k2 = 0.045 s1
T1 = 313 K; T2 = 293 K
R = 8.314 J K1mol1
To find: Activation energy (Ea)
k2 Ea  T2  T1 
Formula: log10 =   [½]
k1 2.303R  T1T2 
Calculation: From formula,
 0.045 s 1  Ea  293K  313 K 
log10  1 
= 1 1   [½]
 0.58 s  2.303 8.314 J K mol  293 K×313 K 
Ea  20 
 log 0.0776 =   [½]
19.147 J mol1  293  313 
E a  20 
 1.110 =  
19.147  293  313 
1.110  19.147  293  313
 Ea = [½]
20
= 97455.34 J mol1
= 97.45 kJ mol1 [1]

109
Board Answer Paper : July 2016
ii. Given: Given equations are,
7
C2H6(g) + O2(g)  2CO2(g) + 3H2O(l), ΔH = 1560 kJ …. (1)
2
1
H2(g) + O2(g)  H2O(l), ΔH = 285.8 kJ …. (2)
2
C(graphite) + O2(g)  CO2(g), ΔH = 393.5 kJ …. (3)
To find: Standard enthalpy of the given reaction
Calculation: Reversing equation (1),
7
2CO2(g) + 3H2O(l)  C2H6(g) + O2(g), ΔH = 1560 kJ…. (4) [½]
2
Multiplying equation (2) by 3 and (3) by 2, then adding to equation (4) [½]
7
2CO2(g) + 3H2O(l)  C2H6(g) + O2(g), ΔH = +1560 kJ
2
3
3H2(g) + O2(g)  3H2O(l), ΔH = 857.4 kJ
2
2C(graphite) + 2O2(g)  2CO2(g), ΔH = 787 kJ
2C(graphite) + 3H2(g)  C2H6(g) , [1]
ΔH = 1560 + (857.4) + (787.0) = 84.4 kJ [1]
iii. Given: Mass of sulphur = W2 = 3.795 g = 3.795  103 kg
Mass of solvent = W1 = 100 g = 100  103 kg
Boiling point of solution = Tb = 319.81 K
Boiling point of pure solvent = Tb = 319.45 K
Molal elevation constant = Kb = 2.42 K kg mol1
Atomic mass of sulphur = 32
To find: Molecular formula of sulphur in solution
K b ×W2
Formula: Molar mass of solute = M2 =
Tb  W1
Calculation: For solution of sulphur in CS2 (solvent),
Tb = Tb – Tb = 319.81 – 319.45 = 0.36 K
From formula,
K b ×W2
M2 = [½]
Tb ×W1
2.42  3.795  103
= [½]
0.36  100  103
= 0.2551 kg mol1
= 255.1 g mol1 [1]
Molar massof sulphur
n= [½]
Atomic mass of sulphur
255.1
 n=
32
= 7.97  8
 Molecular formula of sulphur in CS2 = S8 [½]
iv. Ostwald’s process (Large scale preparation): Ostwald’s process is used to prepare nitric
acid on a large scale.
a. This method is based upon catalytic oxidation of NH3 by atmospheric oxygen.
Pt /Rh gaugecatalyst
4NH3(g) + 5O2(g) 
500K,9bar
 4NO(g) + 6H2O(g)
Ammonia (From air) Nitric oxide
(Reaction + names of reactants, products and reagents + Reaction conditions) [1]
110
Chemistry
b. Nitric oxide reacts with oxygen to form nitrogen dioxide.
2NO(g) + O2(g)   2NO2(g)
Nitric oxide Oxygen Nitrogen dioxide
(Reaction + names of reactants and products) [1]
c. Nitrogen dioxide on dissolution in water gives nitric acid.
3NO2(g) + H2O(l)  2HNO3(l) + NO(g)
Nitrogen dioxide Water Nitric acid

The nitric oxide (NO) thus formed is recycled and the aqueous HNO3 is concentrated
by distillation to give 68% HNO3 by mass. Further, concentration to 98 % can be
achieved by dehydration with concentrated H2SO4.
(Reaction + names of reactants and products) [1]
Q.4. Answer any ONE of the following:
i. a. Third law of thermodynamics: The third law of thermodynamics states that, “The
entropy of a perfectly ordered crystalline substance is zero at absolute zero of
temperature”. [1]
Applications of standard molar entropy:
1. S for a chemical reaction can be calculated by knowing the values of standard
molar entropy of all reactants and products in the reaction. It is obtained by
subtracting total standard molar entropy of all the reactants from those of all the
products. The entropy change for the reaction is thus given by,
ΔS = m S (products)  n S (reactants)
where, m and n are the coefficients of products and reactants respectively in the
balanced equation.
eg. aA + bB  cC + dD
ΔS is given by,
ΔS = ( cSC + dSD )  ( aSA + bSB )
2. The standard molar entropy is useful in comparing the entropies of different
substances under the same conditions of temperature and pressure.
The S values increase with increasing complexity of molecules.
eg. CH3COOH is more complex than CH3OH. Hence S of CH3COOH
(160 J K1 mol1) is larger than that of CH3OH (127 J K1 mol1)
The S values of the heavier substance are larger than those of the lighter substance.
eg. For I(g), S = 180.7 J K1 mol1 whereas for F(g), S = 158.6 J K1 mol1.
(Two applications)[½  2] [1]
b. Battery
+ 

+ 
Impure copper
Pure copper
(blister copper)
2+ as cathode
as anode Cu
Cu2+
Cu2+
Cu2+ 15% CuSO4 + 5% H2SO4
Anode mud solution (electrolyte)
Electrolytic refining of blister copper
(Diagram-1 mark, Labelling-1 mark) [2]
c. Given: Edge length (a) = 412.1 pm = 4.12  108 cm
Molar mass = 133 + 35.5 = 168.5 g mol1
To find: Density (d)
111
Board Answer Paper : July 2016
Molar mass
Formulae: 1. Mass of one molecule =
Avogadro number
2. Volume of unit cell = a3
Mass of unit cell
3. Density =
Volume of unit cell
Calculation: In the bcc type unit cell of CsCl, there is one Cs+ ion at the body centre
position and 8 Cl ions are at the 8 corners.
 Number of Cs+ in unit cell = 1
1
Number of Cl in unit cell =  8 = 1
8
Hence, the unit cell contains one CsCl molecule.
Molar mass 168.5 g mol1
Mass of one CsCl molecule = =
Avogadro number 6.0231023 mol1
= 2.798  1022 g
 Mass of unit cell = 1  2.798  1022 g = 2.798  1022 g [½]
Volume of unit cell = a3 = (4.12  10–8 cm)3 = 6.993  1023 cm3 [½]
Mass of unit cell 2.798  1022 g
 Density = = 23
= 4.0 g cm–3 [1]
Volume of unit cell 6.993  10 cm 3

d. The radius ratio (r+/r–) defines the coordination number of the cation.
rCs+ = 1.69 Å, rCl = 1.81 Å
rCs 1.69
 = = 0.9337 [½]
rCl 1.81
Since, radius ratio is greater than 0.732, the coordination number of cation (Cs+) is 8. [½]

ii. a. When thin copper leaves are thrown in a jar of chlorine they catch fire and form
cupric chloride. [1]
Cu + Cl2  CuCl2
Copper Chlorine Cupric chloride [1]

b. 1. Due to greater electronegativity of O than S, H2O undergoes extensive


intermolecular H-bonding. As a result, H2O exists as an associated molecule in
which each O is tetrahedrally surrounded by four water molecules. Large
amount of energy is required to break these H-bonds. Therefore, H2O is a
liquid at room temperature. [1]
2. In contrast, H2S does not undergo H-bonding. It exists as discrete molecules
which are held together by weak van der Waals forces of attraction. To break
these forces of attraction, only a small amount of energy is required.
Therefore, H2S is a gas at room temperature. [1]

c. Given: C = 0.02 M , k = 2.428  103  1 cm1


To find: Molar conductivity
1000 k
Formula: Molar conductivity = [½]
C
1000 k 1000  2.428  103
Calculation: Molar conductivity = = [½]
C 0.02
= 121.4 1 cm2 mol1 [1]
d. Henry’s law relates solubility of a gas with external pressure. The law states that,
“the solubility of a gas in liquid at constant temperature is proportional to the
pressure of the gas above the solution”. [1]
112
Chemistry

SECTION – II

Q.5. Select and write the most appropriate answer from the given alternatives for each
sub-question:
i. (C) acetic anhydride [1]
CH3COONa + CH3COCl  (CH3–CO)2O + NaCl
Sodium acetate Ethanoyl Acetic anhydride
chloride

ii. (B) diethyl ether and ethanol [1]


Natalite is used as fuel.
iii. (D) 36 [1]
In ferrocyanide ion, [Fe(CN)6]4–
2e 
Fe   Fe2+ + 6CN–  [Fe(CN)6]4–
Formula : for EAN = Z – X + Y
Here, Z = atomic number of iron = 26.
X = number of electrons lost due to oxidation of Fe to Fe2+ = 2.
Y = number of electrons donated by 6CN– = 6  2 = 12.
 EAN = 26 – 2 + 12 = 36
iv. (B) disaccharide [1]
v. (C) +5 [1]
vi. (A) 3-Bromo-3,4-dimethylheptane [1]
vii. (D) (CH3)3C – NH2 [1]
Tertiary alkyl halides on treatment with alcoholic base undergo elimination in preference to
substitution.
Q.6. Answer any SIX of the following:
i. a. 3-Methylcyclohexanone [1]
b. Butane-2,3-dione [1]
ii. a. Citrus fruits, onion, cabbage, amla, green leafy vegetables, tomatoes, etc.
(Any two sources) [½  2] [1]
b. Green leafy vegetables, fish, meat, etc.
(Any two sources) [½  2] [1]
iii.
Lanthanoids Actinoids
a. In lanthanoids, last differentiating In actinoids, last differentiating electron
electron occupies 4f orbital. occupies 5f orbital.
b. They are the elements of first inner transition They are the elements of second inner
series. transition series.
c. They are present in period 6. They are present in period 7.
d. Most of lanthanoids (with exception of Most of actinoids (with exception of
promethium) occur in nature. uranium and thorium) are prepared in
laboratory.
e. Most of lanthanoids (with exception of All the actinoids are radioactive.
promethium) are non-radioactive.
f. Lanthanoids do not form oxo cation. Actinoids form oxo cation such as
UO 22  , PuO2+, UO+.
g. 4f orbitals in lanthanoids have higher binding 5f orbitals in actinoids have lower
energy. binding energy.

113
Board Answer Paper : July 2016
h. Contraction in atomic and ionic radii isContraction in atomic and ionic radii is
relatively less in lanthanoids. relatively greater in actinoids due to poor
shielding of 5f electrons.
i. Lanthanoids show +2, +3 and +4 Actinoids show +3, +4, +5, +6, +7
oxidation states. oxidation states.
j. Lanthanoids have less tendency to form Actinoids have greater tendency to
complexes. form complexes.
k. Some ions of lanthanoids are coloured. Most of the ions of actinoids are deeply
coloured.
l. Lanthanoid hydroxides are less basic in Actinoid hydroxides are more basic in
nature. nature.
(Any four distinguishing points) [½  4] [2]

iv. C6H5  C  N + C6H5 Mg Br  


dry ether
 C6H5  C = NMgBr [1]
Benzonitrile Phenyl
magnesium
C6H5
Adduct
bromide

H3O+
C6H5
C = O + Mg(OH)Br + NH3
C6H5
Benzophenone
(Reaction + names of reactants, products and reagents) [1]
v. a. Raw materials used for the preparation of Buna-S:
CH2 = CH – CH = CH2 [½]
CH = CH2

[½]

b. Raw materials used for the preparation of Dextron:


OH
CH3 – CH – COOH [½]
HO – CH2 – COOH [½]
vi. Sandmeyer’s reaction:
When a primary aromatic amine, dissolved or suspended in cold aqueous mineral acid, is
treated with sodium nitrite, a diazonium salt is formed. Mixing the solution of freshly
prepared diazonium salt with cuprous chloride or cuprous bromide results in the
replacement of the diazonium group by –Cl or –Br. This reaction is known as Sandmeyer’s
reaction.
(Description of reaction) [1]
+ 
NH2 N2X
NaNO 2  HX

273K

Aniline Benzenediazonium halide [½]
+ 
N2X X

Cu 2 X 2 + N2
Aryl halide [½]
Benzenediazonium halide
[X = Cl, Br]

114
Chemistry
vii. a. The reaction of benzenediazonium chloride with phenol in mild alkaline medium
yields p-hydroxyazobenzene (orange dye).

 
OH  
OH
N  NCl + N=N OH + HCl
Benzenediazonium chloride Phenol p-Hydroxyazobenzene
(orange dye)

(Reaction + names of reactants, products and reagents) [1]

b. The reaction of benzenediazonium chloride with aniline in mild alkaline medium


yields p-aminoazobenzene (yellow dye).

 
N  NCl NH2  
OH
+ N=N NH2 + HCl
Benzenediazonium chloride p-Aminoazobenzene
Aniline
(yellow dye)

(Reaction + names of reactants, products and reagents) [1]


viii. Chemical methods used to preserve food:
a. Addition of sugar: In this method, sugar is added and then food is heated. This
method is used for the preparation of jams, jellies and marmalades. Fruits like apples,
mangoes, strawberries, carrots, etc are preserved by this method.
b. Addition of salt: Common salt has antimicrobial activity. Salt is added to food to
control the growth of fungus and micro-organisms. When salt is added, bacterial
cell loses water due to osmosis. Due to this, the cell either remains dormant or
dies. This helps in storage of food. This method of preservation of food by
addition of salt is called salting. It is used for preservation of fish products, meat
and for the preparation of pickles of lemon, chillies, raw mangoes, etc. It is also
used in the preservation of amla, beans, tamarind, etc.
c. Addition of vinegar: Vinegar is added to preserve food like pickles, salad dressings,
mustard, fish, etc.
d. Addition of other chemicals: Chemicals like sodium benzoate, salts of sorbic acid
and propionic acid etc., are used as preservatives.
(Any two methods) [1  2] [2]
Q.7. Answer any THREE of the following:
i. a. Action of bromine water on glucose: The oxidation of glucose with bromine water
(which is a mild oxidizing agent) forms gluconic acid. This indicates presence of
aldehyde group.
CHO COOH
Bromine water
(CHOH)4 + [O] 
 (CHOH)4
CH2OH CH2OH
Glucose Gluconic acid

(Reaction + names of reactants, products and reagents) [1]

b. Action of dil. HNO3 on glucose: The oxidation of glucose with dilute nitric acid
gives saccharic acid (a dicarboxylic acid). Hence, a primary alcoholic group
(CH2OH) is present in glucose.

115
Board Answer Paper : July 2016
CHO COOH
dil HNO3
(CHOH)4  (CHOH)4
CH2OH COOH
Saccharic acid
Glucose
(Reaction + names of reactants, products and reagents) [1]
c. Action of hydroxylamine (NH2OH) on glucose: The reaction of glucose with
hydroxylamine gives an oxime. This indicates the presence of carbonyl group.
CHO CH = N  OH
NH 2 OH
(CHOH)4 
 (CHOH)4 + H2O
CH2OH CH2OH
Glucose Glucoxime
(Reaction + names of reactants, products and reagents) [1]
ii. Ligands:
The molecules or ions which are coordinated to the central atom or ion in the coordination
compound are called ligands or donor groups. [1]
Postulates of Werner’s theory:
a. Two types of valencies are shown by most metallic elements:
1. Primary valence or principal valence
2. Secondary valence.
b. The tendency of every metal is to satisfy both primary and secondary valences.
c. The number of secondary valence shown by each metal is fixed.
d. The secondary valence is always directed towards fixed positions in space.
(Four postulates) [½  2] [2]
iii. Reactions involved in the preparation of potassium dichromate from chrome iron ore
(chromite ore):
a. Conversion of chrome iron ore into sodium chromate (Roasting):
4FeO.Cr2O3 + 8Na2CO3 + 7O2  2Fe2O3 + 8Na2CrO4 + 8CO2
Chrome iron Sodium Oxygen Ferric Sodium Carbon
ore carbonate oxide chromate dioxide
(Reaction + names of reactants and products) [1]
b. Conversion of sodium chromate into sodium dichromate:
2Na2CrO4 + H2SO4  Na2Cr2O7 + Na2SO4 + H2O
Sodium (conc.) Sodium Sodium
chromate dichromate sulphate
(Reaction + names of reactants and products) [1]
c. Conversion of sodium dichromate into potassium dichromate:
Na2Cr2O7 + 2KCl  K2Cr2O7 + 2NaCl
Sodium Potassium Potassium Sodium
dichromate chloride dichromate chloride
(Reaction + names of reactants and products) [1]
iv. a. Metamerism (positional isomerism): Ethers having same molecular formula but
different alkyl groups attached on either side of the oxygen atom are called
metamers of each other. This phenomenon is called metamerism (positional
isomerism). [1]
b. Methyl n-propyl ether:
Structure:
CH3 – O – CH2 – CH2 – CH3 [½]
IUPAC Name: 1-Methoxypropane [½]

116
Chemistry
c. Action of hot HI on methyl n-propyl ether:
CH3 – O – CH2 – CH2 – CH3 + 2HI
Methyl n-propyl ether (Conc.)
Hot /373 K
  CH3I + CH3 – CH2 – CH2 – I + H2O
Methyl Propyl iodide
iodide

(Reaction + names of reactants and products + Reaction condition) [1]


Q.8. Answer any ONE of the following:
i. a. Preparation of orlon:
It is prepared by the addition polymerization of acrylonitrile. A peroxide catalyst is
used during the process.
n CH2 = CH Polymerization
  CH2  CH     CH2  CH 
n
CN CN CN
Acrylonitrile Repeating unit Polyacrylonitrile
(Orlon)
[1]
Preparation of teflon:
Teflon is the addition polymer made from the monomer tetrafluoroethylene.
Tetrafluoroethylene is heated under high pressure in presence of oxygen (which acts as a
catalyst) to form Teflon.
n CF2 = CF2 
Polymerization
  CF2  CF2  
 CF2  CF2 n [1]
Tetrafluoroethylene repeating unit Teflon

b. 1. Ofloxacin – Antibiotic [½]


2. Morphine – Analgesic [½]
3. Ampicillin – Antibiotic [½]
4. Chloramphenicol – Antibiotic [½]
c. 1. CH3 CH3 CH3
NH2 +– Br
N2Br
NaNO 2 /HBr

273 K
 Cu 2 Br2
 + N2 

Diazonium 2–Bromotoluene
2-Methylaniline
bromide (B)
CH3 (A)
+–
N2Br
A is [½]

CH3
Br
B is [½]

2. 2CH3 – CH2 – CH – CH3 + 2KOH


(Alc.)
Br
2-Bromobutane

  H3C – CH = CH – CH3 + H3C – CH2 – CH = CH2+ 2KBr + 2H2O
But-2-ene But-1-ene
(A) (B)

A is H3C – CH = CH – CH3 [½]


B is H3C – CH2 – CH = CH2 [½]
117
Board Answer Paper : July 2016
3. CH3  CH2  N+(CH3)3I 
Ag 2 O/ H 2 O

CH3  CH2  N+(CH3)3OH + AgI
Ethyltrimethyl Ethyltrimethyl
ammonium iodide ammonium hydroxide
(A)

CH3  CH2  N+(CH3)3OH 


  CH2 = CH2 + (CH3)3N + H2O
Ethyltrimethyl Ethylene Trimethyl
ammonium hydroxide (B) amine
(A)

+ –
A is C2H5N(CH3)3OH [½]
B is CH2 = CH2 [½]
ii. a. 1. Conversion of 2-methylbutan-1-ol into 2-methylbutanoic acid:
CH3
K 2 Cr2 O7  dil.H 2SO4
CH3 – CH2 – CH – CH2 – OH + 2[O]  
2-Methylbutan-1-ol
CH3
CH3 – CH2 – CH – COOH + H2O
2–Methylbutanoic
acid

(Reaction + names of reactant, product and reagent) [1]


2. Conversion of phenylethene into benzoic acid:
CH = CH2 COOH

KMnO 4 /dil.H 2SO 4

+ CO2

Phenylethene Benzoic acid

(Reaction + names of reactant, product and reagent) [1]


3. Conversion of benzoic acid into meta-nitrobenzoic acid:
COOH COOH

Conc.HNO3

Conc.H SO
 + H2O
2 4
NO2
Benzoic acid m-Nitrobenzoic acid

(Reaction + names of reactant, product and reagent) [1]


b. 1. Reaction of primary amine with benzene sulphonyl chloride gives
corresponding N-alkyl benzene sulphonylamide.
Owing to the presence of strong electron withdrawing sulphonyl group,
hydrogen attached to nitrogen atom in N-alkyl benzene sulphonylamide is
strongly acidic and dissolves in aqueous KOH. On acidification of this
solution, insoluble amide gets regenerated.

O O

R  N  H + Cl  S  R  N  S + HCl

H O H O
Primary Benzene N-Alkyl benzene
amine sulphonyl sulphonyl amide
chloride

118
Chemistry

O O O

 
RNS  K RN  S  RNS
H

KOH

O O [1]
H H O
N-Alkyl benzene N-Alkyl benzene
sulphonyl amide sulphonyl amide

2. Reaction of secondary amine with benzene sulphonyl chloride gives


N,N-dialkyl benzene sulphonylamide.
N,N-dialkyl benzene sulphonylamide does not contain acidic hydrogen.
Hence, it is not acidic and is insoluble in aqueous KOH. It is also insoluble in
acid.
O O

R  N  H + Cl  S  R  N  S + HCl
[1]
R O R O
Secondary Benzene N,N-Dialkyl benzene
amine sulphonyl sulphonylamide
chloride
3. Due to absence of hydrogen atom directly attached to the nitrogen atom,
tertiary amines do not react with benzene sulphonyl chloride. The unreacted
tertiary amine is insoluble in aqueous KOH but soluble in acid.

C6H5SO2Cl + R3N  C6H5SO2N+R3Cl 
KOH
 Insoluble compound 
H
Clear solution
Benzene Tertiary
sulphonyl amine
chloride
[1]
c. Uses of formaldehyde:
1. Formalin (40% solution of formaldehyde) is used as preservative for biological
specimens.
2. Formaldehyde is used for silvering mirror.
3. Formaldehyde is used for the production of several plastic and resins, bakelite
and binders in plywood.
(Any two uses) [½  2] [1]

119
Board Answer Paper : July 2016

BOARD ANSWER PAPER : JULY 2016


MATHEMATICS AND STATISTICS

Note: Answer to every question must be written on a new page.

SECTION  I

Q.1. (A) Select and write the correct answer from the given alternatives in each of the
following sub-questions:
i. (D)
(p  q)  ( p  q)
Its inverse is
~ (p  q )  ~ ( p  q)
≡ (~p  ~q) (~p  ~q) [2]
ii. (C)
Let a = 2 î  q ĵ + 3 k̂ and b = 4 î  5 ĵ + 6 k̂ .
Since, a and b are collinear.
 there exists a scalar t such that b = t a .
 4 î  5 ĵ + 6 k̂ = t(2 î  q ĵ + 3 k̂ ) = 2t î  qt ĵ + 3t k̂
 By equality of vectors, we get
4 = 2t,  5 =  qt, 6 = 3t
 4 = 2t and 6 = 3t  t=2
  5 =  q(2)
 –5 = – 2q  5 = 2q
5
 q= [2]
2
iii. (B)
a  b  c 18  24  30
s=   36
2 2

sin
A
=
 s  b  s  c  = (36  24)(36  30)
=
12  6
=
1
[2]
2 bc 24  30 24  30 10
(B) Attempt any THREE of the following:
i. Let b1 and b 2 be the vectors in the direction of the lines
r = (3iˆ  2ˆj  4k)
ˆ +  (iˆ  2ˆj  2k)
ˆ and r = (5iˆ  2k)
ˆ +  (3iˆ  2ˆj  6k)
ˆ respectively.

 b1 = ˆi  2ˆj  2kˆ and b 2 = 3iˆ  2ˆj  6kˆ


 b1  b 2 = (iˆ  2ˆj  2k)
ˆ (3iˆ  2ˆj  6k)
ˆ
=13+22+26
= 3 + 4 + 12
 b1  b 2 = 19 [1]
Consider,
b1 = (1) 2  (2) 2  (2) 2 = 1  4  4 = 9 =3

and b 2 = (3) 2  (2)2  (6)2 = 9  4  36 = 49 = 7


Let  be the acute angle between the two given lines.
b1  b 2 19
 cos  = =
b1 b 2 3 7

120
Mathematics and Statistics
19
 cos  =
21
 19 
  = cos1   [1]
 21 
ii. (r  q)  p
 (T  F)  T
 (T  F)  F [1]
 FF
T
Hence, the truth value is ‘T’ [1]
 2 3
iii. Given A = 
3 5 
2 3
 A = = 10 + 9 = 19  0
3 5
 A1 exists
A11 = (1)1 + 1. M11 = 5, A12 = (1)1 + 2. M12 = 3
A21 = (1)2 + 1. M21 = –(–3) = 3, A22 = (1)2 + 2. M22 = 2
Hence, matrix of the co-factors is
A A  5 3
[Aij]2×2 =  11 12
 = 3
A
 21 A 22   2 
T 5 3
Now, adj A =  A ij  =   [1]
2 2
 3 2 
1 1  5 3
A1 =  adj A  =  [1]
A 19  3 2 

iv. Let a, b,c and d be the position vectors of vertices A, B, C, D respectively of □ABCD
 a  ˆi  2ˆj  k,
ˆ b  8iˆ  3jˆ  4kˆ , c  5iˆ  4ˆj  kˆ and d   2iˆ  ˆj  4kˆ

a  c ˆi  2ˆj  kˆ  5iˆ  4ˆj  kˆ


Let e    3iˆ  ˆj ….(i)
2 2
b  d 8iˆ  3jˆ  4kˆ  2iˆ  ˆj  4kˆ
and f    3iˆ  ˆj ….(ii) [1]
2 2
 From (i) and (ii), we get
ef
The mid point of the diagonals AC and BD is same
 The diagonals AC and BD bisect each other.
 The □ABCD is a Parallelogram. [1]
v. sin x = tan x
sin x
 sin x =
cos x
 sin x cos x  sin x = 0
 sin x (cos x  1) = 0 [1]
 sin x = 0 or cos x = 1
 sin x = sin 0 or cos x = cos 0
Since, sin  = 0 implies  = n and cos  = cos  implies  = 2n   , n  Z.
 x = n or x = 2m  0
 the required general solution is x = n or x = 2m, where n, m  Z. [1]
Q.2. (A) Attempt any TWO of the following:
i. Given equation is ax2 + 2hxy + by2 = 0,
Let m1 and m2 be the slopes of the given lines.
2h a
 m1 + m2 = and m1m2 = [1]
b b

121
Board Answer Paper : July 2016
Since, the required lines are perpendicular to these lines.
1 1
 slopes of the required lines are  and 
m1 m2
Required lines also pass through the origin, therefore their equations are
1 1
y=  x and y =  x
m1 m2
 x + m1y = 0 and x + m2y = 0
 the joint equation of the lines is (x + m1y)(x + m2y) = 0 [1]
 x2 + (m1 + m2)xy + m1m2y2 = 0
 2h  a
 x2 +  2
 xy +   y = 0
 b   
b
 bx2 – 2hxy + ay2 = 0 [1]
 1 
ii. Let sin–1   = x
 2
1
 sin x =
2

 sin x = –sin [1]
4
     
The principal value branch of sin–1 x is   ,  and     [1]
 2 2 2 4 2

Hence, the required principal value of x is  . [1]
4

iii. The equation r  a  sb  tc represents a plane passing through a point having position
vector a and parallel to the vectors b and c .
Here, a = ˆi  ˆj , b  ˆi  ˆj  2kˆ and c  ˆi  2ˆj  kˆ
The given plane is perpendicular to the vector n
ˆi ˆj kˆ
= b  c = 1 -1 2
1 2 1

= î (1  4)  ĵ (1  2) + k̂ (2 + 1)
= 5 î + ĵ + 3 k̂ [1]
Vector equation of the plane in scalar product form is r  n = a  n
 r   5iˆ + ˆj + 3kˆ  =  ˆi + ˆj   5iˆ + ˆj + 3kˆ  [1]
 r   5iˆ + ˆj + 3kˆ  = 1(5) + 1(1) + 0(3)
 r   5iˆ + ˆj + 3kˆ  = 4 ....(i)
For cartesian form,
Putting r = x î + y ĵ + z k̂ in (i), we get
 xˆi + yˆj + zkˆ    5iˆ + ˆj + 3kˆ  =  4
 x(5) + y(1) + z(3) =  4
 5x + y + 3z =  4
 5x – y – 3z = 4,
which is the cartesian form of the equation of the plane. [1]
(B) Attempt any TWO of the following:
i. Let p: The switch S1 is closed.
q: The switch S2 is closed.
~p: The switch S1 is closed or the switch S1 is open.
~q: The switch S2 is closed or the switch S2 is open.

122
Mathematics and Statistics
 symbolic form of the given circuit is
(p  ~q)  (~p  q)  (~p  ~q) [1]
 (pq)  [p(qq)] …(Associative and Distributive law)
 (pq)  (pT) ….(Complement law)
 (pq) p ….(Identity law)
 (p  ~p)  (~q  p) ….(Distributive law) [1]
 (p  ~p)  (~p  q) ….(Commutative law)
 T(~p  ~q) …. ( Complement law)
 pq ….(Identity law) [1]
The simplified circuit of the given circuit are as follows:

SS1' 1

SS'22 [1]
+  L

ii. Let , ,  be the direction angles of the line


  = 45,  = 60
1 1
 m = cos  = cos 45 = and n = cos  = cos 60 =
2 2
Since, cos2  + cos2  + cos2  = 1 [1]
2 2
 1  1
 cos2  +      = 1
 2  2
1 1 1
 cos2  = 1   
2 4 4
1
 cos  =  [1]
2
1 1 1
i.e., l =  , m = ,n=
2 2 2
The unit vectors along the direction of line are
û = lˆi  mjˆ  nkˆ
1 1 ˆ 1ˆ
=  ˆi  j k [1]
2 2 2
 The vectors of magnitude 5 are
1 1 ˆ 1 ˆ  1 1 ˆ 1 ˆ
5  ˆi  j  k  and 5   ˆi  j  k [1]
2 2 2   2 2 2 

iii. To draw the feasible region, construct table as follows:

Inequality x + 4y  24 3x + y  21 x+y9
Corresponding equation (of line) x + 4y = 24 3x + y = 21 x+y=9
Intersection of line with X-axis (24, 0) (7, 0) (9, 0)
Intersection of line with Y-axis (0, 6) (0, 21) (0, 9)
Region Origin side Origin side Origin side

123
Board Answer Paper : July 2016
Y

24

21
3x + y = 21
18

15

12 [1]

6 D(0, 6) C(4, 5)

3 B(6, 3)

A(7, 0)
X X
O 3 6 9 12 15 18 21 24
x + 4y = 24

x+y=9
Y
Shaded portion OABCD is the feasible region,
whose vertices are O(0, 0), A(7, 0), B, C and (0, 6)
B is the point of intersection of the lines 3x + y = 21 and x+y = 9.
Solving the above equations, we get x = 6, y = 3
 B  (6, 3)
C is the point of intersection of the lines
x + 4y = 24
and x + y = 9.
Solving the above equations, we get
x = 4, y = 5
 C  (4, 5) [1]
Here, the objective function is Z = 3x + 5y,
 Z at O(0, 0) = 3(0) + 5(0) = 0
Z at A(7, 0) = 3(7) + 5(0) = 21
Z at B(6, 3) = 3(6) + 5(3)
= 18 + 15 = 33 [1]
Z at C(4, 5) = 3(4) + 5(5)
= 12 + 25 = 37
Z at D(0, 6) = 3(0) + 5(6) = 30
 Z has maximum value 37 at C(4, 5).
 Z is maximum, when x = 4, y = 5 [1]
Q.3. (A) Attempt any TWO of the following:
i. Shortest distance between the lines
x  x1 y  y1 z  z1 x  x2 y  y2 z  z 2
  and   is
a1 b1 c1 a2 b2 c2
x2  x1 y2  y1 z 2  z1
a1 b1 c1
a2 b2 c2
d= . [1]
 b1c2  b2c1    c1a 2  c 2a1    a1b 2  a 2 b1 
2 2 2

Equations of the given lines are


x +1 y +1 z  1 x 3 y 5 z 7
  and  
7 6 1 1 2 1

124
Mathematics and Statistics
Here,
x1 =  1, y1 =  1, z1 =  1, x2 = 3, y2 = 5, z2 = 7,
a1 = 7, b1 =  6, c1 = 1, a2 = 1, b2 =  2, c2 = 1
x2  x1 y2  y1 z 2  z1 4 6 8
Now, a1 b1 c1  7 6 1
a2 b2 c2 1 2 1
= 4(6 + 2)  6(7  1) + 8(14 + 6)
=  16  36  64
=  116
and  b1c 2  b 2c1    c1a 2  c2a1    a1b 2  a 2 b1 
2 2 2

= (6 + 2)2 + (1  7)2 + (14 + 6)2


= 16 + 36 + 64
= 116 [1]
116
 shortest distance between the given lines =
116
= 116
= 2 29 units [1]

ii. Let p1 and p2 be the distances of points î  ĵ + 3 k̂ and 3 î + 4 ĵ + 3 k̂ from the plane
r   5iˆ  2ˆj  7kˆ  + 8 = 0
The distance of the point A with position vector a from the plane r  n = p is given by
a n  p
d=
n

(ˆi -ˆj+3kˆ )⋅(5iˆ + 2jˆ-7kˆ )-(-8)


 p1 =
52 + 22 + (-7) 2
1(5)  1(2)  3(7)  8
=
25  4  49
5  2  21 8 10 10
= = = [1]
78 78 78
( )( )
3iˆ + 4jˆ + 3kˆ ⋅ 5iˆ + 2jˆ -7kˆ -(-8)
and p2 = 2 2 2
5 + 2 + (-7)
3(5)  4(2)  3( 7)  8
=
25  4  49
15  8  21 8
=
78
10
= [1]
78
 p1 = p2
Hence, points are equidistant from the plane. [1]
iii. By cosine rule, we have b2 = c2 + a2  2ca cos B
c2  a 2  b2
 cos B =
2ac
b2  c2  a 2
Similarly cos A = [1]
2bc
R.H.S. = a cos B + b cos A
c2  a 2  b2 b2  c2  a 2
= a.  b. [1]
2ac 2bc
125
Board Answer Paper : July 2016
c2  a 2  b2 b2  c2  a 2
= 
2c 2c
c2  a 2  b2  b2  c2  a 2
=
2c
2c2
=
2c
=c
= L.H.S.
 c = a cos B + b cos A [1]

(B) Attempt any TWO of the following:


i. Given equation is px2  8xy + 3y2 + 14x + 2y + q = 0
Comparing with ax2 + 2hxy + by2 + 2gx + 2fy + c = 0, we get
a = p, h =  4, b = 3, g = 7, f = 1, c = q. [1]
The given equation represents a pair of lines perpendicular to each other
 a+b=0
 p+3=0
 p = 3 [1]
Also, the given equation represents a pair of lines
a h g
 h b f =0 [1]
g f c
-3 -4 7
 -4 3 1 = 0
7 1 q
 3(3q  1) + 4(  4q  7) + 7( 4  21) = 0
 9q + 3  16q  28  175 = 0
 25q  200 = 0
 25q = 200
 q = 8 [1]
 p = 3 and q = 8
ii. Let the cost of 1 dozen pencils, 1 dozen pens and 1 dozen erasers be ` x, ` y and ` z
respectively.
According to the given conditions,
4x + 3y + 2z = 60
2x + 4y + 6z = 90 i.e. x + 2y + 3z = 45
6x + 2y + 3z = 70 [1]
Matrix form of the given system of equations is,
é 4 3 2ù é xù é 60ù
ê ú ê ú ê ú
ê 1 2 3ú ê y ú = ê 45ú [1]
ê ú ê ú ê ú
ê 6 2 3ú êzú ê 70ú
ë û ë û ë û
Applying R1  R2,
é 1 2 3ù é x ù é 45ù
ê ú ê ú ê ú
ê 4 3 2ú ê y ú = ê 60ú
ê ú ê ú ê ú
ê 6 2 3ú ê z ú ê 70ú
ë û ë û ë û
Applying R2  R2  4R1, R3  R3  6R1,
é1 2 3ù é xù é 45ù
ê ú ê ú ê ú
ê 0 - 5 -10ú ê y ú = ê-120ú
ê ú ê ú ê ú
ê 0 -10 -15 ú êzú ê-200ú
ë û ë û ë û
126
Mathematics and Statistics
Applying R3  R3  2R2,
é1 2 3ù é xù é 45ù
ê ú ê ú ê ú
ê 0 - 5 -10ú ê y ú = ê-120ú
ê ú ê ú ê ú
ê0 0 5 úû êzú ê 40ú
ë ë û ë û
Hence, the original matrix is reduced to an upper triangular matrix.
 By equality of matrices, we get
x + 2y + 3z = 45 ....(i)
5y  10z = 120
i.e. y + 2z = 24 ....(ii) [1]
5z = 40 ....(iii)
i.e. z = 8
Substituting z = 8 in equation (ii), we get
y + 2(8) = 24
 y=8
Substituting z = 8 and y = 8 in equation (i), we get
x + 2(8) + 3(8) = 45
 x + 16 + 24 = 45
 x=5
 x = 5, y = 8, z = 8
Thus, the cost of pencils is ` 5 per dozen, that of pens is ` 8 per dozen and that of erasers
is ` 8 per dozen. [1]
iii.
A
b c a

 b
B
D
P
O c C

Let a, b and c be the position vectors of points A, B and C respectively with respect to
origin O.
Complete the parallelopiped as shown in the figure with OA , OB and OC as its
coterminous edges.
AP is a perpendicular drawn to the plane of b and c . Let  be the angle made by AP with
OA. Volume of parallelopiped = (Area of parallelogram OCDB)  (height)
Now, area of parallelogram OCDB = b  c ….(i) [1]
Height of parallelopiped = l(AP) = l(OA) cos 
= OA cos 
= a cos  ….(ii) [1]
 From (i) and (ii), we get
volume of parallelopiped = a b  c cos 
= a   bc 
 Volume of parallelopiped = a b c  [1]
Let a = 2iˆ  5jˆ  4k,b
ˆ  5iˆ  7ˆj  5kˆ and c  4iˆ  5jˆ  2kˆ

2 5 4
 a b c  = 5 7 5
 
4 5 2
= 2(–14 – 25) – 5(–10 – 20) – 4(25 – 28)
= –78 + 150 + 12 = 84
 Volume of the parallelopiped is 84 cubic units. [1]
127
Board Answer Paper : July 2016

SECTION  II

Q.4. (A) Select and write the correct answer from the given alternatives in each of the
following sub-questions:
i. (A)
7
  dy  3  3 d2 y
1 +    = 7 2
  dx   dx
7 3
  dy  
3
 d2 y 
Cubing on both sides, we get 1    = 73  2 
  dx    dx 
By definition of degree and order, Degree: 3 ; Order: 2 [2]
ii. (C)
9
9 9
 1
1  x2 

1
9
Let I = 
4
x 4

dx = x dx =    2  x 
1
 
2
4

 2 4
= 2  
9  4 = 2(3  2)
 I=2 [2]

iii. (A)
x
x
y2  y3 
x
x3 1
F(x) =  f ( y )dy   dy =   =  [2]
1 1
3  9  1 9 9
(B) Attempt any THREE of the following:
i. y = sec x

dy
dx

d
dx
sec x  
= sec x tan x
d
dx
 x [1]
1
= sec x . tan x .
2 x
dy sec x tan x
  [1]
dx 2 x
x+1
ii. Let I =  dx
 x+ 2  x+3
x+1 A B
  
      3
x+ 2 x+ 3 x+ 2 x+
 x + 1 = A(x + 3) + B(x + 2) ….(i)
 Putting x = 2 in equation (i) we get
1 = A
 A = 1
 Putting x = 3 in equation (i) we get
2 = B
 B=2 [1]
x+1 1 2
  
 x+ 2  x+3  x+ 2   x+3
 1 2 
 I =     dx
 x  2 x+3 
 I =  log x  2  2log x  3  c [1]

128
Mathematics and Statistics
iii. Let A be the required area
Consider the equation, y2 = 4x
 y= 2 x
4
 A =  y dx
1
4
=  2 x dx [1]
1
4
2 3

= 2.  x 2 
3 1
4
 4   1 2 
3 3
= 
2
3 
4
= 8  1
3
28
 A= sq. units. [1]
3

iv. sec2 x  tan y dx + sec2 y  tan x dy = 0


Dividing throughout by tan x tan y, we get
sec 2 x tan y sec 2 y tan x
dx + dy = 0
tan x tan y tan y tan x
sec 2 x sec 2 y
 dx + dy = 0 [1]
tan x tan y
Integrating on both sides, we get
sec2 x sec2 y
 tan x dx +  tan y dy = 0

 log |tan x| + log |tan y| = log |c|


 log |tan xtan y| = log |c|
 tan x tan y = c [1]
v. Given E(x) = 6, Var. (x) = 4.2
E( x) np
Now 
V( x) npq
6 1

4.2 q
4.2
 q=  0.7
6
p+q=1
p + 0.7 = 1
p = 0.3 [1]
then E(X) = np,
6 = n  0.3
6
n=
0.3
n = 20 [1]
Q.5. (A) Attempt any TWO of the following:
i. f is continuous at x = 0.
 f(0) = lim f(x) [1]
x 0

4  1
2 2
sin x
 4sin x 1  sin 2 x
  2
4  1 
sin x 2

x2  sin x  x
= lim = lim = lim [1]
x 0 x log 1  2 x  x 0 x  log 1  2 x  x 0 log(1 2 x)
2
x2 2x

129
Board Answer Paper : July 2016
2
 4sin x  1 sin x 
 lim  lim 
x  0 sin x x 0 x 
= 
log 1  2 x 
2lim
x 0 2x
 log 4 
2

 f(0) = [1]
2
1
ii. Let I =  3 + 2sin x + cos x dx
 x
Put tan   = t
2  
 x = 2 tan–1 t
2dt 2t 1- t 2
 dx = 2
and sin x = 2
, cos x = [1]
1+ t 1+ t 1 + t2
1 2
 I =  2t

1  t 1+ t 2
2
dt
3+ 2 +
1+ t 2 1+ t 2
2dt dt
=  3 +3t 2
+ 4t +1  t 2
=2  2t 2
+ 4t + 4
2 dt dt
=
2 t 2
+ 2t + 2
=  (t +1)
2
 1+ 2
[1]

dt
=  (t +1) + 1 2 2
= tan1 (t + 1) + c

é
x æ ö ù
 I = tan1 ê tan ççç ÷÷÷ +1ú + c [1]
êë è 2 ø úû

iii. ‘y’ is a differentiable function of ‘x’.


Let there be a small change x in the value of ‘x’.
Correspondingly, there should be a small change y in the value of ‘y’.
As x  0, y  0
x y
Consider,  =1
y x
x 1 y
 = , 0
y y x
x
Taking lim on both sides, we get
x  0

 x  1
lim   = [1]
  lim y 
x  0y 
 
x  0 x
 
Since ‘y’ is a differentiable function of ‘x’
 y  dy
 lim   =
 x  dx
x  0

As x  0, y  0
 x  1
 lim   = ….(i)
  lim y 
y  0y 
 
x  0 x
 
 limits on R.H.S. of (i) exist and are finite. [1]
Hence, limits on L.H.S. of (i) also should exist and be finite.
130
Mathematics and Statistics
 x  dx
 lim   = exists and is finite.
 y  dy
y  0

dx 1 dy
 = , 0 [1]
dy  dy  dx
 
 dx 

(B) Attempt any TWO of the following:


S
i. Let S be the position of source of light.
Let BD be the position of the man at a time t min.
Let AB = x ft. be distance of man from lamp at time t min.
and BC = length of the shadow of man at time t min = y ft. D
30
dx
= 100 ft/min
dt 6
ASC  BDC A x B y C
AS AC
 =
BD BC
30 x+ y
 =
6 y
 5y = x + y
x
y= [1]
4
dy 1 dx
 = .
dt 4 dt
1
= 100 
4
= 25
 The shadow of the man is lengthening at the rate 25 ft/min. [1]
The tip of shadow is C.
Let AC = z
 z=x+y
dz dx dy
  = 100 + 25
dt dt dt
dz
 = 125 ft/min [1]
dt
 The tip of the shadow is moving at the rate 125 ft/min. [1]

ii. E(X) =  x  P( x )
i i

= 0(0.08) + 1(0.15) + 2(0.45) + 3(0.27) + 4(0.05)


= 0 + 0.15 + 0.9 + 0.81 + 0.2
= 2.06 [1]
E(X2) = 
xi2  P( xi )
= 02(0.08) + 12(0.15) + 22(0.45) + 32(0.27) + 42(0.05)
= 0(0.08) + 1(0.15) + 4(0.45) + 9(0.27) + 16(0.05)
= 0 + 0.15 + 1.8 + 2.43 + 0.8
= 5.18 [1]
Var (X) = E(X2)  [E(X)]2 [1]
= 5.18  (2.06)2
= 5.18  4.2436
= 0.9364 [1]
a
iii. Let I =  f  x  dx
0

Put x = a – t
131
Board Answer Paper : July 2016
 dx = – dt
When x = 0, t = a  0 = a
When x = a, t = a  a = 0
a 0
 I=  f ( x)dx =
0
 f (a  t)(dt)
a
[1]
0
=   f (a  t)dt
a

a  b a 
=  f (a  t)dt ….   f  x  dx    f  x  dx 
0  a b

a
 b b

=  f (a  x)dx ….   f  x  dx   f  t  dt  [1]
0  a a 
a a
  f ( x)dx =
0
 f (a  x)dx
0

2 sin x
Let I =  …. (i)
0 sin x  cos x

 

sin   x 
2
2   a a

I =  dx ….   f ( x)dx   f (a  x)dx 
0      
sin   x   cos   x  0 0
2  2 

2 cos x
=  dx ….(ii) [1]
0 cos x  sin x
Adding (i) and (ii), we get

2 sin x+ cos x
2I =  dx
0 sin x  cos x

2
=  1dx
0

=  x 02

= 0
2

2I =
2

I=
4

2 sin x 
  dx  [1]
0 sin x  cos x 4

Q.6. (A) Attempt any TWO of the following:


i. Let y = etan x + (log x)tan x
Put u = etan x and v = (log x)tan x
 y=u+v
dy du dv
 = + ….(i) [1]
dx dx dx
u = etan x
Taking logarithm on both sides, we get
log u = tan xlog e = tan x
Differentiating w. r. t. x, we get
1 du
= sec2 x
u dx

132
Mathematics and Statistics
du
 = usec2 x
dx
du
 = etan x  sec2 x ….(ii)
dx
v = (log x)tan x
Taking logarithm on both sides, we get
log v = tan xlog (log x)
Differentiating w.r.t. x, we get
1 dv 1 1
= tan x + log (log x)sec2 x
v dx log x x
dv é tan x ù
 = vê + log (log x)sec2 xú
dx ê x log x ú
ë û
é tan x ù
= (log x)tan x ê + log (log x) ⋅ sec 2 xú ….(iii) [1]
ê x log x ú
ë û
From (i), (ii) and (iii), we get
dy  tan x 
= etan x  sec2 x + (log x)tan x    log (log x)sec 2 x  [1]
dx  x log x 

ii. Let X be the number of fluorescent lights that have a useful life of at least 800 hours.
P(a light has useful life of at least 800 hours) = p = 0.9, q = 1  0.9 = 0.1
Given n = 20
 X ~ B (20, 0.9)
The p.m.f. of X is given by
P(X = x) = p(x) = 20Cx (0.9)x (0.1)20x , x = 0,1,2, ……,20 [1]
P(at least 2 lights will not have a useful life) = P(at most 18 will have useful life)
= P(X  18) = 1  P(X > 18)
= 1  [P(X = 19) + P(X = 20)]
= 1  [20C19 (0.9)19 (0.1) + 20C20 (0.9)20] [1]
 919 920   919 
= 1   20  20  20  = 1   20  20  9  
 10 10  10 
 919  29 
=1  20 
 10 
29  919
Let M =
1020
 log M = log 29 + 19 log 9  20 log 10
= 1.4624 + 19 × 0.9542  20  1
= 1.4624 + 18.1298  20
= 19.5922  20
= 19.5922  19  1
= 1 .5922
 M = Antilog ( 1 .5922 ) = 0.3910
 P(at least two lights will not have a useful life) = 1  0.3910 = 0.6090 [1]
π π
iii. As function is continuous on [, ], it is continuous at x =  and x =
2 2
 lim f(x) =
-
lim f(x)
+
x - π x - π
2 2

 lim (2sin x) = lim ( sin x + )


- +
x - π x - π
2 2

æ -π ö÷ æ -π ö
 2sin ççç ÷÷ =  sin ççç ÷÷÷ + 
è 2 ø è 2 ø
 2(1) = (1) + 
133
Board Answer Paper : July 2016
 +=2 ….(i) [1]
π
Also, function is continuous at x =
2
 lim f(x) = lim+ f(x)
-
x π x π
2 2

 lim ( sin x + ) = lim+ cos x


-
x π x π
2 2

æπö π
  sin ççç ÷÷÷ +  = cos
è 2ø 2

 +=0 ….(ii) [1]


Solving (i) and (ii), we get
 = 1 and  = 1 [1]

(B) Attempt any TWO of the following:


dy
i. Since, represents the slope of tangent to a given curve at a point (x, y), the given equation is
dx
dy
+5=x+y [1]
dx
dy
  y = (x  5)
dx
dy
The given equation is of the form + Py = Q
dx
where, P = 1 and Q = (x  5)

I.F. = e  = e 
Pdx 1.dx
 = e x [1]
 Solution of the given equation is
y (I.F.) =  Q (I.F.) dx + c
y e x =  ( x  5) e
x
 dx + c


x
  x e  x dx  5 e dx + c

d e x
= x  e x dx     x   e x dx  dx  5 + c
 dx  1
e x
=  x ex   1 dx  5e + c
x

=  x ex +  e  x dx + 5ex + c
=  x ex  ex + 5ex + c
 y ex =  x ex + 4ex + c
 y =  x + 4 + cex [1]
 x + y  4 = cex is the general solution.
Since the curve is passing through the point (0, 2)
 x = 0, y = 2
 0 + 2  4 = ce0
 c=2
 x + y  4 =  2ex
 y = 4  x  2ex is the required equation of the curve. [1]
134
Mathematics and Statistics
ii. Let  vdx  w
dw
 =v
dx
d dw du
Consider, (uw) = u +w [1]
dx dx dx
du
= uv +  vdx 
dx
du
dx 
= uv + vdx

Integrating on both sides w.r.t. ‘x’, we get


 du 
uw =  u vdx     vdx  dx [1]
 dx
 du 
 u  v dx=  uvdx     vdx  dx
 dx 
 d 
  u  v dx = u  vdx    dx (u)   vdx  dx [1]

d 
Hence,  xe x dx  x .  e x dx    x .  e x dx  dx
 d x 
= xex  1  e xdx
= xex  ex + c [1]
log e x
iii. Let f(x) = log10 x = = (log10 e)(loge x)
log e 10
= (04343) log x
0  4343
 f  (x) = [1]
x
x = 1016 = 1000 + 16 = a + h
Here, a = 1000 and h = 16
f(a) = f(1000) = log10(1000) = log10 (10)3
= 3log10 10 = 3 ….[ log10 mn = n log10 m]
f (a) = f (1000) = 0  4343 = 00004343 [1]
1000
f (a + h)  f(a) + hf (a)
 log10 (1016)  3 + 16(00004343) [1]
 3 + 00069488
 log10 (1016)  3006949 [1]

135
Board Answer Paper : July 2016

BOARD ANSWER PAPER : JULY 2016


BIOLOGY

Note: Answer to every question must be written on a new page.

SECTION – I
[BOTANY]

Q.1. Select and write the most appropriate answer from the given alternatives for each
sub-question:
i. (C) ii [1]
ii. (B) Bacillus thuringiensis [1]
iii. (C) Datura stramoneum [1]
iv. (B) Vitamin B [1]
v. (D) 250 [1]
vi. (B) unidirectional [1]
vii. (C) triploid [1]
Q.2. (A) Answer in ‘One’ sentence only :
i. Bacteriophage is virus that infects the bacteria. [1]
ii. Emasculation is the removal of stamens from one of the parents (if bisexual) before they
release their pollen grains. [1]
iii. The branched vesicles forming the arbuscles of endomycorrhizae is known as VAM. [1]
iv. Decomposers act on dead organic matter releasing inorganic nutrients back in environment
by decomposing the complex compounds of the dead or living protoplasm. [1]
v. Edible mushroom are:
a. White button mushroom (Agaricus bisporus)
b. Paddy straw mushroom (Volvariella volvacea)
c. Oyster mushroom (Pleurotus florida)
[Any two names: ½ mark each] [1]
vi. Ecological succession:
The gradual (and predictable) change in the species composition of a given area is called
ecological succession. [1]
(B)

F1 particles or oxysomes
Ribosome
Outer membrane
Matrix DNA
Cristae
Inner membrane

Ultrastructure of Mitochondrion

[Proportionate diagram] [½]


[Any three labels: ½ mark each] [1½]
136
Biology
(C) Answer the following (Any TWO):
i. Floral adaptations for anemophily:
a. Flowers are small, numerous, inconspicuous and not showy.
b. Petals are green or highly reduced.
c. Stamens possess long filaments and anthers are versatile and exposed.
d. When flowers are unisexual, the male flowers are more numerous than female flowers
and are situated at a higher level.
e. Pollen grains are smooth, dry and light in weight.
f. Pollen grains are produced in large numbers to compensate their wastage.
g. To catch pollen grains, the stigmas become sticky, hairy, feathery or branched.
h. Flowers are devoid of fragrance, nectar, bright colour etc.
[Any four points: ½ mark each] [2]
ii.
No. Antibiotic Microbial source
a. Chloromycetin Streptomyces venezuelae
b. Erythromycin Streptomyces erythreus
c. Penicillin Penicillium chrysogenum
d. Streptomycin Streptomyces griseus
[Name of any two antibiotics: ½ mark each] [1]
[Name of any two organisms: ½ mark each] [1]
iii. Helobial type of endosperm:
a. It is an intermediate between nuclear and cellular type of endosperm.
b. First division of primary endosperm nucleus is followed by (division of cytoplasm)
wall formation.
c. The wall formation results in the central cell being divided into a large micropylar
and a small chalazal chamber.
d. In both the chambers the development occurs as seen in nuclear type.
e. This type is generally seen in Helobiae series of monocots, hence the name ‘Helobial
Endosperm’.

Helobial endosperm
[Description] [1½]
[Proportionate diagram] [½]
iv. Use of certain agrochemicals have been associated with some major environmental and
ecological damages. Such as:
a. The pesticides and weedicides are toxic, not only to target organisms but also to many
other non-target organisms, which are important components of the soil ecosystem.
b. Continuous use of inorganic fertilizers changes the chemical nature of soil and
reduces its fertility.
c. In addition, the run off of agrochemical fertilizers into streams, lakes and ponds can
cause an increased productivity of those aquatic ecosystems causing eutrophication
and algal blooms in the water bodies.
d. The pesticide residues can remain in the soil for long and can enter the food chain
and undergo biomagnification.
e. Pesticides also enter the aquatic food chain. They get accumulated in the fatty tissues of
fishes as well as in birds which feed on them. This process is called as bioaccumulation.
f. People handling pesticides in industries also suffer from various disorders like
respiratory diseases, nervous disorders, skin diseases, blindness, etc.
[Any four points: ½ mark each] [2]
137
Board Answer Paper : July 2016
Q.3. (A) Answer the following (Any TWO):
i. Incomplete dominance:
a. Incomplete dominance can be defined as a phenomenon in which neither of the alleles
of a gene is completely dominant over the other and hybrid is intermediate between the
two parents. Incomplete dominance is a deviation of Mendel’s law of dominance.
b. Thus, according to incomplete dominance, F1 phenotype is intermediate between the
parental traits. Incomplete dominance is demonstrated in Mirabilis jalapa (four
o’clock plant) as given below:
Phenotype of Parents Red flower White flower
Genotype RR rr

Gametes R r

F1 generation Rr
Pink flower
Selfing of F1 Rr  Rr
generation
Gametes R r R r

F2 generation
R r
R RR Rr
Red Pink
r Rr rr
Pink White [Graphical representation] [1]
Phenotypic ratio  1:2:1 (1 Red : 2 Pink : 1 White) [1/2]
Genotypic ratio  1:2:1 (1 RR : 2 Rr : 1 rr) [1/2]
This indicates the following facts:
a. Pink is the phenotype of the heterozygous genotype (Rr).
b. This pattern of inheritance is due to non-blending of the characters, because one-
fourth of the F2 progeny are red-flowered and another one-fourth are white-flowered,
which are the parental combinations.
c. The phenotypic and genotypic ratios are the same.
[Explanation] [1]
ii. The plants which are genetically engineered to carry desirable traits like disease resistance,
insect resistance, herbicide resistance are called as transgenic plants. [1]
Examples: Bt cotton, Golden rice, Flavr Savr tomato
a. Bacillus thuringiensis (Bt) is a soil bacterium that produces a protein with
insecticidal property.
Bt toxin proteins occur as inactive protoxin.
The ‘cry’ gene codes for this inactive protoxin.
Crops have now been genetically engineered using rDNA technology, by inserting
the ‘cry’ gene in plants.
After the insects ingest the transgenic crops, they are killed because the inactive
protoxin is converted into the active form due to alkaline pH of the insect gut.
Bt toxin gene has been cloned and introduced in many plants to provide resistance to
insects without the need of insecticides.
Bt cotton is commercially available to control the disease affecting the cotton balls.
b. Golden rice is genetically engineered rice with greater pro-vitamin A [ – carotene]
content.
c. In Flavr Savr variety of tomato, the expression of a native gene that codes for the
enzyme polygalacturonase, has been blocked. The enzyme is responsible for
degradation of pectin during fruit ripening.
In the absence of this enzyme, the fruit ripening or softening is delayed and hence,
the fruit remains fresh for longer periods and flavour is saved.
[Any two examples: 1mark each] [2]
138
Biology
iii. Micropropagation:
It is a type of tissue culture technique by which large number of plants propagules are
produced.
In this technique, shoot apical meristem is used as explants. Using the proper proportion of
growth hormones and tissue culture techniques many shoot apices can be produced.
These shoot apices are called Micropropagules. These are genetically identical and from
them, individual plants can be obtained. eg : Potato, banana and orchids. [11/2]
Somatic hybridization:
The protoplast from two different plants can be made to fuse by using fusogenic agent as
polyethylene glycol (PEG). From the resultant combined protoplast by tissue culture, a new
plant variety can be produced.
e.g. Pomato = Potato + Tomato
Raphanobrassica = Raphanus + Brassica [11/2]
(B)

Diploid Functional megaspore


megaspore Linear tetrad of (Chalazal)
mother cell (2n) haploid Degenerating
megaspores (n) megaspores

a. Megasporogenesis (diagrammatic)

Nucleus (n)
Cytoplasm
Functional megaspore 1st nuclear division 2nd nuclear division 3rd nuclear division
(2 nucleate embryo sac) (4 nucleate embryo sac) (8 nucleate embryo sac)

Antipodals (n)

Female gametophyte
Polars Egg apparatus
Polar nuclei
Egg(n)
Synergids

Shifting of nuclei Organisation of 7 celled


and 8 nucleate embryo sac
b. Development of female gametophyte or embryo sac (diagrammatic)

[3]
Q.4. i. Central dogma of molecular biology can be defined as unidirectional or one way flow of
information from DNA to mRNA (Transcription) and from mRNA to protein (Translation).
This can be represented as:
DNA   mRNA   Protein
Transcription Translation

However, in some retroviruses, reverse transcription takes place due to which DNA is
synthesized from RNA. This can be represented as:
Transcription
 mRNA   Protein
Translation

DNA   [1]
Re verse
Transcription

ii. Translation:
Translation is the process in which the sequence of codons on the mRNA strand is used
(read/decoded) and accordingly the amino acids are joined to each other to form a
polypeptide chain that makes protein.
139
Board Answer Paper : July 2016
The process involves the following steps:
a. Activation of amino acids and formation of AA-tRNA complex:
In presence of an enzyme aminoacyl tRNA synthetase, the amino acid (AA)
molecule is activated and each amino acid is attached to the specific tRNA molecule
at 3/CCA end to form aminoacyl-tRNA complex. The reaction requires ATP. This
process is called charging of tRNA or aminoacylation of tRNA.
b. Formation of the polypeptide chain: It is the actual translation which involves
the following steps:
Initiation:
It begins with the formation of initiation complex which requires the mRNA having
codons for a polypeptide, the smaller (30S) and larger (50S) sub-units of ribosome,
the initial AA1tRNA complex and ATP and GTP as source of energy.
The process of initiation needs initiation factors.
In prokaryotes, the first AA1tRNA complex has amino acid, N-formyl-metheonine
(f-met); In eukaryotes, it is metheonine (met).
The process starts with binding of mRNA on the smaller 30S sub-unit of ribosome.
The start codon AUG is positioned properly.
The AA1-tRNA complex, i.e. f-met-tRNA complex now gets attached to the start
codon AUG. This is done with the help of anticodon UAC of tRNA.
Small and large subunits of ribosome join to form 70S ribosome.
The ribosome has three sites namely, Aminoacyl site (A), Peptidyl site (P) and Exit
site (E).
The empty tRNA leaves from E site.
Only the AA1-tRNA complex binds at P site directly, while all the other incoming
tRNA complexes get attached first at A site and then are shifted to P site.
Polypeptide chain is released from P-site.
In eukaryotes, 40S (smaller sub unit) and 60S (larger sub unit) combine to form 80S
type of ribosome. [2]

Polypeptide chain

Empty t-RNA G
Next t-RNA-AA-complex
U
G U
A Ser A

Ala Val
Ribosome (large unit)

START C G U C A A STOP
A U G U C U C U U G G G U C C G C A G U U A A U U U C U A U C C C U A A
5 3
Ribosome (small unit)

Translation
[Proportionate diagram with correct labelling] [2]
Elongation:
This is done by formation of peptide linkages/bonds in between the successive amino acid
molecules (AA1, AA2, AA3 and so on). The elongation activity is catalyzed by the enzyme
peptidyl transferase.
Each tRNA complex brings a specific amino acid. Due to complementary nature of
anticodons and codons, the amino acids are placed to their proper positions.
During elongation, the ribosome moves along the mRNA in a step wise manner from start
to stop codon (5  3), one codon ahead each time. This movement is called translocation.
140
Biology
In every step of translocation, one amino acid is added in the polypeptide chain causing
elongation. [1]
Termination:
When the mRNA reaches the last termination codon, i.e. either UAA, UAG or UGA,
termination occurs.
In identifying the stop/termination codon and in releasing the polypeptide chain from the site,
the release or termination factors R1, R2 and S play an important role. After termination, the
smaller (30S) and larger (50S) sub units of ribosome get separated from each other.
In order to increase the cellular efficiency of protein synthesis, many ribosomes may bind
to the mRNA strand and form the polypeptide chain for synthesis of protein molecule. Such
a structure with many ribosomes bound to mRNA is called polysomes or polyribosome. [1]
OR
Q.4. Dark Reaction:
It is the second phase of photosynthesis in which CO2 is fixed or reduced to glucose. It
takes place in stroma of chloroplast. The reactions take place independent of light hence it
is known as dark reaction.
It is also known as ‘Blackman’s reaction’, ‘Calvin cycle’ or C3 pathway. [1]
The cycle is divided into the following phases:
i. Carboxylation phase: [1]
RuBP (Ribulose-1,5-bisphosphate) accepts atmospheric CO2 in the presence of enzyme
RuBP carboxylase (RuBisCO) and forms a 6-carbon unstable compound. This unstable
compound soon splits into two molecules of 3-carbon compound called PGA
(phosphoglyceric acid) in presence of the same enzyme:

RuBP + CO2 Mg
RuBP Carboxylase
 Unstable compound
(5C) (1C) (6C)

Unstable compound + H2O Mg
RuBP Carboxylase
 2 molecules of 3-PGA
(6C) (3C)
ii. Reduction phase (utilization of assimilatory power): [1]
a. The phosphoglyceric acid molecules are first phosphorylated by using ATP to
produce 1,3 di-phosphoglyceric acid.
b. It is then reduced by using NADPH2 to produce phosphoglyceraldehyde (PGAL) and
inorganic phosphate is released.
3PGA + ATP  1,3-diphosphoglyceric acid + ADP
1,3-diphosphoglyceric acid + NADPH2  3PGAL + NADP + Pi.
c. Some molecules of 3 PGAL are converted into its isomer dihydroxyacetone
phosphate (DHAP) in presence of enzyme triose phosphate isomerase.
iii. Synthesis of sugar and Regeneration phase: [1]
a. Synthesis of sugar:
1. For the synthesis of one molecule of glucose, six molecules of RuBP and six
molecules of CO2 are required, i.e. six turns of Calvin cycle are required for the
synthesis of one molecule of glucose.
2. Out of 12 molecules of PGAL, 2 molecules (i.e. 1/6th part) are used for the synthesis
of glucose.
3. A molecule of PGAL and DHAP combine together to form a molecule of fructose 1,
6-diphosphate as shown below:
3 PGAL + DHAP  Fructose-1,6-diphosphate
(3C) (3C) (6C)
[Note: Fructose-1,6-diphosphate is the first carbohydrate formed during this process]
4. Fructose-1,6-diphosphate undergoes dephosphorylation to form fructose-6-phosphate
which on isomerization forms glucose-6-phosphate.
5. Glucose-6-phosphate undergoes dephosphorylation to form glucose. Glucose, thus
formed is either utilized or stored as starch.
141
Board Answer Paper : July 2016
b. Regeneration phase:
1. RuBP gets regenerated through many biochemical reactions called sugar phosphate
interconversions.
2. All the intermediate compounds formed like erythrose-4-phosphate, xylulose-5-
phosphate, ribose-5- phosphate, sedoheptulose-7-phosphate, etc. are sugar phosphates.
3. Out of 12 molecules of PGAL, 10 molecules are utilized for the regeneration of 6
molecules of RUMP (ribulose monophosphate) which on phosphorylation form
RUBP as shown below:
12 PGAL  6 RUMP ….(1)
6 RUMP + 6 ATP  6 RUBP ….(2)
4. Thus, RUBP which is necessary for the reduction of CO2 is regenerated to keep the
process going.

RUBP
6CO2
(1C) carboxylase
6 Ribulose
Bisphosphate 6 Unstable
(5C) keto acid (6C)

6 ADP
Carboxylation
6 ATP 12PGA
(3C)
Ribulose
Monophosphate 12 ATP +12 NADPH2
(5C) Regeneration
Reduction 12 ADP +12 iP + 12 NADP
(Sugar phosphate
go to grana
interconversions)
Sedoheptulose
phosphate (7C)

Erythrose  DHAP


3-phosphoglyceraldehyde 
Phosphate (4C)
(3C) (3C)
12

2 Mol.
10 Mol.

Fructose-1, 6-diphosphate
(6C)

iP

Fructose-6-phosphate
(6C)

Glucose-6-phosphate
(6C)

iP

Glucose
(6C)
Diagrammatic representation of Calvin cycle

[Diagram with correct sequential labels] [3]

142
Biology

SECTION – II
[ZOOLOGY]

Q.5. Select and write the most appropriate answer from the given alternatives for each
sub-question:
i. (A) vestigial [1]
ii. (D) Down’s syndrome [1]
iii. (C) haemophilliacs [1]
iv. (C) polyuria [1]
v. (B) retina [1]
vi. (C) AIDS [1]
vii. (A) Banded Krait [1]
Q.6. (A) Answer in ‘One’ sentence each:
i. Two examples of commensalism are:
a. An orchid plant growing as an epiphyte on a mango tree.
b. Sea anemone and hermit crab
c. Barnacles on the back of whale
d. Sea anemone attached to snail’s shell
e. Golden jackals follow a particular tiger upto a safe distance to feed on the tiger’s
kills.
[Any two examples] [1]
ii. The common name for Apis mellifera is European Bee. [1]
iii. Bovine somatotropin is used to enhance dairy productivity. [1]
iv. The tendency of the genes to remain together on the same chromosome and to inherit
together is called linkage. [1]
v. Archaeopteryx is the connecting link between birds and reptiles. [1]
vi. Natality is defined as the number of births per unit time, per unit area, per 1000 individuals
of a population. [1]
(B)
Afferent arteriole
Efferent arteriole

Glomerulus
Parietal layer

Visceral layer
Bowman’s capsule
Podocyte
Capsular space

Neck
Proximal convoluted tubule

Malpighian body

[Proportionate diagram: 1/2 mark]


[Any three labels: 11/2 mark each] [2]
143
Board Answer Paper : July 2016
(C) Attempt any TWO of the following:
i. Propliopithecus

Dryopithecus

Ramapithecus

Australopithecus

Homo habilis
Java Man
Homo erectus
Peking Man
Homo neanderthalensis
(Neanderthal man)

Homo sapiens fossilis


(Cro - Magnon man)

Homo sapiens sapiens


(Modern man)
[Any four sequential evolutionary names: ½ mark each] [2]
ii. Significance of Transgenic animals for betterment of life is as follows:
a. To understand the normal physiology and development:
Specifically designed transgenic animals help us to understand the mechanism of gene
regulation. It is used to study how genes affect the normal functions and development of
the body. For e.g. study of biological role of insulin, role of growth factor in regulating
body’s growth.
b. To test vaccine safety:
Transgenic animals are used as laboratory animals to test efficiency of newly discovered
vaccines before it is used on human beings. Mice are used to test polio vaccines.
c. To test chemical or toxin safety:
For this, transgenic animals with foreign genes are produced so that the transgenic
animals become more sensitive to toxic chemicals than the non-transgenic animals.
These animals are exposed to toxic chemicals and their effects are observed.
d. To serve as a model for study of diseases:
Transgenic animals help us to study how genes contribute to development of disease
and for investigation of new treatments for diseases. Transgenic animals are now
available to study cancer, rheumatoid arthritis, cystic fibrosis, Alzheimer’s disease, etc.
e. To obtain biological products:
Transgenic animals are created to produce biological compounds or molecules that
are useful in various ways. It is accomplished by introducing the gene coding for a
particular molecule, into the animal. For e.g. Rosie, first transgenic cow was
produced which produced milk containing human protein, alpha lactalbumin.
[Any four points: ½ mark each] [2]
iii. Economic importance of lac:
a. It is used in the preparation of sealing wax, paints, varnish, electrical goods.
b. It is used in the preparation of bracelets, buttons, toys and in filling hollow gold
ornaments.
c. It is used in artificial leather and pottery.
d. It is also used in gramophone industry.
e. It is used for coating fruits and vegetables.
f. It is used for making bangles, toys, woodwork, polish, inks, slivering mirrors, etc.
[Any four points: ½ mark each] [2]
144
Biology
iv. Mesoderm forms all types of muscles, connective tissue, dermis of skin, adrenal cortex,
kidney, circulatory system including heart, blood vessels and blood, lymphatic vessels,
middle ear and dentine of teeth. [2]
Q.7. (A) Attempt any TWO of the following:
i. Sex determination in birds:
a. In birds, male is homogametic, whereas female is heterogametic.
b. Male has genotype ZZ and female has genotype ZW.
c. Sperms carry Z chromosome, but egg carries Z and W chromosome approximately in
equal number.
d. Sex of the zygote depends on which egg gets fertilized.
[Description] [11/2]
Male Female
Parents: X
ZZ ZW
Gametes: Z Z Z W

F1: ZZ ZW
Male Female
[Chart] [11/2]
ii. Unique features of acquired immunity are as follows:
a. Specificity:
Acquired immunity is specific for each type of pathogen. It has the ability to
differentiate between various foreign molecules.
b. Diversity:
It has the ability to recognize vast variety of diverse pathogens or foreign molecules.
c. Discrimination between self and non-self:
It is able to differentiate between own cells (self) and foreign cells or molecules
(non-self).
d. Memory:
When the immune system encounters a specific pathogen for the first time, it
generates immune response and eliminates the invader. The immune system retains
the memory of this encounter. As a result, a second encounter with same pathogen
stimulates a stronger immune response.
[Any two features: 1½ mark each] [3]
iii. Growing population:

Post
reproductive
Reproductive

Pre  reproductive

Pyramid of growing population


[Pyramid] [11/2]
The population having larger number of individuals of the pre  reproductive age groups
shows a very rapid growth rate. Thus is known as growing population.
Young individual in the age group of 0-14 years are included in pre-reproductive group,
individuals in age group of 15-59 years are included in reproductive group and individuals
in age group of 60 years and above are included in post reproductive group.
[Description] [11/2]
145
Board Answer Paper : July 2016
(B)
Dorsal root ganglion
White matter Sensory Skin receptor organ
neuron
Synapse
Muscle
Grey matter (effector organ)

Adjustor neuron Motor neuron


Reflex Arc
[Proportionate diagram : 1 mark]
[Any four labels : 1/2 mark each] [3]
Q.8. Process of early cleavage:
Definition: The process of rapid mitotic division of single celled zygote to form hollow,
spherical, multicellular blastula is called cleavage. [1]
Cleavage occurs during its passage through the Fallopian tube to the uterus.
In humans, cleavage is complete or holoblastic and equal.

Inner mass
Blastomere of cells
Zona
pellucida
Blastomeres
Zona Outer layer
pellucida of cells
2 - Celled stage 4 - Celled stage Early morula Morula
Stages of Cleavage

[Proportionate diagram] [1]


[Correct labelling] [1]
1st Cleavage:
It is completed about 30 hours after fertilization.
It is vertical, i.e. it takes place from animal pole to vegetal pole.
Two daughter cells or blastomeres are formed. [1]
2nd Cleavage:
It is completed about 60 hours after fertilization.
The second cleavage is also vertical, but at right angle to the first one.
The second cleavage occurs first in one blastomere resulting in transitory three-celled stage
followed by four cell stage. [1]
3rd Cleavage:
It takes place about 3 days (72 hours) after fertilization.
It is longitudinal and horizontal and forms 8 cells.
The cleavage continues and results in the formation of a solid mass of cells known as
Morula. [1]
Morula:
Morula is about 16 to 32 celled stage, which looks like a small mulberry. It reaches the
uterus 4  6 days after fertilization. It is still surrounded by Zona pellucida. Zona pellucida
prevents implantation at abnormal site. It does not expose trophoblast cells till it reaches
implantation site. [1]

146
Biology
OR
Leucocytes are colourless, nucleated, amoeboid and phagocytic cells.
Leucocytes are of two types: Granulocytes and Agranulocytes. [½]
i. Granulocytes: They are produced in red bone marrow and contain large sized granules in the
cytoplasm.
Granulocytes are of three types:
a. Neutrophils (neutro = neutral, philic = affinity)
They constitute about 70% of total WBCs.
Nucleus is multilobed containing 3 to 5 lobes.
The granules in cytoplasm of these cells take up neutral stain.
These are chief phagocytic cells. [½]
Functions:
They protect the body against invasion of bacteria.
Dead neutrophils along with damaged tissue are removed from the body in the form
of pus. [½]

Multilobed
nucleus

Granules
[½]
Neutrophil

b. Eosinophils (acidophils):
They constitute about 3% of total WBCs.
The nucleus is bilobed.
The granules in the cytoplasm of these cells take up acidic stain.
They are non-phagocytic and their number increases during allergic reactions. [½]
Function:
They show anti-histamine property. [½]

Bilobed nucleus

Eosinophil
[½]
c. Basophils:
They are the smallest white blood cells which constitute about 0.5% of the total
WBCs.
They show twisted nucleus (‘S’ or comma shaped).
The granules in the cytoplasm of these cells take up basic stain.
They are non-phagocytic. [½]
Function:
They secrete heparin, histamine, thus play an important role in local anticoagulation
and formation of ground substance. [½]

S shaped nucleus

[½]
Basophil

147
Board Answer Paper : July 2016
ii. Agranulocytes: They are produced in spleen and do not contain granules in the cytoplasm.
Agranulocytes are of two types:
a. Lymphocytes: They form 30% of total WBCs.
Nucleus is large, spherical and surrounded by thin layer of cytoplasm.
Function: They produce antibodies and are responsible for immune response of the
body. [½]

Cytoplasm

Nucleus

Lymphocytes

b. Monocytes: They are the largest leucocytes and constitute 3 to 9% of total WBC. [½]
They have large amount of cytoplasm and kidney shaped nucleus.
Functions: They are phagocytic in action. They engulf foreign particles. e.g.
bacteria.
They also remove the damaged and dead cells, hence are referred to as scavengers. [½]

Kidney
shaped nucleus
Monocyte [½]

148
Physics

BOARD ANSWER PAPER : MARCH 2017


PHYSICS

Note: Answer to every question must be written on a new page.

SECTION – I

Q.1. Select and write the most appropriate answer from the given alternatives for each
sub-question:
i. (C) [1]
ii. (C) [1]
iii. (C) [1]
iv. (D) [1]
v. (B) [1]
vi. (A) [1]
vii. (B) [1]
Q.2. Attempt any SIX :
i. a. Force acting on a particle performing circular motion along the radius of circle and
directed towards the centre of the circle is called centripetal force.
mv 2
It is given by FCP =
r
where, r = radius of circular path.
b. Example: Electron revolves around the nucleus of an atom. The necessary centripetal
force is provided by electrostatic force of attraction between positively charged
nucleus and negatively charged electron.
c. Unit: N in SI system and dyne in CGS system.
d. Dimensions: [M1L1T2] [2]
ii. Expression for r.m.s velocity:
a. Let, P = pressure exerted by one mole of an ideal gas
V = volume of one mole of the gas
T = absolute temperature
b. Pressure exerted by gas is given by,
1 Mc2
P= [½]
3 V
where M = mass of one mole (molecular weight) of the gas.
1
 PV = Mc2 ....(1)
3
c. But for one mole of an ideal gas, PV = RT [½]
1
 RT = Mc2 ....[From equation (1)]
3
2 3RT
 c =
M
3RT
 c= ….(2) [½]
M
Equation (2) represents expression for r.m.s velocity of gas molecules.
d. As R and M in equation (2) are constant,
 c T [½]
c1 T1
 =
c2 T2

149
Board Answer Paper : March 2017
iii. Differential equation of linear S.H.M:
a. Let a particle of mass ‘m’ undergo S.H.M about its mean position O. At any instant
‘t’, displacement of particle be ‘x’ as shown in the following figure.
Equilibrium position
O F = kx
Q P R
A A
b. By definition, F =  kx .…(1) [½]
where, k is force constant
c. The acceleration of the particle is given by,
 dx 
d  2
a = dv =  dt  = d x [½]
dt dt dt 2
d. According to Newton’s second law of motion,
F = ma [½]
d x
2
 F=m  2 
.…(2)
 dt 
e. From equations (1) and (2),
d2x
m =  kx
dt 2
d2x k
 2
=  x
dt m
d2x k
 2
+ x=0 ….(3)
dt m
k
where, = 2 = constant
m
d2x
 + 2x = 0 ….(4) [½]
dt 2
f. Equations (3) and (4) represent differential equation of linear S.H.M.
iv.


FA
A
P
Air
 Liquid
 FC
FR T

Solid

(Diagram and labelling) (1 + 1) [2]


v. Solution:
R
Given: W = mg = 350 N, d =
2
To find: Weight (Wd)
 d
Formula: gd = g 1  [½]
 R 

150
Physics
Calculation: Since Wd = mgd,
from formula,
 d
Wd = mg 1  [½]
 R 
  R 
  2 
 Wd = 350  1    
 R 
 
1
= 350  = 175
2
 Wd = 175 N
Ans: The body would weight 175 N half way down to the centre of the earth. [1]
vi. Solution:
Given: m = 1 kg, v = 2 m/s
To find: Total K.E
1  K2 
Formulae: E= MV 2 1  2  [½]
2  R 
2
Calculation: For solid sphere, K2 =   R2
5
1   2 2  2 1 7
 E= MV 2 1   5 R  / R   2  5  MV
2
[½]
2    
 E = 2.8 J
Ans: The total kinetic energy of the solid sphere is 2.8 J. [1]
vii. Solution:
Given: l = 0.9 m, n = 324 Hz
To find: Speed (v)
Formula: v = n [½]
Calculation : In 2nd overtone, 3 loops are formed.
3
 l = 
2
2l
 = [½]
3
From formula,
 2l  2 
v = n  = 324   0.9 
 
3  3 
 v = 194.4 m/s
Ans: The speed of the transverse wave is 194.4 m/s. [1]
viii. Solution:
d
Given: 1 = 25 C, 2 = 15 C,   = 0.5 C/min
 dt 1
 d 
To find: Rate of cooling at 2  
 dt  2
d
Formula: = K(  0)
dt
Calculation: Using formula, for 2 = 15 C,
 d / dt 2 2 0
 [½]
 d / dt 1 1 0
 d 
 
  dt 2  15 = 0.3 C/min [½]
0.5 25
Ans: The rate of cooling when it is 15 C above same surroundings is 0.3 C/min. [1]
151
Board Answer Paper : March 2017
Q.3. Attempt any THREE:
i. a. Let, M = mass of earth
m = mass of satellite
R = radius of earth
vc = critical velocity
b. In one revolution, distance covered by satellite is equal to circumference of its
circular orbit. [½]
c. If T is the time period of satellite, then
Circumferenceof the orbit
T=
Critical velocity
2r
 T= .…(1) [½]
vc
GM
But, vc = ….(2) [½]
r
d. Substituting equation (2) in (1),
2r
T= [½]
GM
r
r
= 2 r 2 
GM
r3
 T = 2 ….(3) [½]
GM
Thus, period of a satellite revolving around the earth depends upon mass of the earth. [½]
ii. Expression for torque acting on a rotating body:
a. Suppose a rigid body consists of n particles of masses m1, m2, m3, ......, mn which are
situated at distances r1, r2, r3, …, rn respectively, from the axis of rotation as shown in
figure.
b. Each particle revolves with angular acceleration .
c. Let F1, F2, F3, …., Fn be the tangential force acting on particles of masses, m1, m2,
m3, …, mn respectively.
d. Linear acceleration of particles of masses m1, m2,…, mn are given by,
a1 = r1, a2 = r2, a3 = r3, …, an = rn

m3 


F3 F2
r3 O r m2
2
mn 
rn r1 F1

Fn m1

[Explanation + Diagram](½ + ½) [1]


e. Magnitude of force acting on particle of mass m1 is given by,
F1 = m1a1 = m1r1 ….[ a = r]
Magnitude of torque on particle of mass m1 is given by,
1 = F1 r1 sin  [½]
But,  = 90 [ Radius vector is ar to tangential force]
 1 = F1 r1 sin 90
= F1r1
= m1a1 r1

152
Physics
 1 = m1r12  ….[ a1 = r1 ]
Similarly, 2 = m 2 r22 
3 = m3 r32 ,

.…
.…
n = mn rn2 
f. Total torque acting on the body,
 = 1 + 2 + 3 + …. + n
  = m1r12  + m 2 r22 + m3 r32  + … + m n rn2 
  = (m1r12 + m2r22 + m3r32+.....+ m n rn2 )
 n

  =   mi ri 2  
 i 1 
n
But, m r
i 1
i i
2
=I

  = I [½]
g. Unit: Nm in SI system. [½]
h. Dimensions: [M1L2T2] [½]
iii. Solution:
Given: E = 2T [½]
To find: Diameter of drop (d)
Formula: E = TA [½]
Calculation: From formula,
E
A =
T
2T
 A =
T
 A = 2π ....(1) [½]
We know , A = 4r2
Substituting in equation (1),
 2 = 4r2 [½]
 4r2 = 2
2 1
 r2= =
4 2
r2= 0.5
 r= 0.5 = 0.71 m
d= 2r
= 2(0.71)
 d= 1.414 m
Ans: The diameter of the drop is 1.414 m. [1]
5
v. Given: v = 36 km h1 = 36  = 10 m/s,  = 10
18
To find: Length of circular track (l)
v 2max
Formula: tan  = [½]
rg
Calculation: From formula,
v 2max
r 
g tan 
10 
2

 r = [½]
9.8  tan 10o
= 57.88 m [1]
l = 2πR = 2 3.14  57.88 = 363.7 m
Ans: The length of the circular track is 363.7 m. [1]
153
Board Answer Paper : March 2017
Q.4. A. Conservation of energy in linear S.H.M.:
i. Suppose a particle of mass m performing linear S.H.M. is at point P which is at a
distance x from the mean position O as shown in figure.
O P

X x X
Negative Mean Positive
extremity position extremity

ii. Kinetic energy of particle at point P is given by,


1
K.E. = m2(A2  x2) [½]
2
iii. Potential energy at point P is given by,
1
P.E. = m2x2 [½]
2
iv. Total energy at point P is given by,
T.E. = K.E. + P.E.
1 1
= m2(A2  x2) + m2x2
2 2
1
= m2(A2  x2 + x2)
2
1
 T.E. = m2A2 ….(1) [½]
2
v. If particle is at mean position:
x=0
1
 K.E. = m2A2
2
1
P.E. = m2(0)2 = 0
2
1
 T.E. = K.E. + P.E. = m2A2 . …(2) [½]
2
vi. If particle is at extreme position:
x=A
K.E. =
1
2

m2 A 2  A 2 = 0
1
P.E. = m2A2
2
1
 T.E. = P.E.+ K.E. = m2A2 .…(3) [½]
2
vii. From equations (1), (2) and (3), it is observed that total energy of a particle
performing linear S.H.M. at any point in its path is constant. Hence, total energy of
linear S.H.M. remains conserved. [½]
viii. a. Graph of variation of kinetic energy w. r. t. instantaneous displacement.
K.E

K.Emax

x=A O x=A
x=0
[½]
154
Physics
b. Graph of variation of potential energy w.r.t. instantaneous displacement.

P.E
P.Emax

x=A x=0 x=A


[½]
B. Solution:
83 83
Given: 1 = m, 2 = m, no. of beats = 8/s.
170 172
To find: Velocity (v),
Frequency (n1, n2)
Formula: v = n
Calculation: From formula,
v v
 n1 = and n2 = .
1 2
But 1 > 2
 n2 > n1 [½]
 n2  n1 = 8 [½]
 1 1  172 170 
 v    = 8 or v   = 8
  2 1   83 83 
8  83
 v= = 332 m/s [1]
2
170
 n1 = 332  = 680 Hz and [½]
83
332  172
n2 = = 688 Hz [½]
83
Ans: i. The velocity of sound in air is 332 m/s.
ii. The frequencies of the two notes are 680 Hz and 688 Hz.
OR
A. Formation of stationary waves by analytical method:
i. Consider two identical progressive waves of equal amplitude and frequency
travelling along X axis in opposite direction. They are given by,
2
y1 = A sin (vt  x) along positive X-axis .…(1) [½]

2
y2 = A sin (vt + x) along negative X-axis ….(2) [½]

ii. The resultant displacement ‘y’ is given by the principle of superposition of waves,
y = y1 + y2 ….(3) [½]
2 2
y = A sin (vt  x) + A sin (vt + x) [½]
 
iii. By using,
CD C D
sin C + sin D = 2sin   cos  ,
 2   2 
we get,
 2  vt  x  vt  x    2  vt  x  vt  x 
y = 2A sin    cos   
  2    2 
 2vt   2 
= 2Asin   cos  (  x)  [½]
     

155
Board Answer Paper : March 2017
 2x   v 
 y = 2Asin 2πnt cos   ….  n  and cos ()  cos 
     
 2x 
 y = 2Acos   sin 2πnt
  
 2x 
iv. Let R = 2Acos  
  
 y = Rsin (2πnt) ….(4) [½]
But,  = 2n
 y = R sin t ….(5)
Equation (5) represents the equation of S.H.M. Hence, the resultant wave is a S.H.M.
of amplitude R which varies with x.

A A A A
N N N
N  Node N
A  Antinode
  
2 4 2

Position and displacement of nodes


and antinodes in a stationary wave
(Diagram and labelling) [1]
B. Solution:
Given: L = 4 m,
r = 0.5 mm = 0.5  103 m,
M = 1 kg,
Elastic limit of steel is 2.4  108 N/m2,
Y for steel (Ysteel) = 20  1010 N/m2
To find: Extension in length (l)
Area of cross section (A)
FL
Formulae: i. Y= [½]
Al
F
ii. Elastic limit = [½]
A
Calculation: From formula (i),
FL MgL
l = = 2 [½]
AY r Y
1  9.8  4
 l= [½]
 
2
3.14  0.5  103  20  1010

l = 2.495  104 m,
From formula (ii),
Mg
A=
Elasticlimit
1  9.8
=
2.4  108
 A = 4.083  108 m2 [1]
Ans: The extension produced in length is 2.495  104 m and the area of cross section of
the wire should be 4.083  108 m2.
156
Physics

SECTION – II

Q.5. Select and write the most appropriate answer from the given alternatives for each
sub-question:
i. (C) [1]
ii. (B) [1]
iii. (A) [1]
iv. (B) [1]
v. (C) [1]
vi. (C) [1]
vii. (D) [1]
Q.6. Attempt any SIX:
i.
No. Interference Diffraction
i. Interference is due to superposition Diffraction is due to waves coming
of waves from different wavefronts. from different parts of the same
wavefront.
ii. All bright fringes are of equal Intensity decreases with the order of
intensity. bright band.
iii. Minimum intensity may be zero. Minimum intensity is not zero.
iv. Width of the central maximum is Width of central maximum is broader
same as that of other bright fringes i.e than other maxima and it is double the
fringe width is same for all fringes fringe width.
including central maxima.
v. The waves emitted by two coherent The light waves are bend at the corners
sources travel in straight line. and displaced from their straight line
path.
vi. The resolving power of an optical The resolving power of an optical
instrument does not depend on the instrument depends on the phenomenon
phenomenon of interference. of diffraction.

(Any two points) [2]


ii. a. To use a M.C.G as a voltmeter, its resistance should be increased to a desired value
and an arrangement should be provided to measure large potential difference. This is
achieved by connecting a high resistance in series with the M.C.G. [½]

V = (G + Rs)Ig
V = GIg

Ig G Ig Ig
Ig
G

M.C.G Voltmeter
[½]
b. Let ‘G’ be the resistance of the galvanometer coil and ‘Ig’ be the maximum current
which can be passed through the galvanometer coil for full-scale deflection.
c. Let ‘V’ be the potential difference to be measured.
Let ‘Rs’ be the resistance connected in series with the galvanometer coil.
157
Board Answer Paper : March 2017
d. From Ohm’s law,
V = Ig (G + Rs) [½]
V
G + Rs =
Ig
V
 Rs = G [½]
Ig
Knowing V, Ig and G, value of Rs can be determined.
iii. Advantages of potentiometer over voltmeter:
a. The voltmeter is used to measure terminal P.D of cell while potentiometer is used
to measure small terminal P.D as well as e.m.f of the cell.
b. The accuracy of potentiometer can be easily increased by increasing the length of
wire.
c. A small P.D can be measured accurately with the help of potentiometer. The
resistance of voltmeter is high but not infinity to work as an ideal voltmeter.
d. The internal resistance of a cell can be measured with the help of potentiometer.
e. Potential difference across the wire is greater than E1 or E2 or E1 + E2.
(Any two advantages) [2]
iv.
Receiving
Antenna

Output
Amplifier IF Stage Detector Amplifier
Received
signal
Block diagram of a receiver
(Diagram and labelling) (1+1) [2]
v. Solution:
Given: N = 100, A = 15 cm2 = 15  104 m2,
B = 0.03 Wb/m2, C = 15  1010 Nm/degree
To find: Sensitivity (Si)
NAB
Formula: Si = [½]
C
Calculation: From formula,
100  15  104  0.03
Si = [½]
15  1010
 Si = 3  106 div/A
Ans: The sensitivity of a moving coil galvanometer is 3  106 div/A. [1]
vi. Solution:
Given:  = 6t2 + 7t + 1 (in milliweber), t = 2 s
To find: Magnitude of induced e.m.f. (e)
d
Formula: e= (in magnitude) [½]
dt
Calculation: Using formula,
d
e= (6t2 + 7t + 1)
dt
= 12t + 7 [½]
At t = 2 s
|e| = 12  2 + 7
= 31 mV = 31  103 V
Ans: The magnitude of induced e.m.f. is 31  103 V. [1]

158
Physics
vii. Solution:
Given: R = 50 , lX = 40 cm = 0.4 m, lR = 60 cm = 0.6 m
To find: Unknown resistance (X)
lx
Formula: X=R [½]
lR
Calculation: From formula,
50  40
X= [½]
60
100
= 
3
= 33.33 Ω
Ans: The value of unkown resistance is 33.33 Ω. [1]
viii. Solution:
Given: r2 = 2.14  1010 m, n = 2,
v2 = 1.09  106 m/s
To find: Frequency of revolution (2)
Formula: v = r = r(2)
v
 = [½]
2r
Calculation: From formula,
v2 1.09  106
2 = = [½]
2r2 2  3.142  2.14  1010
 2 = 8.11  1014 Hz
Ans: The frequency of revolution of electron in 2nd Bohr orbit is 8.11  1014 Hz. [1]
Q.7. Attempt any THREE:
i. P-N junction diode as half wave rectifier:
P N Id
_ +
P1 +
S1 D
Vo = Id  RL
d.c out put
a.c input

RL

S2
P2 _
Figure (a): Half wave rectifier
P1P2 : Primary coil of step down transformer
S1S2 : Secondary coil of step down transformer
D : P-N junction diode
RL : Load resistance
[1]
Working:
a. A rectifier, which rectifies only one half cycle of each a.c input supply, is called a
half wave rectifier.
b. The circuit diagram is as shown in figure (a).
c. When a.c input is applied to a junction diode, it gets forward biased during one half
cycle and reverse biased during the next opposite half cycle.
d. The a.c supply is fed across the primary coil P1P2 of a step down transformer. The
secondary coil S1S2 of the transformer is connected to the junction diode D and a
load resistance RL. The output d.c. voltage is obtained across the load resistance RL.
The output d.c. voltage is given by Vo = Id  RL.

159
Board Answer Paper : March 2017
e. Suppose that during the first half of the input a.c voltage terminal S1 becomes
positive w.r.t. S2. Then, P region of diode D becomes positive w.r.t. N region. This
makes the junction diode forward biased, thus the conventional current flows in the
direction of the arrow-heads through RL.
f. During the negative half cycle the terminal S2 is at positive potential w.r.t. S1. Thus,
P region of diode D is negative w.r.t. N region. This makes the diode reverse biased.
Thus diode does not allow current through it and no current will flow through RL.
g. During the next half cycle, output is again obtained as the junction diode gets
forward biased. Thus, a half wave rectifier gives pulsating unidirectional and
intermittent d.c output voltage V across the load resistance RL. [1½]
h. Graph of a.c input and d.c output voltage with respect to time is shown in figure below.

+V
a.c input
voltage +ve + ve + ve
o time
ve ve
V
+V
d.c output +ve +ve +ve
voltage o time
V
Input and out put waveforms
for half wave rectifier
[½]
ii. Derivation of induced e.m.f due to self induction:
L

+  [½]
E K
a. Consider a coil connected with battery E, plug key K and inductor L carrying current of
magnitude I as shown in figure.
b. Since magnetic flux linked with the coil is directly proportional to the current.
 I
  = LI .…(1)
where, L = constant called coefficient of self induction or self inductance of the coil,
which depends upon the material of the core, number of turns, shape and area of the
coil.
c. Induced e.m.f in the coil is given by,
d
e=
dt
dI
e=L ….(2)
dt
ve sign in equation (ii) shows that self induced e.m.f opposes the rate of change of
current.
dI dI
 |e| =  L =L
dt dt
dI
 Magnitude of self induced e.m.f is given by, |e| = L
dt
This is required induced e.m.f. [1]
160
Physics
Derivation of induced e.m.f. due to mutual induction:

+
E
 P S G
K
[½]

a. Consider primary coil P and secondary coil S fitted with galvanometer G are placed
very close to each other as shown in figure. The coil P is connected in series with the
source of e.m.f (battery) and key K.
b. When tap key K is pressed current IP passes through the coil P.
Magnetic flux S linked with secondary coil S at any instant is directly proportional
to current IP through primary coil P at that instant.
 S IP
 S = M IP ….(1)
where M is constant called coefficient of mutual induction or mutual inductance of
the coil.
c. e.m.f induced in S at any instant is given by,
dS d
eS =  =  (MIP) …[From equation (1)]
dt dt
dI P
 eS =  M
dt
 Magnitude of induced e.m.f is given by,
MdIP MdIP
eS =  =
dt dt
eS
 M [1]
dI P / dt

iii. Solution:
Given: l = 1 cm = 102 m, E = 300 Vm1, k = 8
To find: Energy contained in the cube (U)
U
Formula: u=
V
Calculation: Volume of marble,
V = (l3) = (102)3 = 106 m3
1
Energy density, u = 0 kE2 [½]
2
1
=  8.85  1012 8  (300)2 [½]
2
= 3.185  106 J/m3 [½]
From formula,
U = u  V = 3.185  106  106 [½]
 U = 3.185  1012 J
Ans: The energy contained in the cube is 3.185  1012 J. [1]

iv. Solution:
Given: r = 0.53 Å = 0.53  1010 m, f = 9  109 MHz = 9  1015 Hz
To find: Angular momentum (L0)
M0
Formula: L0 = [½]
gyromagnetic ratio

161
Board Answer Paper : March 2017
Calculation: M = IA [½]
1
Since, I = e = fe
T
From formula,
M = feA = fer2 [½]
15 19 10 2
= 9  10  1.6  10    (0.53 10 )
23
= 1.6    0.25  10
 M = 1.270  1023 Am2
Using formula,
1.270  1023
L0 = [½]
8.8  1010
 L0 = 0.1443  1033 kgm2/s
Ans: The oribital angular moementum of the electron is 0.1443  1033 kgm2/s. [1]

Q.8. A. Biprism experiment:


This experiment is used in the laboratory to measure the wavelength of monochromatic
light.
Apparatus:
Monochromatic source (sodium lamp), optical bench, lens, micrometer, biprism etc.
Experimental arrangement:

S B
L E
S

A A

i. The apparatus is arranged as shown in the figure.


ii. A long rail called optical bench is fitted with a scale, a slit (S), biprism (B), convex lens
(L) and micrometer eye piece (E).
iii. They are mounted vertically on the optical bench with the help of stands such that
each stand can be adjusted in a vertical, lateral as well as longitudinal direction so
that perfect axial alignment can be achieved.
iv. Each stand can be rotated about vertical axis passing through itself. [½]
Adjustment:
i. Optical bench is leveled with the help of leveling screws. Stands are leveled at same
horizontal plane.
ii. Slit is made narrow, biprism is kept with its refracting edge parallel to the slit.
Procedure:
i. A narrow vertical slit S is illuminated by a source of monochromatic light (S).
The biprism B is placed close to the slit S.
ii. When light from S falls on the refracting edge of the prism then due to refraction,
two virtual images S1 and S2 of the slit S are formed.
iii. These images are in the plane of the slit, quite close to each other and act as two
coherent sources of light.
iv. The waves from these sources interfere with each other and form interference
fringes.
162
Physics
v. The eye piece (E) carrying micrometer is kept at large distance from the biprism. The
interference pattern is observed through (E).

S1 Interfering region
d S E

S2
B
Ray diagram of biprism experiment
S1, S2: Virtual images of the source
B: Biprism,
E: Eye-piece
(Ray diagram) [1]
Determination of X:
i. To measure the band width, a vertical cross wire of the micrometer is adjusted on one
bright band and the micrometer reading x1 is noted.
ii. Similarly the cross wire is adjusted on 2nd, 3rd and 4th bright bands and the
corresponding micrometer readings x2, x3, x4, are noted.
iii. The differences (x2  x1), (x3  x2), (x4  x3), etc, give the band widths. The mean of
these band widths is the width X of fringe. [½]
Determination of the distance between source and eyepiece (D):
It can be read directly from scale attached to the optical bench. [½]
Determination of d:
i. Conjugate foci method is used in the determination of d. A convex lens (L) of
suitable focal length is fixed between the biprism and eyepiece on the optical bench.
ii. The convex lens is moved, towards the slit till the magnified images of S1 and S2 are
seen. The distance between them is measured say d1.
iii. Now, move the convex lens away from the slit till the diminished images of S1 and
S2 are seen. The distance between them is measured, say d2.
iv. The distance between S1 and S2 is given by the formula, d = d1d 2 [1]
Determination of :
D
Since band width is given by X =
d
Xd
  =
D
Substituting value of d,
X d1d 2
= [½]
D
Putting value of X, d1, d2 and D wavelength can be measured exactly.
B. Solution:

AB = width of incident
B wavefront
Rarer i
medium CD = width of refracted
i C wavefront
A r
Denser r
medium r
D
N1 N2

163
Board Answer Paper : March 2017
Given: i = 65, CD = 2AB
To find: Refractive index ()
cosi AB
Formulae: i.  [1]
cos r CD
sin i
ii. = [½]
sin r
Calculation: From formula (i),
cos65 AB

cos r 2AB
0.4226 1

cos r 2
cos r = 0.845
 r = 3216 [1]
From formula (ii),
sin 65
=
sin (3216)
  = 1.697
Ans: The refractive index for the denser medium is 1.697. [½]
OR
A.

E = 0 n=
n=6
E6 =  0.38
E5 = 0.54 Pfund series n = 5
n=4
E4 = 0.85 Brackett series
E3 = 1.51 n=3
Paschen series

E2 = 3.4 n=2
Balmer series
E(in eV)

E1 = 13.6 n=1
Lyman series Quantum number (n) 
(Diagram and labelling) (1+1) [2]
Paschen series:
i. The spectral lines of this series correspond to the transition of an electron from some
higher energy state to 3rd orbit.
ii. For paschen series, p = 3 and n = 4, 5,...
The wave numbers and the wavelengths of the spectral lines constituting the
1 1 1 
Paschen series are given by,  = =R  2  2
 3 n 

164
Physics
iii. Paschen series lies in the infrared region of the spectrum which is invisible and
contains infinite number of lines.
iv. Wavelengths for n = 4 and 5 are 18750 Å and 12820 Å respectively. [1]
Brackett series:
i. The spectral lines of this series corresponds to the transition of an electron from a higher
energy state to the 4th orbit.
ii. For this series, p = 4 and n = 5, 6, 7,...
The wave numbers and the wavelengths of the spectral lines constituting the Brackett
series are given by,
1  1 1 
 = =R  2 2
 4 n 
iii. This series lies in the near infrared region of the spectrum and contains infinite number of
lines. Wavelengths for n = 5 and 6, are 40518 Å and 26253 Å respectively. [1]

B. Solution:
Given: (W0)P = 2.25 eV = 2.25  1.6  1019 J = 3.6  1019 J,
(W0)C = 2.14 eV = 2.14  1.6  1019 J = 3.424  1019 J,
 = 5180 Å = 5.18  107 m
To find: Will the photoelectric effect occur for either of these elements with
 = 5180 Å
Formula: W0 = h0 [½]
Calculation: From formula,
(W0)P = h(0)P
(W0 )P 3.6  1019
 (0)P = =
h 6.63  1034
 (0)P = 5.430  1014 Hz [1]
Similarly,
 W0 C 3.424  1019
(0)C = =
h 6.63  1034
 (0)C = 5.164  1014 Hz [1]
The corresponding frequency is given by,
c 3  108 3
2 = = =  1015
 5.18  107 5.18
 2 = 5.792  1014 Hz
For potassium,
5.792  1014 Hz > 5.430  1014 Hz
i.e., 2 > (0)P
 Photoelectric emission will take place when light of wavelength  is
incident on it.
For caesium,
5. 792  1014 Hz > 5.16  1014 Hz
i.e., 2 > (0)C
 Photoelectric emission will take place when light of wavelength  is
incident on it.
Ans: For  = 5180Å wavelength, both potassium and caesium will exhibit
photoelectric emission. [½]

165
Board Answer Paper : March 2017

BOARD ANSWER PAPER : MARCH 2017


CHEMISTRY
Note: Answer to every question must be written on a new page.

SECTION – I
Q.1. Select and write the most appropriate answer from the given alternatives for each
sub-question:
i. (B) Babbitt metal [1]
ii. (A) Density, viscosity [1]
2+
iii. (D) [Fe(H2O)5NO] [1]
iv. (C) Fusion [1]
v. (B) A3B [1]
As ‘B’ is present at the 8 corners of the cube, number of atoms of ‘B’ in the unit cell
1
=  8 = 1.
8
As ‘A’ atoms are present at the face centres of the 6 faces of the cube, the number of atoms
1
of ‘A’ in the unit cell = 6=3
2
Ratio of atoms A : B = 3 : 1
The formula of the compound is A3B.
vi. (C) 0.50 [1]
The half reaction for the reduction of Al3+ ion is
Al3+ + 3e–  Al
The reaction indicate that 1 mole of Al is produced by the passage of 3 mole electrons. The
charge of 3 mole electrons is 3 faradays.
 3 F of charge produces 1 mole of Al.
 1.5 F of charge produces 0.50 mole of Al.
vii. (B) 0.5 [1]
Rate1 = k[A]x …(i)
Rate2 = k4x[A]x …(ii)
dividing (ii) by (i)
Rate2
= 4x
Rate1
but Rate2 = 2Rate1 (given)
2Rate1
= 4x
Rate1
2 = 4x
x = 1 2 or 0.5

Q.2. Answer any SIX of the following:


i. a. The galvanic cells in which the energy of combustion of fuels is directly converted
into electrical energy are called fuel cells. (Definition) [1]
b. Cell reactions in a fuel cell
Oxidation at anode (): Oxidation of hydrogen gas to water occurs at anode.

2H2(g) + 4OH (aq)  4H2O(l) + 4e [½]

166
Chemistry
Reduction at cathode (+): Reduction of oxygen gas to OH ions occurs at cathode.
O2(g) + 2H2O(l) + 4e  4OH (aq)

[½]

ii. Relationship between half life and rate constant for a first order reaction:
a. For the first order reaction A  products, the integrated rate law is given as
follows:
2.303 [A]
k= log10 0 ....(1) [½]
t [A]t
where, [A]0 = initial concentration of the reactant at t = 0
[A]t = concentration of the reactant at time t.
[A]0
b. At t = t1/2, [A]t =
2
Equation (1) becomes,
2.303 [A]0
k= log10 ....(2) [½]
t1/ 2 [A]0 / 2
2.303
c.  t1/2 = log10 2 ....(3) [½]
k
2.303
=  0.301
k
0.693
t1/2 = ....(4) [½]
k
d. From the above equation (4) for a first order reaction, half-life period is constant, i.e.,
it is independent of initial concentration of the reacting species.

iii. Magnetic separation process:


a. The magnetic separation process is based on the differences in magnetic properties of
the ore components. If either ore or the gangue is attracted by a magnet, then the ore
can be separated from the impurities with the help of magnetic separation method.
b. It requires an electromagnetic separator which consists of a brass or leather belt
moving over two rollers, one of which is magnetic in nature as shown in the figure.
Powdered ore is dropped over the moving belt at one end. At the other end, the
magnetic portion of the ore is attracted by the magnetic roller and falls nearer to the
roller, while the non-magnetic impurities fall separately farther off.
(Explanation) [1]

Pulverized ore

Magnetic roller
Leather belt

Non-magnetic
particles Magnetic particles

Magnetic separation using electromagnetic separator


(Diagram) [1]
167
Board Answer Paper : March 2017
iv. a. Let W2 g of non-volatile solute of molar mass M2 be dissolved in W1 g of solvent of
molar mass M1.
b. The number of moles of solvent, n1 and number of moles of solute n2, in solution are
given as,
W1 W2  mass of the substance 
n1= and n2 =  Number of moles (n) = 
M1 M2  molar mass of the substance 
[½]
c. The mole fraction of solute, x2 is given by,
n2 W2 / M 2
x2 = = ….(1) [½]
n1  n 2 W1 / M1 + W2 / M 2

d. For a solution of two components A1 and A2 with mole fraction x1 and x2


respectively, if the vapour pressure of pure component A1 is p1 and that of
component A2 is p 2 = 0,
The relative lowering of vapour pressure is given by,
p p  p p x

= 1  = 1  2 = x2 .….(2)
p1 p1 p1
e. Combining equations (1) and (2)
p p  p W2 / M 2

= 1  = x2 = …..(3) [½]
p1 p1 W1 / M1 + W2 / M 2

f. For dilute solutions n1 >> n2. Hence, n2 may be neglected in comparison with n1 in
equation (1) and thus equation (3) becomes,
p n W / M2 WM
= 2 = 2 = 2 1 [½]
p1 n1 W1 / M1 W1M 2

v. a. Enthalpy: Enthalpy of a system may be defined as the sum of the internal energy of
the system and the energy that arises due its pressure and volume. [1]
b. First law of thermodynamics can be given as:
U = q + W
If work is done by the system on the surroundings (work of expansion), then W is
negative (W). The negative value of ‘W’ signifies that energy has left the system as
work. Therefore, internal energy of the system decreases. [1]

vi. a. The electronic configuration of 7N is 1s2 2s2 2p3. It has 3 unpaired electrons which
can form 3 covalent bonds, thus forming NX3 molecule. [1]
b. Valence shell of Nitrogen (n = 2) contains only ‘s’ and ‘p’ orbitals. [½]
c. Therefore, due to absence of d-orbitals in valence shell, nitrogen cannot expand its
octet and its maximum covalence is restricted to 4. It does not form compounds in +5
oxidation state like NCl5 and NF5. [½]

vii. Simple cubic unit cell:


a. In a simple cubic lattice, the atoms are located only on the corners of the cube. The
particles touch each other along the edge.
b. Thus, the edge length or side of the cube ‘a’, and the radius of each particle, ‘r’ are
related as a = 2r
c. The volume of the cubic unit cell = a3 = (2r)3 = 8r3 [½]
d. Since a simple cubic unit cell contains only 1 atom, [½]
4 3
The volume of one atom (occupied space) = r
3

168
Chemistry
Volumeof oneatom
e. Packing efficiency =  100 % [½]
Volumeof cubicunit cell
4 / 3r 3 
=  100 =  100
8r 3 6
= 52.36 %  52.4 %
f. The packing efficiency of simple cubic metal crystal is 52.4 %. [½]

viii. a. Electronic configuration of sulphur (Z = 16) is, 16S  1s22s22p63s23p4


or [Ne]3s23p4 [1]
b. Electronic configuration of krypton (Z = 36) is, 36Kr  1s22s22p63s23p63d104s24p6
or [Ar]3d104s24p6 [1]
Q.3. Answer any THREE of the following:
i. Preparation of phosphine using
a. HCl: The reaction of calcium phosphide with dilute HCl gives phosphine.
Ca3P2 + 6HCl  3CaCl2 + 2PH3
Calcium Calcium Phosphine
phosphide chloride

(Reaction + names of reactants and products) [1]


b. H2SO4: The reaction of aluminium phosphide with dilute H2SO4 gives phosphine.
2AlP + 3H2SO4  Al2(SO4)3 + 2PH3
Aluminium Aluminium Phosphine
phosphide sulphate

(Reaction + names of reactants and products) [1]


c. Caustic soda (NaOH)
White phosphorus is heated with concentrated NaOH solution in an inert atmosphere
of CO2. It is the laboratory method.
P4 + 3NaOH + 3H2O  PH3 + 3NaH2PO2
White Sodium hydroxide Phosphine Sodium
phosphorus (hot) hypophosphite

(Reaction + names of reactants and products) [1]


ii. Given: Cell constant (b) = 0.367 cm–1
Molar concentration (C) = 0.05 M
Resistance (R) = 31.6 
To find: Molar conductivity ()
Formulae: a. Cell constant, (b) = k  R
1000 k
b. Molar conductivity () =
C
Calculation: From formula (a),
Cell constant, b = k  R [½]
0.367 = k  31.6
0.367
k= = 11.61  10–3 –1 cm–1 [½]
31.6
From formula (b),
1000 k
Molar conductivity (m) = [½]
C
1000  11.61  103
= [½]
0.05
= 232.2 –1 cm2 mol–1 [½ + ½]
169
Board Answer Paper : March 2017
iii. Given: The standard enthalpy of combustion of C2H5OH(l) i.e. c H C2H5OH(l)
= –1368 kJ
The standard enthalpy of combustion of ethene i.e. c H C2H4(g)
= –1410 kJ
To find: H for the enthalpy of formation of liquid C2H5OH
Calculation: Given equations are,
C2H5OH(l) + 3O2(g)  2CO2(g) + 3H2O(l) ; cH = –1368 kJ ...(1)
C2H4(g) + 3O2(g)  2CO2(g) + 2H2O(l) ; cH = –1410 kJ ...(2)
The required equation is,
C2H4(g) + H2O(l)  C2H5OH(l), [½]
To get required equation, reverse equation (1) and add to equation (2). [½]
2CO 2(g) + 3H2O(l)  C2H5OH(l) + 3O 2(g) cH = +1368 kJ

C2H4(g) + 3O 2(g)  2CO 2(g) + 2H2O(l) cH = –1410 kJ



C2H4(g) + H2O(l)  C2H5OH(l) H = +1368 kJ – 1410 kJ [½]
= – 42 kJ. [½]
 The calculated H = – 42 kJ is not the enthalpy of formation of liquid
ethanol because the reaction does not involve the formation of liquid ethanol
from its constituent elements. [1]
iv. Given: Frequency factor (A) = 4  1013 sec–1
Energy of activation (Ea) = 98.6 kJ mol–1
Half life period (t1/2) = 10 minutes = 10  60 = 600 sec.
To find: Temperature (T)
0.693
Formulae: a. k=
t1/ 2
Ea
b. log10k = log10A –
2.303RT
Calculation: From formula (a)
0.693
k= [½]
t1/ 2
0.693
k= = 11.55  10–4 sec–1 [½]
600
From formula (b),
Ea
log10 k = log10A – [½]
2.303 RT
98.6 103
log10 11.55  10–4 = log10 4  1013 –
2.3038.314  T
98.6 103
–2.937 = 13.602 – [½]
19.147  T
5149.632
–2.937 = 13.602 –
T
5149.632
= 13.602 + 2.937
T
5149.632
= 16.539 [½]
T
5149.632
T= = 311.36 K
16.539
T = 311.36 K or 38.36C [½]

170
Chemistry
Q.4. Answer any ONE of the following:
i. a. Faraday’s First Law of Electrolysis: It states that “the amount of substance that
undergoes oxidation or reduction at each electrode during electrolysis is directly
proportional to the amount of electricity that passes through the cell.” [1]
b. 1. Uses of sulphuric acid:
i. Sulphuric acid is used in the manufacture of fertilizers.
eg. Ammonium sulphate, superphosphate.
ii. It is used in petroleum refining.
iii. It is used in manufacture of pigments, paints and dyestuff intermediates.
iv. It is used in detergent industry.
v. It is used in storage batteries.
vi. It is used in the manufacture of explosives. eg. T.N.T., nitroglycerine, green
cotton, etc.
vii. It is a laboratory reagent.
viii. Sulphuric acid is used in the preparation of important chemicals like HNO3,
HCl, H3PO4, Na2CO3, sulphates, alums, ether, etc.
ix. It is a dehydrating agent and used for drying wet gases which do not react with
acid.
x. It is used as an oxidizing agent.
xi. Sulphuric acid is also used as a pickling agent. Pickling is an industrial process
for removing layers of basic oxides from metals like Fe and Cu before
electroplating, enamelling, galvanizing and soldering.
(Any two uses) [½  2] [1]
2. Uses of chlorine:
i. Large quantities of chlorine are used industrially for bleaching wood pulp
(required for the manufacture of paper and rayon), bleaching cotton and
textiles.
ii. It is used in the extraction of gold and platinum.
iii. It is used for the manufacture of dyes, drugs and organic compounds such as CCl4,
CHCl3, refrigerants, etc.
iv. It is used in sterilising drinking water.
v. It is used in preparation of poisonous gases such as phosgene (COCl2), tear gas
(CCl3NO2), mustard gas (Cl.C2H4  S  C2H4Cl).
vi. It is used in the manufacture of DDT and BHC which are important
insecticide.
vii. It is used in the manufacture of vinyl chloride which is starting material for
polyvinyl chloride plastics.
viii. It is used in the manufacture of bleaching powder, aluminium chloride,
hydrogen chloride, hypochlorites, chlorates, perchlorates, etc., which are
important industrial compounds.
(Any two uses) [½  2] [1]
c.
Property Crystalline solids Amorphous solids
1. Definition A crystalline solid is a The substances that appear
homogeneous solid in which like solids but do not have
the constituent particles well developed perfectly
(atoms, ions or molecules) ordered crystalline structure
are arranged in a definite are called amorphous (no
repeating pattern. form) solids.
2. Shape They have definite They have irregular shape and
characteristic geometrical shape lack characteristic geometrical
due to the orderly regular long shape due to the short range
range arrangement of orderly arrangement of
constituent particles. constituent particles.

171
Board Answer Paper : March 2017
3. Melting point They have sharp and They do not have sharp
characteristic melting point. melting point. They gradually
soften over a range of
temperature.
4. Cleavage When cut with a sharp edged When cut with a sharp edged
property tool, they split into two pieces tool, they cut into two pieces
and the newly generated with irregular surfaces.
surfaces are plain and smooth.
5. Heat of fusion They have a definite and They do not have definite heat
characteristic heat of fusion. of fusion.
6. Anisotropy They are anisotropic, i.e., have They are isotropic, i.e., have
different physical properties in same physical properties in all
different direction. directions.
7. Nature They are true solids. They are pseudo solids or
super cooled liquids.
8. Order in arrangement They have long range order. They have only short range
of constituent particles order.
eg. Copper, silver, iron, zinc Glass, rubber, plastics, etc.
sulphide, common salt,
potassium nitrate, etc.
(Any four distinguishing points) [½  4] [2]
3
d. Given: Volume (V) = 1dm = 1L
Mass of substance (W2) = 1.8  10–3 kg = 1.8 g
Osmotic pressure () = 0.52 atm
R = 8.314 J K–1 mol–1 or R = 0.082L atm K–1 mol–1
Temperature (T) = 280 K
To find: Molar mass (M2)
W2 RT
Formula: =
M 2V
Calculation: From formula,
W2 RT
M2 = [½]
V
1.8 0.082  280
M2 = [½]
0.52 1
M2 = 79.47 g mol–1 [1]
ii. a. 1. Leaching:Leaching is the process of extracting a soluble material from an
insoluble solid by dissolving out in a suitable solvent. [1]
2. Metallurgy: The process of extraction of a metal in a pure state from its ore is
called metallurgy. [1]
3. Anisotropy: The ability of crystalline solids to change values of physical
properties when measured in different directions is called anisotropy. [1]
b. 1. Consider ‘n’ moles of an ideal gas placed in a cylinder fitted with weightless,
frictionless piston at a constant temperature ‘T’. If the gas expands
isothermally and reversibly from initial volume V1 to final volume V2 and if
the expansion is by a small volume ‘dV’, then opposing pressure ‘P’ decreases
by a small amount ‘dP’ producing small work done ‘dW’. Now, the pressure
(pex) will be ‘P – dP’.
2. This work done is given as:
Work done (dW) = pressure  change in volume
= (P  dP)  dV
= (PdV  dPdV)
 Both dP and dV are small, dP.dV is also very small and is neglected.
 The above equation becomes dW =  PdV ….(1) [½]
172
Chemistry
3. When the expansion of a gas is carried out reversibly, there will be series of such
‘PdV’ terms. As the pressure decreases continuously, volume gradually increases
from initial volume ‘V1’ to final volume ‘V2’.
Thus, in order to obtain maximum work (Wmax), integrate equation (1)
Wmax V2

between the limits V1 to V2 as 


0
dW    PdV
V1
….(2) [½]

4. The general ideal gas equation for n mole is, PV = nRT


nRT
or P  ....(3)
V
Substituting (3) in equation (2),
Wmax V2
nRT

0
dW   
V1 V
dV

5. Since the number of moles is constant, R is a gas constant and the process is
isothermal,
Therefore, T is constant. Hence, nRT = constant.
Wmax V2
1
 
0
dW  nRT 
V1
V
dV [½]

 [W]0max  nRT log e [V]VV12


 dx  x 

 [Wmax  0]  nRT[log e (V2 )  log e (V1 )] ……  1 

  dx  log e x 
 x 
V2
 Wmax   nRT log e
V1
V2
6. Wmax =  2.303 nRT log10
V1
 log e x  2.303 log10 x  [½]

P1
OR Wmax = –2.303 nRT log10 [ According to Boyle’s law, P1V1 = P2V2]
P2
Above equations are expression for work done in reversible isothermal process.
c. Given: Kb for benzene = 2.53 K kg mol–1
Mass of solute = W2 = 1.8 g = 1.8  10–3 kg
Mass of solvent = W1 = 90 g = 90  10–3 kg
Boiling point of solution = Tb = 354.11 K
Boiling point of pure solvent = Tb = 353.23 K
To Find: Molar mass of solute (M2)
K b  W2
Formula: Tb =
M 2  W1

Calculation: Elevation in boiling point (Tb) = Tb  Tb [½]


Tb = 354.11 – 353.23
Tb = 0.88 K [½]
From formula,
K b  W2
M2 =
Tb  W1
2.531.8103 4.554
M2 = 3
= [½]
0.88 90 10 79.2
= 0.0575 kg mol–1
= 57.5 g mol–1 [½]
173
Board Answer Paper : March 2017

SECTION – II

Q.5. Select and write the most appropriate answer from the given alternatives for each
sub-question:
i. (D) an isocyanide [1]
ii. (C) n-butane [1]
CH3 – CH2 – Br 
 CH2 = CH2 
alc.KOH

 CH3 – CH2 – Br HBr

Ethyl Ethene Ethyl


bromide (B) bromide
(C)


Na /ether
Wurtz
 CH3 – CH2 – CH2 – CH3
synthesis
n-butane
iii. (B) cancer (D) [1]
iv. (C) SO2 [1]
Sulphur dioxide gas is oxidized to sulphuric acid when passed through acidified potassium
dichromate solution. The colour of the solution changes from orange to green because
potassium dichromate is reduced to chromic sulphate.
v. (B) ethanol [1]
Ethanol is used in thermometers as it has low freezing point.
vi. (D) Vitamin A [1]
vii. (C) Ketone [1]
Q.6. Answer any SIX of the following:
i. When an excess of ethyl alcohol is distilled with concentrated H2SO4 at 413 K
(140 C), diethyl ether is formed.
This reaction takes place in two steps:
Step-I: Formation of ethyl hydrogen sulphate: When excess of ethyl alcohol is heated
with concentrated H2SO4, ethyl hydrogen sulphate is obtained.
C2H5  OH + H  O  SO3H  C2H5  O  SO3H + H2O

Ethyl alcohol Conc.sulphuric Ethyl hydrogen


acid sulphate
Step-II: Formation of diethyl ether: Ethyl hydrogen sulphate then reacts with excess of
ethyl alcohol at 413 K and forms diethyl ether.
C2H5  O  SO3H + OH  C2H5 
413K
 C2H5  O  C2H5 + H2SO4
Ethyl hydrogen Ethyl Diethyl ether
sulphate alcohol
(Explanation + Reaction) [2]
ii. Hoffmann bromamide degradation:
a. Primary amine can be prepared by reaction of amide with bromine and aqueous or
alcoholic sodium hydroxide. This reaction is known as Hoffmann bromamide
degradation.
b. It involves molecular rearrangement.
c. An alkyl or aryl group migrates from the carbonyl carbon to the adjacent nitrogen
atom.
d. This reaction is useful for decreasing the length of carbon chain by one carbon atom.
O (Explanation) [1]

R  C NH2 + Br2 + 4NaOH  R  NH2 + Na2CO3 + 2NaBr + 2H2O


Amide (aqueous or 1 Amine
alcoholic)
[1]
174
Chemistry
iii. Preparation of ethanoic acid from dry ice:
When solution of methyl magnesium iodide (Grignard reagent) in dry ether is added to
solid carbon dioxide (dry ice), it gives an adduct (magnesium salt of carboxylic acid),
which on acid hydrolysis gives ethanoic acid or acetic acid.
eg.

 CH3  C  OMgI   CH3  C  OH + Mg(OH)I
H3O
O = C = O + CH3  Mg  I  
dry ether

Dry ice Methyl


(Solid carbon O O
magnesium Acetic acid
dioxide) Adduct
iodide (Ethanoic acid)

(Explanation + Reaction) [2]


iv. a. Butylated hydroxy anisole (BHA)
Molecular formula: C11H16O2
OH
C(CH3)3

OCH3
Butylated hydroxy anisole (BHA)

(Molecular formula + Structural formula) [½  2] [1]


b. Butylated hydroxy toluene (BHT)
Molecular formula: C15H24O
OH
(CH3)3C C(CH3)3

CH3
Butylated hydroxy toluene (BHT)

(Molecular formula + Structural formula) [½  2] [1]


v. Preparation of glucose from cane sugar:
Glucose can be prepared in the laboratory by boiling sucrose (cane sugar) with dilute
hydrochloric acid or sulphuric acid for about two hours. This hydrolyzes sucrose to glucose and
fructose. In order to separate glucose from fructose, alcohol is added during cooling.
Glucose is almost insoluble in alcohol. It crystallizes out first, while fructose is more
soluble. It remains in the solution. The solution is filtered to obtain the crystals of glucose.
(Explanation) [1]

C12H22O11 + H2O 


dil.HClor H 2SO 4

 C6H12O6 + C6H12O6
Sucrose Water Glucose Fructose
(Reaction + names of reactants, products and reagents) [1]
vi. Factors which are related to the colour of transition metal ions are as follows:
a. The presence of unpaired electrons in d-orbitals.
b. d-d transitions of electrons due to absorption of radiation in the visible region.
c. Nature of groups (anions or ligands) linked to the metal ion in the compound or a
complex.
d. Type of hybridisation in metal ion in the complex.
e. Geometry of the complex containing transition metal ion.
(Any four points) [½  4] [2]
175
Board Answer Paper : March 2017
vii. a. Homopolymers:
Polymers whose repeating structural units are derived from only one type of
monomer units are called homopolymers.
eg. Polythene (prepared by using only one type of monomer, i.e., ethene), PVC
(monomer: vinyl chloride), Polystyrene (monomer: styrene), etc.
(Definition + Any one example) [1]
b. Elastomers:
The polymers that have elastic character like that of rubber are called elastomers.
eg. Neoprene, vulcanized rubber, etc.
(Definition + Any one example) [1]
viii. a. Racemic mixture or Racemates: A mixture of equimolar amounts of dextro and
laevo rotatory forms of same optically active substance is called racemic mixture or
racemates.
eg. dl-lactic acid or ()-lactic acid. [1]
CH3
b.
CH3  CH2  CH  CHO

IUPAC name is 2-Methylbutanal [1]


Q.7. Answer any THREE of the following:
i. a. Effective atomic number (EAN): Effective atomic number (EAN) is the total
number of electrons around the central metal ion present in a complex. It is the sum
of the electrons of metal ion and the electrons donated by the ligands.
EAN = Z  X + Y

where, Z = Atomic number of the metal


X = Number of electrons lost during oxidation of metal to metal ion
Y = Number of electrons donated by the ligands. (Definition + Formula) [1]
b.
Atomic
Number of Number of
number of EAN
Complex electrons lost electrons from
metal atom (Z – X + Y)
by metal (X) ligands (Y)
(Z)
1. K4Fe(CN)6 26 2 6  2 = 12 26 – 2 + 12 = 36 [1]
2. Cr(CO)6 24 0 6  2 = 12 24 – 0 + 12 = 36 [1]

ii. a. Oxidation states of Fe:


+2, +3, +4, +5, +6 [1]
b. Electronic configuration of Mn2+:1s2 2s2 2p6 3s2 3p6 3d5 4s0
Due to the presence of half filled ‘d’ orbital, the +2 oxidation state of manganese is
more stable.
(Electronic configuration + Explanation) [2]
iii. Aldol condensation (Self/Auto condensation):
a. Two molecules of an aldehyde or ketone (containing -H-atom) in the presence of
dilute basic solution of NaOH, KOH, Ba(OH)2, K2CO3 or Na2CO3 or dil.HCl
undergo addition reaction to give corresponding -hydroxyl aldehyde (aldol) or
-hydroxyl ketone (ketol). The reaction is called aldol condensation/self oxidation.
b. This is a nucleophilic addition of aldehydes (or ketones) in which -carbon of the
first molecule gets attached to carbonyl carbon of second molecule forming a new
C  C bond. The reaction is reversible but the equilibrium is established between
reactants and products.
c. Due to steric factors, yield of ketol is very poor as compared to that of aldol.
176
Chemistry
d. On heating, the aldol or ketol rapidly undergoes dehydration (in the presence of acid
or base) to yield corresponding -unsaturated aldehyde or ketone. This product is
obtained due to stabilization of newly formed C  C double bond via conjugation
with carbonyl group. (Explanation) [2]
eg. Aldehydes: When acetaldehyde is warmed with dil NaOH or Na2CO3 or K2CO3 or
dil. HCl, it gives acetaldol, (3-Hydroxybutanal), which on heating with acid or base, loses
a water molecule and gives crotonaldehyde.

H H H H H H
293 303K, dil Na 2 CO3
CH3  C + 
H  C  C = O  
 CH3  C  C  C = O
|| | | |
O H B OH H

Acetaldehyde Acetaldehyde -Hydroxybutyraldehyde


(acetaldol)
(First molecule) (Second molecule) (3-Hydroxybutanal) [½]

H H H H H H
H / 
CH3  C  C  C = O 
dehydration
 CH3  C = C  C = O + H2O
Crotonaldehyde
[½]
HO H (But-2-enal)
Acetaldol

iv. a. Nucleic acids: Nucleic acids are biologically important polymers present in all
living cells, which direct the synthesis of proteins and are responsible for the transfer
of genetic information i.e., the hereditary characteristics. [1]
b. Complex lipids: The esters of long chain fatty acids which can be easily hydrolyzed
are called complex lipids. [1]
c. Functions of lipids:
1. Oils and fats can supply food energy in plants and animals.
2. Glycolipids are present in bacterial cell wall.
3. In plants, the principal lipid constituents of chloroplasts are glycolipids.
4. Cerebrosides are present in plasma membranes of neural tissues. They are
animal glycolipids. They are abundant in myelin sheath of neurons.
5. In water, phospholipids form membrane like structure.
6. The major components of cell membranes are phospholipids and sterols such
as cholesterol.
7. Waxes are water repelling solids. They provide waterproofing for body
surfaces. They form protective coatings on leaves, fruits, berries, animal fur
and feather of birds.
8. Adrenal hormones, sex hormones and bile acids are steroids. Lipoproteins are
the combination of lipids with proteins. They are found in cell membranes.
9. Bile acids (such as cholic acid) aid digestion of fat in intestine.
10. Prostaglandins have a wide range of biological effects.
11. Vitamin A, E, K and phytol are terpenes. They are present in essential oils
such as menthol and camphor. The glands of certain aromatic plants secrete
essential oils which contains terpenes as the main constituents.
(Any two functions) [½  2] [1]

177
Board Answer Paper : March 2017
Q.8. Answer any ONE of the following:
i. a. Action of nitrous acid i.e., mixture of NaNO2 and dil. HCl on:
1. Ethylamine:
Ethylamine reacts with nitrous acid in cold condition to give ethyl alcohol and
nitrogen gas.
NaNO 2  dil.HCl
C2H5  NH2 + HO  N = O  273 278K
C2H5OH + H2O + N2
Ethylamine Ethyl alcohol [1]
2. Aniline:
Aniline reacts with nitrous acid in cold condition to give benzene diazonium salt.
NH2 N 2 Cl

NaNO 2  dil.HCl
+ HO – N = O 
273 278 K
+ NaCl + 2H2O
Aniline Benzenediazonium chloride [1]
3. Diethylamine:
Diethylamine reacts with nitrous acid in cold condition to give
N-nitrosodiethylamine.
NaNO 2  dil.HCl
(C2H5)2NH + HO – N = O  273 278 K
(C2H5)2N – N = O + H2O
Diethylamine N–Nitrosodiethylamine [1]
b. Preparation of Nylon-6,6:
1. Nylon-6,6 is prepared by the condensation polymerization of
hexamethylenediamine with adipic acid under high pressure and at high
temperature.
2. Equimolar aqueous solutions of both monomers are mixed. Nylon salt is
formed by neutralization.
+ +  
H2N ( CH2 ) NH2 + HOOC ( CH2 )4 COOH   H3N ( CH2 )6NH3OOC ( CH2 )4 COO
6
Hexamethylenediamine Adipic acid Hexamethylene diammonium adipate
(Hexane-1,6-diamine) (Hexane-1,6-dioic acid) (Nylon salt)
(Formation of Nylon salt) [1]
3. The step growth condensation of the nylon salt with elimination of water molecule
gives nylon-6,6 polymer. O O
+ +   
nH3N( CH2) NH3OOC( CH2) COO   H O HN ( CH2 )6 NH  C ( CH2 )4 C 
6 4 2
Hexamethylene diammonium Repeating unit
adipate
(Nylon salt) O O

[ HN ( CH2 ) NH  C ( CH2 )4 C ]n
6
Nylon-6,6
(Formation of polymer) [1]
c. The chemical substances which neutralize excess acid in the gastric juices and give
relief from acid indigestion, acidity, heart burns and gastric ulcers are called
antacids.
eg. Magnesium hydroxide, magnesium carbonate, magnesium trisilicate, aluminium
hydroxide, sodium bicarbonate, etc.
(Definition + Any one example) [1]
d. Side effects of tranquilizers are:
1. It produces drowsiness.
2. It is addictive.
3. It causes headache, fatigue, dizziness and muscle weakness.
4. It also causes visual disturbances (blurring of vision), discomfort and weight gain.
(Any two side effects) [½  2] [1]
178
Chemistry
ii. a.
1. Alkaline hydrolysis of tert-butyl bromide can be explained by unimolecular
nucleophilic substitution S   mechanism.
N1

Reaction:
CH3 CH3
CH3  C  Br + OH  CH3  C  OH + Br
(Nucleophile) (Bromide ion) [½]
CH3 CH3
(tertButyl bromide) (tertButyl alcohol)
2. Kinetics:
Rate  [(CH3)3C  Br]
Rate = k[(CH3)3C  Br]
Hence, it is a first order reaction. [½]
3. Unimolecular nucleophilic substitution (SN1 ) is a two step reaction process. The
first step is a slow step, while the second one is a fast step. The mechanism can be
written as follows:
i. Formation of carbonium ion:
a. The C – Br bond in tertiary butyl bromide slowly dissociates to form bromide ion
(Br) and tertiary butyl carbonium ion [+C(CH3)3].
b. This is a slow process and hence, it is a rate determining step (R.D.S.).
CH3
H3C CH3
* + 
C Slow step

 
 H3C  C Br  + Br
R.D.S. C+
H3C Br
H3C 120
CH3 H3C CH3
tert-Butyl bromide tert-Butyl carbonium ion
Transition state I /carbocation
[½]
ii. Formation of the product:
a. The second step is the attack of OH leading to the C  OH bond formation.
b. It is a fast process.
CH3

Back side
attack C*
CH3 [½]
HO
CH3 CH3
H3C
+  tert-Butyl alcohol
C+ Fast H3C  C OH [Inversion of

 configuration (50%)]
120
H3C CH3 H3C

HO CH3
tert-Butyl carbonium ion
/carbocation Transition state II
Front side
attack C*
H3C OH
CH3
tert-Butyl alcohol
[Retention of
configuration (50%)] [½]

179
Board Answer Paper : March 2017

iii. Stereochemistry of S N1:


The intermediate formed after the bond has broken is known as carbocation (or carbonium
ion). In the carbocation, the carbon atom is sp2 hybridized and therefore, it has planar
geometry. The nucleophile can attack it either from front side or from back side with nearly
same probability. The products obtained in this case due to attack from either side are the
same, as the starting compound itself is optically inactive.
CH3 
HO CH3 OH CH3

+
C*     C*
Back side Front side
attack C attack

CH3 H3C CH3


HO H3C OH
CH3 CH3
tert-Butyl alcohol tert-Butyl alcohol
[Inversion of [Retention of
configuration (50%)] configuration (50%)] [½]
iv. Energy profile diagram:

First step where,


T.S.I Second step Reactants = C(CH3)3Br + OH
T.S.II
E1 = Activation energy for T.S.I.
Potential energy

E1 E2
E2 = Activation energy for T.S. II
Carbocation H = Heat of reaction and
product = C(CH3)3OH
Reactants
H Carbocation = CH3
+
Products C
H3C CH3
Reaction co-ordinate 
Energy profile diagram for S N1 mechanism (Diagram) [1]

b. Carbolic acid: Carbolic acid or Phenol is an organic aromatic, hydroxyl compound,


in which the hydroxyl (OH) group is directly attached to the aromatic nucleus (i.e.,
benzene ring). [1]
c. Preparation of carbolic acid from benzene sulphonic acid:
1. When benzene sulphonic acid is neutralised by aqueous sodium hydroxide
(NaOH), sodium benzene sulphonate is obtained.
 +
SO3H SO3Na

+ NaOH(aq.) 
 Neutralisation
+ H2O

Benzene Sodium benzene


sulphonic acid sulphonate [½]
2. Dry sodium benzene sulphonate when fused with excess of sodium hydroxide
at 573 K, sodium phenoxide is obtained along with sodium sulphite (Na2SO3).
 +  +
SO3Na ONa

+ 2NaOH   fused


573K + Na2SO3 + H2O
Sodium
Sodium benzene (Excess) Sodium
sulphite
sulphonate phenoxide
(Reaction) [½]
(Mentioning temperature) [½]

180
Chemistry
3. Sodium phenoxide when hydrolysed by heating with dilute sulphuric acid, phenol is
obtained.
 +
ONa OH

2 + H2SO4 
Hydrolysis
2 + Na2SO4
(dil.)
Sodium Phenol or
phenoxide carbolic acid

OR
When a current of carbon dioxide is passed through aqueous sodium phenoxide, phenol
is obtained as product.
 +
ONa OH

+ H2O + CO2  + NaHCO3

Sodium Phenol or
phenoxide carbolic acid
(Any one reaction) [½]

181
Board Answer Paper : March 2017

BOARD ANSWER PAPER : MARCH 2017


MATHEMATICS AND STATISTICS

Note: Answer to every question must be written on a new page.

SECTION – I

Q.1. (A) Select and write the appropriate answer from the given alternatives in each of the
following sub-questions:
i. (C)
Let a , b , c be the position vectors of points A, B, C respectively.
 a = 2 î + ĵ + k̂ , b =  ĵ + 4 k̂ , c = k î + 3 ĵ  2 k̂
 AB = b  a
= ( ĵ + 4 k̂ )  (2 î + ĵ + k̂ )
=  2 î  2 ĵ + 3 k̂
and AC = c  a
= (k î + 3 ĵ  2 k̂ )  (2 î + ĵ + k̂ )
= (k 2) î + 2 ĵ  3 k̂
Since points A, B and C are collinear.
 AB and AC are collinear.
 AB  AC = 0
ˆi ˆj kˆ
 -2 -2 3 = 0
k - 2 2 -3

 î (0)  ĵ (3k) + k̂ ( 4 + 2k  4) = 0
 3 ĵ + ( 8 + 2k) k̂ = 0 ⋅î + 0  ĵ + 0  k̂
By equality of vectors, we get
 8 + 2k = 0
 k=4 [2]
ii. (A)
a b  1 1  d b 
If A =   , then A = ad  bc   c
 c d   a 
1  2 5
 A= [2]
13  3 1

iii. (B)
a+b+c 13  14  15
s= = = 21
2 2
A
sin   =
 s  b  s  c  =  21  14  21  15 = 1
[2]
2 bc 14  15 5

(B) Attempt any THREE of the following:


i. Let a = 2iˆ  3jˆ  4kˆ
b = 5iˆ  7ˆj  5kˆ
c = 4iˆ  5jˆ  2kˆ
be the coterminus edges of the parallelopiped.
182
Mathematics and Statistics
volume of the parallelopiped =  a b c 

2 3 4
V= 5 7 5 [1]
4 5 2

= 2  (– 39) –3  (– 30) – 4  (– 3)
= – 78 + 90 +12
= 24
 V = 24 cubic units [1]

ii. L.H.S. = a (b cos c – c cos B)


  a 2  b2  c2   a 2  c2  b2  
= a b    c   [1]
  2ab   2ac 
a 2
L.H.S. = [a + b2 – c2 – a2 – c2 + b2]
2a
1
= [2b2 – 2c2]
2
= b2 – c2 [1]
= R.H.S.

iii. The coordinates of the points A and B are (0, b, c) and (a, 0, c) respectively.
 OA  bjˆ  ckˆ , OB  aiˆ  ckˆ

The plane OAB passess through O  0  and is perpendicular to n  OA  OB


ˆi ˆj kˆ
 n  OA  OB  0 b c [1]
a 0 c

= bciˆ  acjˆ  abkˆ


 The equation of the plane OAB is
 r  0  n  0
 rn  0
  
r̂  bciˆ  acjˆ  abkˆ  0 [1]

iv. The equation of the line passing through A(x1, y1, z1) and B(x2, y2, z2) is
x  x1 y  y1 z  z1
  [1]
x2  x1 y2  y1 z 2  z1
 The quation of the line passing through (3, 4, –7) and (6, –1, 1) is
x 3 y4 z7
 
63 1  4 1  7
x 3 y4 z7
   [1]
3 5 8

v. Let p :  n  N, n2 + n is an even number.


q :  n  N, n2 – n is an odd number.
 Required symbolic form is p  q [1]
Now statement p is always True.
Statement q is always False.
 Truth value of p  q is T  F  F [1]
183
Board Answer Paper : March 2017
Q.2. (A) Attempt any TWO of the following:
i.
1 2 3 4 5 6
p q r pq pr (p  q)  (p  r)
T T T T T T
T T F T F T
T F T F T T
T F F F F F
F T T F F F
F T F F F F
F F T F F F
F F F F F F
In the above truth table, the entries in the last column are a combination of T and F.
 (p  q)  (p  r) is a contingency. [1]
[1 mark each for column 4 or 5 and column 6]
ii. Equations of the lines are
x 1 y  2 z  3
 
2 3 4
x 2 y 4 z 5
 
3 4 5
Here x1 = 1, y1 = 2, z1 = 3
x2 = 2, y2 = 4, z2 = 5
a1 = 2, b1 = 3, c1 = 4
a2 = 3, b2 = 4, c2 = 5
Shortest distance between the lines is
x2  x1 y2  y1 z 2  z1
a1 b1 c1
a2 b2 c2
= [1]
 b1c2  b 2c1    c1a 2  c2a1    a1b 2  a 2 b1 
2 2 2

1 2 2
2 3 4
3 4 5
= [1]
15  16   12  10    8  9 
2 2 2

115  16   2 10  12   2  8  9 
=
1 4 1
1  4  2
=
6
1
= units [1]
6

iii. sin 2x + sin 4x + sin 6x = 0


 (sin 2x + sin 6x) + sin 4x = 0
 6x  2x   6x  2x 
 2sin   cos   + sin 4x = 0
 2   2 
 2sin 4x cos 2x + sin 4x = 0 [1]
 sin 4x (2cos 2x + 1) = 0
1
 sin 4x = 0 or cos 2x =  [1]
2
   2
 sin 4x = 0 or cos 2x =  cos   = cos     = cos  
3  3  3 

184
Mathematics and Statistics
Since, sin  = 0 implies  = n and cos  = cos  implies  = 2n   , n  Z.
2
 4x = n or 2x = 2m 
3
n 
 general solution is x = or x = m  , where n, m  Z. [1]
4 3
(B) Attempt any TWO of the following:
i. Given equations are
x–y+z=4
2x + y – 3z = 0
x+y+z=2
Matrix form of the given system of equations is
1 1 1   x   4 
 2 1 3  y    0  [1]
     
1 1 1   z   2 
1 1 1  x 4
Where, A =  2 1 3 , X =  y , B =
 
0 
 
1 1 1   z   2 
Applying R2  R2 – 2R1, R3  R3 – R1,
1 1 1   x   4 
0 3 5  y    8
     
0 2 0   z   2 
2
Applying R3  R3 – R 2 ,
3
   
1 1 1   x   4 
    
 0 3 5  y    8  [1]
 10   z  10 
0 0   
 3 3
Hence, The original matrix A is reduced to an upper triangular matrix
 By equality of matrices, we get
 x–y+z=4 …(i)
3y – 5z = – 8 …(ii)
10 10
z …(iii) [1]
3 3
i.e., z = 1
Substituting z = 1 in equation (ii), we get
3y – 5 = – 8
 3y = – 3
 y=–1
Substituting y = –1 and z = 1 in equation (i), we get
x+1+1=4
 x=2
 x = 2, y = – 1, z = 1 is the required solution [1]
ii. The given combined equation of lines is ax2 + 2hxy + by2 = 0
Let m1 and m2 be the slopes of the lines represented by ax2 + 2hxy + by2 = 0.
2h
 m1 + m2 =  and m1m2 = a , b  0
b b

If a = 1, then m1m2 = 1.


b
 lines are perpendicular. [1]
So we assume that a  1
b
185
Board Answer Paper : March 2017
Y y = m2x

y = m1x

X X
O

Y
Now, (m1  m2)2 = (m1 + m2)2  4m1.m2
2
4a 4h 2 4a
=    
2h
= 2 
 b  b b b
4h 2  4ab 4  h  ab 
2

 (m1  m2)2 = 2
=
b b2
Taking square root on both the sides, we get
2 h 2  ab
m1  m2 =  [1]
b
Let  be the acute angle between the lines.
2 h 2  ab
m1  m 2 
 tan  = = b , a  1
1  m1.m 2 1
a b
b
2 h 2  ab
 tan  = ,a+b0 [1]
ab
Lines are coincident (parallel), if and only if m1 = m2
i.e., if m1  m2 = 0
2 h 2  ab
i.e., if  =0
b
i.e., if h2  ab = 0
i.e., if h2 = ab
 Lines are coincident if and only if h2 = ab. [1]
iii. Let p, q, r be the position vectors of vertices P, Q, R respectively of PQR.
 p = 4 ĵ , q = 3 k̂ and r = 4 ĵ + 3 k̂ [1]
Now, PQ = q  p = 4 ĵ + 3 k̂
PR = r  p = 3 k̂
QR = r  q = 4 ĵ
Let x = QR , y = PR and z = PQ
 x = 4, y = 3, and z = 5. [1]
If H (h ) is the incentre of PQR, then
x p + y q + zr
h = [1]
x+ y+z
(4)(4ˆj) + (3)(3k)
ˆ + (5)(4ˆj + 3k)
ˆ
 h =
4+3+5
16ˆj + 9kˆ + 20jˆ + 15kˆ 36jˆ + 24kˆ
= =
12 12
 h = 3 ĵ + 2 k̂
 H  (0, 3, 2)
 The incentre of PQR is (0, 3, 2) [1]
186
Mathematics and Statistics
Q.3. (A) Attempt any TWO of the following:
i. Let p : The switch S1 is closed.
q : The switch S2 is closed.
 ~p : The switch S1 is closed or the switch S1 is open.
~q : The switch S2 is closed or the switch S2 is open. [1]
Consider the given statement
(p  q)  (~ p)  (p  ~ q)
p  q : represents that switches S1 and S2 are connected in series.
p  q : represents that switches S1 and S2 are connected in series.
Therefore (p  q)  (~ p)  (p  ~ q) represents that circuits corresponding to (p  q), (~ p)
and (p  ~ q) are connected in parallel with each other [1]
 Switching circuit corresponding to the given statement is

S1 S2

S1

S1 S2 [1]
L
ii. Given equation is 5x2 + 2xy  3y2 = 0.
Comparing with ax2 + 2hxy + by2 = 0, we get
a = 5, 2h = 2, b = 3
Let m1 and m2 be the slopes of the lines represented by 5x2 + 2xy  3y2 = 0.
-2h 2 a -5
 m1 + m2 = = and m1 m2 = = [1]
b 3 b 3
Since, the required lines are perpendicular to these lines.
1 1
 slopes of the required lines are - and - .
m1 m2
Required lines also pass through the origin, therefore their equations are
1 1
y= - x and y = - x
m1 m2
 x + m1y = 0 and x + m2y = 0
 the joint equation of the lines is
(x + m1y)(x + m2y) = 0 [1]
 x2 + (m1 + m2)xy + m1m2y2 = 0
2 5
 x2 + xy  y2 = 0
3 3
 3x2 + 2xy  5y2 = 0 [1]
 4
iii. Let x = cos1  
5
4 
 cos x = and 0 < x <  sin x > 0
5 2
2
4 16 3
Now, sin x = 1 cos 2 x = 1   = 1 =
5 25 5
 12 
Let y = cos1  
 13 
12 
 cos y = and 0 < y <
13 2
 sin y > 0
187
Board Answer Paper : March 2017
2
  12 144 5
Now, sin y = 1 cos 2 y = 1   = 1 = [1]
 13  169 13
4 12 3 5
But, cos (x + y) = cos x cos y  sin x sin y =    [1]
5 13 5 13
48  15 33
= =
65 65
 33 
 x + y = cos1  
 65 
4  12   33 
 cos1   + cos1   = cos1   [1]
5  13   65 
(B) Attempt any TWO of the following:
i. Let AB be a line with direction cosines l, m, n.
Z

B
A
 L
P(x, y, z)

Y
O

L

X
Consider a line L passing through the origin and parallel to the line AB.
Let P(x, y, z) be a point on the line L and l(OP) = r.
The position vector of point P is
OP = x î + y ĵ + z k̂
If , ,  are the direction angles of line OP, then
l = cos , m = cos , n = cos .
Consider, OP  î = (x î + y ĵ + z k̂ )  î
=x ….(i) [1]
Also, OP . î = OP  î  cos 
= r cos  ….(ii)
 x = r cos  ….[From (i) and (ii)]
Similarly, we have
y = r cos , z = r cos  [1]
 OP = r cos  î + r cos  ĵ + r cos  k̂
2
Now, OP  OP = OP
 (r cos )2 + (r cos )2 + (r cos )2 = r2
 r2 (cos2  + cos2  + cos2 ) = r2
 cos2  + cos2  + cos2  = 1
 l2 + m2 + n2 = 1
Given,  = 135,  = 45
Since, cos2  + cos2  + cos2  = 1 [1]
 cos2  + cos2 (135) + cos2 (45) = 1
2 2
 1   1 
 cos2  +      = 1
 2  2
1 1
 cos2  + + =1
2 2

188
Mathematics and Statistics
 cos2  = 0
 cos  = 0
  = 90
 the direction angle of the line with the X - axis is 90. [1]

ii. Let a, b, c be the position vectors of the points A, B, C respectively.


 a = î + ĵ  2 k̂ , b = î + 2 ĵ + k̂ , c = 2 î  ĵ + k̂

B C

Now, AB = b  a = ĵ + 3 k̂
AC = c  a = î  2 ĵ + 3 k̂ [1]
n is perpendicular to AB and AC both.
 n = AB  AC
ˆi ˆj kˆ
= 0 1 3
1 -2 3

= î (3 + 6)  ĵ (0  3) + k̂ (0  1)
= 9 î + 3 ĵ  k̂ [1]
Vector equation of the plane is r.n  a.n [1]

 r   9iˆ + 3jˆ  kˆ  =  ˆi + ˆj  kˆ    9iˆ + 3jˆ  kˆ 


= 1(9) + 1(3)  2(1) = 9 + 3 + 2
 r   9iˆ + 3jˆ  kˆ  = 14

  xˆi + yˆj  zkˆ   9iˆ + 3jˆ  kˆ  = 14


 9x + 3y – z = 14 is the required equation of plane. [1]

iii. To draw the feasible region, construct table as follows:

Inequality x2 x+y3 2x + y  1


Corresponding equation (of line) x=2 x+y=3 2x + y = 1
Intersection of line with X-axis (2, 0) (3, 0)  1 
  ,0 
 2 
Intersection of line with Y-axis  (0, 3) (0, 1)
Region Origin side Origin side Origin side [1]
Shaded portion OABCD is the feasible region, whose vertices are O(0, 0), A(2, 0), B, C and
D(0, 1).
B is the point of intersection of the lines x = 2 and x + y = 3.

189
Board Answer Paper : March 2017
Putting x = 2 in x + y = 3, we get
Y
y=1
 B  (2, 1) 5
C is the point of intersection of the lines
x + y = 3 and 2x + y = 1. 4
Solving the above equations, we get 3
2 7 2 7
x= ,y= C , 
3 3 2 3 3
2 7 1 B(2, 1)
 C=  ,  [1]
3 3 (0, 1)D
A(2, 0)
Here, the objective function is Z = 6x + 4y, X O 1 2 X
4 3 2 1 3 4
 Z at O(0, 0) = 6(0) + 4(0) = 0 1
Z at A(2, 0) = 6(2) + 4(0) = 12
Z at B(2, 1) = 6(2) + 4(1) = 12 + 4 = 16 2 x+y=3
2 7  2 7 3
Z at C  ,  = 6   + 4   x=2
3 3 3 3 2x + y = 1
4
12 28 40
= + = = 13.33
3 3 3 Y [1]
Z at D(0, 1) = 6(0) + 4(1) = 4
 Z has maximum value 16 at B(2, 1).
 Z is maximum, when x = 2 and y = 1. [1]

SECTION – II

Q.4. (A) Select and write the appropriate answer from the given alternatives in each of the
following sub-questions:
i. (B)
Let u = tan3 
du
 = 3 tan2  sec2 

and v = sec3 
dv
 = 3sec2  sec  tan  = 3sec3  tan 

du 3tan 2  sec2  tan 
 = 3
=
dv 3sec  tan  sec 

at  =
3

tan
du
= 3 = 3 [2]
dv  2
sec
3

ii. (A)
Equation of the curve is y = 3x2  x + 1
Differentiating w.r.t. x, we get
dy
= 6x  1
dx
 Slope of tangent at (1, 3) is
 dy 
  = 6(1)  1 = 5
 dx (1,3)
 dy 
Equation of tangent is y  y1 =   (x  x1)
 dx ( x1 , y1 )

190
Mathematics and Statistics
Here, (x1, y1)  (1, 3)
 y  3 = 5(x  1)
 5x – y = 2 [2]
iii. (B)
Three coins are tossed
 S = {HHH, HHT, HTH, THH, TTH, THT, HTT, TTT}
 n(S) = 8
Thus, the probability distribution of X is as follows:
Here X= number of heads obtained
X 0 1 2 3
1 3 3 1
P(X = x)
8 8 8 8
Expected value = E(X) =  x  P( x )
i i

1 3 3 1


= 0   + 1   + 2   + 3   = 1.5 [2]
8 8 8 8

(B) Attempt any THREE of the following:


i. x sin y + y sin x = 0
Differentiating w.r.t. x, we get
d d d d
x (sin y) + sin y  x  + y  sin x  + sin x  y  = 0 [1]
dx dx dx dx
dy dy
 xcos y + sin y + y cos x + sin x = 0
dx dx
dy
 (x cos y + sin x) =  (sin y + y cos x)
dx
dy  (sin y  y cos x)
 = [1]
dx ( x cos y  sin x)

1
ii. f(x) = x  , x  R, x  0
x
1
 f (x) = 1 + 2 [1]
x
Since x2 is always positive, x  0
 f (x) > 0 for all x  R, x  0
Hence, f(x) is an increasing function, for all x  R, x  0. [1]
sin x
iii. Let I =  x
dx

Put x  t
Differentiating w.r.t. x, we get
1
dx = dt [1]
2 x
 I =  sin t  2dt
= 2 sin t dt
= 2 (– cos t) + c
= –2 cos x c [1]

iv. y = Ae5x + Be 5x ....(i)


Differentiating w.r.t. x, we get
dy
= 5Ae5x  5Be 5x [1]
dx
191
Board Answer Paper : March 2017
Again, differentiating w.r. t. x, we get
d2 y d d
2
= 5A (e5x)  5B (e 5x)
dx dx dx
= 5A(5e5x)  5B( 5e 5x)
= 25Ae5x + 25Be 5x
= 25(Ae5x + Be 5x) = 25y ....[From (i)]
d2 y
 = 25y
dx 2
d2 y
  25y = 0 [1]
dx 2

v. Let X be the number of bombs hitting the target.


P(bomb hits the target) = p = 0.8, q = 1  0.8 = 0.2
Given, n = 10
 X ~ B (10, 0.8)
The p.m.f. of X is given by
P(X = x) = p(x) = 10Cx (0.8)x (0.2)10x , x = 0, 1, 2, ….,10 [1]
 P(exactly 4 bombs will hit the target) = P(X = 4)
= 10C4 (0.8)4 (0.2)6
10  9  8  7
=  0.4096  6.4  10–5
4  3 2 1
= 210  0.4096  6.4  10–5
= 550.5024  10–5
–5
 required probability is 550.5024  10 . [1]
Q.5. (A) Attempt any TWO of the following:
dy
i. Given, = cos (x + y) …(i)
dx
Put x + y = v …(ii)
 y=v–x
dy dv
  –1 …(iii)
dx dx
Substituting (ii) and (iii) in (i), we get
dv
 1  cos v [1]
dx
dv
 = 1 + cos v
dx
dv v
 = 2 cos2
dx 2
1
 dv  2dx
v
cos 2
2
2v
 sec dv = 2dx [1]
2
Integrating on both sides, we get
v
 sec dv  2 dx
2

2
v
 2tan = 2x + c
2
v c
 tan = x +
2 2
x y c
 tan   = x + c, where c = [1]
 2  2

192
Mathematics and Statistics
ii. Let  vdx  w
dw
 =v
dx
d dw du
Consider, (uw) = u +w [1]
dx dx dx
du
= uv +  vdx 
dx
du
dx 
= uv + vdx

Integrating on both sides w.r.t. ‘x’, we get


 du 
uw =  u vdx     vdx  dx [1]
 dx 
 du 
 u  v dx=  uvdx     vdx  dx
 dx 
 d 
  u  v dx = u  vdx    dx (u)   vdx  dx [1]

iii. f is continuous at x = 0.
2
e x  cos x
 f(0) = lim f(x) = lim [1]
x 0 x 0 x2
2
e x  1  cos x +1  e x2  1 1  cos x 
= lim = lim   
x 0 x2 x 0  x2 x2 
 
x
2 2sin 2
ex  1 2
= lim + lim [1]
x 0 x2 x 0 4 2
x
4
2
 x
1  sin 2  1 2 1
= log e + lim   = log e + (1) = 1 +
2 x 0  x  2 2
 2 
3
 f(0) = [1]
2

(B) Attempt any TWO of the following:


i. ‘y’ is a differentiable function of ‘x’.
Let there be a small change x in the value of ‘x’.
Correspondingly, there should be a small change y in the value of ‘y’.
As x  0, y  0
x y
Consider,  =1
y x
x 1 y
 = , 0
y y x
x
Taking lim on both sides, we get
x  0

 x  1
lim   = [1]
  lim y 
x  0y 
 
x  0 x
 
Since ‘y’ is a differentiable function of ‘x’,
 y  dy
lim   =
 x  dx
x  0

As x  0, y  0
193
Board Answer Paper : March 2017
 x  1
lim   = ….(i)
 y  lim  y 
y  0
 
x  0 x
 
 limits on R.H.S. of (i) exist and are finite. [1]
Hence, limits on L.H.S. of (i) also should exist and be finite.
 x  dx
 lim   = exists and is finite.
 y  dy
y  0

dx 1 dy
 = , 0
dy  dy  dx
 
 dx 
y = tan1 x, [1]
 x = tan y
Differentiating w.r.t. y, we get
dx
= sec2 y
dy
dy 1
 =
dx sec2 y
dy 1
 =
dx 1  tan 2 y
dy 1
 = [1]
dx 1 x 2

ii. Number of subscribers = 5000


and annual rental charges per subscriber = ` 3000.
For every increase of 1 rupee in the rent, one subscriber will be discontinued.
Let the rent be increased by ` x.
 New rental charges per year = `(3000 + x)
and number of subscribers after the increase in rental charges = 5000  x.
Let R be the annual income of the company.
Then, R = (3000 + x)(5000  x) [1]
= 15000000  3000x + 5000x  x2
= 15000000 + 2000x  x2
dR d 2R
 = 2000  2x and 2 = 2 [1]
dx dx
dR
Now, R is maximum if =0
dx
 2000  2x = 0
 x = 1000 [1]
d R
2
  2 = 2 < 0 and hence, R is maximum when x = 1000.
 dx  x 1000
Thus, the annual income of the company is maximum when the annual rental charges are
increased by ` 1000. [1]
a
ax
iii. Let I = 
a
ax
dx

a
ax ax
= 
a

ax ax
dx

a
ax
= 
a a 2  x2
dx [1]
a a
a x
= 
a a x
2 2
dx  
a a  x2
2
dx

194
Mathematics and Statistics
 a a

  f ( x)dx  2  f ( x)dx if f ( x) is even 
a
1
 I = 2a  dx  0 ….   a 0 
a 2  x2  if f ( x)isodd 
0
=0
= 0 if f(x) is odd [1]
a
 x
= 2a sin 1  [1]
 a 0

= 2a [sin–1 1 – sin–1 0]
 
= 2a   0 
2 
 I = a [1]
Q.6. (A) Attempt any TWO of the following:
x
i. lim f ( x)  lim
x  0 x  0 x
x
= lim
x 0 x
= –1 [1]
x x
lim f ( x)  lim  lim
x  0 x  0 x x 0 x

=1 [1]
 lim f ( x)  lim f ( x )
x  0 x  0

 lim f ( x) does not exist


x 0

 f(x) is discontinuous at x = 0 [1]


ii. Let ‘x’ be the population of the country at time ‘t’.
dx
 x
dt
dx
 = kx
dt
dx
 = kdt
x
Integrating on both sides, we get
dx
 x = k  dt
 log x = kt + c [1]
When t = 0, x = X0
 log X0 = k(0) + c
 c = log X0
 log x = kt + log X0
When t = 60, x = 2X0
 log (2X0) = 60k + log X0
 60k = log (2X0)  log X0
 2X 0 
 60k = log  
 X0 
1
 k= log 2 ....(i) [1]
60
When x = 3X0, we get
log(3X0) = kt + log X0
1
 log (3X0)  log (X0) = t log 2 ....[From (i)]
60
æ 3X ö t
 log ççç 0 ÷÷÷ = log 2
çè X 0 ÷ø 60

195
Board Answer Paper : March 2017
log3
 60 =t
log 2
60(1.0986)
 t= = 95.364  95.4 years
0.6912
 The population of the country will triple approximately in 95.4 years. [1]
iii. Let X be the number of heads out of 8 tosses.
1 1 1
P(getting head) = p = ,q=1p=1 =
2 2 2
Given n = 8
 1
 X ~ B (n, p)  X ~ B  8, 
2  
The p.m.f. of X is given by
x 8 x
1 1
P(X = x) = p(x) = 8 C x     , x = 0, 1, 2, ….,8 [1]
2 2
a. P(exactly 5 heads) = P(X = 5)
5 3
1 1 ! 1 8  7  6  5! 1
 P(X = 5) = 8C5      =  8 = 
2 2 5! 3! 2 3  2  1  5! 256
8 7 7
= = = 0.21875 [1]
256 32

b. P(at least one head) = P(X  1)


0 8
1 1
 P(X  1) = 1  P(no head) = 1  P(X < 1) = 1  P(X = 0) = 1  8C0    
2 2
1
= 1  (1)(1) 
256
255
=
256
= 0.996 [1]
(B) Attempt any TWO of the following:
1
i. Let I =  d
sin   sin 2
1
=  d
sin  + 2sin  cos 
d sin  d
= 
sin (1  2cos )  sin 2 (1  2cos )
=

sin  d
=  (1  cos 2
)(1  2cos )
[1]

sin  d
=  (1  cos )(1  cos )(1  2cos )
Put cos  = t
  sin  d = dt
 sin  d =  dt
 dt
 I =  (1  t)(1  t)(1  2t)
dt
=  (1  t)(1  t)(1  2t)
1 A B C
Let =   [1]
(1  t)(1  t)(1  2t) 1  t 1  t 1  2t
 1 = A(1 + t)(1 + 2t) + B(1  t)(1 + 2t) + C(1  t)(1 + t) ….(i)
Putting in (i), t = 1, we get
1 = A(2)(3) + B(0)(3) + C(0)(2)
196
Mathematics and Statistics
1
 A=
6
Putting t = 1 in (i), we get
1 = A(0)(1) + B(2)(1) + C(2)(0)
1
 B= 
2
1
Putting t =  in (i), we get
2
3 1
1 = A(0) + B(0) + C    
 2  2
4
 C= [1]
3
1  1 4
1      
=   + + 3
6 2

(1  t)(1  t)(1  2t) 1  t 1 t 1  2t
 1   1   4  
 6    2   3  
 I =             dt
 1  t 1  t 1  2t 
 
1 1 1 1 4 1 1 log 1  t 1 4 log 1  2t
6 1 t
=  dt   dt   dt =   + log 1  t   c
2 1 t 3 1  2t 6 1 2 3 2
1 1 2
 I = log 1  cos   log 1  cos   log 1  2cos   c [1]
6 2 3
ii. Y
x2 = 4ay

P(4a, 4a)

X X
O
(0, 0)

y2 = 4ax
Y
The equations of the parabolas are y2 = 4ax and x2 = 4ay
Solving the equations, we get
2
 x2 
  = 4ax
 4a 
 x4 = 64 a3 x
 x[x3 – (4a)3] = 0
 x = 0 or x = 4a [1]
When x = 0, y = 0 and when x = 4a, y = 4a
 The points of intersection of the parabolas are O (0, 0), P(4a, 4a) [1]
 the required area is,
A = (Area under parabola y2 = 4ax) – (Area under parabola x2 = 4ay)
4a 4a
x2
=  0
4ax dx  
0
4a
dx [1]
4a
2 3 1 1 4a
= 4a   x 2     x3 
3   0 4a 3 0

197
Board Answer Paper : March 2017
4 a 1
=  4a 4a   64a 3
3 12a
32 2 16 2
= a  a
3 3
16 2
= a sq. units. [1]
3
iii. Given the p.d.f. of a continuous random variable X as
x2
f(x) = , –1 < x < 2
3
= 0, otherwise.
 c.d.f. of X is given by
x
F(x) =  f ( y )dy
1
x
x
y2  y3 
= 3
1
dy   
 9  1
x3 1
 F(x) =  ,  x  R. [1]
9 9
1 1 2
 P(X < 1) = F(1) =   [1]
9 9 9
 P(X > 0) = 1 – P (X  0)
1 8
= 1 – F(0) = 1 – = [1]
9 9
 P(1 < x < 2) = F(2) – F(1)
8 1 1 1
=   –  
9 9 9 9
2
=1–
9
7
= [1]
9

198
Biology

BOARD ANSWER PAPER : MARCH 2017


BIOLOGY

Note: Answer to every question must be written on a new page.

SECTION – I
[BOTANY]

Q.1. Select and write the most appropriate answer from the given alternatives for each
sub-question:
i. (B) IBi [1]
ii. (A) fragmentation [1]
iii. (B) Pusa Gaurav [1]
iv. (D) Streptomyces venezuelae [1]
v. (C) restriction endonuclease [1]
vi. (C) 0.9 [1]
vii. (C) CFCs [1]
Q.2. (A) Answer in One sentence each :
i. The bacterium, Thermus aquaticus is the source of thermostable enzyme DNA polymerase. [1]
ii. Toad stools are non-edible or poisonous mushrooms. [1]
iii. Vincristin and vinblastin are the secondary metabolites in Catharanthus roseus. [1]
iv. The gradual (and predictable) change in the species composition of a given area is called
ecological succession. [1]
v. Yeast and group of enzymes collectively called as zymase can bring about alcoholic
fermentation of sucrose. (Name of organism ½ mark + enzyme ½ mark) [1]
vi. Floral adaptations in Salvia:
a. Flowers are bisexual and protandrous, i.e. anthers mature earlier than stigma.
b. It shows lever mechanism, to favour insect pollination. [1]
(B)
CO2

Sugars,
Starches
Fossil
Fuels

Decomposition

Carbon Cycle
(Correct schematic representation with correct labels) [2]
199
Board Answer Paper : March 2017
(C) Answer the following (Any TWO):
i. The cross between F1 hybrid and the recessive parent is called test cross. [1]
Significance of test cross:
a. It helps to determine whether individuals exhibiting dominant character are
genotypically homozygous or heterozygous.
b. Purity of the parents can be determined.
c. It helps to determine the genotype of the individual.
d. It has wide application in plant breeding experiments.
e. In rapid crop improvement programmes, test cross is used to introduce useful
recessive trait in the hybrids.
(Any two significance) [½  2] [1]
ii. Wobble Hypothesis:
a. According to Wobble hypothesis, in codon-anticodon pairing the third base may not
be complementary.
b. The third base of the codon is called wobble base and this position is called Wobble
position.
The actual base pairing occurs at first two positions only.

Perfect pairing Wobble pairing

G U U G U C G U A G U G
C A A C A A C A A C A A

5 G C 5 G C 5 G C 5 G C
C C C C
A A A A
val val val val
Wobble Hypothesis
c. In the above example, though the codon and anticodon do not match perfectly, then
also the required amino acid is brought perfectly.
d. This enables the economy of tRNA. GUU, GUC, GUA and GUG code for amino
acid – Valine. So, a single tRNA can interact with all the four codons which code for
amino acid Valine. (Explanation - 1 mark + Correct diagram - 1 mark) [2]
iii. a. A biopatent is a patent granted by the government to the inventor for biological
entities, processes and for products obtained from them. [1]
b. Examples:
1. Basmati rice known for its unique aroma and flavour has been grown in India
for centuries.
2. Turmeric and margosa plants.
(Any one example) [1]
iv. (W)
Suspensor
(Y)
Radicle
(Z)
Cotyledons
(X)
Plumule

(Correct labels to W, X, Y, Z - ½ mark each) [2]


200
Biology
Q.3. (A) Answer the following (Any TWO):
i. Replication of bacteriophages, (lytic cycle) inside the specific host/bacterial cell takes place in
the following steps:
a. Attachment: Bacteriophages attach to specific receptors on the surface of bacteria. As
phages do not move independently, they rely on random encounters with the right receptors.
b. Penetration: After attachment, the tail fibres bring the base plate closer to the
surface of the cell. Once attached completely, the tail contracts, injecting genetic
material (DNA) through the bacterial membrane. (Capsid – protein coat remains
outside and is called ‘ghost’)

Attachment

Penetration

Lysis of host cell


and release of
virions

Ghost (Capsid)
Assembly of Viral DNA
virions within
host cell
Degradation of host DNA

Synthesis of viral DNA


and protein coat
Lytic cycle

(Schematic representation) [1]


c. Degradation of host DNA: Once the viral DNA enters the host cell, the degradation
of host DNA starts.
d. Synthesis of proteins and nucleic acid: The host’s normal synthesis of proteins and
nucleic acids is disrupted, and it is forced to manufacture viral DNA and proteins
instead. These products are the parts of new virions within the cell or proteins
involved in cell lysis.
e. Virion assembly: The base plates are assembled with the tails first. The head
(capsids) are constructed separately and then are joined with the tails. The DNA is
packed efficiently within the head. The whole process takes about 15 minutes.
f. Release of virions: Phages are released via. lysis of cell. It is achieved by an enzyme
called endolysin, which breaks down the cell wall. Released virions are capable of
infecting a new bacterium.
(Any four correct steps) [½  4] [2]
ii. The biofertilizers are mostly nitrogen fixing microbes which enrich soil with nutrients. [1]
Examples of biofertilizers:
a. Bacterial biofertilizers: Rhizobia are symbiotic nitrogen fixing bacteria associated
with nodules of leguminous plants. Azotobacter and Azospirillum are free-living
nitrogen fixing bacteria used as biofertilizers.
b. Cyanobacterial biofertilizers: Cyanobacteria are autotrophic microbes, also known as
blue green alga which are used as biofertilizers. Some species of Anabaena are
symbiotic. Anabaena is associated with aquatic fern Azolla, whereas Nostoc, Aulosira
are cyanobacteria which carry out nitrogen fixation with the help of heterocysts.
201
Board Answer Paper : March 2017
c. Fungal biofertilizers: Mycorrhiza is a fungus and forms symbiotic association with
the roots of higher plants. They are of two types, Ectomycorrhiza and Endomycorrhiza.
(Any two examples) [1  2] [2]
iii.
No. Anemophily Entomophily
a. Wind pollinated flowers Insect pollinated flowers.
b. Size of flowers is small. Size of flowers is large or present in groups
to show conspicuous appearance.
c. Not brightly coloured. Brightly coloured.
d. Odourless. Usually odour/fragrance present.
e. Nectar is not produced. Nectar or edible pollen usually present.
f. Number of pollen grains produced in large Number of pollen grains are less.
quantity.
g. Stigma is branched or hairy. Stigma is usually unbranched and sticky.
h. Pollination is non-directional. Pollination is directional and highly specific.
i. Pollen grains are light and smooth. Pollen grains are heavier and sticky.
j. e.g. grass, maize, etc. e.g. jasmine, rose, etc.
(Any six correct differences) [½  6] [3]
(B)
Chalaza

Raphe
Nucellus

Integuments
Antipodal cells
Definitive nucleus
Embryo-sac
(secondary nucleus)
Egg-cell
Hilum
Funiculus Synergids
Micropyle

V.S. of mature Anatropous ovule


(Proportionate diagram) [1]
(Any four correct labels) [½  4] [2]
Q.4. Photophosphorylation:
Formation of ATP molecules from ADP and inorganic phosphate (Pi) in presence of light
and chlorophyll during the photochemical phase of photosynthesis is called
photophosphorylation or photosynthetic phosphorylations. [1]
It can be represented as follows:
ADP + Pi light ATP
Chl.a
Non-cyclic photophosphorylation involves the following steps:
i. Photoexcitation of PS-II:
PS-II absorbs light and gets excited, which results in ionisation of chlorophyll-a and a high
energy electron is expelled from PS-II.
ii. Flow of electrons from PS-II to PS-I through ETS:
The expelled energy rich electron is first accepted by CO-Q (co-enzyme quinone).
Electron from CO-Q moves down through various electron carriers and releases energy.
From CO-Q, electrons are transferred to plastoquinone (PQ) (it is an iron containing protein).
From PQ, electrons are transferred to cytochrome complex (cytochrome-b and cytochrome-f).
From cytochrome complex, the electrons are transferred to plastocyanin (PC) (it is a copper
containing protein). From PC, the electrons are finally accepted by ionized chlorophyll of PS-I.
iii. Synthesis of ATP :
One ATP is synthesized when electron passes from cytochrome b6 to cytochrome f.
202
Biology
iv. Photoexcitation of PS-I :
The light energy absorbed by PS-I is transferred to reaction centre P700. It gets excited and
expels energy rich electrons. The electrons are first accepted by unknown electron acceptor
called FRS (Ferredoxin Reducing Substance).
Electron from FRS moves down through various electron carriers and release energy.
From FRS, electrons are transferred to ferredoxin (it is an iron containing protein).
The reduced ferredoxin transfers electron to NADP to form NADP  in the presence of
enzyme Fd-NADP-reductase.
v. Photolysis of water:
Splitting of water into H+ and OH in presence of ligh7t and chlorophyll is called photolysis
of water.
Manganese, calcium and chloride ions present in PS-II play an important role in photolysis
of water.
Photolysis of water occurs in order to satisfy the electron need of PS-II and proton need of
NADP .
vi. Assimilatory power:
ATP and NADPH2 are together called as assimilating power by Calvin, as it is required for
assimilation of CO2.
(Explanation - Any two points) [1  2] [2]
Primary
Acceptor CO-Q
FRS

4e 4e

PQ Fd
4e 2NADP
4e
Cyt. b6
2NADP2
ADP + iP
Photoexcitation

4e ATP
4e 4e
Cyt. f

4e
PC

4e
P-700
PS-I
4 photons
LIGHT
P-680 
4H2O
4OH
PS-II 4e 4H+
CO-Q: Co-enzyme quinone
4 photons 4OH O2 2NADPH2 FRS: Ferredoxin Reducing
Substance
LIGHT PC: Plastocyanin
2H2O PQ: Plastoquinone
Cyt: Cytochrome
Non-Cyclic Photophosphorylation Fd: Ferredoxin

(Schematic representation) [3]


Significance of non-cyclic photophosphorylation:
i. ATP molecules as well as NADPH2 are produced. [½]
ii. Photolysis of water takes place to release O2. [½]
203
Board Answer Paper : March 2017
OR
Q.4. RNA (Ribose Nucleic Acid) is a type of nucleic acid found in the nucleus as well as in the
cytoplasm.
Types of RNA: There are two main types of RNA:
i. Genetic RNA: It acts as genetic material in some viruses.
ii. Non-genetic RNA: It is mainly involved in protein synthesis. There are three
different types of non-genetic RNA:
a. Messenger RNA (m-RNA) b. Ribosomal RNA (r-RNA)
c. Transfer RNA (t-RNA) or Soluble RNA (s-RNA)
a. Messenger RNA (m-RNA) or Informational RNA:
1. It is called messenger RNA because it carries the message for protein synthesis
from DNA to the ribosomes (site for protein synthesis) in the form of codons.
2. It is produced on the DNA strand inside nucleus by a process called
transcription and then transferred to the cytoplasm.
3. It constitutes about 3-5% of the total RNA content of the cell.
4. It is long RNA and the molecular weight of an average sized m-RNA is about
5,00,000.
5. It is always single stranded, linear and straight (unfolded).
6. It has two ends  5 end and 3 end.
7. A triplet of nucleotides on m-RNA is called codon.
8. Each codon on m-RNA specifies one amino acid. This is called m-RNA
language or genetic code or cryptogram.
9. The codon present at 5 end of mRNA is called initiation codon or start codon.
The common initiation codon is AUG or in some cases GUG. AUG and GUG
specify amino acids methionine and valine respectively.
10. The codon present at 3 end is called termination codon or stop codon or
non-sense codon (as they do not specify any amino acid). The termination
codon may be UAA (Ochre) or UAG (Amber) or UGA (Opal).
11. m-RNA is short lived and is degraded soon after protein synthesis.
5 3
AUG UAA

Initiation Triplet Termination


codon codons codon
Structure of m-RNA
Functions of m-RNA:
1. It carries genetic information from DNA to ribosomes during protein synthesis.
2. The genetic code of m-RNA gets translated into the sequences of amino acids
to form proteins.

Unpaired
bases

Paired Coiled region


bases
Uncoiled region
Structure of r-RNA
b. Ribosomal RNA (r-RNA):
1. It is present in ribosomes, hence the name ribosomal RNA.
2. It consists of a single strand.
The single strand is folded upon itself in certain regions.
3. In folded regions, complementary base pairing occurs, while in unfolded
regions, it is absent. Hence, r-RNA does not show purine-pyrimidine equality.
4. It constitutes about 80% of the total RNA content of the cell.
5. The molecular weight ranges from 40,000 to 100,000.
204
Biology
Functions of r-RNA:
1. It provides proper binding site for m-RNA on the ribosome.
2. It orients m-RNA molecule in such a way that all the codons are properly read.
3. It releases t-RNA molecule after transfer of activated amino acid to
polypeptide chain.
4. It also protects the protein molecule under construction.
5. It also protects m-RNA from Rnase enzyme.
c. Transfer RNA (t-RNA) or Soluble RNA (s-RNA) or Supernatant RNA or adapter
RNA.
1. It is the smallest of the three types of non-genetic RNA.
2. It transfers activated amino acids to the site of protein synthesis. Hence, it is
called transfer RNA.
3. It is also called soluble RNA as these molecules cannot be separated from
cytoplasm even by ultra centrifugation technique.
4. The t-RNA molecule consists of a single strand folded upon itself.
5. It constitutes about 10-20% of the total RNA content of the cell.
6. It is made up of 73-93 nucleotides with molecular weight of about 23,000-30,000.
7. There are more than 20 different types of t-RNA.
8. Structure of t-RNA can be explained by two models as:
Hairpin model:
The t-RNA molecule is folded in such a manner that there is formation of one loop
having a triplet of unpaired bases called anticodon.
The 5 end has G-nucleotide, while at 3 end, there is a sequence of CCA nucleotides.
Clover leaf model (trifoliate leaf model):
i. The clover leaf model of t-RNA shows presence of three arms, namely – DHU
arm, middle arm and TC arm.
ii. These arms have loops at their ends such as amino acyl binding loop,
anticodon loop and ribosomal binding loop respectively.
iii. The anticodon loop has anticodon which is a triplet of unpaired nucleotides.
iv. The anticodons present on t-RNA are complementary to codons present on the
m-RNA (anticodons are also referred to as nodoc).
v. In addition, it also shows a small lump called variable arm or variable lump.
vi. Like the hair-pin model of t-RNA, it has G nucleotide at 5 end and CCA
nucleotides at 3 end.
Functions of t-RNA:
1. It carries specific type of amino acid at CCA end to the ribosomes during
protein synthesis.
2. It places the required amino acid properly in the sequence. (This becomes
possible because of complementary nature of codons and anticodons).
3 3
A A
C Acceptor end C
C C
5 5
G TC arm G
DHU Loop TC Loop

DHU arm Variable arm/lump


Middle arm
or Anticodon loop
Anticodon arm
Anticodon
Clover-leaf model Hairpin model
Structure of t-RNA
(Correct definition - 1 mark + Correct explanation of each type of RNA - 1  3
+ Correct diagram with labels - 1  3) [7]
205
Board Answer Paper : March 2017

SECTION – II
[ZOOLOGY]

Q.5. Select and write the most appropriate answer from the given alternatives for each
sub-question:
i. (C) XC Xc [1]
ii. (B) Y chromosome [1]
iii. (D) progesterone [1]
iv. (C) Seminal vesicles [1]
v. (B) ANF [1]
vi. (A) Diabetes mellitus [1]
vii. (D) Gene therapy [1]
Q.6. (A) Answer in One sentence each:
i. Blood, semen, hair roots or other cells of the body are used for isolation of DNA in
fingerprinting technique. [1]
ii. Podocytes produce foot like processes, which form intimate contact with glomerulus and
the gaps between these processes are filtration slits which help in ultrafiltration. [1]
iii. Commensalism is an interspecific interaction in which one species is benefited and the
other one is neither benefited nor harmed. [1]
iv. Acrosome secretes an enzyme hyaluronidase which helps in the penetration of egg during
fertilization. [1]
v.
No. X Chromosome Y Chromosome
a. These chromosomes are metacentric, These chromosomes are acrocentric,
hence appear X shaped. hence appear Y shaped.
b. They are longer than ‘Y’ They are shorter than ‘X’ chromosomes.
chromosomes.
c. It contains large amount of It contains large amount of
euchromatin and small amount of heterochromatin and small amount of
heterochromatin. euchromatin.
d. Found in both male and females. Found only in males.
e. Non-homologous part shows more Non-homologous part contains few genes
genes than Y chromosome. as compared to X chromosome.
f. X – linked genes are present on X Y linked genes (Holandric genes) are
chromosome. present on Y chromosome.
g. Genes present on X chromosome Genes present on Y chromosome show
show criss-cross inheritance. straight inheritance.

(Any two points) [½  2] [1]


vi. Endangered species:
i. Plant species
a. Psilotum nudum b. Osmunda regalis
ii. Animal species
a. Asiatic wild ass b. White-eyed duck
c. Red Panda d. Crocodile
(Any two examples) [½  2] [1]
206
Biology
(B)

Glycoprotein

Lipid membrane
Single Stranded RNA

Outer and Reverse transcriptase


Inner protein coats

Structure of HIV
(Proportionate diagram) [1/2]
(Any three labels) [1/2  3] [11/2]
(C) Attempt any TWO of the following:
i. RBC (Red Blood Corpuscles) or Erythrocytes:
Erythrocytes are circular, biconcave and non-nucleated cells.
Their diameter is 7 and are 2.5 thick.
In adult male, RBC count is 5.1 to 5.8 million per cubic millimeter, while in female, it is
4.3 to 5.2 million per cubic millimeter.
Normal life span of a single RBC is about 120 days.
Formation of RBCs is called erythropoiesis.
Erythropoiesis in foetus, takes place in yolk sac, kidney, spleen and liver while in adults, it
takes place in red bone marrow.
Stroma of RBC contains haemoglobin which helps in carrying respiratory gases.
Old and worn out RBCs are destroyed in spleen and liver.
(Explanation) [1½]

RBC (Diagram) [½]


ii. a. Vaccine is an antigenic preparation used to stimulate the production of antibodies.
b. Vaccines induce immunity against several diseases.
c. Vaccines stimulate immune system to act against genuine toxins.
d. Vaccines or vaccination programme is used to eradicate a particular disease.
e.g. Now, the disease smallpox is totally eradicated by vaccination programme.
(Any two applications) [1  2] [2]
iii. a. Budding is a type of asexual reproduction seen in Coelenterates (e.g. Hydra) and
Ascidians.
b. These multicellular animals form a small bud which grows gradually.
c. The bud grows slowly into a young animal and then detaches itself from the parent
body.
d. This bud then grows into an independent new organism.
(Explanation) [1]

Parent Parent Parent Parent


body body body Hydra

Young
bud Growing Mature Daughter
bud bud Hydra
Budding in Hydra
(Diagram) [1]
207
Board Answer Paper : March 2017
iv. Bombyx mori is the species used in sericulture. [1]
Adult female

Cocoon

Eggs on
Mulberry leaf

Head
Thoracic legs

Mature Spiracles
caterpillar
Life cycle of silk moth

(Cyclic representation with arrows) [1]


Q.7. (A) Attempt any TWO of the following:
i. On the surface of plasma membrane of RBC’s, certain glycoprotein molecules called antigens
are present. These antigens differ in different persons and give blood grouping properties to
them.
ABO blood group system:
a. ABO blood group system was discovered by Karl Landsteiner.
b. In ABO system, blood groups are determined by the presence or absence of
antigen A and antigen B.
c. Antigen: Blood groups are based on two antigens present on the membranes
of RBC’s, namely antigen A and antigen B. Person may have neither of them
or one of them or both of them.
d. Antibody: It is γ-globulin protein present in blood plasma, so is called plasma
factor. There are two types of antibodies a and b. A person may have neither of
them, one of them or both of them.
e. Antigen A and antibody a and antigen B and antibody b are incompatible to
each other and causes self-clumping. On the basis of this, four blood groups
are recognized.

Blood Can give Can receive


Antigen Antibody Genotype
Group blood to blood from
A A b A, AB A,O IAIA or IAi
B B a B, AB B,O IBIB or IBi
AB A,B None AB All (Universal IAIB
(Both) recipient)
O None a,b (Both) All (Universal O ii
donor)
f. Person with blood group O is called Universal Donor as it has no antigen and
can donate blood to any person.
g. Person with blood group AB is called Universal Recipient as it has no
antibody in their plasma, so can receive blood from any blood group.
(Explanation - 1½ marks + Chart - 1 ½ marks) [3]
208
Biology
ii. a. The relative proportion of individuals of various age groups in the population is
called age structure of the population.
b. Age structure of a population is the percentage of individuals of different age groups
such as young (0-14 years), adults (15-59 years) and old (60 years and above).
c. The distribution of these age groups determine the trend of population.

Post-
reproductive
Reproductive

Pre-
reproductive

Age structure showing declining population


d. In declining population, number of post reproductive age group is large whereas the
number of pre reproductive age group is small.
e. Thus, the age structure shows declining population.
(Explanation - ½ mark + Diagrammatic representation - 1½ marks) [3]
iii. Reflex arc: The pathway of nerve fibres along which the reflex impulse travels is known as
reflex arc.
It is always unidirectional from receptor organs to the effector organ, via. CNS.
Reflex arc is the structural and functional unit of reflex action.
Components of simple reflex arc:
Simple reflex arc is formed of five components as given below:
a. Receptor organ:
It is a specialized part of body called sense organ that receives the stimulus and
converts it into the impulse.
e.g. skin, eye, tongue, nose and ears.
Dorsal root ganglion
White matter Sensory Skin receptor organ
neuron
Synapse
Muscle
Grey Matter (effector organ)

Adjustor neuron Motor neuron


Reflex Arc
b. Sensory or Afferent neuron:
It carries sensory nerve impulse from receptor organ to CNS.
Its cyton is located in dorsal root ganglion.
Its dendron is long and connected to receptor, while the axon enters in the grey
matter of spinal cord to form a synapse.
c. Associated or Intermediate neuron:
It is present in the grey matter of spinal cord.
It receives sensory impulse, interprets it and generates motor impulse.
d. Motor or effector neuron:
Its cyton is present in the ventral horn of grey matter and axon travels through ventral
root.
It conducts motor impulse from spinal cord to effector organ.
209
Board Answer Paper : March 2017
e. Effector organ:
It is a specialized part of the body which is excited by receiving the motor impulse.
It gives proper response to the stimulus.
e.g. Muscles and glands.
(Explanation -1½ marks + Diagrammatic representation - 1½ marks) [3]
(B)

Urinary bladder

Seminal vesicle

Prostate gland
Vas deferens
Cowper’s gland

Urethra

Epididymis
Testis

Scrotum
Gubernaculum
Glans
Prepuce

Human Male Reproductive System

(Diagram) [1]
(Any four labels) [½  4] [2]

Q.8. Human endocrine glands:


Hypothalamus, pituitary, parathyroid, thymus, adrenal, pancreas, testis and ovary are the
human endocrine glands. (Any four names) [½  4] [2]
Histological structure of thyroid gland:
Thyroid gland is externally covered by connective tissue sheath or capsule.
From the capsule, number of septa called trabeculae arise, which divide the thyroid gland
into several lobules.
The lobules contain about 3 million thyroid follicles.
Each thyroid follicle is oval in shape and varies in size.
Larger follicles are present towards periphery, whereas smaller ones are present in the
interior.
The follicles are surrounded by a connective tissue called interfollicular tissue which
contains blood vessels and nerve fibres.
Each follicle is lined by a single layer of cuboidal glandular epithelium which rest on a very
thin basement.
The follicular cavity or acinus is filled by dense amorphous semisolid substance called
colloid, which is thyroglobulin, a precursor of thyroid hormone, thyroxine.
210
Biology
Other cells bigger than follicular cells are also present singly or in groups in the connective
tissue. These are called parafollicular or ‘C’ cells.
They secrete hormone thyrocalcitonin. (Explanation) [1½]

Basement membrane
Cuboidal epithelium
Colloid

Para follicular cells

Thyroid follicle

Inter follicular connective tissue


T.S. of Thyroid Gland

(Diagram with three labels) [1½]


Deficiency of thyroxine:
a. Cretinism:
In childhood, deficiency of thyroxine causes cretinism.
It leads to retardation of physical and mental growth of the child.
Patient has low I.Q. (mentally retarded), delayed puberty, dwarfism and sterility.
b. Myxoedema (Gull’s disease):
In adults, deficiency of thyroxine causes myxoedema.
It causes thickening and puffiness of the skin and subcutaneous tissue, specifically of
face extremities.
It also causes low BMR, mental dullness, memory loss and slow action.
c. Simple goiter (Iodine deficiency goiter or endemic goiter):
Deficiency of iodine in diet or drinking water causes simple goiter.
It causes enlargement of thyroid gland (15 times or more) for synthesis of thyroxine
hormone. It is commonly found in hilly regions.
(Any two disorders) [2]

OR

i. Evolution:
Organic evolution is a slow, gradual, continuous and irreversible changes through which
the present day complex forms have descended from their simple, pre-existing forms of
the past. [1]
ii. Principles of Darwinism:
Darwin’s theory of organic evolution by natural selection is based on the following
principles:
a. Over production or prodigality of production:
All organisms have a natural tendency to over produce.
If this tendency is not checked, then even a single species of a plant or animal will
occupy the entire space available on the earth. [1]
b. Struggle for existence: Organisms multiply in geometric ratio, but space and food
remain constant leading to competition for survival. Increase in the number of
species leads to a competition called struggle for existence. [1]
211
Board Answer Paper : March 2017
c. Variations and Heredity:
The differences which occur between the closely related organisms are called
variations.
It is universal law of nature. Variations may be favourable or unfavourable. Useful
variations are preserved and passed on to offsprings. This plays an important role
in evolution. [1]
d. Survival of the fittest or natural selection:
According to Darwin, in the struggle for existence, the fittest individuals survive
and reproduce, while the unfit individuals perish. [1]
e. Origin of new species:
According Darwin, useful variations appear in every generation and are inherited
from one generation to another. [1]
iii. Objections to Darwin’s natural selection theory:
a. Natural selection theory explained “survival of the fittest” but not “arrival of the
fittest”.
b. Darwin did not take into account the hereditary principles.
c. He could not provide a satisfactory explanation for the cause, origin and
inheritance of variations.
d. Certain useless characters are also inherited. He could not explain the inheritance
of useless variations.
e. He was unable to differentiate variations as hereditary and environmental
variations.
(Any one objection) [1]

212
Physics

BOARD ANSWER PAPER : JULY 2017


PHYSICS

Note: Answer to every question must be written on a new page.

SECTION – I

Q.1. Attempt any SIX:


i. Solution:
Given: R = 6.4  106 m,  = 7.26  10–5 rad/s
To find: Change in weight (W)
Formulae: a. g = gp  geq = R2 [½]
b. W = mg [½]
Calculation: From formula (a) and (b),
W = m(R2)
= 1  6.4  106  (7.26  10–5)2 = 3373  10–5 N
Ans: Change in weight of the mass is 3373  10–5 N. [1]
ii. Solution:
Given: m = 0.3 kg, r = l = 0.5 m, v = 2 m/s
To find: Tension (T)
mv 2
Formula: T= + mg cos [½]
r
Calculation: From formula,
0.3  4
T= + 0.3  9.8  cos 60 [½]
0.5
= 3.87 N
Ans: The tension in the string is 3.87 N. [1]
iii. Solution:
Given: r1 : r2 = 4 : 3
To find: W1 : W2
Formula: W = 2TdA [½]
Calculation: From formula,
Work done to blow both bubbles,
 
W1 = 2T 4r12 ; and W2 = 2T 4r22  [½]
2
W1  r1 
   [½]
W2  r2 
2
4
=   = 16 : 9
3
Ans: The ratio of work done to blow the bubbles is 16 : 9. [½]
iv. Solution:
3
Kinetic energy of gas = RT
2
 K.E.  T
(K.E.) 2 T
 = 2 [½]
(K.E.)1 T1
1 T
 = 2 [½]
2 273
 T2 = 136.5 K
Ans: At 136.5 K, the average kinetic energy of the gas will be exactly half of its value at
N.T.P. [1]
213
Board Answer Paper : July 2017
v. a. Surface tension is defined as the force per unit length acting at right angles to an
imaginary line drawn on the free surface of liquid. [1]
b. The extra energy that a liquid surface holds under isothermal condition is called surface
energy. [1]
vi. Variation of g due to altitude:
a. Let, M = mass of the earth
R = radius of the earth
h = height at which acceleration due to gravity is to be found.
g = acceleration due to gravity at the surface of the earth.
gh = acceleration due to gravity at height h.
b. On the surface of the earth,
GM
g= ….(1) [½]
R2
At height ‘h’ above the earth surface,
GM
gh = ….(2) [½]
(R  h) 2
c. Dividing equation (2) by (1),
gh GM / (R  h) 2
=
g GM / R 2
GM R2
= 
(R  h)2 GM
gh R2
 = ….(3) [½]
g (R  h)2
gh R2
 = 2
g  h
R 2 1  
 R
gh 1
 = 2
g  h
1 
 R 
2
gh  h
 = 1   ….(4)
g  R
d. Expanding equation (4) by using binomial expansion and neglecting higher power of
h
gives,
R
g h  2h 
= 1   ( h << R) [½]
g  R 
 2h 
 gh = g  1   ....(5)
 R 
Above equation represents acceleration due to gravity at altitude h.
vii. Physical significance of K:
a. Radius of gyration is a measure of distribution of mass about the given axis of
rotation.
b. If the particles of the body are distributed close to the axis of rotation, the radius of
gyration is less.
c. If the particles are distributed away from the axis of rotation, the radius of gyration is
more.
d. The knowledge of mass and radius of gyration of the body about a given axis of
rotation gives the value of its moment of inertia about the same axis, even if we do
not know the actual shape of the body. [2]

214
Physics
viii.
N N cos 


N sin 
G C
F cos  

F h
A  F sin 
B

W = mg
AC : inclined road surface
AB : horizontal surface
BC : height of road surface
G : centre of gravity of vehicle
W : (mg) weight of vehicle
N : normal reaction exerted on vehicle
 : angle of banking
(Diagram and labelling) (1 + 1) [2]
Q.2. Attempt any THREE:
i. Theoretical proof:
a. Consider an ordinary body O and perfectly black body B of same dimension suspended
in a uniform temperature enclosure as shown in the figure.
b. At thermal equilibrium, both the bodies will have same temperature as that of the
enclosure.
c. Let, E = emissive power of ordinary body O
Eb = emissive power of perfectly black body B
a = coefficient of absorption of O
e = emissivity of O
Q = radiant energy incident per unit time per unit area on each body [½]

Uniform
temperature
enclosure
O B
[½]
d. Quantity of heat absorbed per unit area per unit time by body O = aQ.
Quantity of heat energy emitted per unit area per unit time by body O = E.
Since there is no change in temperature
 E = aQ
E
 Q= .…(1) [½]
a
e. Quantity of heat absorbed per unit area per unit time by perfectly black body,
B=Q
The radiant heat energy emitted per unit time per unit area by perfectly black body,
B = Eb
Since there is no change in temperature.
 Eb = Q .…(2) [½]
f. From equations (1) and (2),
E E
= Eb  =a
a Eb
E
But, =e [½]
Eb
 a=e [½]

215
Board Answer Paper : July 2017
ii. Expression for strain energy:
a. Let, L = length of wire
A = area of cross section of wire
r = radius of cross section of wire
l = elongation of the wire by applying load. L
b. If the wire is perfectly elastic then,
Young’s modulus,
F/A 0 A
Y= l
l/L
F L F
= 
A l
YAl
 F= ….(1)
L
c. Let ‘f’ be the restoring force and ‘x’ be its corresponding extension at certain instant
during the process of extension.
YAx
 f= ….(2)
L
d. Let ‘dW’ be the work done for the further small extension ‘dx’.
 dW = fdx [½]
YAx
 dW = dx ….(3) [½]
L
e. The total amount of work done in stretching the wire from 0 to l can be found out by
integrating equation (3).
l l l
YAx YA
W= 0
dW = 0 L dx = L  xdx
0
[½]

l
YA  x 2 
 W=   [½]
L  2 0
YAl 2
 W= [½]
2L
YAl l
 W= 
L 2
YAl
But, =F
L
1
 W= Fl .…(4) [½]
2
Equation (4) represents the work done by stretching a wire.
iii. Solution:
Given: N = 12, nL = 2 nF , n5 = 90 Hz
To find: Number of beats (x),
Frequency of last fork (nL)
Formula: nL = nF + (N  1) x [½]
Calculation: From formula,
nL = nF + (5  1) x
 nF + 4x = 90 ….(1)
nL = nF + (12  1) x [½]
 nL = nF + 11x ….(2)
nL = 2nF
nF = 11x ….From (2) [½]
Substituting in equation (1),
15x = 90 or x = 6 beat/s [½]
 nF = 11  6 = 66 Hz [½]
 nL = 2  66 = 132 Hz [½]
Ans: The frequency of the first and last tuning fork is 66 Hz and 132 Hz respectively.

216
Physics
iv. Solution:
Given: R = 0.2m,  = 8000 Kg/m3
To find: Moment of inertia (I)
Formulae: a. I0 = Ic + MR2 [½]
7
 I0 = MR2
5
b. Mass (M) = volume  density
Calculation: From formula (b),
4 
M = V =  R 3   [½]
3 
From formula (a),
7 4 3  2
I=  R   R [½]
5 3 
28
= R5
15
28
=  3.14  (2  101 )5  8000 [½]
15
 I = 15.02 kg m2
Ans: M.I. of the uniform solid sphere about a tangent to its surface is 15.02 kg m2. [1]
Q.3. A. Linear S.H.M is defined as the linear periodic motion of a body, in which the restoring
force (or acceleration) is always directed towards the mean position and its magnitude
is directly proportional to the displacement from the mean position. [1]
Displacement of particle is given by, x = A cos t
Velocity time graph:
i. At extreme position,  = /2
 Velocity of a particle is v =  A sin t [½]
ii. Table:
2
Substituting  = in above equation,
T

Time Phase Velocity


(t) (t) (v)
0 0 0
T/4 /2  A
[½]
T/2  0
3T/4 3/2 A
T 2 0
iii. Graph:

+A
Velocity

0 T T 3T 5T 3T
t
T
4 2 4 4 2
A
[½]

Acceleration-time graph:
i. At extreme position,  = /2
 Acceleration of a particle is,
a = A2 cos t [½]
217
Board Answer Paper : July 2017
ii. Table:
2
Substituting  = in above equation,
T

Time (t) Phase Acceleration


(t) (a)
0 0  A2
T/4 /2 0 [½]
T/2  A2
3T/4 3/2 0
T 2  A2

iii. Graph:

+A2
Acceleration

0 T T 3T 5T 3T
t
T
4 2 4 4 2
A2

[½]
Conclusions:
i. Displacement, velocity and acceleration of S.H.M. are periodic functions of time.
ii. The displacement and acceleration curves are sine curves whereas velocity curve is cosine
curve ( = 0).
iii. The phase difference between displacement and acceleration is of  radian.
iv. The phase difference between displacement and velocity and velocity and acceleration is of
/2 radian.
v. The displacement and acceleration is maximum at extreme position whereas velocity is
minimum at the same position.
vi. All curves repeat same path after phase of 2 radian.
[Explanation of variation of velocity-time and acceleration-time graph] [1]

B. Solution:
Given: L = 1.005 m, g = 9.8 m/s2
To find: Loss in period (T)
L
Formula: T = 2 [½]
g
Calculation: From formula,
1.005
T = 2  3.14 
9.8
1.005
= 6.28
9.8
= 2.012 s
The period of a seconds pendulum is 2 second.
Hence, the given pendulum clock will lose 0.012 s in 2.012 s
(during summer). [½]
 Loss in period per day
24  3600  0.012
T =
2.012
 T = 515.3 s
Ans: The clock will gain or lose 515.3 s in one day. [1]
218
Physics
OR
A. i. First mode or fundamental mode:
In this mode of vibration, there is one node at the centre of the
pipe and two antinodes, one at each open end as shown in
figure (a).
Let, v = wave velocity in air A
n = fundamental frequency
 = wavelength L N 
L = length of air column 2
v = n ….(1)

Also L = A
2
  = 2L ….(2) Figure (a)
From equation (1) and (2),
v = n 2L [Diagram  ½]
v
 n= ….(3) [½]
2L
Equation (3) represents fundamental frequency or lowest frequency of vibration.
ii. Second mode or first overtone:
In this mode of vibration, there are two nodes and three antinodes as shown in figure (b).
Let, v = wave velocity in air
[As the medium is same, wave velocity remains same]
n1 = next higher frequency
1 = corresponding wavelength A
L = length of tube Velocity of wave is given by,
N
v = n11 .…(4)
Also, L = 1 .…(5) 1
L A
From equation (4) and (5),
v = n1 L
N
v
 n1 = A
L
v
 n1 = 2  ….(6) Figure (b)
2L
From equation (3) and (6) [Diagram  ½]
 n1 = 2n .…(7) [½]
Thus, frequency of first overtone (second harmonic) is twice the fundamental frequency.
iii. Third mode or second overtone:
In this mode of vibration, there are 3 nodes and 4 antinodes as shown in figure (C).
Let, v = wave velocity in air [As the medium is same, wave velocity remains same]
n2 = next higher frequency
2 = corresponding wavelength
L = length of tube
Velocity of wave is given by, A
v = n22 .…(8) N
3 2 A
Also L =
2
2L L N 3 2
2 = .…(9) A 2
3
From equation (8) and (9), N
2L
v = n2 A
3
3v
 n2 = ….(10) Figure (c)
2L
From equation (3) and (10), [Diagram  ½]
n2 = 3n [½]
Thus, frequency of second overtone is thrice the fundamental frequency.
219
Board Answer Paper : July 2017
Causes:
End correction arises because air particles in the plane of the open end of tube are not free to
move in all directions, hence reflection takes place at the plane, small distance outside the tube. [1]
End correction in open pipe:
i. Let, l1 and l2 = Vibrating lengths of pipe
n1 and n2 = Resonating frequency
v = Velocity of sound
e = End correction
ii. For the first resonance,
v
n1 =
2(l1 + 2e)
 v = 2n1 (l 1 + 2e) .…(1)
iii. For the second resonance,
v
n2 =
2(l2 + 2e)
 v = 2n2 (l 2 + 2e) .…(2)
iv. From (1) and (2),
n1 (l1 + 2e) = n2 (l 2 + 2e)
 n1 l 1 + 2e n1 = n2 l 2 + 2e n2
 2e n1  2e n2 = n2 l 2  n1 l 1
 2e (n1  n2) = n2l2  n1l1
n 2l2  n1l1 n l n l
 e= = 11 2 2 [1]
2  n1  n 2  2(n 2  n1 )

B. Solution:
Given: l = 1m, M =10g = 10103kg, n= 50 Hz
To find: Tension (T)
1 T
Formula: n= [½]
2l m
Calculation: Linear density of wire,
M 10  103
m= = = 10  103 kg/m
l 1
From formula,
1 T
50 = [½]
2 1 10  103
T
 100 =
10  103
T
 (100)2 =
10  103
 T = 104  10  103
 T = 100 N
Ans: The tension to be applied to the wire is 100 N. [1]
Q.4. Select and write the most appropriate answer from the given alternatives for each
sub-question:
i. (D) [1]
ii. (C) [1]
iii. (C) [1]
iv. (A) [1]
v. (A) [1]
vi. (D) [1]
vii. (D) [1]

220
Physics

SECTION – II

Q.5. Attempt any SIX:


i. When an -particle is emitted by an atom, its atomic number decreases by 2 and mass
number decreases by 4. [½]
When 90Th232 disintegrates to 82Pb200, 8  particles are emitted. [½]
When -particle is radiated, the atomic number increases by 1 and mass number does not
change. [½]
When 90Th232 disintegrates to 82Pb200, 8  particles are emitted. [½]
ii. Solution:
Given: W0 = 3eV = 3  1.6  1019 = 4.8  1019 J, h = 6.63  1034 J – s,
e = 1.6  1019 C, c = 3  108 m/s.
To find: Threshold wavelength (0),
hc
Formula: 0 = [½]
W0
Calculation: From formula,
hc 6.631034  3108
(0) = = [½]
W0 4.8 1019
 0 = 4.14  10–7 m
Ans: Threshold wavelength of the metal is 4.14  10–7 m. [1]
iii. Solution:
Given: C1 = 8 F, C2 = 8 F, C3 = 4 F, V = 120 volt
To find: Charge on the capacitor C3
1 1 1 1
Formulae: a.    [½]
CS C1 C 2 C3
Q
b. C=
V
Calculation: Using formula (a),
1 1 1 1
=  
CS C1 C 2 C3
1 1 1
= + +
8 8 4
 CS = 2 F = 2  10–6 F [½]
In series combination,
Q1 = Q2 = Q3 [½]
Using formula (b),
Q = CS V
 Q = 2  10–6  120 = 240  10–6 C
Ans: The charge on the capacitor C3 is 240  10–6 C. [½]
iv. Solution:
Given: E = 6.63 J,  = 1014 Hz, h = 6.63  10–34 Js.
To find: Number of photons (n)
E
Formulae: n= [½]
h
Calculation: Using formula,
6.63
n= [½]
6.63  1034  1014
 n = 1020
Ans: The number of photons emitted in the radiation are 1020. [1]

221
Board Answer Paper : July 2017
v.
No. Diamagnetic substance Paramagnetic substance
a. In an external magnetic field, a In an external magnetic field, a
substance gets weakly magnetised in paramagnetic substance gets weakly
the direction opposite to that of the magnetised in the same direction as
field. that of the field.
b. When placed in a non-uniform magnetic When placed in a non-uniform magnetic
field, it tends to move from the stronger field, it tends to move from the weaker to
to the weaker part of the field. the stronger part of the field.
c. It is weakly repelled by a magnet. It is weakly attracted by a magnet.
d. Magnetic moment of every atom of a Every atom of a paramagnetic substance
diamagnetic substance is zero. is a magnetic dipole having a certain
resultant magnetic moment.
e. When a rod of diamagnetic substance is When a rod of paramagnetic substance
suspended in a uniform magnetic field, it is suspended in a uniform magnetic
comes to rest with its length field, it comes to rest with its length
perpendicular to the direction of the field. parallel to the direction of the field.
f. There is no effect of temperature on There is effect of temperature on
diamagnetic substance. paramagnetic substance.
(Any two points) [2]
vi.
Exosphere (> 500 km)

Appleton layer
Ionosphere

Kennelly heavyside layer

400 km
Thermosphere Edge of
140 km
atmosphere
Mesosphere
80 km
Ozone layer
Stratosphere 50 km

Troposphere 12 km

Earth’s surface (Diagram + labelling) (1 + 1) [2]


vii. Huygens’ construction of plane wavefront:

ct
P P1 N1

ct
Q Q1 N2

ct
R R1 N3

PQR: Plane wavefront at any instant,


P1Q1R1 : Plane wavefront after time ‘t’,
PP1N1, QQ1N2, RR1N3 : wave normals at PQR [1]

222
Physics
a. A plane wavefront is formed when point of observation is very far away from the
primary source.
b. Let PQR represent a plane wavefront at any instant. According to Huygens’
principle, all the points on this wavefront will act as secondary sources of light
sending out secondary wavelets in the forward direction.
c. Draw hemispheres with P, Q, R…. as centres and ‘ct’ as radius. The surface
tangential to all such hemispheres is P1Q1R1…. at instant ‘t’. It is a new wavefront at
time ‘t’.
d. The plane wavefronts is propagated as plane waves in homogeneous isotropic
medium. They are parallel to each other.
e. PP1N1, QQ1N2, RR1N3 are the wave normals at P, Q, R respectively. These wave
normals show the direction of propagation of plane wavefront.
f. The new wavefront P1Q1R1 is parallel to primary wavefront PQR. [1]

viii. Expression for electric intensity due to uniformly charged infinite plane sheet:
a. Consider an infinite thin plane sheet of positive charge having a uniform surface charge
density  on both sides of the sheet.
b. By symmetry, it follows that the electric field is perpendicular to the plane sheet of
charge and is directed in outward direction.
c. Electric field intensity has same magnitude at a given distance on either sides of the sheet.

Gaussian
surface

P
ds
E1 = E E1 = E

[½]
PS

d. To find electric field intensity at a point P due to uniformly charged infinite thin
plane sheet, construct an imaginary cylinder around P with its axis perpendicular to
plane sheet carrying charge with ends having cross sectional area ds.
e. The plane sheet passes through the middle of cylinder’s length so that the ends of
cylinder are equidistant from the plane sheet carrying charge.

f. Electric field intensity, E is perpendicular to the ends of cylinder, hence the electric flux
through each end is Eds.

g. Since E is perpendicular to plane sheet, it is parallel to the curved surface of
Gaussian cylinder. Hence, electric flux does not pass through the curved surface of
Gaussian cylinder. [½]
h. Now, Total Normal Electric Induction over Gaussian surface =  E (2ds)
where, ds is surface area of end faces of the cylinder.
Algebraic sum of charges enclosed by Gaussian cylinder = ds
i. According to Gauss’ law,
 E (2ds) = ds [½]

 E= [½]
2
This is the expression for electric field intensity at a point outside uniformly charged
thin plane sheet.
j. Above equation shows that the magnitude of electric field intensity is independent of
the distance of point from plane sheet.
223
Board Answer Paper : July 2017
Q.6. Attempt any THREE:
i.
+
mA
IC 
C +
N-P-N
R I +
A B B V
+   VCE VCC

+ + T E
VBB  V
 VBE
IE

N-P-N transistor in CE configuration


[1]
Output characteristics of transistor:

IB = 50 A
IB = 40 A
IC (mA)
IB = 30 A
IB = 20 A
IB = 10 A
IB = 0 A
0 0.3 1 2 3 4 5
VCE (volt)
Transistor output characteristics [½]

a. To study output characteristics, IB is kept constant, VCE is increased in small


steps and IC is recorded at every step.
b. For the plot IC versus VCE, output characteristics are as shown in figure.
c. From output characteristics it is clear that:
1. When IB = 0, then IC is nearly zero. Thus the transistor is in cut off region.
2. For VCE < 0.3 V,
IC increases with VCE and can be controlled by VCE.
3. When VCE > VBE (0.7), C-E junction gets reverse biased and IC gets saturated.
i.e., IC doesn’t depend on VCE but can be controlled by IB. In this case the
transistor is in active region. It has large current gain in this region.
4. When VCE < 0.2 V, both the junctions gets forward biased and IC no longer can
be controlled by IB. In this case relation between IC and IB gets lost.
5. The dynamic output resistance of transistor is given by,
 VCE 
ro =  
 IC  I B = constant
6. The output resistance in this mode varies from 1 k to 100 k. [1½]
ii. Expression for magnetic induction at a point along the axis of a toroid:
a. A toroid is a long solenoid bent in the shape of a ring.

O P
r

I I [½]

224
Physics
b. The magnetic field around the toroid consists of concentric circular lines of force
around it. Magnetic field is produced, when a steady current ‘I’ flows through toroid.
c. The direction of magnetic field at a point is along the tangent to the circular path at
that point.
d. Let r be the radius of the Amperian loop. This loop is concentric with the axis of
toroid. P is a point on the loop. We have to determine magnetic induction at P. [½]
e. Applying Ampere’s law,
 

 B  dl = 0NI .…(1) [½]


where, N = total number of turns in the toroid.
NI = total current flowing through toroid.
 
Now,  B  dl =  B  dl cos  ….2
 
f. But, B and dl are in same direction
  = 0
 cos  = 1
 Equation (2) can be written as,
 

 B  dl =  Bdl = B  dl .…(3)
g. Also  dl = 2r
From equation (3),
 

 B  dl = B (2r) .…(4) [½]


h. From equation (1) and (4),
B (2r) = 0NI
 0 NI
 B= .…(5) [½]
2r
i. If ‘n’ is the number of turns per unit length of toroid then
N
n= ….(6)
2r
j. From equation (5) and (6),
B = 0nI ….(7) [½]
Equation (5) and (7) both represent magnetic induction at a point along the axis of
toroid.
iii. Solution:
Given: R1 = 12 , l2 = 120 cm, R2 = 18 , l2 = 150 cm
To find: a. Balancing length (l1), b. Internal resistance (r)
 l1  l2 
Formula: r=R   [½]
 l2 
Calculation: From first condition,
 l  120 
r = 12  1  .…(1)
 120 
From second condition,
 l  150 
r = 18  1  .…(2)
 150 
From equation (1) and (2),
 l1  120   l1  150 
12   = 18   [½]
 120   150 
 5(l1  120) = 6(l1  150)
 l1 = 300 cm [1]

225
Board Answer Paper : July 2017
Using equation (1) we get
Internal resistance,
 l  120 
r = 12  1 
 120 
300  120 
= 12  
 120 
 r = 18  [1]
Ans: a. The balancing length is 300 cm.
b. The internal resistance of the cell is 18 .

iv. Solution:
Let, S = shortest wavelength
L = longest wavelength
1  1 1 
=R  2 2 [½]
 p n 
Longest wavelength in Paschen series is obtained when p = 3, n = 4
For longest wavelength,
1 1 1
 = R  2  2 [½]
L 3 4 
1 1 1 
 =R   
L  9 16 
1 16  9 
 =R  9  16 
L  
1 7R
 =
L 9  16
9  16 144
 L = = [½]
7R 7R
Shortest wavelength in Balmer series is obtained when p = 2, n = 
For shortest wavelength,
1 1 1 
= R  2  2 [½]
S 2  
4
 S = [½]
R
L 144 R 36
 =  =
S 7R 4 7
L
 = 5.131
S
Ans: The ratio of longest wavelength in Paschen serires to shortest wavelength in Balmer
series is 5.131. [½]

Q.7. A. Principle:
It is based on the principle of mutual induction i.e., whenever the magnetic flux linked
with a coil changes, an e.m.f is induced in the neighbouring coil. [½]
Construction:
i. A transformer consists of two sets of coils P and S insulated from each other. The coil P is
called the primary coil and coil S is called the secondary coil.
ii. The two coils are wound separately on a laminated soft iron core.
iii. The a.c input voltage is applied across the primary and the induced output a.c voltage is
obtained across the secondary, which is used to drive current in the desired circuit.
iv. The two coils are electrically insulated from each other but they are magnetically linked.
226
Physics
v. To minimise eddy currents, the soft iron core is laminated. [1]
Soft iron core

Input a.c.
voltage  P S
Load

P S

core
[½]
Working:
i. When an alternating voltage is applied to the primary coil the current through the coil goes
on changing. Hence, the magnetic flux through the core also changes.
ii. As this changing magnetic flux is linked with both the coils, an e.m.f is induced in each
coil.
iii. The amount of the magnetic flux linked with the coil depends upon the number of turns of
the coil.
iv. Let, ‘’ be the magnetic flux linked per turn with both the coils at certain instant ‘t’.
v. Let ‘NP and ‘NS’ be the number of turns of primary and secondary coil,
NP = magnetic flux linked with the primary coil at certain instant ‘t’
NS = magnetic flux linked with the secondary coil at certain instant ‘t’ [1]
vi. Induced e.m.f produced in the primary and secondary coil is given by,
d P d
eP =  =  NP ….(1)
dt dt
d d
eS =  S =  NS ….(2)
dt dt
vii. Dividing equation (2) by (1),
eS N
 = S ….(3) [1]
eP NP
Equation (3) represents equation of transformer.
NS
The ratio is called turns ratio (transformer ratio) of the transformer.
NP
viii. For an ideal transformer,
Input power = Output power
 ePIP = eSIS
eS I
 = P ….(4)
eP IS
ix. From equation (3) and (4),
eS N I
= S = P [1]
eP NP IS

B. Solution:
Given: Mnet = 2 Am2, l = 10 cm = 10  10–2 m,
area of cross section = 4 cm2 = 4  10–4 m2
To find: magnetisation (MZ)
M net
Formula: MZ = [½]
V
Calculation: Since, V = l  cross sectional area
= (10  10–2  4  10–4)
= 4  10–5 m3
227
Board Answer Paper : July 2017
From formula,
2
MZ = [½]
4  105
 MZ = 50000 A/m
Ans: The intensity of magnetisation is 50000 A/m. [1]
OR
A. Theory of interference bands:
i. Let S1 and S2 be the two coherent monochromatic sources which are separated by short
distance d. They emit light waves of wavelength .
ii. Let D = horizontal distance between screen and source.
iii. Draw S1M and S2N  AB
OP = perpendicular bisector of slit.
Since S1P = S2P, the path difference between waves reaching P from S1 and S2 is zero,
therefore there is a bright point at P.
iv. Consider a point Q on the screen which is at a distance x from the central point P on the
screen. Light waves from S1 and S2 reach at Q simultaneously by covering path S1Q and
S2Q, where they superimpose. [½]
A
Q

x
S1 M
d/2
d O P
x d/2
K
S2 N
D

B
[½]

v. In  S1MQ, (S1Q)2 = (S1M)2 + (MQ)2


2
 d
(S1Q)2 = D2 +  x   ….(1)
 2
vi. In  S2NQ, (S2Q)2 = (S2N)2 + (NQ)2
2
 d
 (S2Q)2 = D2 +  x   ….(2)
 2
vii. Subtract equation (1) from (2),
 d  
2
d 
2

(S2Q)2  (S1Q)2 =  D2   x      D2   x   
  2    2 
   
2 2
 d  d
= D2   x    D2   x  
2  2   
2 2
 d  d
= x   x  
 2 2   
 d2
  d2 
=  x2   xd    x 2   xd 
 4   4 
d2 d2
= x2   xd  x 2   xd
4 4
(S2Q)2  (S1Q)2 = 2xd
 (S2Q + S1Q) (S2Q  S1Q) = 2xd
2xd
 S2Q  S1Q = ….(3)
S2Q  S1Q

228
Physics
viii. If x << D and d << D then,
S1Q  S2Q  D
S2Q + S1Q = 2D
 Equation (3) becomes,
2xd
S2Q  S1Q =
2D
xd
 S2Q  S1Q =
D
xd
 x = ….(4) [1]
D
Equation (4) gives the path difference of two interfering light waves.
ix. Point Q will be bright if,

x = n = 2n
2
xd 
 = n = 2n ….[From equation (4)]
D 2
D
 x=n ….(5) [½]
d
th
Equation (5) represents distance of n bright fringe from central bright fringe.
where n = 0, 1, 2,….
x. Point Q will be dark point if,

x = (2n 1)
2
where n = 1, 2, 3,……
xd 
 = (2n 1)
D 2
D
x = (2n 1) .…(6) [½]
2d
th
Equation (6) represents distance of n dark fringe from central maximum.
Expression for the fringe (band) width:
i. The distance between the centre of two consecutive bright or dark fringe is the fringe
(band) width.
ii. Let xn = distance of nth bright fringe and xn+1 = distance of (n + 1)th bright fringe
D
 xn = n
d
D
 xn+1 = (n + 1)
d
iii. If X is the fringe width between two bright fringes then,
D D
X = xn+1  xn = (n + 1 ) n
d d
D
 X = ….(7) [1]
d
Equation (7) represent fringe (band) width of bright fringes.
iv. Similarly if xn and x(n + 1) represent distance of nth and (n + 1)th dark fringe then,
D
xn = (2n  1)
2d
D
x(n + 1) = [2(n + 1)  1]
2d
 width of dark fringe or dark band is given by,
D D
X = x(n + 1)  xn = [2 (n + 1)  1]  (2n  1 )
2d 2d
D
 X = ….(8) [1]
d
Equation (8) represents fringe (band) width of dark fringes.

229
Board Answer Paper : July 2017
B. Solution:
3 4
Given: ag = , aw =
2 3
To find: wg
g
wg
a
Formula: = [½]
a w
va v
Calculation: ag = and aw = a
vg vw
vw
 wg = [½]
vg
From formula,
3/ 2
wg = [½]
4/3
 wg = 1.12
Ans: The refractive index of glass w.r.t. water is 1.12. [½]
Q.8. Select and write the most appropriate answer from the given alternatives for each
sub-question:
i. (C) [1]
ii. (D) [1]
iii. (B) [1]
iv. (D) [1]
v. (B) [1]
vi. (A) [1]
vii. (D) [1]

230
Chemistry

BOARD ANSWER PAPER : JULY 2017


CHEMISTRY
Note: Answer to every question must be written on a new page.

SECTION – I
Q.1. Select and write the most appropriate answer from the given alternatives for each
sub-question:
i. (C) MgO [1]
The oxide which combines with water to form a base is termed as basic oxide.
MgO + H2O  Mg(OH)2(aq)
Magnesium Magnesium
oxide hydroxide

ii. (B) comma [1]


eg. Fe3+, Fe2+ | Pt
iii. (A) 6 [1]
iv. (B) mol dm–3 t–1 [1]
v. (D) flux [1]
vi. (C) 1.0 M [1]
10
Molarity of NaOH solution = = 1.0 M
40  0.25

vii. (B) 5.5 atm [1]


W = – Pex.V
55 =  Pex(18.5 – 28.5)
55
 Pex = = 5.5 atm
10

Q.2. Answer any SIX of the following:


i. a. Henry’s law states that “the solubility of a gas in liquid at constant temperature is
proportional to the pressure of the gas above the solution.” [1]
b. The solubility of a gas in a liquid decreases with increase in temperature.
Reason: According to Charles’ law, volume of a given mass of a gas increases with
increase in temperature. Therefore, volume of a given mass of dissolved gas in
solution also increases with increase in temperature, so that it becomes impossible
for the solvent in solution to accommodate gaseous solute in it and gas bubbles out.
Hence, solubility of gas in liquid decreases with increase in temperature. [1]
ii. a. This method is based upon catalytic oxidation of NH3 by atmospheric oxygen.
Pt /Rh gaugecatalyst
4NH3(g) + 5O2(g) 
500K,9bar
4NO(g) + 6H2O(g)
Ammonia (From air) Nitric oxide [1]
b. Nitric oxide reacts with oxygen to form nitrogen dioxide.
2NO(g) + 

O2(g) 
 2NO2(g)
Nitric oxide Oxygen Nitrogen dioxide [½]

c. Nitrogen dioxide on dissolution in water gives nitric acid.


3NO2(g) + H2O(l)  2HNO3(l) + NO(g)
[½]
Nitrogen dioxide Water Nitric acid

231
Board Answer Paper : July 2017
iii. a. Ammonium phosphate  Ionic solid [½]
b. Brass  Metallic solid [½]
c. S8 molecule  Molecular solid [½]
d. Diamond  Covalent or network solid [½]

iv. In the given representation of the cell, a standard Zn2+(1M)|Zn electrode is combined with
standard hydrogen electrode to form the cell:
Zn | Zn2+(1M) || H+(1M) | H2(g, 1 atm) | Pt
e e

salt bridge
() (+)
Zinc anode
H2 gas (1 atm)
SHE
1M
ZnSO4
solution 1 M H+ ion solution

Standard zinc electrode combined with SHE


(Diagram + labelling) [2]
v. a. ZnO reacts with acid to form salt. This shows that ZnO acts as a basic oxide. [½]
eg. ZnO(s) + 2HCl(aq)  ZnCl2(aq) + H2O(l) [½]
(Basic)

b. ZnO reacts with base to form salt. This shows that ZnO acts as an acidic oxide. [½]
eg. ZnO(s) + 2NaOH(aq)  Na2ZnO2(aq) + H2O(l) [½]
(Acidic) (excess)

ZnO acts as both acidic as well as basic oxide. Hence, zinc oxide is an amphoteric oxide.

vi. Minerals of aluminium:


Name of mineral Formula [1]
a. Bauxite Al2O3.2H2O
[1]
b. Cryolite Na3AlF6
vii. a. Role of N2O: N2O acts as an intermediate since it is produced in first step and
consumed in second step. [1]
b. The first step is trimolecular/termolecular. [½]
The second step is bimolecular. [½]
viii. The mathematical expression of first law of thermodynamics is U = q + W.

Name of process Condition Mathematical expression of the first law


a. Isothermal T = 0 hence W = q [½]
U = 0
[½]
b. Adiabatic q=0 U = +W
[½]
c. Isochoric V = 0, W = 0 U = qV [V is constant, hence q = qv]
d. Isobaric P = Constant, U = qP + W
hence, q = qP U = qp  pex.V [ W =  pex.V]
[½]
or [qP = U + pex.V]

232
Chemistry
Q.3. Answer any THREE of the following:
i. Calculation:
We require to calculate the rate constant at different time intervals.
2.303 [A]
k= log10 0 ....[Integrated rate law for first order reaction]
t [A]t
a. [A]0 = 0.624
[A]t1 = 0.446
t1 = 600 s
2.303 [A]0
k1 = log [½]
t1 [A]t1

2.303 0.624
k1 = log
600 0.446
 k1 = 3.838  103 log [1.399]
 k1 = 3.838  103  0.1458
 k1 = 5.596  104 s1 [½]
2.303 [A]0
b. k2 = log10
t2 [A]t 2

[A]0 = 0.624
[A]t 2 = 0.318
t2 = 1200 s
2.303 0.624
k2 = log
1200 0.318
 k2 = 1.919  103 log (1.962)
 k2 = 1.919  103  0.2927
 k2 = 5.617  104 s1 [½]
2.303 [A]0
c. k3 = log10
t3 [A]t3

[A]0 = 0.624
[A]t3 = 0.226
t3 = 1800 s
2.303 0.624
k3 = log
1800 0.226
 k3 = 1.279  103 log [2.7610]
 k3 = 1.279  103  0.4411
 k3 = 5.641  104 s1 [½]
4 4 4
k1  k 2  k 3 (5.596  10 )  (5.617  10 )  (5.641  10 )
k= =
3 3
k = 5.618  104 s1 [½]
All the k values calculated at different time intervals are almost the same. This implies that
the reaction obeys the integrated rate equation of first order reaction. Hence, the reaction is
a first order reaction. [½]
233
Board Answer Paper : July 2017
ii. Given: C(s) + O2(g)  CO2(g) ; fH = –393.3 kJ mol–1 ...(1) [½]
1
H2(g) + O2(g)  H2O(l) ; fH = –285.8 kJ mol–1 ...(2) [½]
2
2C(s) + 2H2(g) + O2(g)  CH3COOH(l) ; fH = 483.2 kJ mol–1...(3) [½]
To find: The standard enthalpy of combustion of CH3COOH(l) (cH)
Calculation: The required equation is,
CH3COOH(l) + 2O2(g)  2CO2(g) + 2H2O(l) [½]
Multiplying equation (1) and equation (2) by 2, then adding to reverse of
equation (3). [½]
–1
2C(s) + 2O2(g)  2CO2(g) ; fH = –786.6 kJ mol ...(4)
–1
2H2(g) + O2(g)  2H2O(l) ; fH = –571.6 kJ mol ...(5)
–1
CH3COOH(l)  2C(s) + 2H2(g) + O2(g) ; fH= 483.2 kJ mol ...(6)
–1
2O2(g)  2CO2(g) + 2H2O(l) ; cH = –875 kJ mol
CH3COOH(l) +[½]
[½]

iii. Given: The Cell reaction at 25 C,



Ni(s) + 2Ag (aq) (1 M)  Ni (aq)
2
(1 M) + 2Ag(s)
E Ni = –0.25 V
E Ag = 0.799 V
To find: a. Cell representation
b. Equilibrium constant (K)
Calculation: Cell representation:
2 
Ni(s) | Ni (aq) (1 M) || Ag (aq) (1 M) | Ag(s) [1]
Calculation of equilibrium constant
0.0592
E cell = log10 K [½]
n
E cell = E Ag – ENi [½]
= 0.799  (–0.25) = 1.049 V
0.0592
Hence, 1.049 = log10 K
2
1.049  2
 log10 K = = 35.44 [½]
0.0592
 K = antilog (35.44)
= 2.754  1035 [½]

iv. a. Concentrated sulphuric acid reacts with PCl5 to form chlorosulphuric acid.
HOSO2OH + PCl5  ClSO2OH[½]+ POCl3 + HCl [½]
Sulphuric Phosphorus Chlorosulphuric
acid pentachloride acid

Chlorosulphuric acid further reacts with PCl5 to form sulphuryl chloride.


ClSO2OH + PCl5  ClSO2Cl[½] + POCl3 + HCl
Chlorosulphuric Phosphorus Sulphuryl
acid pentachloride chloride [½]

234
Chemistry
b. Sulphuric acid oxidizes metal Cu.
Cu + 2H2SO4  CuSO4 + SO2[1]
+ 2H2O [1]
Copper Sulphuric Copper Sulphur Water
acid sulphate dioxide

c. Sulphuric acid reacts with KClO3 to form KHSO4, HClO4 and ClO2.
3KClO3 + 3H2SO4  3KHSO4 + HClO4 + 2ClO2 + H2O [1]
Potassium Sulphuric Potassium Perchloric
chlorate acid hydrogen acid
sulphate

Q.4. Answer any ONE of the following:


i. a. Molality (m): Molality is defined as the number of moles of the solute dissolved in one
kilogram (kg) of the solvent. [1]
b. Osmotic pressure: The excess of pressure on the side of the solution that stops the
net flow of solvent into the solution through a semipermeable membrane is called
osmotic pressure. [1]
c. Advantages of calomel electrode:
1. It is convenient to handle, easy to construct and transport.
2. No separate salt bridge is required for its combination with other electrode.
3. The potential of the electrode is reproducible and remains constant.
(Any two advantages) [½  2] [1]
d. Given: Edge length of FCC unit cell (a1) = 3.5 Å
Edge length of BCC unit cell (a2) = 3.0 Å
d1 (FCC)
To find: Ratio of density of FCC and BCC =
d 2 (BCC)
ZM
Formula: Density of unit cell = d = g cm–3
NA  a3
4M
Calculation: Density of FCC unit cell = d1 = [½]
N A  (3.5Å)3
2M
Density of BCC unit cell = d2 = [½]
N A  (3.0Å)3

 Ratio of densities of FCC and BCC unit cell is,


d1 M N A  (3.0Å)3
=  [½]
d2 N A  (3.5 Å)3 2M

d1 54
= = 1.259  1.26 [½]
d2 42.875

e. 1. Increasing order of thermal stability:


HClO < HOClO < HOClO2 < HOClO3
OR
HClO < HClO2 < HClO3 < HClO4 [1]
2. Increasing order of oxidizing power:
HOClO3 < HOClO2 < HOClO < HClO
OR
HClO4 < HClO3 < HClO2 < HClO. [1]
235
Board Answer Paper : July 2017
ii. a. Given: T = 273.15 + 12 = 285.15 K
V = 2000 cm3 = 2 L [½]
M2 = 169 g mol–1
R = 0.0821 L atm K–1 mol–1
 = 0.54 atm
To find: Mass of solute (W2)
W2 RT
Formula: = [½]
M 2V
Calculation: From formula,
M 2 V
W2 =
RT
0.54  169  2
W2 = [½]
0.0821  285.15
182.52
W2 = = 7.796 g [½]
23.4108

b. 1. In face-centred cubic unit cell, eight constituent particles (spheres) are present
at eight corners of unit cell. Six constituent particles (spheres) are present at
centres of six faces.
2. The total number of atoms in a FCC unit cell
1
= 8 corner atoms  atom per unit cell + 6 atoms at the faces
8
1
 atom per unit cell
2
= 1 +3 = 4 [1]
c.
No. Isothermal process Adiabatic process
1. The temperature of system remains The temperature of system changes i.e.
constant (T = 0). increases or decreases (T  0).
2. System can exchange heat with the System cannot exchange heat with the
surroundings (q  0). surroundings (q = 0).
3. The internal energy of the system The internal energy of the system changes
remains constant (U = 0). (U  0).
4. System is not thermally isolated from System is thermally isolated from its
its surroundings. Hence, open or closed surroundings. Hence, isolated system is
system is required. required.
5. In this U = 0, hence q = W. In this q = 0, hence U = W.
6. Enthalpy remains constant (H = 0). Enthalpy changes in this process (H  0).
7. Expansion takes place by absorbing Expansion takes place by utilizing internal
heat from surroundings. energy of the system.
8. Heat supplied is used only for doing Work done is only at the cost of internal
work. energy.
9. Compression takes place by losing heat Compression takes place by adding heat into
to the surroundings. internal energy of the system.
eg. Fusion of ice. Expansion of gas in vacuum.
(Any four points) [½  4] [2]
d. The various steps involved in the extraction of pure metals from their ores are as follows:
1. Concentration of ores [½]
2. Conversion of ores into oxides or other desired compounds. [½]
3. Reduction of ores to form crude metals. [½]
4. Refining of metals [½]
236
Chemistry

SECTION – II

Q.5. Select and write the most appropriate answer from the given alternatives for each
sub-question:
i. (A) Propan-1-amine [1]
H H

C2H5  C = O + NH2OH  C2H5  C = NOH 


Na /C2 H5OH
Δ
 C2H5  CH2  NH2 + H2O
Propanal (oxime) Propan-1-amine
[A] [B]

ii. (B) 3 > 2 > 1 [1]


iii. (D) number of possible ligands around metal ion in complex [1]
EAN rule can be used to determine the number of possible ligands around a metal ion as the
ligands are added to a metal ion until the number of electrons around the metal ion reaches
to its EAN.
iv. (D) V2+ [1]
V (Z = 23): [Ar] 3d3 4s2
V2+ : [Ar]3d3
Due to the presence of 3 unpaired electrons, V2+ is coloured.
v. (C) 2,4,6-trinitrophenol [1]
OH OH
O2N NO2
+ 3HNO3 
conc.H 2SO4
+ 3H2O

Phenol (conc.)
NO2
Picric acid
(2,4,6-Trinitrophenol)
vi. (C) hydrogen bond [1]
vii. (B) acetaldehyde [1]
H H H

CH3  C  Cl + 2NaOH(aq) 
2 NaCl
CH3  C  OH  CH3  C = O + H2O
Cl OH Acetaldehyde
Ethylidene
dichloride Unstable

Q.6. Answer any SIX of the following:


i. Preparation of phenol from cumene:
a. When an alkaline solution of cumene (isopropyl benzene or 2-Phenylpropane) in
sodium carbonate is oxidised by passing air or oxygen in the presence of cobalt
naphthenate as a catalyst at 423 K, cumene hydroperoxide is obtained. [½]
CH3 CH3

H3C  C  H H3C  C  O  O H
[½]
O2  
Cobalt naphthenate
+ 423K /(alkaline medium)
(Air)
Cumene or Cumene hydroperoxide
(Isopropyl benzene
or 2-Phenylpropane)

237
Board Answer Paper : July 2017
b. Auto oxidation: Cumene hydroperoxide on heating with dilute H2SO4 decomposes
forming phenol and acetone. In this method, acetone is obtained as an important
byproduct which is separated by distillation.
CH3
H3C  C  O  O  H OH
O
[1]

dil.H 2SO 4

 + CH3  C  CH3
Phenol Acetone
Cumene
hydroperoxide

ii. a. Self oxidation and reduction (disproportionation) of aldehydes (which do not possess
an hydrogen atom) in the presence of concentrated alkali (aqueous or alcoholic) is
known as Cannizzaro reaction. The reaction products are alcohol and salt of
carboxylic acid. [½]
b. In this reaction, alcohol is obtained due to reduction of one molecule of aldehyde and
carboxylic acid is obtained due to oxidation of another molecule of an aldehyde. The
carboxylic acid forms a salt with alkali. [½]
eg. H H H O
 
H  C = O + H  C = O + NaOH  H  C OH + H  C  ONa
[1]
Formaldehyde (50%) Sodium formate
H
Methyl alcohol

iii. Nitrogen atom of amines contains a lone pair of electrons which can be donated. Thus, amines
act as Lewis bases. Amines are Lowry-Bronsted bases as they accept a proton. Thus, amines act
as bases and nucleophiles.

R  NH2 + HX  R NH 3 X  [1]



Amine Alkyl ammonium halide
Halogen acid
eg. a. The reaction of ethylamine with dilute hydrochloric acid results in the
formation of ethyl ammonium chloride.
 CH3  CH2  NH 3 Cl
CH3  CH2  NH2 + HCl 
Ethylamine Ethyl ammonium
(Ethanamine) chloride

b. The reaction of aniline with dilute hydrochloric acid results in the formation of
anilinium chloride.
NH2 NH 3 Cl

+ HCl 


Aniline Anilinium chloride


(Any one example) [1]
iv. Antiseptics: Drugs which are applied to the living tissues to kill the bacteria and to stop
their growth in wound, thus preventing its infection are called antiseptics. [1]
eg. Dettol, phenol (dilute solution), bithionol, boric acid, hydrogen peroxide, iodoform,
soframycin, etc.
(Any two examples) [½  2] [1]

238
Chemistry
v. a. Action of hydroxylamine (NH2OH) on glucose: The reaction of glucose with
hydroxylamine gives an oxime. This indicates the presence of carbonyl group.
CHO CH = N  OH
NH OH
(CHOH)4 
2  (CHOH)4 + H2O
CH2OH CH2OH
Glucose Glucoxime [1]

b. Action of hydrogen cyanide (HCN) on glucose: The reaction of glucose with


hydrogen cyanide gives cyanohydrin. This indicates the presence of carbonyl group.
CN
CHO CHOH
(CHOH)4 HCN
 (CHOH)4
CH2OH CH2OH [1]
Glucose Glucose cyanohydrin

vi. Structure of chromate and dichromate ions:


O 2

Cr
O
O
O [1]
Chromate ion

2
O O O
O Cr 126 Cr O
O O [1]
Dichromate ion
vii. a. Dacron (Terylene) is obtained by the condensation polymerization of ethylene glycol
(1,2-Ethanediol) and dimethylterephthalate.
b. Monomers are heated at 503 K. The catalyst used is a mixture of zinc acetate and
antimony trioxide.
c. Transesterification followed by polymerization gives terylene.
O O
Catalyst,Δ
n HO  CH2  CH2  OH + n H3C O  C C  O  CH3 
CH OH

3

Ethylene glycol DMT (Dimethylterephthalate)


O O
Polymerisation
O  CH2  CH2  O C C  

O O
Repeating unit
O  CH2  CH2  O C C
n
Polyester

(Explanation + Two steps) [1  2] [2]


239
Board Answer Paper : July 2017
viii. CH3  Br + Mg 
dry ether
 CH3  MgBr
Methyl bromide Methyl magnesium
bromide (A)

CH3  MgBr + CO2 
dry ether
 CH3  C  OMgBr 
H / H2O
 CH3COOH + Mg(Br)OH
(A) (dry ice) Acetic acid (B)
O
Adduct

A  CH3MgBr (Methyl magnesium bromide) [1]


B  CH3COOH (Acetic acid) [1]
Q.7. Answer any THREE of the following:
i. Ligands are classified as mono or unidentate, poly or multidentate and ambidentate ligands:
a. Mono or unidentate ligands: The ligand molecule or ion which has only one donor
atom or one point of attachment and can coordinate with the metal ion at only one
site in a complex is called unidentate or monodentate ligand.
eg. Cl, OH–, NH3, H2O, etc. (Explanation + Example) [1]
b. Poly or multidentate ligands: The ligand molecule or ion which has two or more
donor atoms or points of attachments and can be linked to the same metal in a
complex using two or more donating sites is called poly or multidentate ligands.
Thus, multiple sites of ligands are used in the coordination with metal. Based on the
number of donor atoms, polydentate ligands are further classified as bidentate,
tridentate, tetradentate, etc.
1. Bidentate ligands have two donor atoms.
eg. Ethylenediammine (en)
2. Tridentate ligands have three donor atoms.
eg. Diethylenetriammine (dien)
3. Tetradentate ligands have four donor atoms.
eg. Triethylenetetraammine (trien)
4. Hexadentate ligands have six donor atoms.
eg. Ethylenediamminetetraacetate (EDTA)
(Explanation + Examples) [1]
c. Ambidentate ligands: Ambidentate ligands are the ligands which have two or more
donor atoms capable of forming coordinate bonds; however only one donor atom is
utilized during complex formation.
eg. NO 2 group can form complexes by utilizing either N or O as donor atom, but
not both. This results in formation of either MNO2 or MONO complex
respectively.
(Explanation + Example) [1]
ii. a. Lanthanoid contraction:
The atomic and ionic radii of lanthanoids show gradual decrease with increase in
atomic number. It is known as Lanthanoid contraction. [1]
0 1 2
b. 57La  Electronic configuration is [Xe]4f 5d 6s [½]
After losing 3 electrons La forms La3+ ion which is stable due to empty 4f-orbitals. [½]
2 0 2
58Ce  Electronic configuration is [Xe]4f 5d 6s [½]
4+
After losing 4 electrons Ce forms Ce ion which is stable due to empty 4f-orbitals. [½]
Thus, lanthanum (Z = 57) forms La3+ ion, while cerium (Z = 58) forms Ce4+ ion.
iii. a. Propanone on reaction with phenyl hydrazine produces acetonephenylhydrazone.
CH3 CH3

CH3  C = O + H2N  NH  C6H5  CH3  C = N  NH  C6H5 + H2O [1]


Propanone Phenyl hydrazine Acetonephenylhydrazone

240
Chemistry
b. Propanone on reduction with zinc-amalgam and concentrated hydrochloric acid gives
propane.
Zn  Hg  conc.HCl
CH3  CO  CH3 + 4[H]  
 CH3  CH2  CH3 + H2O
Propane [1]
Propanone

c. Propanone, when shaken with a saturated aqueous solution of sodium bisulphite


gives acetone sodium bisulphite.
CH3 CH3
CH3  C = O +  CH3  C  OH
NaHSO3(aq) 
Propanone Sodium bisulphite [1]
SO3Na
Acetone sodium bisulphite

iv. a. Enzymes are defined as biocatalysts, produced by the living cells which catalyse
many biochemical reactions in animals and plant bodies. [1]
b. 1. -Amino acids are bifunctional compounds containing a carboxylic acid group
and an amino group on -carbon.
H2N  CHCOOH
R
-Amino acid
(R = H or alkyl or aryl group)
2. The reaction of the COOH group of one amino acid molecule and NH2 group
of the neighbouring amino acid molecule forms peptide having CONH
linkage by elimination of water.
R1 O H R2 O R1 R2
 H2O
H2N  CH C OH + H N  CH  C  OH 
 H2N  CH CO  NH  CH  COOH
-Amino acid -Amino acid Peptide linkage
(Dipeptide)
3. Above reaction repeats itself to give tri, tetra, penta and finally polypeptides
(i.e., protein).
(Explanation + Reaction) [2]
Q.8. Answer any ONE of the following:
i. a. 1. Nitroethane reduced to ethylamine by the action of tin and conc. HCl.
CH3CH2NO2 + 6[H]  [1]  CH3CH2NH2 + 2H2O
Sn /conc.HCl

Nitroethane Ethylamine [1]

2. When nitroethane is reduced in a neutral medium with metal, it forms


N-ethylhydroxylamine.
CH3CH2NO2 + 4[H]  [1]
Zn/NH 4Cl
 CH3CH2NHOH + H2O
Nitroethane N-Ethylhydroxylamine [1]

3. Nitroethane undergoes hydrolysis on boiling with hydrochloric acid to give


acetic acid along with hydroxylamine.
CH3CH2NO2 + H2O 
HCl
 CH3COOH + NH2OH [1]

Nitroethane Acetic acid Hydroxylamine
b. Preparation of Buna-N:
Name of monomers Formula
1. Buta-1,3-diene CH2 = CH  CH = CH2
2. Acrylonitrile CH2 = CH  CN
(Names) [½  2] [1]
(Formulae) [½  2] [1]
241
Board Answer Paper : July 2017
c. Soaps are sodium or potassium salts of higher fatty acids which contain more than
12 carbon atoms. [1]
Preparation of soaps:
1. By hydrolysis of fat using sodium or potassium hydroxide solution
(saponification):
When oils and fats are heated with solution of sodium hydroxide or potassium hydroxide,
they are hydrolysed to glycerol and sodium (or potassium) salt of fatty acid (known as
soap). This process is called saponification.
O
CH2  O  C  R
O CH2OH O
 

CH  O  C  R + 3NaOH   CHOH + 3RC ONa
Soap
O CH2OH
CH2  O  C  R Glycerol
Fat/oil
After saponification, common salt (NaCl) is added to the aqueous solution. This
decreases the solubility of soap due to which it precipitates. Soap being lighter, floats
on the surface of water and can be separated.
2. Direct neutralization of fatty acids: Soaps are prepared by direct neutralization of
fatty acids by using sodium hydroxide or sodium carbonate.
O O
 
RCOH + NaOH  RC ONa + H2O
Higher fatty acid Soap (Any one method) [1]
ii. a. 1. Methanal reacts with methyl magnesium iodide in presence of dry ether to give
an addition compound, which on acid hydrolysis gives ethanol.

H H
H  C = O + CH3  Mg  I 
dry ether
 CH3  C  O  Mg  I
Methanal Methyl
magnesium H
iodide Addition compound


H /H  OH
I
  CH3  CH2  OH + Mg
Ethanol OH [1]
2. Ethanal reacts with methyl magnesium iodide in presence of dry ether to give
an addition compound, complex which on acid hydrolysis gives propan-2-ol.

CH3
CH3  C = O + CH3  Mg  I 
dry ether
 CH3  C  O  Mg  I
Methyl
H magnesium H
Ethanal iodide Addition compound
CH3
H  /H  OH
I

 CH3  CH  OH + Mg
Propan-2-ol OH
[1]

242
Chemistry
3. Propanone reacts with methyl magnesium iodide in presence of dry ether to give
an addition compound, which on acid hydrolysis gives 2-methylpropan-2-ol.
CH3 CH3
CH3  C = O + CH3  Mg  I 
dry ether
 CH3  C  O  Mg  I
Propanone Methyl
magnesium CH3
iodide Addition compound

CH3
H  /H  OH
I

 CH3  C  OH + Mg
OH
CH3
2-Methylpropan-2-ol
[1]
b. Optical activity (Chirality): The property of a substance to rotate the plane of a
plane polarised light towards the right (clockwise) or towards the left (anticlockwise)
is called optical activity. [1]
c. The optical activity of lactic acid can be discussed as:
1. Presence of asymmetrical carbon atom:
i. Lactic acid contains one asymmetrical carbon atom.
(Number of asymmetric carbon atom) [½]
ii. According to van’t Hoff’s rule: a = 2n, where, a is the number of isomers and n
is the number of asymmetric carbon atom.
iii. Thus, two isomers of lactic acid are possible. (Number of isomers) [½]
2. Non-superimposable mirror image structures:

COOH COOH

H  C  OH HO  C*  H

CH3 CH3
d or (+) lactic acid  or () lactic acid
Mirror

(Two enantiomer structures) [1]


Hence, lactic acid can exist as d-form and l-form which are non-superimposable mirror
images of each other.
(Mentioning ‘d’ and ‘l’ forms) [½]
3. (dl) Racemic mixture and its optical inactivity:
A mixture containing equal moles of the d and l forms of lactic acid is a racemic mixture
which is optically inactive i.e., dl or () form. This inactivity arises due to external
compensation.
(Explanation of racemic mixture) [½]

243
Board Answer Paper : July 2017

BOARD ANSWER PAPER : JULY 2017


MATHEMATICS AND STATISTICS

Note: Answer to every question must be written on a new page.

SECTION - I

Q.1. (A) Select and write the appropriate answer from the given alternatives in each of the
following sub-questions:
i. (B)
a b  1 1  d b 
If A =   , then A = ad  bc   c
 c d   a 
1  3 1
 A= [2]
5  2 1
ii. (C)
3 1 4

a b  c =  2 3 1
5 2 3
= 3(9 + 2) + 1 (6 – 5) + 4 (4 + 15)
= 33 + 1 + 76
= 110 [2]
iii. (D)
Let , ,  be the angles made by the line with positive directions of X, Y, Z axes
respectively.
  = 90,  = 135,  = 45
 l = cos 90, m = cos 135, n = cos 45
Now, m = cos 135 = cos(180 – 45)
1
= cos 45 = 
2
1 1
 l = 0, m =  ,n=
2 2
1 1
 Direction cosines of the line are 0,  , [2]
2 2
(B) Attempt any THREE of the following:
i. b = 2iˆ  ˆj  2kˆ
n̂ = 3iˆ  2ˆj  pkˆ
Since, line is parallel to the plane
b  n̂ = 0 [1]
  2iˆ  ˆj  2kˆ    3iˆ  2ˆj  pkˆ  = 0
 6 – 2 + 2p = 0
 p=–2 [1]
ii. Consider cos 2 + cos 2 + cos 2
= (2cos2 1)+(2cos2 1)+(2cos2   1)
= 2(cos2  + cos2  + cos2 )  3 [1]
= 2(l)  3 …[  cos2  + cos2  + cos2  = 1]
= 1
 cos 2 + cos 2 + cos 2 = –1.
i.e., cos 2 + cos 2 + cos 2 + 1 = 0 [1]
244
Mathematics and Statistics
iii. (a)  n  N such that n + 7  6 [1]
(b) The kitchen is not neat or it is not tidy. [1]
iv. Let  be the acute angle between the lines whose direction ratios are 4, –3, 5 and 3, 4, 5.
Then,
a1a 2  b1b 2  c1c 2
cos  =
a  b12  c12  a 22  b 22  c 22
2
1

4  3   3 4   5  5 
 cos  = [1]
4   3  52  32  42  52
2 2

12  12  25
=
16  9  25  9  16  25
25 1
= =
50 2

 cos  = cos
3

 =
3

 The angle between the lines is . [1]
3

v. 3 a + 5 b – 8c = 0 ….(given)
 3a = 8c 5 b
8c - 5b
 a= [1]
3
8c - 5b
 a=
8-5
 A  a  divides BC externally in the ratio 8 : 5. [1]

Q.2. (A) Attempt any TWO of the following:



i. tan1 2x + tan1 3x =
4
  x  3x  
 tan1   = [1]
 1   2 x  3 x   4
5x 
 = tan
1  6x2 4
5x
 =1
1  6x2
 5x = 1  6x2
 6x2 + 5x 1 = 0 [1]
 6x2 + 6x x  1 = 0
 6x(x + 1)  1(x + 1) = 0
 (x + 1)(6x  1) = 0
1
 x = 1 or x =
6

But x = 1 does not satisfy tan1 2x + tan1 3x =
4
1
 x= [1]
6
ii. Let p : It rains, q : the match will be cancelled.
 The symbolic form of the given statement is p  q.
Converse: q  p
245
Board Answer Paper : July 2017
i.e., If the match is cancelled then it rains. [1]
Inverse: ~p  ~q
i.e., If it does not rain then the match will not be cancelled. [1]
Contrapositive: ~q  ~p
i.e. If the match is not cancelled then it does not rain. [1]
iii. Given equation is px2  8xy + 3y2 + 14x + 2y + q = 0
Comparing with
ax2 + 2hxy + by2 + 2gx + 2fy + c = 0, we get
a = p, h =  4, b = 3, g = 7, f = 1, c = q.
The given equation represents a pair of lines perpendicular to each other
 a+b=0
 p+3=0
 p = 3 [1]
Also, the given equation represents a pair of lines
a h g
 h b f =0
g f c
-3 -4 7
 -4 3 1 = 0 [1]
7 1 q
 3(3q  1) + 4(  4q  7) + 7( 4  21) = 0
 9q + 3  16q  28  175 = 0
 25q  200 = 0
 25q = 200
 q = 8 [1]
 p = 3 and q = 8
(B) Attempt any TWO of the following:
i. The equation of plane passing through the intersection of the planes 3x + 2y – z + 1 = 0
and x + y + z – 2 = 0 is
(3x + 2y – z + 1) + (x + y + z – 2) = 0 …. (i) [1]
It passes through the point (2, 2, 1)
 (6 + 4 – 1 + 1) +  (2 + 2 + 1 – 2) = 0 [1]
 10 + 3 = 0
10
 = [1]
3
 10 
Now, (3x + 2y – z + 1) +   (x + y + z – 2) = 0 .…[From (i)]
 3 
 9x + 6y – 3z + 3 – 10x – 10y – 10z + 20 = 0
 –x – 4y – 13z + 23 = 0
 x + 4y + 13z – 23 = 0 [1]
 The equation of plane is
x + 4y + 13z = 23
ii. O

b
a r

A m n
R B
Consider a line segment AB.
Let R be any point on it such that point R divides AB internally in the ratio m : n.
246
Mathematics and Statistics
OA = a, OR = r and OB = b are the position vectors of points A, R, B respectively.
Since point R divides AB internally in the ratio m : n,
AR m
 = [1]
RB n
 n(AR) = m(RB)
AR and RB are in the same direction.
 n (AR) = m (RB) [1]
 n (r  a) = m(b  r)
 nr  na = mb  mr
 r (mn) = mb na

mb  na
 r= ….(i) [1]
mn
This is the section formula for internal division.
Let P. V. of point A a = i  2j  k
P. V. of point B b = i  4j  2k
m 2
Given, 
n 1
2(i  4j  2k)
 1(i  2j  k)

Now, r = ....[From (i)]
2 1
3i  6j  3k
 r =
3
P. V. of R is r  i  2j  k [1]

iii. A, B, C are in A.P. and b : c = 3: 2


 2B = A + C [1]
 2B = 180  B ….[ A + B+ C = 180]
 3B = 180
 B = 60 [1]
In ABC by sine rule, we have
sin B sin C
=
b c
sin B b
 =
sin C c
sin 60 3
 =
sin C 2
3 3
 =  sin C
2 2
1
 sin C = [1]
2
 C = 45
 A = 180  60  45 = 75
Thus, the angles of ABC are A = 75, B = 60, C = 45. [1]
Q.3. (A) Attempt any TWO of the following:
i. The vector equation of a line passing through the points having position vectors a and b is
given
by r  a    b  a 
Here, a  3iˆ + 4 ˆj  7 kˆ and b  6iˆ  ˆj  kˆ
247
Board Answer Paper : July 2017
 the vector equation of the line passing through A (3, 4, 7) and B (6, 1, 1) is
r  (3iˆ + 4 ˆj  7 kˆ )   [(6iˆ  ˆj  kˆ )  (3iˆ+ 4 ˆj  7 k)]
ˆ

 r  (3iˆ + 4 ˆj  7 k)ˆ   (3iˆ  5 ˆj  8k) ˆ ….(i) [1]


For cartesian form:
Putting r  x ˆi + y ˆj+ z kˆ in (i), we get
x ˆi + y ˆj  z kˆ  (3iˆ  4 ˆj  7 k)
ˆ   (3iˆ  5 ˆj  8k)
ˆ

 xˆi + yˆj + zkˆ = (3 + 3 )iˆ + (4  5 )ˆj + ( 7  8 )kˆ [1]


Equating the coefficients of ˆi, ˆj and k̂ , we get
x = 3 + 3, y = 4  5, z =  7 + 8
x 3 y4 z7
  ,  , 
3 5 8
x 3 y 4 z 7
   , which is the cartesian form of the equation of line. [1]
3 5 8
ii. cot x + tan x = 2cosec x
cos x sin x 2
 + =
sin x cos x sin x
cos 2 x  sin 2 x 2
 
sin x  cos x sin x
 cos2x + sin2 x = 2cos x
 2 cos x = 1
1
 cos x = [1]
2

 cos x = cos   [1]
3
Since, cos  = cos  implies  = 2n  , n  Z.

 the required general solution is x = 2n  , where n  Z. [1]
3
iii. Let p : The switch S1 is closed
q : The switch S2 is closed
p : The switch S1 is open
q : The switch S2 is open
The logical expression of the given switching circuit is (p  q)  (p  q) [1]
The switching table is:
p q p q p  q p  q (p  q)  (p  q)
1 1 0 0 1 0 1
1 0 0 1 1 0 1
0 1 1 0 0 1 1
[1]
0 0 1 1 1 0 1
From the last column of switching table we conclude that the current will always flow
through the circuit. [1]
(B) Attempt any TWO of the following:
é 1 -1 2ù
ê ú
i. A = êê3 0 -2úú
ê1 0 3úû
ë
1 -1 2
 |A| = 3 0 -2 = 1(0) + 1(9 + 2) + 2(0)
1 0 3
= 0 + 11 + 0
248
Mathematics and Statistics
 |A| = 11 [1]
0 -2
A11 = (1)1 + 1 M11 = 1 = 1(0  0) = 1  0 = 0
0 3
3 -2
A12 = (1)1 + 2 M12 = 1 = 1(9 + 2) = 11
1 3
3 0
A13 = (1)1 + 3 M13 = 1 = 1(0  0) = 0
1 0
-1 2
A21 = (1)2 + 1 M21 = 1 = 1(3  0) = 3
0 3
1 2
A22 = (1)2 + 2 M22 = 1 = 1(3  2) = 1
1 3
1 -1
A23 = (1)2 + 3 M23 = 1 = 1(0 + 1) = 1
1 0
-1 2
A31 = (1)3 + 1 M31 = 1 = 1(2  0) = 2
0 -2
1 2
A32 = (1)3 + 2 M32 = 1 = 1(2  6) = 8
3 -2
1 -1
A33 = (1)3 + 3 M33 = 1 = 1(0 + 3) = 3
3 0
Hence, matrix of the co-factors is
é A11 A12 A13 ù é 0 -11 0ù
ê ú ê ú
ê A 21 A 22 A 23 úú =ê3 1 - 1ú =  A ij 
ê ê ú  3  3
êA A 33 úû ê2 ú
ë 31 A 32 ë 8 3û
é 0 3 2ù
T ê ú
Now, adj A =  Aij  3  3 = ê -11 1 8úú
ê [1]
ê 0 -1 3ú
ë û
é 1 -1 2ù é 0 3 2ù
ê ú ê ú
 A(adj A) = êê3 0 -2úú êê-11 1 8úú
ê1 0 3úû êë 0 -1 3úû
ë
é 0 +11+ 0 3 -1 - 2 2 - 8 + 6ù
ê ú
= êê 0 + 0 + 0 9 + 0 + 2 6 + 0 - 6úú
ê 0 + 0 + 0 3 + 0 - 3 2 + 0 + 9ú
ë û
é11 0 0 ù
ê ú
= êê 0 11 0 úú .…(i) [1]
ê 0 0 11ú
ë û
é 1 0 0ù é11 0 0 ù
ê ú ê ú
|A|  I = 11 ê0 1 0ú = êê 0 11 0 úú
ê ú ….(ii)
ê 0 0 1ú ê 0 0 11ú
ë û ë û
From equations (i) and (ii), we get
A(adj A) = |A| I [1]
ii. Let x number of bicycles and y number of tricycles be manufactured by the company.
 Total profit Z = 180x + 220y
This is the objective function to be maximized. [1]
The given information can be tabulated as shown below:
Bicycles (x) Tricycles (y) Maximum availability of time (hrs)
Machine A 6 4 120
Machine B 3 10 180

249
Board Answer Paper : July 2017
 The constraints are 6x + 4y  120, 3x + 10y  180, x  0, y  0
 Given problem can be formulated as
Maximize Z = 180x + 220y
Subject to, 6x + 4y  120, 3x + 10y  180 , x  0, y  0. [1]
To draw the feasible region, construct the table as follows:

Inequality 6x + 4y  120 3x + 10y  180


Corresponding
6x + 4y = 120 3x + 10y = 180
equation (of line)
Intersection of line with X-axis (20, 0) (60, 0)
Intersection of line with Y-axis (0, 30) (0, 18)
Region Origin side Origin side

Shaded portion OABC is the feasible region, whose vertices are O  (0, 0), A  (20, 0), B and
C  (0, 18)
B is the point of intersection of the lines 3x + 10y = 180 and 6x + 4y = 120.
Solving the above equations, we get
Y
B  (10, 15)
Here the objective function is, 40
Z = 180x + 220y
 Z at O(0, 0) = 180(0) + 220(0) = 0 30
Z at A(20, 0) = 180(20) + 220(0) 20
= 3600 B(10, 15)
C(0,18)
Z at B(10, 15) = 180(10) + 220(15) 10
= 5100 A(20, 0)
X X
Z at C(0, 18) = 180(0) + 220(18) O 10 20 30 40 50 60
3x + 10y = 180
= 3960

 Z has maximum value 5100 at 6x + 4y = 120


Y [1]
B(10, 15)
 Z is maximum when x = 10, y = 15
Thus, the company should manufacture 10 bicycles and 15 tricycles to gain maximum
profit of `5100. [1]
iii. The given combined equation of lines is
ax2 + 2hxy + by2 = 0
Let m1 and m2 be the slopes of the lines represented by ax2 + 2hxy + by2 = 0
2h
 m1 + m2 =  and m1m2 = a , b  0
b b
If a = 1, then m1m2 = 1.
b
 lines are perpendicular. [1]
So we assume that a  1
b
Y
y = m2x
y = m1x

X X
O

Y
250
Mathematics and Statistics
Now, (m1  m2)2 = (m1 + m2)2  4m1.m2
2
4a
=    
2h
 b  b
4h 4a
2
= 2 
b b
4h 2  4ab 4  h 2  ab 
 (m1  m2)2 = =
b2 b2
Taking square root on both the sides, we get
2 h 2  ab
m1  m2 =  [1]
b
Let  be the acute angle between the lines.
2 h 2  ab
m1  m 2  a
 tan  = = b ,  1
1  m1.m 2 1
a b
b
2 h 2  ab
 tan  = ,a+b0 [1]
ab
The given pair of lines are x2 – 4xy + y2 = 0
Comparing with ax2 + 2hxy + by2 = 0, we get
a = 1, h = –2, b = 1
2 h 2  ab 2 4 1
tan  = 
ab 1 1

2 3
=
2
 tan  = 3

  = 60 or [1]
3

SECTION - II

Q.4. (A) Select and write the appropriate answer from the given alternatives in each of the
following sub-questions:
i. (D)
 2x , x < 0
f(x) = 
 0, x  0
lim f(x) = lim 2x = 0
x  0 x  0

lim f(x) = lim 0 = 0


x  0 x  0

and f(0) = 0
 lim f(x) = lim f(x) = f(0)
 
x 0 x 0

Hence, f(x) is continuous at x = 0.


Now we find left hand derivative and right hand derivative of f(0) at x = 0
Right hand derivative at x = 0
f  0 + h   f  0 00
i.e., f (0+) = lim = lim =0
h 0 h h 0 h
Left hand derivative at x = 0
f  0 + h   f  0 h0
i.e., f (0) = lim = lim
h 0 h h 0 h
h
= lim = 1
h 0 h

251
Board Answer Paper : July 2017
 f (0+)  f (0)
Hence, f(x) is not differentiable at x = 0. [2]
ii. (B)

  3x 
2
 2 x  1 dx = 14
0

  x3  x 2  x  = 14
 0
 3 + 2 +  – 14 = 0
 ( – 2) (2 + 3 + 7) = 0
But 2 + 3 + 7 = 0 does not have real roots
 =2 [2]

iii. (A)
f (x) = x3 – 3x2 + 3x – 100, x  R
 f (x) = 3x2 – 6x + 3
= 3(x2 – 2x + 1)
= 3(x – 1)2
Since, (x – 1)2 is always positive x ≠ 1
 f (x) > 0 for all x  R, x ≠ 1
Hence, f (x) is an increasing function, for all x  R, x ≠ 1 [2]
(B) Attempt any THREE of the following:
i. Let u = 3x
du
 = 3x log 3
dx
log x
Let v = log3 x =
log3
dv 1 d
 = (log x)
dx log3 dx
1 1 1
=  = [1]
log3 x x log3
æ du ö÷
çç ÷
du çè dx ÷ø 3x log 3
 = = = 3xx (log 3)2 [1]
dv æ dv ö÷ 1
çç ÷
çè dx ø÷ x log3

ii. f(x) = (x  1)(x  2)(x 3), x  [1, 3]


= x3  6x2 + 11x  6
As f(x) is a polynomial function, it is continuous and differentiable everywhere on its
domain. Thus,
a. f(x) is continuous on [1, 3]
b. f(x) is differentiable on (1, 3) [1]
Further, f(1) = 0 and f(3) = 0
 f(1) = f(3)
Thus, all the conditions of Rolle’s theorem are satisfied. [1]

tan x
tan x cos 2 x dx
iii. Let I =  dx =  sin x  cos x
sin x  cos x
cos 2 x
sec 2 x tan x
=  tan x
dx [1]

252
Mathematics and Statistics
sec 2 x
=  tan x
dx

 f (x) 
 I = 2 tan x + c …. 

 f ( x)
dx  2 f ( x )  c 

[1]

iv. Y

y=6

y=2

X X
O
Y
y varies from y = 2 to y = 6.
Equation of parabola x2 = 16y
 x=4 y
b
Required area =  xdy
a
6
=  4 y dy [1]
2
6
 3
 y2 
= 4 
3
 
 2  2

2 3 3
= 4  (6) 2  2 2 
3 
8  2 32 
3
= 6  2  sq. units [1]
3 
v. Given, n = 10, p = 0.4
q = 1 – p = 1 – 0.4 = 0.6
Now, E(X) = np = 10  0.4 = 4 [1]
Var(X) = npq = 10  0.4  0.6 = 2.4 [1]
Q.5. (A) Attempt any TWO of the following:
i. f is continuous at x = 0.
 f(0) = lim f(x) [1]
x 0

5  1
2 2
sin x
 5sin x 1  sin 2 x
  2
5  1 
sin x 2
sin x  x
 f(0) = lim = lim x2 = lim  [1]
x 0 x log 1  2 x  x 0 x  log 1  2 x  x 0 log(1 2 x)
2
x2 2x
2
 5sin x  1 sin x 
 lim  lim 
x  0 sin x x 0 x 
= 
log 1  2 x 
2lim
x 0 2x
 log5
2

 f(0) = [1]
2
ii. E(X) =  x  P( x )
i i

= 0(0.08) + 1(0.15) + 2(0.45) + 3(0.27) + 4(0.05)


= 0 + 0.15 + 0.9 + 0.81 + 0.2
= 2.06 [1]
253
Board Answer Paper : July 2017
E(X2) = x 2
i  P( xi )
= 02(0.08) + 12(0.15) + 22(0.45) + 32(0.27) + 42(0.05)
= 0(0.08) + 1(0.15) + 4(0.45) + 9(0.27) + 16(0.05)
= 0 + 0.15 + 1.8 + 2.43 + 0.8
= 5.18 [1]
Var (X) = E(X2)  [E(X)]2
= 5.18  (2.06)2
= 5.18  4.2436
= 0.9364 [1]
iii. X = Number of families who own a television set.
P = Probability of families who own a television set.
80 4
P = 80% = =
100 5
4 1
q=1–p=1– =
5 5
 4
Given, n = 5, X  B  5, 
5  
The p.m.f. or X is given as
P(X = x) = nCx px qn  x
= nCx (p)x (q)5  x [1]
a. P(families own television set)
= P(X = 3)
3 53
4 1
= 5C3    
 5 5
128
=
625
= 0.2048 [1]
b. P(At least two families own television set)
= P(X  2) = 1  P(X < 2)
= 1 – [P(X = 0) + P(X = 1)]
 0
4 1
5 1
4 1 
4

= 1   5 C0      5C1      
 5 5  5   5  
 1 20 
=1–   
 55 55 
21 3104
=1–  [1]
3125 3125
(B) Attempt any TWO of the following:
i. Let f(x) = cos x
 f (x) = – sin x [1]

1
x = 60 30 = 60 + =a+h
2

Here, a = 60 =
3
1  0.0175
and h = = = 0.00875
2 2
  1
f(a) = f   = cos   = = 0.5
3 3 2
 
f (a) = f    = –sin   = – 0.8660 [1]
 3 3  
f(a + h)  f(a) + hf(a) [1]
254
Mathematics and Statistics
 cos(60 30)  0.5 + (0.00875) (– 0.8660)
 0.5 – 0.0075775
 0.4924 [1]
ii. Let ‘N’ be the number of bacteria at time’t ’
dN
 N
dt
dN
 = kN
dt
dN
 = kdt
N
Integrating on both sides, we get
dN
 N
= k  dt
 log N = kt + c [1]
when t = 0, N = 1000
c = log 1000 [1]
 log N = kt + log 1000
 N 
 log   = kt
 1000 
 N = 1000 ekt ….(i)
when t = 1 hour, N = 2000
 ek = 2 [1]
 N = 1000  (2)t ….[From(i)]
1
when t = 2 hours, we get
2
5
N = 1000   2  2
= 1000  4  2 = 4000  1.414
N = 5656
1
Number of bacteria present after 2 hours is 5656. [1]
2
iii. Since, – a < 0 < a
a 0 a
  f ( x)dx =
a
 f ( x)dx +
a
 f ( x)dx
0
[1]
a a
  f ( x)dx
a
= I +  f ( x)dx ….(i)
0
0
Now, I =  f ( x)dx
a

Put x = t
 dx =  dt
When x = a, t = a and when x = 0, t = 0
0 0
 I =  f ( t)(dt) =   f ( t)dt
a a
a
 b a

=  f ( t)dt ….   f ( x)dx    f ( x)dx 
0  a b 
a
 b b

=  f (  x)dx ….   f ( x)dx   f (t)dt 
0  a a 
 Equation (i) becomes
a a a


a
f ( x)dx =  f ( x)dx +
0
 f ( x)dx
0

255
Board Answer Paper : July 2017
a
=  f ( x)  f ( x) dx
0
….(ii) [1]

If f(x) is an even function, then f(x) = f(x).


Thus, equation (ii) becomes
a a a

 f ( x)dx =  f ( x)  f ( x) dx = 2  f ( x)dx


a 0 0
[1]

If f(x) is an odd function, then f(x) = f(x).


Thus, equation (ii) becomes
a a

 f ( x)dx =
a
  f ( x)  f ( x) dx = 0
0
[1]

Q.6. (A) Attempt any TWO of the following:


i. Since f is continuous on [4, 2],
 f is continuous on x = 2
 lim f  x   lim f  x  [1]
x 2 x 2

 lim 6b  3ax  lim 4 x +1


x 2 x 2

 6b  3a(2) = 4(2) + 1 [1]


 6b + 6a = 7
 6(a + b) = 7
7
 a+b= [1]
6
ii. Let  vdx  w
dw
 =v
dx
d dw du
Consider, (uw) = u +w [1]
dx dx dx
du
= uv +  vdx 
dx
du
dx 
= uv + vdx

Integrating on both sides w.r.t. ‘x’, we get


 du 
uw =  u vdx     vdx  dx [1]
 dx
 du 
 u  v dx=  uvdx     vdx  dx
 dx 
 d 
  u  v dx = u  vdx    dx (u)   vdx  dx [1]

iii. By definition cummulative distribution function at x is


P(x ≥ 2) = 0.3 + 0.4 + 0.2 = 0.9 [1]
f (xi) = P1 + P2 + P3 + ……. + Pi where, i = 1, …, x
Thus f (x1) = P1 = 0.1
f (x2) = P1 + P2 = 0.1 + 0.3 = 0.4
f (x3) = P1 + P2 + P3 = 0.1 + 0.3 + 0.4 = 0.8
f (x4) = P1 + P2 + P3 + P4 = 0.1 + 0.3 + 0.4 + 0.2 = 1 [1]
4
 f (x4) = p
i 1
i =1

 Cumulative distribution function of X is


1 2 3 4
F(x) 0.1 0.4 0.8 1
[1]

256
Mathematics and Statistics
(B) Attempt any TWO of the following:
dy
i.  y = ex
dx
dy
The given equation is of the form + Py = Q
dx
Where, P = –1 and Q = ex
I.F. = e   e 
pdx 1 dx
 e x [1]
 Solution of the given equation is
y (I.F.) =  Q (I.F.) dx + c
 y. e x =  e x .e  x dx  c
 y e x = x + c [1]
put x = 0 and y = 1, we get
c=1 [1]
y. e x = x + 1
y = (x + 1) ex is a particular solution of D.E. [1]
ii. ‘y’ is a differentiable function of ‘x’.
Let there be a small change x in the value of ‘x’.
Correspondingly, there should be a small change y in the value of ‘y’.
As x  0, y  0
x y
Consider,  =1
y x
x 1 y
 = , 0
y  y x
x
Taking lim on both sides, we get
x  0

 x  1
lim   = [1]
  lim  y 
x  0y
 
x  0 x
 
Since ‘y’ is a differentiable function of ‘x’,
 y  dy
lim   =
 x  dx
x  0

As x  0, y  0
 x  1
lim   = ….(i)
 y  lim  y 
y  0
 
x  0 x
 
 limits on R.H.S. of (i) exist and are finite. [1]
Hence, limits on L.H.S. of (i) also should exist and be finite.
 x  dx
 lim   = exists and is finite.
 y  dy
y  0

dx 1 dy
 = , 0 [1]
dy  dy  dx
 
 dx 
 
y = sin1 x, 1 ≤ x ≤ 1,   y 
2 2
 x = sin y
Differentiating w.r.t. y, we get
dx
= cos y
dy
dy 1
 =
dx cos y

257
Board Answer Paper : July 2017
dy 1
 =
dx  1  sin 2 y
dy 1
 =
dx  1  x2
 
Since   y  , y lies in I or IV quadrant.
2 2
 cos y is positive.
dy 1
 = , x < 1 [1]
dx 1  x2

8
iii. Let I =   x  2  x 2
 4
dx

8 A Bx + C
Let =  2 [1]
 x  2  x 2
 4   x  2   x  4
 8 = A (x2 + 4) + (Bx + C) (x + 2)
 8 = Ax2 + 4A + Bx2 + 2Bx + Cx + 2C
 8 = (A + B) x2 + (2B + C) x + (4A + 2C)
Comparing the coefficients of x2 , x and the constant term, we get
A + B = 0, 2B + C = 0 and 4A + 2C = 8
On solving these equations, we get
A = 1, B = –1, C = 2 [1]
8 1 x + 2
 =  2
 x  2  x  4  x  2  x  4
2

 1 x  2 
I =   2  dx
x  2 x  4
1 1 2x 1
=  x  2 dx  2  x 2
4
dx  2  x   2
2 2
dx [1]

1  x
= log x  2 – log x 2  4 + tan–1   + c
2 2
x2  x
= log  tan 1    c [1]
x2  4 2

258
Biology

BOARD ANSWER PAPER : JULY 2017


BIOLOGY

Note: Answer to every question must be written on a new page.

SECTION – I
[BOTANY]

Q.1. Select and write the most appropriate answer from the given alternatives for each sub-
question:
i. (C) endonucleases [1]
ii. (D) rice [1]
iii. (C) DNA [1]
iv. (A) CFCs [1]
v. (D) humification [1]
vi. (B) 140 [1]
vii. (C) Acetobacter aceti [1]
Q.2. (A) Answer in One sentence each :
i. The Blue Green Algae (BGA) have some specialized colourless cells called heterocysts
which are the sites of nitrogen fixation. [1]
ii. Plasmids are small, circular, double stranded, extrachromosomal, self-replicating DNA
molecules. [1]
iii. Chl-a (Chlorophyll-a) absorbs and converts solar energy into chemical energy, hence it is
called as essential pigment. [1]
iv. Global warming is an increase in global mean temperature caused by green house effect. [1]
v. Nif gene is a Nitrogen (N2) fixing gene. [1]
vi. The aquatic fern commonly used in paddy field as biofertilizer is Azolla. [1]
(B)
Outer membrane
Inner
membrane
DNA
Stroma
1 to 2 

Grana

Stroma lamellae
Ribosome

4 to 6 

Ultrastructure of Chloroplast

(Diagram - ½ mark + Any three labels  outer membrane, inner membrane,


grana, stroma, stroma lamellae, DNA, ribosome - 1½ marks) [2]
259
Board Answer Paper : July 2017
(C) Answer the following (Any TWO):
i. Advantages of biogas:
a. Biogas is a cheap, safe and renewable source of energy.
b. It burns with a blue flame and without smoke.
c. It can be used for cooking, domestic lighting, street lighting and small scale
industries.
d. It is eco-friendly and does not cause pollution and imbalance of the environment.
e. It helps to improve sanitation of the surroundings.
f. It can be easily generated, stored and transported.
(Any four points) [½  4] [2]
ii. Pusa Gaurav, Pusa Sawni, Pusa A-4 are insect-resistant crop varieties.
(Any two varieties) [1  2] [2]
iii. The causes of deforestation are:
a. Intensive collection of firewood.
b. Clearing of land for agriculture.
c. Developing pastures for grazing animals.
d. Carrying out mining activities.
e. Obtaining land for construction of roads, dams and buildings.
f. Selling wood for earning foreign exchange.
g. Meeting the increasing demand of timber wood.
h. Natural calamities like wildfire, floods, storms, volcanic eruptions, earthquakes, etc.
(Any four points) [½  4] [2]
iv. Floral adaptations for entomophily:
a. Flowers are large and attractive. When small, they are clustered into an inflorescence
(e.g. sunflower).
b. Flowers have attractive bright colors, with pleasant fragrance and nectar gland.
c. In some plants, additional modifications are made to attract insects. e.g. Corona in
passion flower and petalloid bracts in Bougainvillea.
d. Pollen grains possess spiny or rough exine.
e. Stigma has rough and sticky surface.
f. To favour insect pollination, some plants develop special mechanism e.g. lever
mechanism in Salvia. e.g. Jasmine, Rose, Salvia, Bougainvillea.
(Any four points) [½  4] [2]
Q.3. (A) Answer the following (Any TWO):
i. Wobble Hypothesis:
According to Wobble hypothesis, in codon-anticodon pairing the third base may not be
complementary.
The third base of the codon is called wobble base and this position is called Wobble position.
The actual base pairing occurs at first two positions only.

Perfect pairing Wobble pairing

G U U G U C G U A G U G
C A A C A A C A A C A A

5 G C 5 G C 5 G C 5 G C
C C C C
A A A A
val val val val
Wobble Hypothesis

In the above example, though the codon and anticodon do not match perfectly, then also the
required amino acid is brought perfectly.
260
Biology
This enables the economy of tRNA. GUU, GUC, GUA and GUG code for amino acid – Valine.
So, a single tRNA can interact with all the four codons which code for amino acid Valine.
(Meaning and explanation - 1 mark + Diagram - 2 marks) [3]
ii. a. Acetylation of Pyruvate is a connecting link between glycolysis and Krebs cycle.
b. Each pyruvic acid molecule formed in glycolysis undergoes oxidative
decarboxylation (i.e. oxidation by dehydrogenation and decarboxylation by loss of
CO2), to form acetyl group (2C).
Pyruvic dehydrogenase (oxidase) catalyses this reaction in the presence of co-enzyme NAD
and co-enzyme-A.
c. The acetyl fraction combines with Co-A to form acetyl Co-A.
d. Thus, pyruvic acid is converted to Acetyl Co-A. This is called ‘acetylation of
pyruvate’.
e. It involves several reactions, many enzymes and co-factors.
f. Hydrogen released combines with NAD to form reduced co-enzyme NADH2, which
has the potential to form 3ATP in ETS.
g. The conversion of pyruvate to acetyl co-enzyme A occurs in the mitochondrial matrix.
h. Summary is represented as:
Enzyme Complex ( Pyruvic dehydrogenase )
2 Pyruvate + 2 Co-A + 2 NAD            
(3C)
2 Acetyl Co-A + 2 CO2 + 2 NADH2
(2C)
(Explanation) [1½]
Schematic representation of formation of Acetyl Co-A:

2NAD
(3C) Pyruvic acid (2M)
2NADH2 2H 2CO2

Acetyl + Coenzyme - A

Acetyl Co-A (2M) (2C) Co -A

(Schematic representation) [1½]


iii.

NADH2
ADP + iP
ATP

NAD
FMN Red.
Co.Q 2e–
NADH2 Cyt.bc1
OR
Co.Q ADP + iP
FADH2 FMNH2 2e–
Oxi ATP
From substrate 2e– ADP + iP
FADH2 Cyt.a
Cyt.c ATP
2H+ Cyt.a3 1/2O2
2e–
FAD
2H+ H2O
2H+
Metabolic
water

Diagrammatic representation of Electron Transport System (Terminal oxidation)

(Diagram of ETS - 1 mark + Any four labels – ½ 4) [3]


261
Board Answer Paper : July 2017
(B)
Attachment

Penetration
Lysis of host cell
and release of
virions
Ghost (Capsid)
Assembly of Viral DNA
virions within
host cell
Degradation of host DNA

Synthesis of viral DNA


and protein coat
Lytic cycle

(Diagram) [1]
(Any four correct labels) [½  4] [2]

Q.4. Neo Mendelism:


a. Different patterns of inheritance or gene interactions were discovered based on
Mendelian ratios.
b. These interactions can be explained by modifying Mendel’s laws thus, this is called
Post Mendelian genetics or Neo Mendelism.
c. These gene interactions are of two types, intragenic and intergenic.
d. Quantitative inheritance is also known as polygenic inheritance in which characters
are determined by two or more gene pairs which have additive or cumulative effect.
These genes are called cumulative gene or polygenes.
e. This quantitative inheritance can be explained by wheat kernel colour which is
controlled by two pairs of genes Aa and Bb.
f. H. Nilsson Ehle crossed red kernelled variety of wheat with white kernelled variety.
g. Gene A and B determine the red colour of kernel (responsible for producing red
pigment) and are dominant over their recessive allele a and b which do not produce
red colour pigment and the expression is white, if dominant gene is not present.
Thus, genotype of a parent with red kernels is AABB and that of parent with white
kernels is aabb. Genotype of F1 hybrids is AaBb.
h. Continuous variations in expressions is observed as each dominant gene produces
specific amount of pigment and the shade of red colour, depends on number of
dominant gene present as shown in Punnett square.

262
Biology
Phenotype
Red kernels White kernels
of Parents
Genotype AABB  aabb

Gametes AB ab

AaBb
F1 generation
Intermediate Red
Selfing of F1 AaBb  AaBb
generation

Gametes AB Ab aB ab

AB AABB AABb AaBB AaBb


Red darkest Medium Medium Intermediate
Ab AABb AAbb AaBb Aabb
F2 generation Medium Intermediate Intermediate Light
aB AaBB AaBb aaBB aaBb
Medium Intermediate Intermediate Light
ab AaBb Aabb aaBb aabb
Intermediate Light Light White
i. In F2 generation, five different phenotypic expressions appeared in the ratio 1 : 4 : 6 :
4:1

[Meaning of Neo-Mendelism - 1 mark + Meaning of quantitative inheritance - 1 mark


+ Example and parental cross - 1 mark + F1 hybrids and gametes - 1 mark
+ Checker board of F2 generation with phenotypes - 2 marks
+ Ratio obtained in F2 generation - 1 mark] [7]
OR
Q.4. Advantages of cross pollination:
i. Cross pollination involves genetic recombination which results in variations.
ii. Possibility of variations results in new desired varieties of plants, hence favours the
process of evolution.
iii. Improved varieties like hybrids can be produced by hybridization.
iv. The offsprings formed are healthier and are well adapted to environment.
v. The offsprings show better vigour (hybrid vigour) and vitality.
Disadvantages of cross pollination:
i. The plants have to depend upon various external agencies.
ii. There is expenditure of energy due to the adaptations such as bright colour, nectar,
fragrance, etc. in attracting insects.
iii. There is considerable wastage of pollen grains.
iv. Since cross pollination result in the formation of new genotypes, it may also develop
undesirable characters in the progeny. Some desirable characters may get eliminated.
v. Genetic purity is not maintained.

Pollination mechanism in Salvia:


i. In Salvia, flower is bisexual and protandrous, i.e. anthers mature earlier than stigma.
ii. The two stamens of flower have long bifurcated connective.
iii. Upper branch of connective bears fertile anther lobe, while lower has sterile anther lobe.
iv. When an insect enters the flower, it pushes the lower sterile lobes. As a result, the
upper fertile anther lobe bends down.
263
Board Answer Paper : July 2017
v. The fertile anther lobe comes in contact with back side of insect body and pollen grains
are dusted there.
vi. When the same insect visits another flower with matured gynoecium, the pollen
grains are picked up by the receptive stigma.
vii. This mechanism is called lever-mechanism or turn-pipe mechanism.

Fertile anther lobe


Connective
Sterile anther lobe
B

C
Pollination is Salvia flower

(Three points of advantages - ½  3 + Three points of disadvantages - ½  3


+ Explanation of pollination mechanism in Salvia - 2 marks
+ Labelled diagram of pollination in Salvia flower - 2 marks) [7]

SECTION – II
[ZOOLOGY]

Q.5. Select and write the most appropriate answer from the given alternatives for each sub-
question:
i. (B) Theory of organic evolution [1]
ii. (A) complete sex-linkage [1]
iii. (B) Sporozoite [1]
iv. (D) acetylcholine [1]
v. (B) tunica vaginalis [1]
vi. (B) against concentration gradient [1]
vii. (D) Human growth hormone producer gene [1]
Q.6. (A) Answer in One sentence each:
i. The first vaccine was developed against small pox. [1]
ii. The two types of maps generated by H.G.P. are genetic linkage maps and physical maps. [1]
iii. Appearance of different types of symptoms is called syndrome. [1]
iv. Killer-T cells secrete perforins. [1]
v. The full form of IVF is In vitro fertilization. [1]
vi. The origin of life on the earth is called protobiogenesis. [1]
264
Biology
(B)
Afferent arteriole
Efferent arteriole

Glomerulus
Parietal layer

Visceral layer

Bowman’s capsule
Podocyte

Capsular space

Neck
Proximal convoluted tubule

Malpighian body

(Proportionate diagram) [1/2]


(Any three labels) [1/2  3] [11/2]
(C) Attempt any TWO of the following:
i. Organisms showing characters of two groups (kingdoms, phyla, classes, etc.) are called
connecting links between the two groups.
Archaeopteryx shows characters of birds as well as of reptiles.
Avian characters of Archaeopteryx:
a. It has skull bones completely fused: cranium rounded with large orbits and a single
condyle.
b. Jaws modified into beak.
c. Forelimbs modified into wings.
d. Limb bones have their first toe opposable.
e. Foot with clawed digits.
f. Feathery exoskeleton
Reptilian characters of Archaeopteryx:
a. Jaw with homodont teeth.
b. Ribs with single head.
c. Abdominal ribs somewhat like those seen in crocodiles.
d. Forelimbs modified into wings and three digits ending in distinct claws.
e. The hind limb had four digits ending with claws.
f. Long tail.
Thus, Archaeopteryx is called connecting link between reptiles and birds.
(Any two Avian characters - 1 mark + Any two Reptilian characters - 1 mark) [2]

ii. a. Production of vaccines involves growing the organism in a growth medium to


produce large amounts of toxins. [½]
b. These toxins are then altered to give toxoids which are no longer toxic. These toxoids
become antigens which the immune system recognises, and hence produces
antibodies to act against genuine toxins. [½]
c. The live organisms in the culture are inactivated, and the resulting mixture is diluted
and mixed with an adjuvant. [½]
d. The blended vaccine is then filled and packed. [½]
265
Board Answer Paper : July 2017
iii. a. Lac culture is a branch of applied zoology which deals with the rearing of lac insects to
obtain lac.
b. Lac is a pink coloured resin which hardens on coming in contact with air forming lac.
c. Tacchardia lacca is most commonly used for lac culture.
d. Lac insects feed on succulent twigs of certain plants like Ber, peeple, palas, kusum,
babool, etc.
e. Natural lac is always contaminated, so washing and filtering is needed to obtain shellac
in pure form.
f. India contributes upto 85% of total lac production in world.
g. Lac is the byproduct of lac culture which is used in bangles, toys, woodwork, polish
inks, silvering mirrors etc.
(Any four points) [½  4] [2]
iv. Commensalism:
Commensalism is an interspecific interaction in which one species is benefited and the
other one is neither benefited nor harmed. [1]
e.g. a. An orchid plant growing as an epiphyte on a mango tree.
In this association, the orchid plant is benefited (gets its shelter), while the
mango tree is neither benefited nor harmed.
b. The cattle egrets and grazing cattle.
The egrets forage close to where the cattle are grazing because as the cattle
move, they stir out the insects from the grass, that otherwise might be difficult
for the egrets to find and catch (the egrets are benefited and the cattle are
unaffected). (Any one example) [1]

Q.7. (A) Attempt any TWO of the following:


i. Chromosomes can be classified into four types on the basis of position of centromere:
a. Metacentric:
Centromere is in the middle of the chromosome and hence, the two arms are nearly equal.
Chromosome looks ‘V’-shaped in anaphase.
b. Submetacentric:
Centromere is not in the center but some distance away from the centre of
chromosomes and therefore arms are unequal in length.
The chromosome appears ‘L’ shaped in anaphase.
c. Acrocentric:
Centromere is near one end of the chromosome. One arm is very short, while other is long.
The chromosome appears ‘J’ shaped during anaphase.
d. Telocentric:
Centromere is at the extreme tip of the chromosome.
The arm is only on one side.
Chromosome remains rod shaped even in anaphase.
Chromosome appears ‘i’ shaped.

Metacentric Sub-metacentric Acrocentric Telocentric


Types of Chromosomes

(Explanation - 2 marks + Diagram - 1 mark) [3]


266
Biology
ii. a. The characteristic symptom of typhoid is a slowly progressing fever (as high as
40C or 104 F), profuse sweating and gastroenteritis.
b. The course of untreated typhoid fever is generally divided into four individual
stages, each lasting approximately for one week.
c. Slowly rising temperature with bradycardia, malaise, headache and cough.
d. Bleeding nose (epistaxis) is seen in certain cases and abdominal pain may also be
present.
e. Leukopenia, (decrease in the number of circulating white blood cells) with
eosinopenia and relative lymphocytosis may also occur.
f. In some cases, in the 2nd week, a rash of flat, rose coloured spots called “rose spots”
appear on the front of the chest.

‘O’ antigen

Flagella
with antigen
‘H’

Salmonella typhi
(Any four symptoms) [½  4] [2]
(Diagram) [1]
iii. The relative proportion of individuals of various age groups in the population is called age
structure of the population.
Role of age structure:
Age structure (distribution of age groups) determines the trend of the population.
a. Growing population:
Population having larger number of individuals of the pre-reproductive age groups
shows a very rapid rate of growth.
b. Steady population:
Population having same number of pre-reproductive and post-reproductive age
groups is stable.
c. Declining population:
Large number of post-reproductive and small number of pre-reproductive groups
make the population decline.

Post
Post-reproductive
reproductive
Reproductive
Post Reproductive Pre-
reproductive reproductive
Reproductive
Pre -
Pre - reproductive reproductive

Age structure showing declining


Pyramid of growing population population
Pyramid of steady
population
(Explanation - 1½ marks + diagram - 1½ marks) [3]
267
Board Answer Paper : July 2017
(B)
Basement membrane
Cuboidal epithelium
Colloid

Para follicular cells

Thyroid follicle
Inter follicular connective tissue
T.S. of Thyroid Gland

(Proportionate Diagram) [1]


(Any four labels) [½  4] [2]
Q.8. Histology of Testis:
i. Externally, each testis is covered by three layers. These are:
a. Tunica vaginalis:
It is the outermost incomplete peritoneal covering made up of connective tissue and
epithelium.
b. Tunica albuginea:
It is the middle layer formed by fibrous connective tissue.
c. Tunica vasculosa:
It is the innermost layer formed of delicate connective tissue, supporting a network of
blood capillaries.

Tunica albuginea
Interstitial cells / cells of Leydig

Sperm bundle Seminiferous tubule


Germinal epithelium
Sertoli cell

Basement membrane
Connective tissue
Different stages of
spermatogenesis
T.S. of testis

ii. Each testis contains about 200-300 lobules. Each lobule has 1 to 4 convoluted loops
called seminiferous tubules.
iii. These are lined by a single layer of cuboidal germinal epithelium which undergo
spermatogenesis.
iv. In the germinal epithelium, various stages of spermatogenesis such as spermatogonia,
primary spermatocytes, secondary spermatocytes, spermatids and sperms are seen.
v. Between germinal epithelium, few large and pyramidal cells called nurse cells or Sertoli
cells are present.
vi. Sperm bundles are seen attached to Sertoli cells with their heads.
vii. These cells provide nourishment to the sperms till maturation.
viii. Between seminiferous tubules, few groups of interstitial cells (Cells of Leydig) are
present.
ix. Interstitial cells produce hormone testosterone after puberty.
268
Biology
Spermatogenesis:
i. Spermatogenesis is the process of formation of haploid, microscopic and motile male
gametes, called spermatozoa, from the diploid spermatogonia of the testis of male
organism.

Primordial cell

Mitosis

Growth phase Primay spermatocyte

Secondary spermatocyte
Meiotic division
Spermatid

4 Spermatozoa
Spermatogenesis

ii. Each testis has seminiferous tubules which are lined by cuboidal epithelium called germinal
epithelium.
iii. Germinal cells in testes are known as primary germinal cells. Primordial cells passes through
three phases, namely:
a. Multiplication phase: Primordial cells undergo repeated mitotic divisions to
produce large number of spermatogonia.
Each spermatogonium is diploid (2n).
b. The Growth phase: Spermatogonium cell accumulates food and grows in size.
Now, it is called primary spermatocyte.
c. The Maturation phase: The primary spermatocyte undergoes first meiotic or
maturation division. The homologous chromosomes start pairing.
Each homologous chromosome splits longitudinally. Chiasma formation results in
exchange of genetic material.
iv. At the end of 1st meiotic division, two haploid, secondary spermatocytes are formed. Each
secondary spermatocyte undergoes 2nd meiotic division and produces spermatids. So, at the
end of maturation phase, each spermatogonium produces four haploid spermatids.
v. Spermatid is non-motile, so it has to undergo spermiogenesis to become functional, motile
male gamete, i.e. spermatozoan.
(Description of histology of testis - 2 marks + Diagram of T.S. of testis - 1
mark + Three phases of spermatogenesis - 3 marks + Diagram of spermatogenesis - 1 mark) [7]
OR
Working of Heart:
Human heart is composed of cardiac muscles which are capable of alternate contraction
(systole) and relaxation (diastole).
Human heart is myogenic as its contraction is initiated and conducted by the modified
muscle plexuses called nodal tissues. [1]
269
Board Answer Paper : July 2017
It consists of the following components:
a. Sinoauricular or sinuatrial node or SA node or SAN:
1. A specialised patch of cardiac muscle present in the wall of right atrium
near the right side of right atrium [opening of precaval vein] is known as
SA node or SAN.
2. Sinoatrial node has the highest degree of rhythmicity.
3. It is also called pacemaker, as it has the power of generation of wave of
contraction. It initiates and maintains the rhythmicity of the heart. [1]
b. Atrioventricular node or AV node:
1. It lies in the right atrium near the junction of inter-auricular and inter-
ventricular septum, close to groove which separates atria and ventricles.
2. It receives the wave of contraction initiated by SA node.
3. It helps in conduction of impulses to Bundle of His and Purkinje fibres. [1]
c. Bundle of His:
1. It arises from AV node and spreads the wave of contraction at the speed of
5 mts/sec.
2. It divides into two branches which descends along two sides of inter-
ventricular septum. [1]
d. Purkinje fibres:
1. Purkinje fibres arise from the bundle branches.
2. They penetrate into the myocardium of ventricles. [1]

Sino auricular node

Right auricle Left auricle

Auriculo ventricular
node Bundle of His

Purkinje fibres
Left ventricle

Right ventricle

Auriculoventricular septum
Conducting System of Human Heart [2]

270
Physics

BOARD ANSWER PAPER : MARCH 2018


PHYSICS

Note: Answer to every question must be written on a new page.

SECTION – I

Q.1. Select and write the most appropriate answer from the given alternatives for each
sub-question:
i. (B) [1]
ii. (D) [1]
iii. (A)
P.E. = mgh
At highest point, h = 2r
 P.E. = 2 mgr [1]
iv. (B) [1]
v. (C) [1]
vi. (B) [1]
vii. (A)
Ic = MR2 = 0.25  0.52 = 0.0625 kgm2 [1]
Q.2. Attempt any SIX:
i. a. Kepler’s first law (Law of orbit):
Every planet revolves around the sun in an elliptical orbit with the sun situated at one
of the focii of the ellipse. [1]
b. Kepler’s second law (Law of equal areas):
The radius vector drawn from the sun to any planet sweeps out equal areas in equal
intervals of time, i.e., areal velocity of the radius vector is constant. [1]
ii. Basic assumptions of kinetic theory of gases:
a. A gas consists of very large number of extremely small particles known as
molecules.
b. The molecules are rigid and perfectly elastic spheres of very small diameters of the
order 1 Å i.e., 1010 m.
c. All the molecules of the same gas are identical in shape, size and mass.
d. The actual volume occupied by the gas molecules is negligible as compared to the
total volume occupied by the gas.
e. The intermolecular force of attraction between gas molecules are negligible.
f. Molecules are always in the state of random motion, i.e., they are moving in all possible
directions with all possible velocities. This state is called molecular chaos.
g. Due to their random motion, the molecules constantly collide with each other and
with the walls of the container. Such collisions are perfectly elastic, i.e., there is no
loss of kinetic energy during the collisions.
h. Between any two successive collisions, a molecule travels in a straight line with
constant velocity. It is called free path.
i. The time of impact, i.e., the time during which collision occurs, is very small as
compared to the time interval between successive collisions.
j. Though the molecules are constantly moving from one place to another, the average
number of molecules per unit volume of the gas remains constant.
(Any four assumptions) [2]
271
Board Answer Paper : March 2018
iii. a. Definition:
Moment of inertia of a rigid body about an axis of rotation is defined as the sum of
product of the mass of each particle and the square of its perpendicular distance from
the axis of rotation. [1]
b. Unit: kg m2 in SI system and g cm2 in CGS system.
Dimensions: [M1L2T0] [1]
iv. Difference between centripetal force and centrifugal force:
Sr.
Centripetal force Centrifugal force
No.
Centripetal force is directed along the Centrifugal force is directed along the
i.
radius towards the centre of a circle. radius away from the centre of a circle.
ii. It is a real force. It is a pseudo force.
It is considered in inertial frame of It is considered in non-inertial frame of
iii.
reference. reference.
In vector form, it is given by In vector form, it is given by
iv.  mv 2  mv 2
F =  r̂0 F =+ r̂0
r r [2]
v. Solution:
Given: p1 = 3, p2 = 4, T1 = 10 g-wt
To find: Tension (T2)
Formula: T1p12 = T2 p 22
Calculation: From formula,
2
T2 p 
=  1
T1  p2 
2
T2 3
 = 
T1 4
9
 T2 =  10
16
 T2 = 5.625 g-wt
= 5.625  9.8  10–3 N = 0.055 N
Ans: The tension applied to the string is 0.055 N. [2]
vi. Solution:
Given: r1 = 1 cm, r2 = 2 cm, T = 30 dyne/cm
To find: Work (W)
Formula: W = TA
Calculation: A1 = 4 r12 = 4  12 = 4 cm2
A2 = 4 r22 = 4  22 = 16 cm2
A = A2  A1 = (16  4 ) = 12 cm2
Since soap bubble has two surfaces
From formula,
W = 2T  A
= 2  30  12
= 2  30  12  3.14
 W = 2.26  103 erg
Ans: The work done in increasing the radius of the soap bubble is 2.26  103 erg. [2]
vii. Solution:
Given: v = 32 m/s, r = 400 m, g = 9.8 m/s2
To find: Coefficient of friction ()
v2
Formula: =
rg

272
Physics
Calculation: From formula,
322
= = 0.26
400  9.8
Ans: The value of coefficient of friction between tyre and ground is 0.26. [2]
viii. Solution:
Given: vmax = 25 cm/s, amax = 100 cm/s2
To find: Amplitude (A), Period (T)
2
Formulae: a. vmax = A b. amax = A2 c. =
T
Calculation: From formulae (a) and (b),
a max 100
= = = 4 rad/s
v max 25
From formula (c),
2 2  3.142
 T= = = 1.571 s
 4
From formula (a),
v max 25
 A= = = 6.25 cm
 4
Ans: Amplitude is 6.25 cm and time period is 1.571 s [2]
Q.3. Attempt any THREE:
i. Expression for excess pressure inside a drop:

Po
Pi

a. Free surface of drops or bubbles are spherical in shape.


Let, Pi = inside pressure of a drop or air bubble
Po = outside pressure of bubble
r = radius of drop or bubble.
b. Let the radius of drop increases from r to r + r so that inside pressure remains
constant.
c. Initial area of drop A1 = 4r2, Final surface area of drop A2 = 4(r+r)2
Increase in surface area,
A = A2  A1 = 4[(r + r)2  r2]
= 4[r2 + 2rr + r2  r2]
= 8rr + 4r2
d. As r is very small, the term containing
r2 can be neglected.
 A = 8rr
e. Work done by force of surface tension,
dW = TA = (8rr)T ….(1)
But,
 dF = (Pi  Po)A
dW = Fr = (Pi  Po) Ar
From equation (1),
(Pi  Po) Ar = (8rr) T
8πrrT
 Pi  Po = ….[ A = 4r2]
4πr 2 r
2T
 Pi  Po = ….(2)
r
Equation (2) represents excess pressure inside a drop or air bubble. It is also called
Laplace’s law of spherical membrane. [3]
273
Board Answer Paper : March 2018
ii. Statement:
The angular momentum of a body remains constant, if resultant external torque acting on
the body is zero. [1]
Proof:
a. Consider a particle of mass m, rotating about an axis with torque ‘’.
 
Let p be the linear momentum of the particle and r be its position vector.
  
b. By definition, angular momentum is given by, L = r  p ....(1)
c. Differentiating equation (1) with respect to time t, we get,

dL d  
= ( r  p)
dt dt
  
dL  dp  dr
 = r + p ….(2)
dt dt dt
 
dr  dp  
d. But, = v, = F and p = mv
dt dt
 Equation (2) becomes,

dL    
= r  F+ 0 ….[ v  v = v2 sin 0 = 0]
dt
  
e. Also,  = r  F

dL 
 = 
dt

dL
f. If resultant external torque () acting on the particle is zero, then = 0.
dt

 L = constant
 I = constant
Hence, angular momentum remains conserved. [2]

iii. Solution:
Given: L = 3 m, F = 6 kg-wt = 6 × 9.8 N = 58.8 N,
A = 0.6 mm2 = 0.6  106 m2, l = 3 mm = 3  103 m
To find: Strain energy per unit volume (u)
F l
Formulae: a. Stress = b. Strain =
A L
1
c. u=  Stress  Strain
2
Calculation: From formula (a),
F 58.8
Stress = = = 98  106 N/m2
A 0.6  106
From formula (b),
l 3  103
Strain = = = 103
L 3
From formula (c),
1
u=  Stress  Strain
2
1
=  98  106  103
2
u = 49  103 J/m3
Ans: Strain energy per unit volume is 49  103 J/m3 [3]
274
Physics
iv. Solution:
Given: m = 500 kg, g = 9.8 m/s2, d = 1000 km = 106 m,
R = 6400 km = 6.4  106 m
To find: Decrease in weight of body (dW)
 d
Formula: gd = g 1 
 R 
Calculation: Weight of body on the earth’s surface,
W = mg = 500  9.8 = 4900 N
Multiplying both sides of formula by ‘m’,
 d
mgd = mg 1 
 R 
 106 
 Wd = 4900 1  
 6.4  106 
 Wd  4134 N
 dW = W  Wd = 4900 – 4134 = 766 N
Ans: Decrease in weight of the body is 766 N. [3]
Q.4. A.
i. Differential equation of linear S.H.M. is given as,
d2x
+ 2x = 0,
dt 2
d2x
=  2x ….(1)
dt 2
d2x
ii. But in linear motion, linear acceleration is given by, =a ….(2)
dt 2
From equations, (1) and (2),
a =  2 x ….(3)
Equation (3) gives acceleration in linear S.H.M.
Negative sign in equation (3) shows that direction of acceleration is opposite to that of
displacement x.
d2x
iii. a. Also, =  2x
dt 2
d  dx  2
  =   x
dt  dt 
dv  dx 
 =  2x  dt  v 
dt  
dv dx
  =  2x
dx dt
dv
 v = 2x
dx
 v dv = 2xdx .…(4)
b. Integrating both sides of equation (4),
v dv =   2xdx
v2 2 x 2
=  +C ….(5)
2 2
where, C is the constant of integration.
c. At extreme position, x =  A and v = 0.
Substituting these values in equation (5),
2A 2
0=  +C
2
2A 2
 C= ….(6)
2

275
Board Answer Paper : March 2018
d. Substituting C from equation (6) in equation (5),
v2 2 x 2 2 A 2
= +
2 2 2
 v2 = 2A2  2x2
 v2 = 2(A2  x2)
 v =   A2  x 2 ….(7)
This is the required expression for velocity in linear S.H.M.
dx
iv. a. In linear motion, v =
dt
Substituting for v in equation (7),
dx
   A2  x 2
dt
dx
 =  dt
A2  x 2
b. Integrating both sides, we get
dx
 A  x2
2
=   dt

x
 sin 1   = t +  ….(8)
A
where,  is constant of integration which depends upon initial condition
(phase angle)
x
 = sin  t +  
A
 x = A sin (t + ) ….(9)
This is required expression for displacement. [4]
B. Solution:
80 + 70 70  62
Given: 1 = = 75 C, t1 = 5 minutes, 2 = = 66 C, t2 = 5 minutes
2 2
 d  80  70 10
Also   = = = 2 C/min.
 1
dt 5 5
 d  70  62 8
  = = = 1.6 C/min
 dt  2 5 5
To find: Temperature of surroundings (0)
 d 
Formula:   = K(  0)
 dt 
Calculation: From formula,
 d 
  = K (1  0) .…(1)
 dt 1
 d 
  = K (2  0) .…(2)
 dt  2
Dividing equation (1) by (2),
 d 
 
  dt 1 = K  1  0  = 1  0
 d  K   2  0   2  0
 
 dt 2
2 75  0
 =
1.6 66  0
 1.6 (75  0) = 2 (66  0)
 120  1.6 0 = 132  20
 20  1.6 0 = 132  120
 0.4 0 = 12
276
Physics
12
 0 =
0.4
 0 = 30 C
Ans: The temperature of the surrounding is 30 C. [3]
OR
A. i. Harmonics: The frequencies which are the integral multiples of the fundamental
frequency are known as harmonics.
ii. Consider a pipe closed at one end. A vibrating tuning fork is held at the open end.
Longitudinal wave is set up in the air column which gets reflected from closed end.
Superposition of incident wave and reflected wave sets up a longitudinal stationary
wave in the air column. The layer of air in contact with closed end remains
stationary, hence displacement node is formed at closed end. The layer of air at open
end can vibrate freely, hence displacement antinode is formed. In this mode of
vibration, reflection takes place from denser medium.
iii. First mode or Fundamental mode:
In this mode of vibration, there is one node at the closed end and one antinode at the
open end as shown in figure (a).
Let, L = length of air column
v = wave velocity in air A
n = fundamental frequency
 = wavelength
Velocity of wave in a tube is given by, 
L
v = n .…(1) 4

But L=
4
From equation (1),
N
v = 4nL
v Figure (a)
 n= .…(2)
4L
Equation (2) represents fundamental frequency or first mode of vibration.
iv. Second mode or first overtone:
In this mode of vibration, there are two nodes and two antinodes as shown in figure
(b).
Let, L = length of air column
v = wave velocity in air
As the medium is same, wave velocity is same A
n1 = next higher frequency
1 = wavelength
N
v = n11 .…(3) 31
L
31 4
L=
4 A
4L
 1 =
3
From equation (3), N
4L Figure (b)
v = n1
3
3v
 n1 = .…(4)
4L
From equations (2) and (4),
n1 = 3n
Thus, frequency of first overtone is three times the fundamental frequency.
Hence, the 1st overtone corresponds to the 3rd harmonic.
277
Board Answer Paper : March 2018
v. Third mode or second overtone:
In this mode of vibration there are three nodes and three antinodes, as shown in
figure (c).
Let, L = length of air column
v = wave velocity in air A
n2 = next higher frequency
2 = corresponding wavelength N
v = n22 .…(5) L A
5 2
5 2 4L 4
L= or 2 =
4 5 N
From equation (5), A
4L
v = n2 .
5
N
5v
 n2 = .…(6) Figure (c)
4L
From equations (2) and (6),
 n2 = 5n
Thus, the frequency of second overtone is five times the fundamental
frequency. Hence, the 2nd overtone corresponds to the 5th harmonic.
vi. Similarly, frequency of pth overtone,
np = (2p + 1)n
where, p = 0, 1, 2, 3……
vii. Thus, possible frequencies of the air column are n, 3n, 5n ……
It means that only odd harmonics are present as overtone in the mode of
vibration of air column closed at one end. [5]
B. Solution:
81 81
Given: 1 = m, 2 = m, beat frequency, N = 10 beats per second
170 173
To find: Velocity (v)
Formula: v = n
Calculation: From formula,
v v
 n1 = and n2 =
1 2
But 1 > 2
 n2 > n1
 n2  n1 = 10
 1 1
 v   = 10
  2 1 
 173 170 
 v   = 10
 81 81 
10  81
 v= = 270 m/s
3
Ans: The velocity of sound in air is 270 m/s. [2]

SECTION – II

Q.5. Select and write the most appropriate answer from the given alternatives for each
sub-question:
i. (B) [1]
ii. (A) [1]
iii. (C) [1]
iv. (D) [1]
278
Physics
v. (B) [1]
vi. (B) [1]
vii. (C)
ip = tan1 (1.732) = 59.99  60 [1]
Q.6. Attempt any SIX:
i. a. Conditions for constructive interference:
1. Two waves should be in same phase.
2. The phase difference between the interfering waves should be even multiple of .
 = 0, 2π, 4π, 6π, .…
 = 2nπ where, n = 0, 1, 2, .…
3. The path difference between the interfering waves should be an integral

multiple of  or even multiple of .
2
i.e. x = 0, λ, 2λ, 3λ, ….
  
 x = 0, 2 , 4 , 6 ,.......
2 2 2

 x = nλ = 2n where, n = 0, 1, 2, …
2
Thus, constructive interference is obtained when path difference is an even
multiple of half wavelength. [1]
b. Conditions for destructive interference:
1. Two waves should be in opposite phase.
2. The phase difference between the interfering waves should be odd multiple of .
 = , 3π, 5π, 7π, …
  = (2n – 1)π where, n = 1, 2, 3, …
3. The path difference between the interfering waves should be odd multiple of

.
2
 3 5
x = , , ,...
2 2 2

 x = (2n  1) where, n = 1, 2, 3, …
2
Thus, constructive interference is obtained when path difference is an odd
multiple of half wavelength. [1]
ii. Statement:

The line integral of magnetic field of induction B around any closed path in free space is
equal to absolute permeability of free space 0 times the total current flowing through area
bounded by the path.
 
Mathematically,  B  dl = 0I
Explanation:
a. Consider a long straight current-carrying conductor XY, placed in vacuum. A steady
current ‘I’ flows through it from the end Y to X as shown in the figure.

X
I
 
B B
r Q

P dl
 
B B

279
Board Answer Paper : March 2018
b. Imagine a closed curve (Amperian loop) around the conductor having radius ‘r’.
The loop is assumed to be made of large number of small elements, each of length

dl . Its direction is along the direction of traced loop.

c. Let B be the strength of magnetic field around the conductor.
  
 B  d l   B cos   d l
 
where,  = angle between B and dl .
Taking line integral,
 

 B  dl =  B dl cos  = 0I [2]

iii.
Receiving
Antenna

Output
Amplifier IF Stage Detector Amplifier
Received
signal
Block diagram of a receiver
[2]
iv. a. Definition:
The net magnetic dipole moment per unit volume, in the magnetic material is called
as magnetization,
M net
i.e., MZ = [1]
V
b. Unit: Am1 and dimensions: [M0L1T0I1] [1]

v. Solution:
Given: v = 2.3  106 m/s, r = 0.53 Å = 0.53  10–10 m, n = 1
To find: Period of revolution (T)
2r
Formula: T=
v
Calculation: Using formula,
2r
T=
v
2  3.14  0.53  1010
T=
2.3  106
T = 1.44  10–16 s
Ans: Period of revolution of electron is 1.44  10–16 s. [2]

vi. Solution:
Given: C = 0.5 F = 0.5  10–6 F, f = 100 Hz
To find: Capacitive reactance (XC)
1
Formula: XC =
2f C
Calculation: Using formula,
1
XC =
2  3.14  100  0.5  106
XC = 3.18  103 Ω
XC = 3.18 k
Ans: The capacitive reactance of a capacitor is 3.18 k. [2]
280
Physics
vii. Solution:
1 1
Given: v=  c =  3  108 = 0.6  108 m/s, h = 6.63  10–34 J.s
5 5
m = 9  10–31 kg
To find: de-Broglie wavelength ()
h
Formula: =
mv
Calculation: Using formula,
6.63  1034
=
9  1031  0.6  108
 = 1.228  10–11 m
 = 0.1228 Å
Ans: The de-Broglie wavelength of an electron is 0.1228 Å. [2]
viii. Solution:
Given: Charge on proton qp = 1.6  10–19 C, Mass of proton mp = 1.67  10–27 kg,
B = 1.4 Wb/m2, qp = 1.6  10–19 C, mp = 1.67  10–27 kg
m p
Formula: t=
Bq p
Calculation: Using formula,
3.142  1.67  1027
t=
1.4  1.6  1019
t = 2.342  10–8 s.
Ans: The electric field between the dees should be reversed in 2.342  10–8 s. [2]
Q.7. Attempt any THREE:
i. Determination of unknown resistance:
a. Metrebridge works on the principle of Wheatstone’s bridge.
b. Unknown resistance X is connected in the left gap and a resistance box R in right gap.
c. A galvanometer is connected between points B and D through a Jockey (J). A battery
is connected between A and C.
d. A suitable resistance is introduced in the circuit from the resistance box and the
jockey is tapped on the wire till a point D is located such that the galvanometer
deflection is zero as shown in the figure below.
X
R

T1 B
T2
T3
G
lX DJ lR
A C

0 10 20 30 40 50 60 70 80 90 100

+  ( )
E K Rh
AC : One metre long uniform wire
X : Unknown resistance
R : Resistance from resistance box
G : Galvanometer
T1, T2, T3 : Metal strips
D : Null Point
J : Sliding key (jockey)
Rh : Rheostat

281
Board Answer Paper : March 2018
e. The distance of the point D from A is measured on the scale say lX.
The distance of the point D from C is measured on the scale say lR.
By adjusting the value of R, the neutral point is obtained in the middle of the wire.
f. In the balanced condition of bridge,
X Resistance of length lX
=
R Resistance of length lR
l
Since, R = 
A
where,  is specific resistance of the material of wire
l X
X l
 = A = X
R  lR lR
A
 lX + lR = 100 cm
 lR = 100  lX
 l 
 X=R  X 
 100  lX 
Hence the value of unknown resistance ‘X’ can be determined. [3]
ii. A zener diode is a heavily doped P-N junction diode intentionally manufactured to operate
in breakdown region.
Zener diode as voltage regulator:

RS
(+) (+)
I
IZ IL
Unregulated
input Vi Zener Vo=VZ Regulated
RL
voltage diode output
voltage
() ()

a. The circuit diagram of zener diode as voltage regulator is shown in the figure.
b. The input voltage Vi is connected across zener diode through a series resistance Rs.
c. The load resistance RL is connected in parallel with zener diode (Vo = Vz). The
output voltage Vo is taken across RL.
d. The zener diode is reverse biased i.e., P-side of diode is connected to negative
terminal and N-side of diode is connected to positive terminal.
Working:
a. When voltage is applied to the circuit, current I flows through it. I is divided into IZ
(current flowing through zener diode) and IL (current flowing through load resistance).
b. From the figure, I = Iz + IL
Vi = VS + Vz
 Vi = IRS + Vz ….[ V = IR]
But, I = Iz + IL
 Vi = (Iz + IL) RS + Vz
 Vi = (Iz + IL) RS + Vo ….[ Vo = Vz]
c. If input voltage Vi is increased beyond zener voltage, I increases such that current Iz
through zener diode increases but current IL through load resistance remains same.
Therefore, output voltage Vo across load resistance remains the same.
d. If input voltage Vi is decreased below zener voltage, I decreases such that current Iz
through zener diode decreases but current IL through load resistance remains same.
Therefore, output voltage Vo across load resistance remains same.
e. Thus, whether the input voltage increases or decreases, the output voltage remains
constant. So, zener diode acts as a voltage regulator. Once the breakdown voltage is
observed, it remains constant, though current is increased. [3]

282
Physics
iii. Solution:
Given:  = 5200 Å = 5200  10–10 m, X2 – X1 = 1.3 mm = 1.3  10–3 m
D2 – D1 = 50 cm = 50  10–2 m
To find: Distance between two virtual images of the slit (d).
D
Formula: X=
d
Calculation: Using formula,
  D 2  D1 
(X2 – X1) =
d
  D 2  D1 
d=
X 2 – X1
5200  1010   50  102 
d=
1.3  103
–4
d = 2  10 m = 0.2 mm
Ans: The distance between two virtual images of the slit is 0.2 mm. [3]
iv. Solution:
4
Given: w = , d = 2.42, c = 3  108 m/s
3
To find: a. speed of light in water (vw)
b. speed of light in diamond (vd)
c
Formula: =
v
Calculation: Using formula,
c
For water, w =
vw
c
vw =
w
3  108
vw =
4
3
vw = 2.25  108 m/s
c
For diamond, d =
vd
c
vd =
d
3  108
vd =
2.42
vd = 1.24  108 m/s
Ans: a. speed of light in water is 2.25  108 m/s.
b. speed of light in diamond is 1.24  108 m/s. [3]
Q.8. A. Theoretical proof of Lenz’s law:
i. Consider a rectangular loop of conducting wire ‘PQRS’ partly placed in uniform magnetic
field of induction ‘B’ as shown in figure.
      
F1
 B  PP     I Q
      
F       v
       l
      
  x
  S  F    dx R
 2   

283
Board Answer Paper : March 2018
ii. Let ‘l’ be the length of the side PS and ‘x’ be the length of the loop within the field.
 A = lx = area of the loop, which lies inside the field.
iii The magnetic flux () through the area A at certain time ‘t’ is  = BA = Blx
iv. The loop is pulled out of the magnetic field of induction ‘B’ to the right with a uniform
velocity ‘v’.
v. The rate of change of magnetic flux is given by,
d d
= (Blx)
dt dt
d
= Bl  
dx

dt  dt 

But,   = v
dx
 dt 
d
 = Blv .…(1)
dt
vi. Due to change in magnetic flux, induced current is set up in the coil. The direction of
this current is clockwise according to Lenz’s law. Due to this, the sides of the coil
experiences the forces, F1, F2 and F as shown in figure. The directions of these forces is
given by Flemings left hand rule.
vii. The magnitude of force ‘F’ acting on the side PS is given by, F = BIl.
 
viii. The force F1 and F2 are equal in magnitude and opposite in direction, therefore they

cancel out. The only unbalanced force which opposes the motion of the coil is F . Hence,
work must be done against this force in order to pull the coil.
ix. The work done in time ‘dt’ during the small displacement ‘dx’ is given by,
dW = Fdx
 ve sign shows that F and ‘dx’ are opposite to each other.
 dW =  (BIl) dx ….(2)
x. Mechanical power is given by,
dW  dx 
P= = BIl  
dt  dt 
dx
 P = BIlv ….[ = v]
dt
xi. This external work provides the energy needed to maintain the induced current I through
the loop (coil).
xii. If ‘e’ is the e.m.f induced then,
dW
electric power = = eI
dt
 dW = eIdt .…(3)
xiii. From equations (2) and (3),
eIdt =  BIl dx
e =  Bl 
dx 
 
 dt 
 e =  Blv .…(4)
xiv. From equation (1) and (4),
d
e=  [4]
dt
B. Solution:
Given: C1 = 20 F, d2 = 2d, k1 = k2 = k (air), k3 = 8
To find: a. The new capacity if distance between two plates is doubled (C2)
b. The new capacity if a marble slab is introduced between two plates
(C3)
A 0 k
Formula: C=
d

284
Physics
Calculation: From formula,
A0 k A0 k A0 k
C1 = and C2 = =
d1 d2 2d1
C2 A0 k / 2d1
 =
C1 A0 k / d1
C2 1
 =
C1 2
1 1
 C2 = C1  = 20 
2 2
 C2 = 10 F
A0 k 3
C3 =
d
A0  8
= ….[Distance between plates remains same]
d
= 8C1 = 8  20 = 160 F
Ans: a. The new capacity will be 10 F if distance between two plates is doubled
b. The new capacity will be 160 F if a marble slab is introduced between two
plates. [3]
OR
A. Different wavelength in hydrogen spectrum can be calculated by formula,
1 1 1
=  .
  p2 n 2 
The transition of electrons from different outer orbits to a certain inner orbit is explained
with following series as shown in energy level diagram.

E = 0 n=
n=6
E6 =  0.38 n=5
E5 = 0.54 Pfund series
n=4
E4 = 0.85 Brackett series
E3 = 1.51 n=3
Paschen series

E2 = 3.4 n=2
Balmer series
E(in eV)

E1 = 13.6 n=1
Lyman series Quantum number (n) 

285
Board Answer Paper : March 2018
Balmer series:
i. The spectral lines of this series correspond to the transition of an electron from some higher
energy state to 2nd orbit.
ii. For Balmer series, p = 2 and n = 3, 4, 5,
The wave numbers and the wavelengths of spectral lines constituting the Balmer series are
1 1 1 
given by,  = =R  2  2
 2 n 
iii. There are infinite number lines in this series out of which four lines are seen called H, H,
H, H.
iv. This series lies in the visible region. Wavelengths for n = 3 and 4 are 6563 Å and 4868 Å
respectively.
Brackett series:
i. The spectral lines of this series correspond to the transition of an electron from a higher
energy state to the 4th orbit.
ii. For this series, p = 4 and n = 5, 6, 7,...
The wave numbers and the wavelengths of the spectral lines constituting the Brackett series
are given by,
1  1 1 
 = =R  2  2
 4 n 
iii. This series lies in the near infrared region of the spectrum and contains infinite number of
lines. Wavelengths for n = 5 and 6, are 40518 Å and 26253 Å respectively. [4]

B. Solution:
Given: W0 = 2.2 eV = 2.2  1.6  10–19 J,
 = 5000 Å = 5000  10–10 m
h = 6.63  10–34 J.s, c = 3  108 m/s
To find: a. Threshold frequency (0) b. Incident frequency ()
c. If photoemission is possible?
Formulae: a. W0 = h0 b. c = 
Calculation: Using formula (a),
2.2  1.6  10–19 = 6.63  10–34  0
2.2 1.6 1019
0 =
6.63  1034
0 = 5.31  1014 Hz
Using formula (b),
3  108 =   5000  10–10
3108
=
5000 1010
 = 6  1014 Hz
As,  > 0, there will be emission of photoelectrons.
Ans: a. Threshold frequency vs 5.31  10–14 Hz
b. Incident frequency is 6  1014 Hz
c. There will be emission of photoelectrons. [3]

286
Chemistry

BOARD ANSWER PAPER : MARCH 2018


CHEMISTRY

Note: Answer to every question must be written on a new page.

SECTION – I

Q.1. Select and write the most appropriate answer from the given alternatives for each
sub-question:
i. (B) Isothermal [1]
In isothermal process, temperature is constant (T = 0). Hence, the internal energy change
of a system is constant (U=0).
ii. (B) 4 [1]
r
According to radius ratio rule, the value of radius ratio,  = 0.320 corresponds to
r
tetrahedral structure which has a coordination number of 4.
iii. (B) KOH [1]
iv. (D) Zn2SiO4 [1]
v. (C) +3 [1]
Dinitrogen trioxide (N2O3)
Oxidation state of nitrogen in N2O3
N2O3
2x + (–2  3) = 0
2x – 6 = 0
2x = 6
x = +3
vi. (A) Al2(SO4)3 [1]
Colligative properties are proportional to the number of solute particles present in the solution.
Number of particles after
Solute Ionization
ionization of one molecule
(A) Al2(SO4)3  2Al3+ + 3SO 24 
Al2(SO4)3  5
(B) Na2SO4  2Na+ + SO 24
Na2SO4  3
(C) MgCl2  Mg2+ + 2Cl–
MgCl2  3
(D) KCl  K + Cl
KCl  + –
2

Osmotic pressure is a colligative property, hence according to the formula  = iCRT,


Al2(SO4)3 exerts the highest osmotic pressure.
 A 0
vii. (C) [1]
2K
Q.2. Answer any SIX of the following:
i. a. The elevation in the boiling point of a dilute solution is directly proportional to the
molal concentration (expressed in mol kg1) of the solution.
Hence, if Tb is the elevation in the boiling point of a dilute solution of molal
concentration m, then
Tb  m
 Tb = Kb m …..(1)
where, Kb is called molal elevation of boiling point constant (ebullioscopic constant).
287
Board Answer Paper : March 2018
b. From equation (1), Kb = Tb/m.
Tb is expressed in Kelvin and m in mol kg1. Hence, unit of Kb is K kg mol1.
For m = 1 molal, Kb = Tb. Hence, Kb is defined as the elevation of boiling point
produced, when one mole of solute is dissolved in 1 kg of solvent.
c. If W2 gram of solute with molar mass M2 is dissolved in W1 gram of solvent then,
Mass of solutein kg
Molality (m) =
Molar mass of solutein kg mol1 ×Mass of solvent in kg
W2
m = …..(2)
M 2 W1
d. Substituting the value of molality from equation (2) in equation (1),
W2
Tb = Kb
M 2 W1
K b W2
Therefore, M2 = …..(3)
Tb W1
Equation (3) shows the relationship between molar mass and elevation of boiling
point.
[2]
ii. a. Third law of thermodynamics: The third law of thermodynamics states that, “The
entropy of a perfectly ordered crystalline substance is zero at absolute zero of
temperature”.
b. Uses of third law of thermodynamics:
1. Third law is used to determine the absolute entropy of any substance
either in solid, liquid or gaseous state at any desired temperature.
2. During the heating process, the heat capacity of the substance is
determined at various temperatures above 0 K from which the absolute
entropy of substance in solid, liquid or gaseous state can be calculated.
3. The standard molar entropy (S) of a pure substance, elements, ions,
etc., can be measured at 25 C and 1 atmosphere.
4. Standard entropy change (S) for a reaction can be calculated and the
spontaneity of the reaction can be predicted. (Any two uses)
[2]
iii.
Anode Cathode
() (+)

38% H2SO4
Pb plates Acid proof container
packed with Pb plates
spongy lead packed with
PbO2
Lead Storage Battery
[2]
iv. a. In ionic crystalline solids, constituent particles are positively charged cations and
negatively charged anions placed at alternate lattice points.
b. The ions are held by strong coulombic electrostatic forces of attraction compensating
opposite forces. Hence, they are hard.
c. Since there are no free electrons, ionic solids are not malleable.
d. On applying a shearing force, ionic crystals break into small units, hence they are
brittle.
[2]
288
Chemistry
v. 1. In both the elementary reactions, two reactant molecules take part in the reaction.
Hence, they are bimolecular (i.e., Molecularity = 2).
2. The overall reaction is obtained by adding the two elementary steps of the
mechanism. Thus,
Cl(g) + O3(g)  ClO(g) + O2(g) …..(1)
ClO(g) + O(g)  Cl(g) + O2(g) …..(2)
O3(g) + O(g)  2O2(g) (overall reaction)
The species ClO(g) is a reaction intermediate because it is produced in step (1) and is
consumed in step (2).
[2]
vi. a. Semipermeable membrane: Semipermeable membrane is a membrane which
allows the solvent molecules, but not the solute molecules, to pass through it.
eg. Cellulose, cellulose nitrate are used as semipermeable membrane.
b. Reference electrode: The reference electrode is an electrode whose potential is
arbitrarily taken as zero or is exactly known.
eg. Standard hydrogen electrode is a primary reference electrode
[2]
vii. a. Action of Cl2 on CS2:
1. Chlorine reacts with CS2 to form CCl4 and S2Cl2.
CS2 + 3Cl2  CCl4 + S2Cl2
Carbon Chlorine Carbon
disulphide tetrachloride
2. S2Cl2 then reacts with CS2 to give CCl4 and S.
2S2Cl2 + CS2  CCl4 + 6S
Carbon Carbon Sulphur
disulphide tetrachloride
b. Action of Cl2 on excess of NH3:
Action on ammonia: With excess ammonia, chlorine gives nitrogen and ammonium
chloride.
3Cl2 + 8NH3  6NH4Cl + N2
Chlorine Ammonia Ammonium Nitrogen
(excess) chloride [2]
viii. The van Arkel method for refining zirconium:
a. The impure zirconium metal (Zr) is heated in a vessel with little iodine to form
iodide of metal, which is covalent and thus volatizes.
Zr(S) + 2I2(g)  870K
 ZrI2(g)
Impure Iodine Volatile iodide
zirconium of zirconium
b. The volatile iodide of zirconium is electrically heated using tungsten filament above
1700 K. The pure metal gets deposited on the filament.
ZrI4(g)  2075K
 Zr(s) + 2I2(g)
Volatile iodide Zirconium Iodine
or zirconium
[2]
Q.3. Answer any THREE of the following:
i. a. Phosphorous reacts with magnesium:
Phosphorous (on heating) combines with magnesium to form magnesium phosphide.

6Mg + P4   2Mg3P2
Magnesium Phosphorus Magnesium phosphide
b. Flowers of sulphur boiled with calcium hydroxide:
Sulphur is boiled with calcium hydroxide (milk of lime) to obtain a mixture of
calcium pentasulphide (CaS5) and calcium thiosulphate (CaS2O3).
3Ca(OH)2 + 12S  2CaS5 + CaS2O3 + 3H2O
Calcium Sulphur Calcium Calcium Water
hydroxide pentasulphide thiosulphate
289
Board Answer Paper : March 2018
c. Action of ozone on hydrogen peroxide:
Ozone reduces H2O2 to water.
H2O2 + O3  H2O + 2O2
Hydrogen Ozone Water Oxygen
peroxide
[3]
ii. Given: Density (d) = 8.54 g.cm–3
Edge length (a) = 2.8 Å = 2.8  10–8 cm
Molar mass = 56 g mol–1
Avogadro’s number = 6.022  1023
1 Å = 1  10–8 cm
To find: The number of atoms in the unit cell
Atomic mass
Formulae: a. Mass of one atom =
Avogadro's number
b. Volume of unit cell = a3
Mass of unit cell
c. Density =
Volume of unit cell
56
Atomic Mass of Fe = Antilog10 (log10 56  log 6.022)
Calculation: Mass of one Fe atom = 6.022
Avogadro 's number = Antilog10 (1.7482  0.7797)
56 = Antilog10 (0.9685)
= = 9.301
6.022 1023
= 9.3  10–23 g
(2.8)3 = Antilog10 (log102.83)
Volume of unit cell = a3
= (2.8  10–8 cm)3 = Antilog10 (3log10 2.8)
= 2.195  10–23 cm3 = Antilog10 (3  0.4472)
Mass of unit cell = Antilog10 (1.3416)
Density (d) = = 21.96
Volume of unit cell
Number of atomsin unit cell  Mass of one atom
=
Volume of unit cell
Number of atomsin unit cell  9.31023 8.54  21.95
8.54 = 9.3
2.195 1023
= Antilog10 (log10 8.54 +
log10 21.95 –log 9.3)
8.54  2.195 1023 = Antilog10 (0.9315 + 1.3414 – 0.9685)
Number of atoms in unit cell = = Antilog10 (1.3044)
9.31023 = 20.15
= 2.016  2
Ans: The number of atoms per unit cell in the given iron crystal is 2.
[3]
Note: The above problem can be alternatively be solved by using the formula:
Z.M
d= 3
a .N A
This numerical has been solved using log-tables to help students with the logarithmic
calculations.
iii. Given: Mass of Al = 13 g
Molar mass of Al = 27 g mol–1
To find: Quantity of electricity (in terms of Faraday)
Mass of Al produced
Formula: Moles of Al produced =
Molar massof Al
13
Calculation: Moles of Al produced =
27
= 0.48 mole
290
Chemistry

The half reaction is: Al3(aq) + 3e   Al(s)
Thus,1 mole of Al is produced by passage of 3 mole electrons.
 0.48 mole of Al is produced by passage of 0.48  3 mole electrons
= 1.44 mole electrons = 1.44 Faradays
Ans: The quantity of electricity (in terms of Faraday) required to produce 13 g of Al from
aluminium chloride solution is 1.44 Faradays.
[3]

1 3
iv. Given: Reaction: N2(g) + H2(g)  NH3(g)
2 2
Enthalpy change (H) for above reaction = – 42.0 kJ mol–1
=  42000 J mol–1
T = 298 K
R = 8.314 J K–1 mol–1
To find: Internal energy change (U) for the formation of one mole of ammonia.
Formula: H = U + nRT
Calculation: For the reaction
1 3
N2(g) + H2(g)  NH3(g)
2 2
1 3
n = 1 –   
2 2
n = – 1 mol
From formula,
H = U + nRT
– 42000 J = U + (–1 mol)  (8.314 J mol–1 K–1)  (298 K)
– 42000 J = U – 2477.57
 U = – 42000 + 2477.57
U = –39522.4 J = –39.52 kJ
Ans: The internal energy change (U) for the formation of one mole of ammonia
is –39.52 kJ.
[3]

Q.4. Answer any ONE of the following:


i. a. Enthalpy of atomization (atomH):
The enthalpy change accompanying the dissociation of all the molecules in one mole
of gas-phase substance into gaseous atoms is called enthalpy of atomization. It is
denoted as atomH.
eg. Cl2(g)  Cl(g) + Cl(g) , atomH = 242 kJ mol1

b. Enthalpy of vaporization (vapH):


The enthalpy change that accompanies the vaporization of one mole of liquid without
changing its temperature at constant pressure is called enthalpy of vaporization. It is
denoted as ΔvapH.
eg. H2O(l)  H2O(g), ΔvapH = + 40.7 kJ mol1 at 100 ºC

ii. IF7: IF7 is the only known interhalogen compound of the type XX7. It has a pentagonal
bipyramidal structure due to sp3d3 hybridization.

291
Board Answer Paper : March 2018

5s 5p 5d F7
Ground F



state 5 4 F
5s 5p 5d
F 1 I
Third excited 3
state F
5d 2
F
Hybridized F6
state
Molecular shape
sp3d3 hybridisation
Pentagonal Bipyramidal Structure of Interhalogen compounds of Type XX7 (IF7)

iii. a. Henry’s law relates solubility of a gas with external pressure. The law states that,
“the solubility of a gas in liquid at constant temperature is proportional to the
pressure of the gas above the solution”. If S is the solubility of the gas in mol dm3,
then according to Henry’s law,
SP
S = KP
where, P is the pressure of the gas measured in atmosphere, K is constant of
proportionality and has the unit of mol dm3 atm1.

b. Given: Mass of urea = 22.22 g.


Molar mass of urea = 60 g mol–1
Mass of water = 300 g = 0.3 kg
To find: 1. Number of moles of urea (solute)
2. Molality of urea solution
Mass of thesolute
Formulae: 1. Number of moles of solute =
Molar mass of solute
Number of moles of solute
2. Molality =
Mass of solvent in kg
Calculation: From formula (1),
22.22
Number of moles of urea = = 0.37 mol
60
From formula (2),
0.37
Molality = = 1.23 mol kg–1
0.3
Ans: Number of moles of urea is 0.37 mol and molality of the urea solution is 1.23 mol kg–1.
[7]
OR

i.
a. Reaction of carbon with Fe2O3 in blast furnace.
Fe2O3 + 3C 
1500K
 2Fe + 3CO
Ferric Carbon Iron Carbon
oxide monoxide

b. Reaction of carbon with ZnO in vertical retort furnace.


ZnO + C 
1673K
 Zn + CO(g)
Zinc oxide Carbon Zinc Carbon
monoxide

292
Chemistry
ii. a. Thiosulphuric acid:
Molecular formula: H2S2O3
S
Structural formula: HO – S – OH

O
b. Dithionous acid:
Molecular formula: H2S2O4
O O

Structural formula: HO – S – S – OH
iii. A + B  Products
As the reaction is first order in each of the reactants, the rate law for the above reaction is
Rate1 = k [A][B]
a. If the concentration of A is increased by factor 3, the new rate law becomes
Rate2 = k [3A][B]
Rate2 = 3 k[A][B]
Rate2 = 3 Rate1
The rate of reaction increases three times.
b. If the concentration of A is halved and concentration of B is doubled, the new rate
law becomes
1 
Rate3 = k  A  [2B]
2 
1
Rate3 = × 2 k [A] [B]
2
Rate3 = k [A] [B]
Rate3 = Rate1
The rate of reaction remains the same.
[7]

SECTION – II

Q.5. Select and write the most appropriate answer from the given alternatives for each
sub-question:
i. (D) glyptal [1]
ii. (A) 1, 3. [1]
5 O
OH Secondary
HOH2 C
β
4
H H 1
Primary H
H
3 2
OH OH
Secondary
Secondary β-D-ribose

iii. (C) tridentate [1]


The ligand diethylenetriamine (dien) has three donor N atoms.
iv. (A) Propan-1-ol [1]
Hydroboration-oxidation of alkenes follow anti-Markownikoff’s rule.
v. (C) Alkaline potassium permanganate [1]
293
Board Answer Paper : March 2018
vi. (B) 1-Bromo-2,2-dimethylpropane [1]
CH3 CH3
CH3 – C – CH2 – Br + 2Na + Br – CH2 – C – CH3

CH3 CH3 CH3 CH3


1-Bromo-2, 2-dimethylpropane
 CH3 – C – CH2 – CH2 – C – CH3 + 2NaBr

CH3 CH3 Sodium


2, 2, 5, 5-Tetramethylhexane bromide

vii. (A) novestrol [1]

Q.6. Answer any SIX of the following:


i.
a. On addition of hydrochloric acid to the yellow coloured solution of potassium
chromate, orange-red coloured solution of potassium dichromate is obtained.
2K2CrO4 + 2HCl  K2Cr2O7 + 2KCl + H2O
Potassium Hydrochloric Potassium Potassium
potassium Water
chromate acid dichromate chloride
(yellow) (orange-red)

b. On addition of potassium hydroxide solution to the orange red coloured solution of


potassium dichromate, yellow coloured solution of potassium chromate is obtained.
K2Cr2O7 + 2KOH  2K2Cr2O4 + H2O
Potassium Potassium Potassium Water
dichromate hydroxide chromate
(orange-red) (yellow)
[2]

ii. Structure of [Co(NH3)6]3+:


a. The oxidation state of Co is +3. The electronic configuration of Co3+ is 3d6.
b. d2sp3 hybridisation of two d, one s and three p orbitals results in octahedral geometry.
c. Coordinate bonds are formed by overlap of metal d2sp3 hybrid orbitals and ligand
orbitals.

Co3+     
6
3d 4s 4p

Spin pairing and d2sp3   


hybridisation
4s 4p
3d6

[Co(NH3)6]3+         
Inner orbital or 3d
low spin complex d2sp3 hybrid orbitals
(6 electrons of Co)
(12 electrons of NH3 ligands)

d. Due to absence of unpaired electron, [Co(NH3)6]3+ is diamagnetic.


[2]
294
Chemistry
iii. a. Ethanol has higher boiling point than ethane
due to the presence of intermolecular C2H5 C2H5
hydrogen bonding in it. Hydrogen bonding  +
arises due to the presence of electronegative OH OH OH
oxygen atom in OH group of ethanol.  +  +
C2H5
b. Oxygen atom attracts electron density of OH
Intermolecular hydrogen bonds
bond towards itself and hence, it gets partial
negative charge, while H atom gets partial in ethanol molecules
positive charge.
 +
C2H5 – O – H

Hence, in ethyl alcohol, –OH group becomes polar.


c. This results in strong intermolecular attraction between oxygen atom of one molecule
of ethanol and ‘H’ atom of another ethanol molecule, giving rise to strong hydrogen
bonding which is not present in ethane.
d. Hence, higher thermal energy is required to separate or evaporate ethanol molecules.
Therefore, ethanol has higher boiling point than ethane.
[2]
iv.
C6H5  C  N + C6H5 Mg Br  
dry ether
 C6H5  C = NMgBr
Benzonitrile Phenyl
magnesium
C6H5
bromide Adduct

H3O+
C6H5
C = O + Mg(OH) Br + NH3
C6H5
Benzophenone

[2]

v. a. Reaction of ethylamine with p-toluenesulphonyl chloride gives


N-ethyl-p-toluenesulphonamide.
O O
CH3 S – Cl + H – N – C2H5  CH3 S – N – C2H5 + HCl
O H O H
p-Toluenesulphonyl Ethylamine N-Ethyl-p-toluene
chloride (Primary amine) sulphonamide
(Soluble in aqueous KOH)

b. Reaction of diethylamine with p-toluenesulphonyl chloride gives N,N-diethyl-p-


toluenesulphonamide.
O O
CH3 S – Cl + H – N – C2H5  CH3 S – N – C2H5 + HCl
O C2H5 O C2H5
p-Toluenesulphonyl Diethylamine N,N-Diethyl-p-toluenesulphonamide
chloride (Insoluble in aqueous KOH)
[2]
295
Board Answer Paper : March 2018
vi. a. Amino acids are bifunctional compounds containing a carboxylic acid group and an
amino group along with a side chain (R group) specific to each amino acid.
H2N  CH  COOH

R
-Amino acid
(R = H or alkyl or aryl group)

b. The reaction of the COOH group of one amino acid molecule and NH2 group of
the neighbouring amino acid molecule forms peptide having CONH linkage by
elimination of water.
R1 O H R2 O R1 R2
 H2 O
H2N  CH C OH + H N  CH  C  OH 
 H2N  CH CO  NH  CH  COOH
-Amino acid Peptide linkage
-Amino acid
(Dipeptide)

[2]
vii. a. Antiseptic: Drugs which are applied to the living tissues to kill the bacteria and to
stop their growth in wound, thus preventing infection are called antiseptics.
eg. Dettol
b. Antioxidant: Antioxidant is a substance which when added to food, retards or
prevents oxidative deterioration of food.
eg. Butylated hydroxy anisole (BHA)
[2]
viii. Mechanism of alkaline hydrolysis of tert-butyl bromide:
a. Alkaline hydrolysis of tert-butyl bromide can be explained by unimolecular, first
order nucleophilic substitution (SN1 ) mechanism.
Reaction:
CH3 CH3
CH3  C  Br + OH  CH3  C  OH + Br
(Nucleophile) (Bromide ion)
CH3 CH3
(tertButyl bromide) (tertButyl alcohol)

b. Unimolecular nucleophilic substitution (SN1 ) is a two step process. The first step is a
slow step, while the second one is a fast step. The mechanism can be written as follows:

1. Formation of carbonium ion:


i. The C–Br bond in tertiary butyl bromide slowly dissociates to form bromide
ion (Br) and tertiary butyl carbonium ion [+C(CH3)3].
ii. This is a slow process and hence, it is a rate determining step (R.D.S.).
CH3 CH3
H3C
+ Br
C*
+ 
Slow step

  H3C  C Br  C+
R.D.S.

H3C Br 120
CH3 H3C H3C CH3
tert-Butyl bromide tert-Butylcarbonium ion
Transition state I
/carbocation

296
Chemistry
2. Formation of the product:
i. The second step is the attack of OH leading to the COH bond formation.
ii. It is a fast process. CH3

Back side
attack C*
HO CH3
CH3 CH3
H3C
+  tert-Butyl alcohol
C+ Fast H3C  C OH [Inversion of

 configuration (50%)]
120
H3C CH3 H3C CH3
HO
tert-Butyl carbonium ion
Front side
/carbocation Transition state II
attack C*
H3C OH
CH3
tert-Butyl alcohol
[Retention of configuration (50%)]

[2]
Q.7. Answer any THREE of the following:
i. a. Chloro group of chlorobenzene can be replaced by –CN group at higher temperature
and pressure.
Cl CN

NaCN  CuCN
+ CN– 
473K, Pr essure
 + Cl–
Chlorobenzene Cyanobenzene Chloride ion
b. Isobutyraldehyde (2-methylpropanal) undergoes Cannizzaro reaction, even though it
contains an -hydrogen atom.
CH3 CH3 CH3
2CH3 – CH – CHO + KOH  CH3 – CH – COOK + CH3 – CH – CH2 – OH
Isobutyraldehyde 50% Potassium isobutyrate Isobutyl alcohol
Potassium
hydroxide
c. Butanone reacts with 2,4-dinitrophenyl hydrazine in presence of acid catalyst to form
2,4-dinitrophenyl hydrazone
CH3 O2N

CH3 – CH2 – C = O + H2N – NH NO2

Butanone 2,4-Dinitrophenyl hydrazine


CH3 O2N
H

 CH3 – CH2 – C = N – NH NO2 + H2O

2,4-Dinitrophenyl hydrazone
[3]
ii. a. Preparation of carbolic acid or phenol from benzene sulphonic acid:
Phenol is prepared from benzene sulphonic acid as follows:
1. When benzene sulphonic acid is neutralised by aqueous sodium hydroxide
(NaOH), sodium benzene sulphonate is obtained.
297
Board Answer Paper : March 2018
 +
SO3H SO3Na

+ NaOH(aq.) 
 Neutralisation
+ H2O
Benzene Sodium benzene
sulphonic acid sulphonate

2. Dry sodium benzene sulphonate when fused with excess of sodium hydroxide
at 573 K, sodium phenoxide is obtained along with sodium sulphite (Na2SO3).
 +  +
SO3Na ONa

+ 2NaOH   fused


573K + Na2SO3 + H2O
Sodium
Sodium benzene (Excess) Sodium sulphite
sulphonate phenoxide

3. Sodium phenoxide when hydrolysed by heating with dilute sulphuric acid,


phenol is obtained.
 +
ONa OH

2 + H2SO4 
Hydrolysis
2 + Na2SO4
(dil.)
Sodium phenoxide Phenol
OR
When a current of carbon dioxide is passed through aqueous sodium
phenoxide, phenol is obtained as product.
 +
ONa OH

+ H2O + CO2  + NaHCO3


Sodium phenoxide Phenol

b. Action on phenol: When phenol is treated with aqueous neutral FeCl3, it gives red to
violet colouration.
The colour obtained is due to the formation of violet coloured ferric phenoxide complex.

3C6H5OH + FeCl3  Fe(C6H5O)3 + 3HCl


Phenol (aq./neutral) Ferric phenoxide
(Violet colour)
[3]
iii. a. Preparation of nylon-2-nylon-6:
The condensation polymerization of glycine and -amino caproic acid gives nylon-
2-nylon-6.
nH2N  CH2  COOH + NH2 ( CH2 )5 COOH 
H2O
 CCH2NHC ( CH2 )5 NH n

Glycine -Amino caproic acid O O
Nylon-2-nylon-6

Thus, it is a copolymer containing an amide linkage.


b. Uses: Nylon-2-nylon-6 is a biodegradable polymer and is used as orthopedic devices,
implants, sutures and drug release matrices.
[3]

298
Chemistry
iv. a. Preparation of glucose from cane sugar:
1. Glucose can be prepared in the laboratory by boiling sucrose (cane sugar) with dilute
hydrochloric acid or sulphuric acid for about two hours. This hydrolyzes sucrose to glucose
and fructose.
C12H22O11 + H2O 
dil.HClor H 2SO 4

 C6H12O6 + C6H12O6
Sucrose water Glucose Fructose
(cane sugar)
2. In order to separate glucose from fructose, alcohol is added during cooling.
3. Glucose is almost insoluble in alcohol. It crystallizes out first, while fructose is more
soluble. It remains in the solution.
4. The solution is filtered to obtain the crystals of glucose.
b. Complex: Copper (II) hexacyanoferrate (II)
Formula: Cu2[Fe(CN)6]
[3]
Q.8. Answer any ONE of the following:
i. a. Lanthanoid contraction: The atomic and ionic radii of lanthanoids show gradual
decrease with increase in atomic number. It is known as Lanthanoid contraction.
There are 14 lanthanoids from Ce to Lu. As we move from one element to other
element, the contraction in size is small. However for fourteen elements from Ce to
Lu the net contraction is appreciable
b. Causes of the lanthanoid contraction:
1. The nuclear positive charge increases by +1, when the atomic number
increases by 1. An electron is added in the partly filled 4f orbital.
2. The shielding of 4f electrons is less effective than the shielding of 5d electron
as 4f orbital is more diffused in shape than 5d orbital.
Thus, greater effective nuclear charge is experienced with the increase in the
atomic number. This results in slight pull of the valence shell towards the
nucleus.
3. Thus, the atomic and ionic radii decrease slightly with increase in the atomic
number, thereby causing lanthanoid contraction.
4. In a given lanthanoid series, atomic radii decrease by 10 pm and the ionic radii
decrease by 18 pm.
Thus, the extent of decrease is very small.
c. Structure of chloroxylenol
OH

H3C CH3
Cl
Chloroxylenol (C8H9ClO)

Structure of adenine
NH2
C
N
C N

HC
C CH
N N
H
Adenine (A)

299
Board Answer Paper : March 2018
d. Ethylamine (primary amine) and ethylmethylamine (secondary amine) react
differently with nitrous acid (HNO2), hence nitrous acid is used to distinguish
between them.
Ethylamine reacts with nitrous acid to give ethyl alcohol with liberation of nitrogen.
NaNO 2  HCl
C2H5 – NH2 + HNO2 
273 278K
 C2H5 – OH + H2O + N2 
Ethylamine Nitrous Ethyl Water Nitrogen
acid alcohol
Ethylmethylamine reacts with nitrous acid to give yellow oily
N-nitrosoethylmethylamine.
NaNO 2  HCl
CH3 – NH + HO – N = O 
273 278K
 CH3 – N –N = O + H2O
C2H5 C2H5
Ethylmethylamine Nitrous N-Nitrosoethylmethylamine Water
acid (yellow oily)
[7]
OR
i. a.
1. Action of LiAlH4/H3O+ on ethanoic acid:
Strong reducing agent LiAlH4 reduces ethanoic acid to ethyl alcohol.
CH3COOH 
LiAlH 4 / Ether
  CH3CH2OH
H 3O
Ethanoic acid Ethyl alcohol

2. Action of PCl3 on ethanoic acid:


Ethanoic acid reacts with phosphorus trichloride to give ethanoyl chloride.

3CH3COOH + PCl3   3CH3COCl + H3PO3
Ethanoic Phosphorus Ethanoyl Phosphorous
acid trichloride chloride acid
3. Action of P2O5 on ethanoic acid:
When ethanoic acid is distilled over phosphorus pentoxide, it gives ethanoic
anhydride. This reaction is reversible.
O O O O
P2 O5 , 

CH3 – C – OH + H – O – C – CH3  
 CH3 – C – O – C – CH3 + H2O
Ethanoic Ethanoic Ethanoic anhydride Water
acid acid
b. Ethyl bromide reacts with alcoholic potassium cyanide (KCN) to form ethyl cyanide.
Ethyl cyanide on reduction by sodium and alcohol gives n-propylamine (primary
amine).

CH3 – CH2 – Br + KCN(alc.)   CH3 – CH2 – CN + KBr
Ethyl bromide Potassium Ethyl cyanide (A) Potassium
cyanide bromide

4[H] Na + C2H5OH

CH3 – CH2 – CH2 – NH2


n-Propylamine (B)
(Primary amine)
c. Hoffmann bromamide degradation reaction:
1. Primary amine can be prepared by reaction of amide with bromine and
aqueous or alcoholic sodium hydroxide.
O

R  C NH2 + Br2 + 4NaOH  R  NH2 + Na2CO3 + 2NaBr + 2H2O


Aqueous or Sodium Sodium
Amide 1 Amine Water
alcoholic carbonate bromide
sodium hydroxide
300
Chemistry
eg. Ethanamine is prepared by reaction of propanamide with bromine and
aqueous or alcoholic sodium hydroxide.
O

CH3  CH2  C  NH2 + Br2 + 4NaOH  CH3  CH2  NH2 + Na2CO3 + 2NaBr + 2H2O
Propanamide (Aqueous Ethanamine Sodium Sodium Water
or alcoholic carbonate bromide
sodium hydroxide)
2. This reaction is known as Hoffmann bromamide degradation. It involves molecular
rearrangement.
3. An alkyl or aryl group migrates from the carbonyl carbon of amide to the adjacent
nitrogen atom.
4. This reaction is useful for decreasing the length of carbon chain by one carbon
atom.
[7]

301
Board Answer Paper : March 2018

BOARD ANSWER PAPER : MARCH 2018


MATHEMATICS AND STATISTICS

Note: Answer to every question must be written on a new page.

SECTION – I
Q.1. (A) Select and write the most appropriate answer from the given alternatives in each of
the following sub-questions:
i. (A)
M11 = 1 and A11 = (1)1 + 1 (1) = 1
M12 = 4 and A12 = (1)1 + 2 (4) =  4
M21 = –3 and A21 = (1)2 + 1 (3) = 3
M22 = 2 and A22 = (1)2 + 2 (2) = 2
A A  1  4 
 Matrix of co - factors = [Aij]2  2 =  11 12
 = 3
A
 21 A 22   2 
T 1 3
 adj A =  Aij  =   [2]
2 2
 4 2
ii. (B)
2
sec x =
3
    11 
 sec x = sec   and sec x = sec  2   = sec  
6  6  6 
 11
such that 0 < < 2 and 0 < < 2
6 6
 11
 the required principal solutions are x = and x = . [2]
6 6
iii. (D)
a1a 2  b1b 2  c1c 2
cos  =
a  b12  c12  a 22  b 22  c 22
2
1

Here, a1 = 3, b1 = 2, c1 = 6 and a2 = 2, b2 = 1, c2 = 2


3  2   2 1  6  2 
 cos  =
3  22  62  ( 2) 2  12  22
2

6  2  12
=
9  4  36  4  1  4
8
=
7  3
8
 cos  =
21
1  8 
  = cos   [2]
 21 
(B) Attempt any THREE of the following:
i. a. Some students of this college do not live in the hostel. [1]
b. Let p: 6 is an even number.
q: 36 is a perfect square.
 The given statement is of the form p  q.
Its negation is p  q
i.e. 6 is not an even number and 36 is not a perfect square. [1]
302
Mathematics and Statistics
ii. Since, ,  and  are the angles made by the line with the coordinate axes.
 cos2  + cos2  + cos2  = 1 …. (i)
cos 2 + cos 2 + cos 2 + 1
= (2cos2 1)+(2cos2 1)+(2cos2   1) + 1
= 2(cos2  + cos2  + cos2 )  3 + 1
= 2(l)  3 + 1 …[From (i)]
=0
 cos 2 + cos 2 + cos 2 + 1 = 0 [2]
iii. The distance of a point P(x1, y1, z1) from the plane ax + by + cz + d = 0 is given by
ax1  by1  cz1  d
.
a 2  b2  c2
Here, x1 = 1, y1 = 2, z1 = 1, a = 1, b = 2, c = 4, d = 10
11  (2) (2)  4(1)  10
 required distance =
12   2   42
2

1  4  4  10
=
1  4  16
17
=
21
17
 required distance = units [2]
21

iv. Let a  4iˆ  ˆj  2kˆ and b   2iˆ  ˆj  kˆ


The vector equation of the line passing through a point with position vector a and parallel to b
r  a  b .
 the vector equation of the required line is
r  (4iˆ  ˆj  2k)
ˆ   (2iˆ  ˆj  k)
ˆ [2]

3 -2 7
v. a ⋅ (b ´ c) = 5 1 -2
1 1 -1
= 3(1 + 2) + 2(5 + 2) + 7(5  1)
= 3  6 + 28
 a ⋅ (b ´ c) = 25 [2]
Q.2. (A) Attempt any TWO of the following:
i. Consider  ABC.
Let P, Q, R be the midpoints of the sides BC, CA, AB respectively.
Let a , b , c , p , q , r , g be the position vectors of the points A, B, C, P, Q, R, G respectively.
Since P, Q, R are the mid-points of the sides BC, CA, AB respectively
 By midpoint formula, we get
A
bc
p= ….(i)
2
R Q
ca G
q= ….(ii)
2
ab B C
r= ….(iii)
2 P
From (i), (ii) and (iii), we get
2p = b + c  2p + a = a + b + c

303
Board Answer Paper : March 2018
2q = c + a  2q + b = a + b + c
2r = a + b  2r + c = a + b + c
2p  a 2q  b 2r  c a  b  c
 = = =
3 3 3 3
2p  a 2q  b 2r  c a bc
 = = = = g (say)
2 1 2 1 2 1 3
This shows that the point G whose position vector is g lies on the three medians AP, BQ,
CR dividing them internally in the ratio 2:1.
Hence, the three medians are concurrent. [3]
ii.
1 2 3 4 5 6 7 8
p q pq ~p ~q p  q ~p  ~q (p  q)  (~p  ~q)
T T T F F T F T
T F F F T F F F
F T F T F F F F
F F T T T F T T

The entries in the columns 3 and 8 are identical.


 p  q  (p  q)  (~p  ~q) [3]

iii. Let a , b and c be the position vectors of points A, B and C respectively.


 a = 2 î + p ĵ  3 k̂ , b = q î  2 ĵ + 5 k̂ and c =  5 î + ĵ + r k̂
Given, origin is the centroid of ABC.
 By centroid formula, we get
abc
0 =
3
 0 = a + b + c
 0 = 2 î + p ĵ  3 k̂ + q î  2 ĵ + 5 k̂  5 î + ĵ + r k̂

 0. î + 0. ĵ + 0. k̂ = (2 + q  5) î + (p  2 + 1) ĵ + (3 + 5 + r) k̂
 0. î + 0. ĵ + 0. k̂ = (q  3) î + (p  1) ĵ + (r + 2) k̂
 By equality of vectors, we get
q  3 = 0, p  1 = 0, r + 2 = 0
 p = 1, q = 3, r = 2 [3]

(B) Attempt any TWO of the following:


i. Let ax2 + 2hxy + by2 = 0 ….(i)
be a homogeneous equation of degree 2 in x and y.
Case I:
If b = 0 (i.e., a  0, h  0), then the equation (i) reduces to ax2 + 2hxy = 0
i.e., x(ax + 2hy) = 0
This represents two lines, x = 0 and ax + 2hy = 0, both passing through the origin.
Case II:
If a = 0 and b = 0 (i.e., h  0), then the equation (i) reduces to 2hxy = 0, i.e., xy = 0 which
represents the coordinate axes and they pass through the origin.
Case III:
If b  0,

304
Mathematics and Statistics
Multiplying both sides of equation (i) by b, we get
abx2 + 2hbxy + b2y2 = 0
 b2y2 + 2hbxy = abx2
To make L.H.S. a complete square, we add h2x2 on both the sides.
 b2y2 + 2hbxy + h2x2 = abx2 + h2x2
 (by + hx)2 = (h2  ab)x2

 
2
 (by + hx)2 =  h 2  ab x 
 

(by + hx)   h  ab  x  = 0


2
2
 2
 

    
 by  hx   h  ab x   (by  hx ) 
2
 
h 2  ab x  = 0

This is the joint equation of two lines
(by + hx) +  
h 2  ab x = 0 and (by + hx)   
h 2  ab x = 0

i.e., h    
h 2  ab x + by = 0 and h  h 2  ab x + by = 0

These lines pass through the origin.


From the above three cases, we conclude that the equation ax2 + 2hxy + by2 = 0 represents a
pair of lines passing through the origin, if h2  ab  0. [4]
ii. In ABC by sine rule, we have
a b c
  k
sin A sin B sin C
 a = k sin A, b = k sin B and c = k sin C
c  a k sin C  k sin A
Now, consider 
c  a k sin C  k sin A
sin C  sin A
=
sin C  sin A
CA CA
2cos    sin  
=  2   2 
CA C  A
2sin    cos  
 2   2 
C  A C A
= cot    tan  
 2   2 
 B C A
= cot     tan   ….[A + B + C = ]
2 2  2 
B CA
= tan  tan  
2  2 
CA
tan  
ca  2 
  [
ca B
cot
2
CA  ca  B
 tan    cot [4]
 2  c a 2

é 1 2 -2ù
ê ú
iii. A = ê -1 3 0úú
ê
ê 0 -2 1úû
ë
1 2 -2
 A = -1 3 0 = 1(3 – 0) – 2(–1 – 0) – 2(2 – 0) = 3 + 2 – 4 = 1 ≠ 0
0 -2 1
 A1 exists.
305
Board Answer Paper : March 2018
Consider, AA1 = I
é 1 2 -2ù é 1 0 0ù
ê ú 1 ê ú
 ê -1 3 0úú A = ê 0 1 0ú
ê ê ú
ê 0 -2 1úû ê 0 0 1ú
ë ë û
Applying R2  R2 + R1,
é1 2 -2ù é 1 0 0ù
ê ú 1 ê ú
ê0 5 -2 úA = ê 1 1 0ú
ê ú ê ú
ê 0 -2 1úû ê 0 0 1ú
ë ë û
Applying R2  R2 + 2R3,
é1 2 -2ù é 1 0 0ù
ê ú 1 ê ú
ê0 1 0úú A = ê 1 1 2ú
ê ê ú
ê 0 -2 1úû ê 0 0 1ú
ë ë û
Applying R1  R1  2R2 and R3  R3 + 2R2,
é 1 0 -2ù é-1 -2 -4ù
ê ú 1 ê ú
ê0 1 0 úA = ê 1 1 2 ú
ê ú ê ú
ê0 0 1úû ê 2 2 5úû
ë ë
Applying R1  R1 + 2R3,
é 1 0 0ù é 3 2 6ù
ê ú ê ú
ê 0 1 0ú A1 = ê 1 1 2ú
ê ú ê ú
ê 0 0 1ú ê 2 2 5ú
ë û ë û
é 3 2 6ù
ê ú
 A1 = êê 1 1 2úú [4]
ê 2 2 5ú
ë û
Q.3. (A) Attempt any TWO of the following:
i. Given equation is 5x2 + 2xy  3y2 = 0.
Comparing with ax2 + 2hxy + by2 = 0, we get
a = 5, 2h = 2, b = 3
Let m1 and m2 be the slopes of the lines represented by 5x2 + 2xy  3y2 = 0.
-2h 2 a -5
 m1 + m2 = = and m1 m2 = =
b 3 b 3
Since, the required lines are perpendicular to these lines.
1 1
 slopes of the required lines are - and - .
m1 m2
Required lines also pass through the origin, therefore their equations are
1 1
y= - x and y = - x
m1 m2
 x + m1y = 0 and x + m2y = 0
 the joint equation of the lines is
(x + m1y)(x + m2y) = 0
 x2 + (m1 + m2)xy + m1m2y2 = 0
2 5
 x2 + xy  y2 = 0
3 3
 3x + 2xy  5y2 = 0
2
[3]
x 1 y  3 z
ii. Let a and b be the vectors in the direction of the lines   and
4 1 8
x  2 y 1 z  4
  respectively.
2 2 1
 a  4iˆ  ˆj  8kˆ and b  2iˆ  2ˆj  kˆ

306
Mathematics and Statistics
 a  b = 4  2 + 1  2 + 8  1 = 8 + 2 + 8 = 18
and a  42  12  82  16  1  64  81 = 9

b  22  22  1  4  4  1  9 = 3
Let  be the acute angle between the two given lines.
ab 18 2
 cos  =  
ab 93 3

 2
  = cos1   [3]
3 

iii. Let p : An angle is a right angle,


q : Its measure is 90.
 The symbolic form of the given statement is p  q.
Converse: q  p
i.e. If the measure of an angle is 90, then it is a right angle.
Inverse: ~p  ~q
i.e. If an angle is not a right angle, then its measure is not 90.
Contrapositive: ~q  ~p
i.e. If the measure of an angle is not 90, then it is not a right angle. [3]
(B) Attempt any TWO of the following:
3
i. Let sin1   = x
5
3 
 sin x = and 0 < x <
5 2
 cos x > 0
Now, cos x = 1 sin 2 x
2
3
= 1  
5
9
= 1
25
4
=
5
 12 
Let cos1   = y
 13 
12 
 cos y = and 0 < y <
13 2
 sin y > 0
2
 12 
Now, sin y = 1 cos 2 y = 1  
 13 
144 5
= 1 =
169 13
3  12  4 5
But, sin (x + y) = sin x cos y + cos x sin y =   +  
5  13  5  13 
36  20 56
= =
65 65
 56 
 x + y = sin–1  
 65 
3  12   56 
 sin–1   + cos–1   = sin–1   [4]
5  13   65 
307
Board Answer Paper : March 2018
ii. Let a , b , c be the position vectors of the points A, B and C respectively.
 a = ˆi  kˆ , b = ˆi  ˆj  kˆ , c = 4 î  3 ĵ + 2 k̂
The vector equation of the plane passing through the points A  a  , B  b  and C  c  is

  
r  AB  AC = a  AB AC 
   
Now, AB = b  a = ˆi  ˆj  kˆ  ˆi  kˆ =  ĵ

and AC = c  a =  4iˆ  3jˆ  2kˆ    ˆi  kˆ  = 3 î  3 ĵ + k̂


ˆi ˆj kˆ
 AB  AC = 0 1 0
3 3 1

= î (1  0)  ĵ (0) + k̂(0  3) =  ˆi  3kˆ


 Vector equation of the required plane is
    
r  ˆi  3kˆ  ˆi  kˆ  ˆi  3kˆ

 r   ˆi  3kˆ  = 1(–1) + 1(3)

 r   ˆi  3kˆ  = –1 + 3

 r   ˆi  3kˆ  = 2 [4]

iii. To draw the feasible region, construct table as follows:


Inequality 5x + y  5 x+y3
Corresponding equation (of line) 5x + y = 5 x+y=3
Intersection of line with X-axis (1, 0) (3, 0)
Intersection of line with Y-axis (0, 5) (0, 3)
Region Non-origin side Non-origin side
Shaded portion XABCY is the feasible region, Y
whose vertices are A(3, 0), B and C (0, 5).
B is the point of intersection of the lines 7
x + y = 3 and 5x + y = 5
6
Solving the above equations, we get
1 5 1 5 5 C(0, 5)
x= ,y=  B  , 
2 2 2 2 4
Here, the objective function is
Z = 7x + y 3 1 5
B , 
Z at A(3, 0) = 7(3) + 0 = 21 2
2 2
 
1 5 1 5 7 5
Z at B  ,  = 7   + =  = 6 1
2 2 2 2 2 2
A(3, 0)
Z at C (0, 5) = 7(0) + 5 = 5 X X
O 1 2 3 4 5 6
 Z has minimum value 5 at C(0, 5).
 Z is minimum, when x = 0 and y = 5.
Y 5x + y = 5 x+y=3
[4]

SECTION – II

Q.4. (A) Select and write the most appropriate answer from the given alternatives in each of
the following sub-questions:
i. (C)
x 1 0 1 2
P(x) 0.4 0.3 0.2 0.1
E(X) = xi . P(xi) = (1) (0.4) + 0(0.3) + 1(0.2) + 2(0.1) = 0 [2]
308
Mathematics and Statistics
ii. (A)
k
1 
 2  8x2 dx  16
0
k
1 1 

2   2 x 2  12 dx  16
0
k
1 1  
 1
 2 tan (2 x )   16
2  0
1 
 tan 1 (2k) 
4 16

 2k = tan
4
1
 k= [2]
2

iii. (D)
dy 2
 y = x log x
dx x
2
 x dx 2
 I.F. = e = e2log x = elog x = x2 [2]
(B) Attempt any THREE of the following:
 cos x  sin x 
i. Let I =  e   dx
x

sin 2 x 
 cos x sin x 
=  e  sin  dx
x
2
x sin 2 x 

=  e  cot x  cosec x  cosec x  dx


x

= (1)  e  cosec x  cot x  cosec x  dx


x

Put f(x) = cosec x


 f (x) = – cot x cosec x
 
I =  e x f  x   f   x   dx
= – ex f(x) + c
= – ex cosec x + c [2]

ii. Let y = tan2 (log x3)


dy d
 =  tan 2  log x 3  
dx dx  
d
= 2tan (log x3). tan (log x3)
dx
d
= 2 tan (log x3 ).sec 2 (log x3 ) (log x3 )
dx
1 d 3
= 2tan (log x3).sec2 (log x3).  3 
 x 
 x  dx
1
= 2tan (log x3).sec2(log x3).  3   3x 2
x 
dy 6 tan (log x 3 ).sec2 (log x 3 )
 = [2]
dx x

309
Board Answer Paper : March 2018
iii. Y
Q(0, 2)

X X
O P(1, 0)

Y
By the symmetry of the ellipse, required area of the ellipse is 4 times the area of the region
OPQO.
x2 y2
Given equation of ellipse is + =1 ….(i)
1 4
where a = 1, b = 2
For the region, the limits of integration are x = 0 and x = 1.
From (i), we get
y2 x2
=1
4 1
 y2 = 4(1 – x2)
 y =  2 1 x 2
 y = 2 1  x2 ….[ In first quadrant, y > 0]
 required area = 4(area of the region OPQO)
1 1

= 4  y dx = 4  2 1  x 2 dx
0 0

1
x 1 
= 4  2  1  x 2  sin 1 ( x) 
2 2 0
1
= 8[0 + sin1 (1)]
2
1 
=8  = 2
2 2
 Area of ellipse is 2 sq. units. [2]
iv. y = c1e2x + c2e2x ...(i)
Differentiating w.r.t.x, we get
dy
= 2c1e2x  2c2e2x
dx
Again, differentiating w.r.t.x, we get
d2 y d 2x d
2
= 2c1 (e )  2c 2 (e 2 x )
dx dx dx
= 2c1 (2e2x)  2c2 ( 2e 2x)
= 4c1e2x + 4c2e 2x
= 4(c1e2x + c2e2x)
d2 y
 = 4y ...[From (i)]
dx 2
d2 y
  4y = 0 [2]
dx 2

310
Mathematics and Statistics
v. n = 10 and p = 0.4 ...(given)
q = 1  p = 1  0.4 = 0.6
E(X) = np = (10) (0.4) = 4
Var (X) = npq = (10) (0.4) (0.6) = 2.4 [2]
Q.5. (A) Attempt any TWO of the following:
1
i. Let I =  3 + 2sin x + cos x dx
 x
Put tan   = t
2  
 x = 2 tan–1 t
2dt 2t 1- t 2
 dx = 2
and sin x = 2
, cos x =
1+ t 1+ t 1 + t2
1 2
 I =   dt
2t 1  t 1+ t 2
2
3+ 2 +
1+ t 2 1+ t 2
2dt dt
=  =2  2
3 +3t 2 + 4t +1  t 2 2t + 4t + 4
2 dt dt
=
2 t 2
+ 2t + 2
=  (t +1)
2
 1+ 2
dt
=  (t +1) + 12 2
= tan1 (t + 1) + c

æ æ xö ö
 I = tan1 çç tan ççç ÷÷÷ +1÷÷÷ + c [3]
çè è 2 ø ø÷

ii. x = a cos3 t
Differentiating w.r.t. t, we get
dx d 3 2 d
 a (cos t) = a3(cos t) (cos t)
dt dt dt
= 3acos2 t (sin t) = 3acos2 t . sin t
y = a sin3 t
Differentiating w.r.t. t, we get
dy d d
= a (sin t)3 = a3 (sin t)2 (sin t)
dt dt dt
= 3asin2 tcos t
 dy 
dy  dt  3a sin 2 t cos t sin t
 = = = ….(i)
dx  dx 3a cos t sin t
2
cos t
 
 dt 
Now, x = a cos3 t
x
 cos3 t =
a
1
 x 3
 cos t =  
a
y = a sin3 t
y
 sin3 t =
a
1
 y 3
 sin t =  
a

311
Board Answer Paper : March 2018
From (i), we get
dy  sin t
=
dx cos t
1
y3
1 1
 y 3
=  a1 =   
3
[3]
x
x3
1
a3
100
iii. f(0) = ….(given)
3
log100  log  0.01  x 
lim f(x) = lim
x 0 x 0 3x
log 100  0  01  100 x  log 1  100 x 
= lim = lim
x 0 3x x 0 3x
100 log 1  100 x 
= lim
3 x 0 100 x
100  log 1  x  
= 1 ….  lim  1
3  x  0 x 
100
= = f(0)
3
Since, lim f(x) = f(0), f is continuous at x = 0. [3]
x 0

(B) Attempt any TWO of the following:


i. f(x) = 2x3  21x2 + 36x  20
 f (x) = 6x2  42x + 36 and f (x) = 12x  42
 6(x2  7x + 6) = 0
 6(x2  6x  x + 6) = 0
 6[x (x  6)  1(x  6)] = 0
 6(x  1) (x  6) = 0
 (x  1) (x  6) = 0
 x = 1 or x = 6
For x = 1,
f (1) = 12(1)  42 = 12  42 = 30 < 0
 f(x) attains maximum at x = 1.
 Maximum value = f(1)
= 2(1)3  21(1)2 + 36(1)  20
= 2  21 + 36  20
= 19  20 + 36
= 39 + 36 = 3
The function f(x) has maximum value –3 at x = 1.
For x = 6,
f (6) = 12(6)  42 = 72 – 42 = 30 > 0
 f(x) attains minimum at x = 6.
 Minimum value = f(6)
= 2(6)3  21(6)2 + 36(6)  20
= 432  756 + 216  20
= 128
 the function f(x) has minimum value – 128 at x = 6. [4]
dx 1 2a
ii. a 2
x 2
=
2a  a 2
 x2
dx

1  (a  x)  (a  x) 
2a   (a  x)(a  x)
=  dx 

312
Mathematics and Statistics
1   1 1  
=     dx
2a   a  x a  x  

1  1 1 
2a   a  x
= dx   dx
a  x 
 log a  x 
= 1  log a  x  c
2a 1  
1
=
 log a  x  log a  x  c
2a 
dx 1 ax
  a 2  x 2 = 2a log a  x  c [4]

iii. L.H.S. becomes


a 0 a

 f ( x)dx =
a a
 f ( x)dx +  f ( x)dx
0
a a

 f ( x)dx
a
= I +  f ( x)dx
0
….(i)
0
Now, I =  f ( x)dx
a

Put x = t  dx =  dt
When x = a, t = a and when x = 0, t = 0
0 0
 I =  f ( t)(dt) =   f ( t)dt
a a
a
 b a

=  f ( t)dt ….   f ( x)dx    f ( x)dx 
0  a b 
a
 b b

=  f ( x)dx ….   f ( x)dx   f (t)dt 
0  a a 
 Equation (i) becomes
a a a

 f ( x)dx
a
=  f ( x)dx +  f ( x)dx
0 0
a
=  f ( x)  f ( x) dx
0
….(ii)

If f(x) is an even function, then f(x) = f(x).


Thus, equation (ii) becomes
a a a

 f ( x)dx =  f ( x)  f ( x) dx = 2  f ( x)dx


a 0 0

If f(x) is an odd function, then f(x) = f(x).


Thus, equation (ii) becomes
a a

 f ( x)dx =
a
  f ( x)  f ( x) dx = 0
0
[4]

Q.6. (A) Attempt any TWO of the following:


i. f(x) is continuous at x = 3. ….(given)
 f(3) = lim f(x) = lim (2x2 + 3x + )
x 3 x 3
2
 5 = 2(3) + 3(3) +  = 18 + 9 + 
  = 22
f(3) = lim f(x)
x 3

x2  9 ( x  3)( x  3)
 5 = lim +  = lim +  = lim (x + 3) +  = (3 + 3) + 
x 3 x 3 x 3 ( x  3) x 3

 5=6+
313
Board Answer Paper : March 2018
  = 1
  = 1,  = 22 [3]
 5x  1 
ii. y = tan 1  2 
 3  x  6x 
 5x  1 
= tan 1  2 
 1  2  x  6x 
 5x  1 
= tan 1  
 1  (3 x  2)(2 x  1) 
 (3x  2)  (2 x  1) 
= tan 1  
 1  (3x  2)(2 x  1) 
= tan1 (3x + 2) + tan1 (2x  1)
dy 3 2
 = 
dx 1  (3x  2) 1  (2 x  1) 2
2

3 2
= 
1  9 x 2  12 x  4 1  4 x 2  4 x  1
3 1
= 2  [3]
9 x  12 x  5 2 x 2  2 x  1
iii. Let X be the number of heads.
1 1 1
P(outcome is head) = p = ,q=1 =
2 2 2
Given n = 9
 1
 X ~ B  9, 
2 
The p.m.f. of X is given by
x 9 x
1 1
P(X = x) = p(x) = 9Cx     , x = 0, 1, 2, …., 9
2 2
5 4
1 1 98 7 6 1 126
 P(exactly 5 heads) = P(X = 5) = 9C5     =  = = 0.2461 [3]
 2 2 4  3  2 1 512 512
(B) Attempt any TWO of the following:
i. f(x) = x2  4x + 10 , x  [0, 4]
As f(x) is a polynomial function,
a. f(x) is continuous on [0, 4]
b. f(x) is differentiable on (0, 4)
f(0) = 02  4(0) + 10 = 10
f(4) = 42  4(4) + 10 = 16  16 + 10 = 10
 f(0) = f(4)
Thus, all the conditions of Rolle’s theorem are satisfied.
The derivative of f(x) should vanish for at least one point c  [0, 4]
To obtain the value of c,
f(x) = x2  4x + 10
 f (x) = 2x  4
f (x) = 0
 2x  4 = 0
4
 x= =2
2
x = 2 lies in [0, 4]
Thus, Rolle’s theorem is verified. [4]
dx
ii. y(1 + log x)  x log x = 0
dy
dx
 y(1 + log x) = x log x
dy

314
Mathematics and Statistics
 y(1 + log x)dx = x log x dy
1 1  log x
 dy = dx
y x log x
Integrating on both sides, we get
1 1  log x
 y dy =  x log x dx

 log |y| = log |x log x| + log |c|


 log |y| = log |cx log x|
 y = cx log x
When x = e, y = e2, we have
e2 = ce log e
 e2 = ce
 c=e
 Particular solution is y = ex log x [4]
iii. E(X) =  x . P( x )
i i

1  3 3 1


= (0)   + (1)   + 2   + 3  
8 8 8 8
3 6 3
=0+ + +
8 8 8
12 3
= =
8 2
  x
E X2 = i
2
. P( xi )

1 3 3 1


= (0)2 + (1)2   + (2)2   + (3)2  
8 8 8 8
3  3 1
=0+ + 4  + 9 
8  
8 8
3 12 9
= + +
8 8 8
24
= =3
8
 Var (X) = E(X2)  [E(X)]2
2
3
=3  
2
9
=3
4
3
=
4
3 3
 = Var  X  = =
4 2
3
 Standard deviation of X = [4]
2

315
Board Answer Paper : March 2018

BOARD ANSWER PAPER : MARCH 2018


BIOLOGY

Note: Answer to every question must be written on a new page.

SECTION – I
[BOTANY]

Q.1. Select and write the most appropriate answer from the given alternatives for each sub-
question:
i. (C) 1 : 2 : 1 [1]

ii. (D) phosphate group [1]

iii. (A) Agaricus bisporus [1]

iv. (C) fungi [1]

v. (B) Chl – a, 680 [1]

vi. (A) oxysomes [1]

vii. (B) primary consumers [1]

Q.2. (A) Answer each question in ‘One’ sentence only :


i. The precipitation of water soluble inorganic nutrients in the form of salts inside the soil
horizon is called leaching. [1]

ii. Organisms which prepare their food by chemosynthesis, i.e. they use energy released
during chemical reactions for synthesis of food are called as chemoautotrophs.

iii. Krebs cycle occurs in the matrix of mitochondria. [1]

iv. Deforestation is the permanent removal, decrease or deterioration of forests and woodlands.

v. The microbial source of Vit. B12 is Pseudomonas denitrificans. [1]

vi. The primary treatment of sewage is a physical process in which large pieces of floating
debris, oily substances, etc. are removed through filtration and sedimentation. [1]

(B)
Exine
Intine
Germpore

Nucleus

Pollen grain (microspore)


[2]

(C) Attempt any TWO of the following:


i. Following are the steps involved in genetic engineering or r-DNA technology:
Isolation of desired gene, fragmentation, screening, selection of vector, formation of
recombinant DNA, gene transfer to the host, cloning. [2]
316
Biology
ii. Microbial Pesticides:
Pathogen Host Range
Bacteria
Bacillus thuringiensis (Bt) Caterpillars (larvae of moths and butterflies),
larvae of Aedes, black flies, some adult beetles,
wax moths, etc.
Fungi
Beauveria bassiana Aphids, mealy bugs, mites, white flies, etc.
Protozoans
Nosema locustae Grasshoppers, caterpillars, some corn-borers
and crickets.
Viruses
Nucleopolyhedrovirus or NPV (125 types Gypsy moths and caterpillars.
known)
(Any two examples) [2]
iii. Significance of respiration:
a. It converts chemical energy into metabolically usable form of energy, i.e. ATP.
b. Along with photosynthesis, it helps to maintain the carbon cycle in nature, i.e. CO2
released during respiration is used in photosynthesis.
c. Anaerobic respiration or fermentation is applied in the preparation of a number of
commercially important products like alcohol, antibiotics and vitamins. It is also
used in various industries like the paper industry, leather industry, dairies, bakeries,
distilleries, etc.
d. The intermediate products formed during respiration are also used in the synthesis of
organic compounds like fats and proteins.
e. Respiration provides energy for cell division, growth and repair, replacement of worn
out parts, movement, locomotion, etc.
f. Processes like biosynthesis of cellular materials such as carbohydrates, proteins, fats,
etc., utilize energy derived from respiration.
g. Insoluble substances are converted into soluble form using energy of respiration.
(Any four significance) [2]
iv. Energy pyramid:
a. It is a graphical representation of the amount of energy trapped per unit time and area
in different trophic levels of a food chain.
b. The pyramid of energy in any ecosystem is always upright. This is because producers
forming the base of the pyramid have maximum amount of energy.
c. The amount of energy transferred to successive higher trophic levels decreases. This
is because at each level, 80-90% energy is utilized for metabolic activities and only
10% energy is available to the next trophic level.

Trophic level Energy


TC 10 J

SC 100 J

PC 1000 J

PP 10,000 J

1,000,000 J of Sunlight
Pyramid of energy [2]

317
Board Answer Paper : March 2018
Q.3. (A) Attempt any TWO of the following:
i. a. The nucleosomes in chromatin are seen as ‘beads-on-string’.
b. Around the octamer, DNA molecule is wrapped as 1 and 3/4th turn. This DNA is
called core DNA and it consists of about 146 bp (base pairs).
c. Adjacent nucleosomes are linked with small segments of DNA called linker DNA
consisting of about 54 bp. The diameter of linker DNA is 2nm or 20Å.
d. This ‘beads-on-string’ structure gets condensed into nucleosome fiber, which is 10 nm
(100Å) in diameter.
e. H1 histone is present in the linker region, and as DNA makes two complete turns, it is
present where the DNA starts wrapping the octamer and leaves it. Each nucleosome
contains 200 bp of DNA helix.

H2B
H2B H2A H2A
DNA Helix

H4 H4 H1
H3 H3

Nucleosome [3]

ii. One cycle of Polymerase Chain Reaction involves three basic steps:
3 5
ds DNA
5 3
Denaturation 91C

3 5

P1 P1 ~55C
Annealing

5 3

72C

Polymerization
(Extension)

Steps of PCR

a. Heat denaturation:
The desired DNA is heated to a high temperature of about 91C. It results in the
separation of the two strands of DNA, each of which would function as a template
for the synthesis of a new molecule of DNA. DNA with more G-C pair require a
higher temperature.
b. Annealing:
1. It is the process in which the two primers (oligonucleotides) hybridise to each of the
strands of DNA.
2. It requires a temperature of about 55C.
318
Biology
c. Polymerization:
In this step, the Taq polymerase carries out the synthesis of the DNA region between
the two primers by using deoxyribonucleotides (dNTPs) and Mg2+. The optimum
temperature for this polymerization reaction is 72C.
[3]
iii. The different steps involved in tissue culture technique are:
a. Explant culture:
Tissue or plant part that is excised from the original plant and is used for initiation of
a culture is known as explant. e.g. root apex, shoot apex, pollen grains, etc. The
explants are sterilized properly and placed on solid nutrient medium. The cells from
explants absorb nutrients and start multiplying.
b. Callus formation and its culture:
Callus is an unorganized mass of loosely arranged parenchyma cells which develop
from explant tissue due to proliferation of cells.
All the cells of callus are identical because they are produced by mitosis only.
c. Organogenesis:
Growth hormones like auxins and cytokinins in proper proportion are provided to the
callus to induce formation of organs.
If auxins are more, roots are formed (rhizogenesis) and if the cytokinins are in more
quantity, then the shoot system begins to develop (caulogenesis).
d. Formation of cell or suspension culture:
Callus is transferred to liquid nutrient medium and then it is agitated constantly at
100-250 rpm. Due to agitation, cells from callus get separated.
Agitation prevents the aggregation of cells, also serves the purpose of aeration and
mixing of medium.
Suspension culture grows much faster than callus culture. They need to be sub-
cultured every week.
e. Transplanting:
The healthy plantlets are transferred to soil in pots. They are kept in growth
chambers and then in glass houses. Gradual exposure of plantlets to the environment
is called hardening. The hardened plants are then transferred to the field. [3]
(B)
Upper Epidermis

Mesophyll cells with


granal chloroplast

Xylem
Phloem Vascular Bundle
Bundle sheath cells with
agranal chloroplast

Lower Epidermis Stoma


T.S. of leaf showing Kranz anatomy
[3]
Q.4. Development of female gametophyte of angiosperms:
i. In a young ovule, a single hypodermal cell differentiates to form archesporial cells.
ii. These cells are diploid and collectively called as nucellus.
319
Board Answer Paper : March 2018
iii. At maturity, one of the archesporial cells, act as megaspore mother cell and undergoes
meiosis to form four haploid megaspores, arranged in a linear tetrad.
iv. The formation of megaspore is called megasporogenesis.
v. Out of the four haploid megaspore, upper 3 (towards micropylar end) degenerate and one at
the base remains functional. It is the 1st cell of female gametophyte.
vi. The functional megaspore enlarges and undergoes mitotic nuclear division to produce
two nuclei.
vii. These nuclei, migrate to opposite poles of the megaspore.
viii. At each pole, nucleus divides twice to form 4 nuclei at each pole.
ix. The functional megaspore enlarges gradually and becomes 8 nucleated embryo sac.

Diploid Functional megaspore


megaspore Linear tetrad of (Chalazal)
mother cell (2n) haploid Degenerating
megaspores (n) megaspores

a. Megasporogenesis (diagrammatic)

Nucleus (n)
Cytoplasm
Functional megaspore 1st nuclear division 2nd nuclear division 3rd nuclear division
(2 nucleate embryo sac) (4 nucleate embryo sac) (8 nucleate embryo sac)

Antipodals (n)

Female gametophyte
Polars
Egg apparatus
Polar nuclei
Egg(n)
Synergids

Shifting of nuclei Organisation of 7 celled


and 8 nucleate embryo sac
b. Development of female gametophyte or embryo sac (diagrammatic)

x. The two nuclei, one from each pole (polar nuclei) migrate to the centre and fuse to form
diploid secondary nucleus.
xi. The three nuclei at the chalazal end form antipodal cells, while the three at the micropylar
end form egg apparatus of which one in the centre is the egg and the other two are
synergids.
xii. The embryo sac is 7-celled and 8-nucleated called polygonum type.
xiii. The development of female gametophyte in angiosperms is completely endosporic, i.e.
within the megaspore and monosporic as female gametophyte develops from a single
megaspore. (However, in some angiosperms, it may be bisporic or tetrasporic). [7]
OR
Q.4. The law of independent assortment states that “when two parents differing from each
other in two or more pairs of contrasting characters are crossed, then the inheritance of one
pair of character is independent of the other pair of character.”
For example, when we cross a pure tall, red flowered pea plant with a pure dwarf white
flowered pea plant, we get 9 tall red, 3 tall white, 3 dwarf red and 1 dwarf white plants in
the F2 generation. A cross between two homozygous individuals differing in two pairs of
contrasting characters is called dihybrid cross.
320
Biology

Phenotype of parents Tall Red  Dwarf White


Genotype TTRR ttrr

Gametes TR tr

F1 generation TtRr
(Tall Red)
Selfing of F1 generation TtRr  TtRr

Gametes TR Tr tR tr TR Tr tR tr

F2 generation TR Tr tR tr
TR TTRR TTRr TtRR TtRr
Tall red Tall red Tall red Tall red
Tr TTRr TTrr TtRr Ttrr
Tall red Tall white Tall red Tall white
tR TtRR TtRr ttRR ttRr
Tall red Tall red Dwarf red Dwarf red
tr TtRr Ttrr ttRr ttrr
Tall red Tall white Dwarf red Dwarf white

Result: Tall red = 9 ;Tall white = 3; Dwarf red = 3; Dwarf white = 1


Phenotypic ratio  9 : 3 : 3 : 1
Genotypic ratio  1 : 2 : 2 : 4 : 1 : 2 : 1 : 2 : 1
TTRR TTRr TtRR TtRr ttRR ttRr TTrr Ttrr ttrr

From the above results, it is obvious that the inheritance of character of tallness in no way
linked with the red colour of the flower. Similarly, the character of dwarfness is not linked
with the white colour of the flower. This is due to the fact that in the above cross, the two
pairs of characters segregate independently. In other words, there is independent assortment
of characters during inheritance. [7]

SECTION – II
[ZOOLOGY]

Q.5. Select and write the most appropriate answer from the given alternatives for each sub-
question:
i. (A) Reverse osmosis [1]
ii. (C) Heart [1]
iii. (D) Jersey [1]
iv. (A) Thyroxine [1]
v. (C) Bird [1]
vi. (B) Gene flow [1]
vii. (A) antigen [1]

Q.6. (A) Answer in One sentence each:


i. Longer toes and long prehensile tail indicate arboreal adaptations. [1]
ii. IUCD indicates Intra uterine contraceptive device. [1]
iii. Bicuspid or mitral valve is present between left atrium and left ventricle, which prevents the
backflow of blood from left ventricle to left atrium. [1]
iv. Bovine growth hormone is used to increase cattle and dairy yields. [1]
321
Board Answer Paper : March 2018
v. Certain characteristics of Down’s syndrome are as follows:
a. The person has short stature, small skull, small and round head with flat back and
straight and sparse hair.
b. The face is broad, flat and is moon like with standing eyes, rounded flat nose and
malformed ears.
c. Children with Down’s syndrome have typical facial features with a prominent fold of
skin (epicanthal skin fold) over the inner corner of the eye, thus giving an appearance
of downward slanting eyelids.
d. They have small and arched palate and protruding tongue that causes the mouth to
remain partially open.
e. The hands and feet are short and broad with short fingers and toes. Hands are flat
with a characteristic crease running across the palm termed as simian crease.
f. Severe mental retardation is often seen.
(Any two symptoms) [1]
vi. a. Apes have prognathous face, whereas man has orthoghathus face.
b. Apes have semi-erect posture and man has fully erect posture.
c. Forelimbs in apes are longer than hindlimbs, while in man the forelimbs are shorter
than the hindlimbs.
d. In apes, foramen magnum in skull is directed backward and in man it is directed
downward.
e. Pelvic girdle is narrow and elongated in apes and broad and flattened in man.
f. Lumbar curve is absent in apes and it is present in man.
(Any one difference) [1]
(B)
Antigen binding site Antigen binding site
Variable region of the
N heavy chain N
N N

s s Variable region of the


s s ss s light chain
Light chain s SS
SS Constant region of the
C C
light chain

Hinge Disulfide bond


Heavy chain Constant region of the
C C heavy chain

Structure of Antibody [2]

(C) Attempt any TWO of the following:


i. The causative organism of typhoid is Salmonella typhi.

‘O’
antigen

Flagella
with antigen
‘H’

Salmonella typhi
[2]
322
Biology
ii. Economic importance of lac:
a. Lac is used in the preparation of sealing wax, paints, varnish, electrical goods.
b. Lac is used in the preparation of bracelets, buttons, toys and in filling hollow gold
ornaments.
c. Lac is used in artificial leather and pottery.
d. It is also used in gramophone industry.
e. It is used for coating fruits and vegetables.
f. It is used for making bangles, toys, woodwork, polish, inks, silvering mirrors, etc.
(Any four points) [2]
iii. a. All organisms have great potential for reproduction.
b. They produce more young ones than those which can actually survive up to maturity.
c. In other words, any population tends to increase in geometrical proportion.
For e.g An elephant can produce 6 young ones in its life span. If all these young ones
of a single female elephant survive and reproduce, about 19 million elephants will be
produced in 750 years.
d. The only purpose of the organisms is to survive and perpetuate the species.
[2]
iv. Agranulocytes: They are produced in spleen and do not contain granules in the cytoplasm.
Agranulocytes are of two types:
a. Lymphocytes: They form 30% of total
WBCs. Cytoplasm
Nucleus is large, spherical and surrounded by
Nucleus
thin layer of cytoplasm.
Functions: They produce antibodies and are
responsible for immune response of the body. Lymphocyte
b. Monocytes: They are the largest leucocytes
and constitute 3 - 9% of total WBCs.
They have large amount of cytoplasm and
kidney shaped nucleus.
Functions: They are phagocytic in function. Kidney
They engulf foreign particles. e.g. bacteria. shaped nucleus
Monocytes enlarge and differentiate into Monocyte
macrophages, at the site of infection.
They also remove the damaged and dead cells,
hence are referred to as scavengers.
[2]

Q.7. (A) Attempt any TWO of the following:


i. Parasitism: Parasitism is an interspecific interaction in which one of the species (parasite)
is benefited, while the other one (host) is harmed (may even be killed). (1)
Types of parasitism:
a. In Ectoparasitism, parasites live on the surface of host. e.g. Leeches, bed bugs.
b. In Endoparasitism, parasites live inside the host’s body and are called
endoparasites. e.g. Plant aphids, Scab mites. Endoparasites can be of two forms:
intercellular (inhabiting intercellular spaces) or intracellular (living within the
cells in the host’s body).
c. Parasites involving carriers/vectors: These parasites (bacteria, viruses, etc.)
depend on a third organism known as a carrier or a vector to be transmitted to
their host. e.g. Plasmodium, the causative agent of malaria is transmitted by a
Anopheles mosquito to its human host.
d. Epiparasite or hyperparasitism is a relationship in which one parasite feeds on
another parasite which in turn feeds on its host. e.g. A protozoan (the
hyperparasite) living in the digestive tract of a flea, living on a dog.
e. Social Parasitism: Social parasites take advantage of interactions between
members of social organisms such as ants or termites.

323
Board Answer Paper : March 2018
f. In kleptoparasitism, parasites steal food gathered by the host.
g. In Brood parasitism certain species depend on other species to raise their young
ones. i.e. Female of one species lays its eggs in the nest of another species. Eggs
are incubated and young ones are fed by the foster mother. e.g. Koel and cuckoo
females lay their eggs in the crow’s nest.
h. In Adelphoparasitism, the host species is closely related to the parasite, often
being a member of the same family or genus. e.g. unmated females of the Citrus
blackfly parasitoid, Encarsia perplexa, may lay haploid eggs in the fully
developed larvae of the their own species, resulting into production of male
offsprings. The marine worm, Bonellia viridis, has a similar reproductive
strategy.
(Any two types) (2)
ii. Structure of chromosome:
a. During metaphase, a chromosome has two identical
halves called sister chromatids. Telomere
b. Each chromatid is made up of sub-chromatids called
chromonemata (singular – chromonema). Chromomere
c. The chromatids lie side by side and are held together
at one point called the centromere. The centromere is
also called the primary constriction.
Chromonemata
d. Spindle fibres attach at the centromere during cell
division.
e. Besides the primary constriction, additional narrow
areas called secondary constrictions are present.
Primary
f. Some of the secondary constrictions are called constriction
nucleolar organizers because they are necessary for (Centromere)
formation of nucleolus.
g. The short and spherical part of the chromosome
Secondary
beyond the nucleolar organizer is called satellite.
constriction
h. The tip of the chromosome is called telomere.
i. The telomere prevents the ends of the chromosomes Satellite
Structure of Chromosome
from sticking together, but attaches to the nuclear
envelope.
j. The surface of a chromosome bears number of small swellings called chromomeres.
[3]
iii. Genomics:
a. It is a scientific discipline of mapping, sequencing and analyzing the genome.
b. Thus, Genomics is the study of the molecular organization of genomes, their
information contents and the gene products they encode. (1)
Applications of genomics:
a. The number, size, location and organization of all the genes which are required for
formation of an organism can be known.
b. Genome information is very much helpful for solving many questions arising in biology.
c. Helps to identify faulty genes for various genetic diseases and seek a cure for them.
d. Functional genomics includes the study of gene expression, regulation and actual
production of a phenotype.
e. It also provides researchers with the potential of gaining more insight on the molecular
biology of genes, its evolution and mutation.
(Any two applications) (2)
324
Biology
(B)
Afferent arteriole
Efferent arteriole

Glomerulus
Parietal layer

Visceral layer

Bowman’s capsule
Podocyte

Capsular space

Neck
Proximal convoluted tubule

Malpighian body
[3]
Q.8. Histology of human testis:
i. Externally, each testis is covered by three layers. These are:
a. Tunica vaginalis:
It is the outermost incomplete peritoneal covering made up of connective tissue and
epithelium.
b. Tunica albuginea:
It is the middle layer formed by fibrous connective tissue.
c. Tunica vasculosa/vascularis:
It is the innermost layer formed of delicate connective tissue, supporting a network of
blood capillaries.

Tunica albuginea
Interstitial cells / cells of Leydig

Sperm bundle Seminiferous tubule


Germinal epithelium
Sertoli cell

Basement membrane
Connective tissue
Different stages of
spermatogenesis
T.S. of testis

ii. Each testis contains about 200-300 lobules. Each lobule has 1 to 4 convoluted loops called
seminiferous tubules.
iii. These are lined by a single layer of cuboidal germinal epithelium which undergoes
spermatogenesis.
iv. In the germinal epithelium, various stages of spermatogenesis such as spermatogonia,
primary spermatocytes, secondary spermatocytes, spermatids and sperms are seen.

325
Board Answer Paper : March 2018
v. Between germinal epithelium, few large and pyramidal cells called nurse cells or Sertoli cells
are present.
vi. Sperm bundles are seen attached to Sertoli cells with their heads.
vii. These cells provide nourishment to the sperms till maturation.
viii. Between seminiferous tubules, few groups of interstitial cells (Cells of Leydig) are present.
ix. Interstitial cells produce hormone testosterone after puberty.
Structure of human sperm:
i. The human sperm is a haploid, elongated and motile male gamete.
ii. It is microscopic, measuring about 0.055 mm in length.
iii. Sperms are produced by the process of spermatogenesis in testes.
iv. The body of sperm is divided into four parts, viz. head, neck, middle piece and tail.
a. Head: It is flat and oval consisting of large haploid nucleus and an acrosome formed
from Golgi complex.
The acrosome secretes an enzyme hyaluronidase which helps in penetration of egg during
fertilization.
The anterior half of the nucleus is covered by a fibrillar sheath.
a. Neck: It is a narrow constricted region consisting of two centrioles (proximal and
distal).
The proximal centriole plays an important role in first cleavage of the zygote. The
distal centriole forms the axial filament present in the tail of the sperm.
c. Middle piece: Middle piece has many mitochondria spirally coiled (Nebenkern)
around the axial filament.
The mitochondria provides energy for the sperm during its movement, hence it is
called the powerhouse of the sperm. Posterior half of nucleus, neck and middle piece
of sperm are covered by a sheath.
d. Tail: The tail is long, slender and tapering.
It is made up of cytoplasm.
The axial filament arises from the distal centriole and traverses the middle piece and
tail. [7]
OR
In humans, photoreceptors or the organs of sight are the two large eyes.
Location: Human eyes are located in the sockets of skull called orbits.
Structure: Each human eyeball is nearly spherical and formed of three layers.
i. Sclera:
It is formed by the outer layer of dense connective tissue. It is white and opaque and
popularly called white of the eye.
Function: It protects and maintains the shape of the eye ball.
ii. Cornea:
The transparent anterior portion of sclera is termed as cornea.
It is non-vascular and convex anteriorly.
The cornea is transparent because of the collagen fibres in this region.
Conjuctiva is the thin transparent membrane present in front of cornea.
Function: It refracts the incident light rays to focus on the retina.
iii. Choroid is the middle layer, bluish in colour and contains many blood vessels.
The posterior two third region of choroid layer is thin, whereas the anterior part is thick
forming the ciliary body.
iv. The forward segment of the ciliary body is termed as iris which is pigmented and opaque.
v. Lens is the transparent crystalline structure, present anteriorly and is held by ligaments of
ciliary body.
vi. The aperture surrounded by the iris in front of the lens is known as pupil. The muscle fibres
of iris regulate the movement of the pupil.
326
Biology

Vitreous
chamber

Ciliary
body Optic nerve
Aqueous Blind spot
Chamber
Cornea
Pupil Fovea
Iris Choroid
Retina
Lens Sclera

Anatomy of an eye

vii. The innermost layer of the eye is the retina. It is formed of three sub-layers ganglion cells,
bipolar cells and photo-receptor cells. These cells are sensitive to light. Photo-receptor cells
are of two types called as rods and cones. They contain light sensitive proteins termed as
photopigments. Cones are responsible for photopic (day light) vision and colour vision
while rods are responsible for scotopic (dim-light) vision. Rods contain photosensitive
pigment rhodopsin. Cones are of three types, containing characteristic photopigments that
respond to red, green and blue lights.
viii. Blind spot or optic disc: The optic nerves leave the eye at a point slightly away from the
median posterior pole of the eyeball. The rods and cones are absent in this region hence, it
is known as blindspot. Therefore no image is seen here.
ix. Macula lutea, a yellowish pigmented spot is present lateral to the blind spot.
x. Fovea is the central pit located in the macula of the retina. It is the thinned out portion of
retina where only the cones are densely packed and thus, the visual acuity (resolution) is
the greatest at this point.
xi. The space between the cornea and the lens is the aqueous chamber. It contains a thin
watery fluid known as aqueous humor.
xii. The space between the lens and retina is termed as vitreous chamber which is filled with the
transparent gelatinous fluid called as vitreous humor.
xiii. The aqueous humor and vitreous humor maintain the shape of eyeball.
Mechanism of vision:
i. Light rays from the objects pass through the cornea and the lens.
ii. Rods and cones contain photopigments which are conjugated proteins composed of opsin (a
protein) and retinol (vitamin A derivative).
iii. Light induces dissociation of retinol from opsin, resulting in a change in the structure of
opsin.
iv. It ultimately changes permeability of retinal cell and generates action potential.
v. Action potential is carried, via. bipolar cells and ganglion cells, and further by optic nerves to
visual cortex.
vi. Neural impulses are analyzed and image formed on retina is recognized, based on earlier
memory and experience.
[7]

327
Board Answer Paper : July 2018

BOARD ANSWER PAPER : JULY 2018


PHYSICS

Note: Answer to every question must be written on a new page.

SECTION – I

Q.1. Select and write the most appropriate answer from the given alternatives for each
sub-question:
i. (B)
a = 2 x
when x = 0, a = 0 [1]
ii. (D)
g = g – R2 cos2 
As  increases, cos  decreases.
 g will increase when we move from equator towards the pole. [1]
iii. (B) [1]
iv. (B)
L = mvr
m
 unit of angular momentum: kg  m
s
Dimensions: [L2 M1 T–1] [1]
v. (B)
Three translatory and two rotational motion. [1]
vi. (B)
For wave travelling in the negative direction of X-axis,
 2x 
y = a sin  2nt  
  
 x 
 y = a sin 2n  t  
 n 
 x
 y = a sin 2n  t   ….(  v = n) [1]
 v 

vii. (D)
A1 = 5  10–2  4  10–2 = 20  10–4 m2
A2 = 7  10–2  5  10–2 = 35  10–4 m2
 W = T  2A
W 0.06 0.06
 T=   = 20 J/m2 [1]
2A 2  35  10  20  10  2  15  104
4 4

Q.2. Attempt any SIX:


i. An artificial satellite which is orbiting the earth in same direction as direction of earth’s
spin and having the same period as the period of rotation of earth (i.e., 1 day = 24 hours) is
called as geostationary or communication satellite. [1]
Uses of geo-stationary satellite:
a. For the transmission of television and radiowave signals over large areas of earth’s
surface.
b. For broadcasting telecommunication.
c. For military purposes.
d. For weather forecasting and meteorological purposes.
328
Physics
e. For astronomical observations.
f. For study of solar and cosmic radiation.
g. To relay distress signals from ships.
h. To transmit cyclone warnings to coastal villages.
i. For geopositions system (GPS).
(Any two uses) [1]
ii. Radius of gyration of a body about its given axis is defined as the distance between the axis
of rotation and a point at which the whole mass of the body is supposed to be concentrated,
so as to possess the same moment of inertia as that of body about the same axis.
Physical significance of radius of gyration:
a. Radius of gyration is a measure of distribution of mass about the given axis of
rotation.
b. If the particles of the body are distributed close to the axis of rotation, the radius of
gyration is less.
c. If the particles are distributed away from the axis of rotation, the radius of gyration is
more.
d. The knowledge of mass and radius of gyration of the body about a given axis of
rotation gives the value of its moment of inertia about the same axis, even if we do
not know the actual shape of the body. [2]
iii. The phenomenon of rise or fall of liquid level inside a capillary tube when it is dipped in
the liquid is called capillary action or capillarity. [1]
Applications of capillarity:
a. Blotting paper contains small pores. These pores act as capillaries and hence quickly
absorb ink.
b. Wick in oil lamp contains threads, which act as capillaries and oil rises up in the
wick of the lamp.
c. Sap and water rise upto the topmost leaves in the tree by capillary action.
d. Towel used in everyday life is made up of cotton. It absorbs water or moisture from
the body due to capillarity of towel.
e. Piece of sponge retains water, due to capillarity.
(Any two applications) [1]
iv.
Double walled hollow
metal sphere

Incident radiation

Aperture
Lamp black
Conical
projection Evacuated space

Ferry’s perfectly black body


[2]
v. Solution:
60 180
Given: n1 = 60 r.p.m = = 1 rev/s, n2 = 180 r.p.m = = 3 rev/s,
60 60
t = 20 s
To find: Angular acceleration ()
2  1
Formula: =
t
Calculation: From formula,
2n 2  2n1 2 (3  1) 2  3.142  2
= = =
t 20 20
  = 0.6284 rad/s2
Ans: Angular acceleration of the particle is 0.6284 rad/s2. [2]

329
Board Answer Paper : July 2018
vi. Solution:
25 g
Given: gh = 25% of g =  g = , R = 6400 km = 6.4  106 m
100 4
To find: Height (h)
 2h 
Formula: gh = g  1  
 R 
Calculation: From formula,
g  2h 
 g 1  
4  R 
1 2h
1
4 R
2h 1 3
 1 
R 4 4
3
 2h =  R
4
3
 2h =  6.4  106
4
 2h = 3  1.6  106 = 4.8  106 m
 h = 2.4  106 m = 2400 km
Ans: At a height of 2400 km, the acceleration due to gravity reduces to 25% from the
value at the surface. [2]
vii. Solution:
Given: l = 1 m, m = 10 g = 10  10–3 kg = 10–2 kg, A = 2 cm = 2  10–2 m
To find: Potential energy (P.Emax)
1
Formula: P.Emax = m2A2
2
Calculation: From formula,
1 mg 2  g
P.Emax = A ….    
2 l  l 
1  102  9.8  –2 2
 P.Emax =    (2  10 )
2  1 
 P.Emax = 1.96  10–5 J
Ans: The potential energy of the simple pendulum at extreme positions is
1.96  10–5 J. [2]
viii. Solution:
Given: l1 = 16 cm = 0.16 m, l2 = 50.25 cm = 0.5025 m
To find: End correction (e)
l2  3l1
Formula: e=
2
Calculation: From formula ,
0.5025  3  0.16
e=
2
0.0225
 e= = 0.01125 m
2
Ans: The end correction is 0.01125 m or 1.125 cm. [2]
Q.3. Attempt any THREE:
i. a. Consider two sound waves of equal amplitude but of slightly different frequencies n1
and n2 (n1 > n2), passing simultaneously through a given point in space.
b. Suppose that the two waves start in phase, the displacement y1 and y2 of the waves
may be represented by,
y1 = A sin 1t = A sin 2 n1t .…(1)
y2 = A sin 2t = A sin 2 n2t .…(2)
330
Physics
c. By the principle of superposition of waves, the resultant displacement ‘y’ is given by,
y = y1 + y2 = A sin 2 n1 t + A sin 2 n2 t
 y = A (sin 2 n1 t + sin 2 n2 t)
By using,
CD CD
sin C + sin D = 2 sin   cos  
 2   2 
 n1 + n 2   n1  n 2 
y = 2A sin 2   t cos 2  t
 2   2 
n1  n 2   n1  n 2 
 y = 2Acos2   t sin2  t ....(3)
 2   2 
 n1  n 2 
d. Let, R = 2A cos 2  t ....(4)
 2 
n1  n 2
and n =
2
 y = R sin (2nt) ….(5)
This is the equation of simple harmonic motion.
e. Waxing will be possible, if R is maximum.
i.e., R =  2A
From equation (4),
 n1  n 2 
R = 2A cos 2   t =  2A
 2 
 n1  n 2 
 cos2   t = ±1
 2 

2 
n1 - n 2 
  t = 0, , 2, 3…
 2 
  (n1  n2) t = 0, , 2, ……….
Time at which maximum intensity is produced, is
1 2 3
t = 0, , , , …….
n1  n 2 n1  n 2 n1  n 2
Time interval between two successive waxing is,
1 2 1 3 2
T= 0= – =  and so on.
n1  n 2 n1  n 2 n1  n 2 n1  n 2 n1  n 2
1
 T=
n1  n 2
1 1
Frequency of waxing = = = n1  n2
Period T
f. Waning will be possible, if R is minimum, i.e., R = 0.
From equation (4),
 n1  n 2 
2A cos 2  t = 0
 2 
 n1  n 2 
 cos 2  t = 0
 2 
 n1  n 2   3 5
 2  t= , , ,…
 2  2 2 2
Time at which minimum intensity is produced, is
1 3 5
t= , , ……
2  n1  n 2  2  n1  n 2  2  n1  n 2 
The time interval between two successive waning is,
3 1 5 3
T=  = 
2(n1  n 2 ) 2(n1  n 2 ) 2(n1  n 2 ) 2(n1  n 2 )

331
Board Answer Paper : July 2018
1
 T=
(n1  n 2 )
Period of waxing= Period of waning = Period of beats = T
1
 Frequency of beats, N = = n1  n2 [3]
T
ii. Expression for strain energy:
a. Let, L = length of wire
A = area of cross section of wire
r = radius of cross section of wire
l = elongation of the wire by applying load.
b. If the wire is perfectly elastic then, Young’s modulus,
F/A
Y=
l/L
F L
= 
A l
YAl
 F= ….(1)
L
c. Let ‘f’ be the restoring force and ‘x’ be its corresponding extension at certain instant
during the process of extension.
YAx
 f= ….(2)
L
d. Let ‘dW’ be the work done for the further small extension ‘dx’.
 dW = fdx
YAx
 dW = dx ….(3)
L
e. The total amount of work done in stretching the wire from 0 to l can be found out by
integrating equation (3).
l l l
YAx YA
W =  dW = 0 L dx = L  xdx
0 0
l
YA  x  2
 W=  
L  2 0
YAl 2
 W =
2L
YAl l
 W = 
L 2
YAl
But, =F
L
1
W= Fl .…(4)
2
Equation (4) represents the work done by stretching a wire.
 Work done in stretching a wire
1
W=  load  extension
2
f. Work done by stretching force is equal to strain energy gained by the wire.
1
 U= W= Fl
2
1
 U=  load  extension
2
g. If u is the strain energy per unit volume of stretched wire then,
U 1 F  l 1 Fl
u=   
V 2 V 2 AL
where A = area of cross section of wire
332
Physics
1 F l 1
 u=   =  stress  strain
2 A L 2
1
 u=  stress  strain
2
1 stress
u =  stress 
2 Y
1  stress 
2

 u= 
2 Y
Hence, strain energy per unit volume is directly proportional to the square of the stress. [3]
iii. Solution:
Given: T1 = 627 + 273 = 900 K, T2 = 327 + 273 = 600 K,
T0 = 27 + 273 = 300 K
 dQ  dQ  
To find: Ratio of loss of heat     
 dt 1  dt 2 
dQ
Formula: = Ae (T4  T04 )
dt
Calculation: From formula,
 dQ 
 =  Ae ( T1  T0 )
4

4
….(1)
 dt 1
 dQ 
 =  Ae ( T2  T0 )
4

4
….(2)
 dt 2
Dividing equation (1) by (2),
 dQ 
 dt 1 = Ae  T1  T0 
  4 4

 dQ  Ae  T24  T04 


 
 dt 2
 900    300 
4 4
T4  T4
= 14 04 =
T2  T0  600    300 
4 4

16
= = 5.33
3
 dQ   dQ 
     = 5.33 : 1
 dt 1  dt 2
Ans: The rate of loss of heat by the body at temperature 627 C and 327 C is 5.33 :1. [3]
iv. Solution:
Given: Radius of curve, r = 500 m,
5
Speed of train, v = 72 km/hr = 72  = 20 m/s
18
To find: Elevation of rails (h)
v2
Formulae: a. tan  = b. h = lsin
rg
Calculation:

l=1m
h

From formula (a),


 v2  1  202 
 = tan1   = tan 
1
 = tan (0.0816)
rg
   500  9.8 
  = 440’
333
Board Answer Paper : July 2018
From formula (b),
h = l sin 
h = 1  sin (440’)
≈ 0.08 m
 h = 8 cm
Ans: The elevation of the outer rail above the inner rail is 8 cm. [3]
Q.4. A. An ideal simple pendulum is defined as a heavy point mass suspended by a weightless,
inextensible and twistless string from a perfectly rigid support.
To show motion of the bob of simple pendulum is S.H.M:
i. Consider a simple pendulum of mass ‘m’ and length ‘L’.
L=l+r
ii. Let OA be the initial position of pendulum and OB, its instantaneous position when the
string makes an angle  with the vertical.
In displaced position, two forces are acting on the bob:
a. Gravitational force (weight) ‘mg’ in downward direction.
b. Tension T in the string.
O

T T

mg sin 
B
A x 
mg cos 
mg
mg
L : Length of simple pendulum = l + r
l : Length of string,
r : radius of bob
T : Tension in the string
 : Small angular displacement
mg : Weight of bob
mgcos : Radial component along string
mgsin : Tangential component along path

iii. Weight ‘mg’ can be resolved into two rectangular components:


a. Radial component mg cos  along OB and
b. Tangential component mg sin  perpendicular to OB and directed towards mean
position.
iv. mg cos  is balanced by tension T in the string, while mg sin  provides restoring force.
 F =  mg sin 
where, negative sign shows that force and angular displacement are oppositely directed.
Hence, restoring force is proportional to sin  instead of . So, the resulting motion is not
S.H.M.
x
v. If  is very small then, sin    =
L
x
 F =  mg
L
F x
 =g
m L

334
Physics
ma x
 =g
m L
g
 a= x .…(1)
L
 g 
 ax .…   constant 
 L 
Hence, motion of the bob of simple pendulum is simple harmonic.
vi. Expression for time period:
In S.H.M,
a =  2 x ….(2)
Comparing equations (1) and (2),
g
2 =
L
2
But,  =
T
2
 2  g
   =
T
  L
2 g
 =
T L
L
 T = 2 ….(3)
g
Equation (3) represents time period of simple pendulum. Thus, period of simple pendulum
depends on the length of the pendulum and acceleration due to gravity. [4]
B. Solution:
Given: Diameter (D) = 50 cm, Radius (R) = 25 cm = 0.25 m
M = 25 kg, 1 = 2 rad/s, 2 = 12 rad/s, t = 5 s
To find: Torque ()
2
Formulae: i. M.I of sphere, I = MR2 ii.  = I
5
Calculation: From formula (i),
2
 25   0.25 
2
I=
5
 I = 0.625 kg m2
From formula (ii),
 2  1 
=I  
 t 
 12  2 
= 0.625   
 5 
= 1.25
  = 1.25 N m
Ans: The torque applied is 1.25 N m. [3]
OR
A. i. Consider a pipe closed at one end. A vibrating tuning fork is held at the open end.
Longitudinal wave is set up in the air column which gets reflected from closed end.
Superposition of incident wave and reflected wave sets up a longitudinal stationary
wave in the air column. The layer of air in contact with closed end remains
stationary, hence displacement node is formed at closed end. The layer of air at open
end can vibrate freely, hence displacement antinode is formed. In this mode of
vibration, reflection takes place from denser medium.
335
Board Answer Paper : July 2018
a. First mode or fundamental mode:
In this mode of vibration, there is one node at the closed end and one antinode at the
open end.
Let, L = length of air column
v = wave velocity in air
n = fundamental frequency A
 = wavelength
Velocity of wave in a tube is given by,
v = n .…(1)

 L
But L= 4
4
  = 4L
From equation (1),
v = 4nL N
v
 n= .…(2)
4L
Equation (2) represents fundamental frequency or first mode of vibration.
b.
Second mode or first overtone: Third mode or second overtone:

A A

N
N L 5 2
L 31 A
4
4 N
A
A

N N

ii. Consider a pipe open at both ends. A vibrating tuning fork is held at one of its open
end. Longitudinal wave is set up in the air column which gets reflected from the
other open end. Superposition of incident wave and reflected wave sets up a
longitudinal stationary wave in the air column. The layer of air at open ends can
vibrate freely, hence displacement antinodes are formed. In this mode of vibration,
reflection takes place from rarer medium.
a. First mode or fundamental mode:
In this mode of vibration, there is one node at the centre of the pipe and two
antinodes, one at each open end.
Let, L = length of air column
v = wave velocity in air A
n = fundamental frequency
 = wavelength
Velocity of wave in a tube is given by, L N 
v = n ….(1) 2

But L =
2 A
 = 2L
From equation (1),
v = 2nL
v
n= ….(2)
2L
Equation (2) represents fundamental frequency or lowest frequency of vibration.
336
Physics
b. Second mode or first overtone: Third mode or second overtone:

A A
N
N
A
1
L A N 3 2
L
A 2
N
A N
A
[5]
B. Solution:
Given: r = 0.02 m,
F = 0.0113 N
To find: Surface tension (T)
Formula: F = Tl
Calculation: As there are two sides of a circular loop acted upon by the force,
l = 2  2r.
From formula,
F = 4rT
F
 T=
4r
0.0113
 T=
4  3.142  0.02
 T = 0.04495 N/m
Ans: The surface tension of crude oil is 0.04495 N/m. [2]

SECTION – II

Q.5 Select and write the most appropriate answer from the given alternatives for each
sub-question:
i. (A) [1]
ii. (D) [1]
iii. (D) [1]
iv. (B) [1]
v. (B) [1]
vi. (C)
R 2 +  XL  XC 
2
Z =
32 +  8  4 
2
Z =
Z = 32 + 4 2
Z = 9 +16
Z = 25
Z =5 [1]
vii. (D) [1]

337
Board Answer Paper : July 2018
Q.6 Attempt any SIX:
i. Statement:
The Total Normal Electric Induction (T.N.E.I) over any closed surface with any number of
electric charges situated at any position inside it, is equal to algebraic sum of charges
enclosed by that surface.
Mathematically,
n
T.N.E.I. = q
i 1
i =Q
The expression for electric field intensity at a point outside an infinitely long charged
cylinder is given by,
λ
E=
2πkε 0 r
If  is surface charge density, then,
σR
E= [2]
ε 0r

ii. a. The temperature at which the domain structure is destroyed and a ferromagnetic
substance looses its magnetism is called Curie temperature.
b. Above the Curie temperature, a ferromagnetic substance is converted into
paramagnetic substance. The Curie temperature is different for different substances. [2]
iii.
Exosphere (> 500 km)

Appleton layer
Ionosphere

Kennelly heavyside layer

400 km
Thermosphere Edge of 140 km
atmosphere
Mesosphere
80 km
Ozone layer
Stratosphere 50 km

Troposphere 12 km

Earth’s surface
[2]
iv. Solution:
Given: Optical Path difference (x) = 371
Path difference = 0.24 mm = 0.24 × 10–3 m
To find: a. Whether the point is bright or dark
b. Wavelength of light used ()

Calculation: Path difference, x = 371 = 742 ×
2

Since path difference is even multiple of , point is a bright point
2
Also, Path difference, x = 0.24 × 10–3m

742 × = 0.24 × 10–3
2
0.24  2  103
=
742
= 6.469 × 10–7m
= 6469 Å
Ans: The point is bright and wavelength of light is 6469 Å. [2]

338
Physics
v. Kirchhoff’s first law (Current law or junction law):
Statement:
The algebraic sum of electric currents at any junction is always equal to zero.
i.e.,  I = 0 [1]
Kirchhoff’s second law (Voltage law or loop theorem):
Statement:
In a closed loop of electrical network, the algebraic sum of potential differences for all
components plus the algebraic sum of all e.m.f’s is equal to zero.
i.e., IR + E = 0 [1]
vi. Solution:
Given: E1 = 1.5 V, l1 = 2.5 m, E2 = 1.2 V
To find: Balancing length of the wire (l2)
E1 l1
Formula: 
E 2 l2
Calculation: From formula,
l1  E 2 2.5  1.2
l2 = 
E1 1.5
 l2 = 2 m
Ans: The balancing length of the wire for another cell is 2 m. [2]
vii. Solution:
Given: Number of turns (N) = 1000,
Area of Cross section (A) = 2 × 10–4 m2
Current (I) = 1 A, Angle () = 30,
Magnetic field (B) = 0.16 T
To find: Torque acting on solenoid ()
Formula:  = INAB cos
Calculation: From formula,
 = 1 × 1000 × 2 × 10–4 × 0.16 × cos30
3
= 1000 × 2 × 0.16 × 10–4 ×
2
  = 0.0277 N-m
Ans: The torque acting on the solenoid is 0.0277 N-m. [2]
viii. Solution:
Given: h = 6.63 × 10–34 J.s, E1 = –13.6 eV, En = –0.85 eV
To find: Angular momentum of the electron (L)
E1
Formulae: a. En =
n2
nh
b. L=
2
Calculation: Using formula (a)
13.6
0.85 =
n2
13.6
n2 =
0.85
n=4
Using formula (b)
4  6.63  1034
L=
2  3.142
 L = 4.22 × 10–34 kgm2/s
Ans: The angular momentum of the electron in the fourth orbit in a hydrogen atom is
4.22 × 10–34 kgm2/s. [2]
339
Board Answer Paper : July 2018
Q.7 Attempt any THREE:
i. Transistor as a switch:
N-P-N
C
RB RL
B
IC
E
VBB VCC
IE

IBB IC

Figure (a)

Working:
a. A base-biased N-P-N common emitter transistor is as shown in figure (a).
b. On applying Kirchhoff’s law in input circuit, we get, VBB  IBRB = VBE
 VBB = VBE + IBRB
c. Considering VBB as the d.c input voltage ‘Vi’ then the above equation may be
written as,
Vi = VBE + IBRB ….(1)

Applying Kirchhoff’s law in output circuit, we get,


VCE = VCC  ICRL
d. Considering VCC as the dc output voltage ‘Vo’ then the above equation may be
written as,
Vo = VCC  ICRC ….(2)
e. As Vi is increased from 0 to 0.6 V (for silicon transistor), IC = 0 and the transistor is
said to be in the cut-off state.
 IC = 0
From equation (2),
Vo = VCC = constant.
f. As ‘Vi’ is increased from 0.6 V to 1 V, transistor will be in active state therefore IC
increases linearly. From equation (2) ‘Vo’ decreases linearly. The transistor in this
range is called in active state.
g. If ‘Vi’ is increased further, ‘Vo’ becomes non-linear and decreases further and tends
to become zero, though its value nearly reaches zero. The transistor in this range is
said to be in saturation state. The above characteristics are shown in figure (b).

Vo Cut-off
region Active
region

Saturation
region

Vi
Figure (b)
h. When Vi is low(< 0.6 V), Vo is high and when Vi is high (> 1 V), Vo is low. By
defining low voltage state as cut off state and high voltage level as saturation state,
transistor can be used as a switch.
i. A low voltage input keeps the transistor in cut-off region (non-working) and the
transistor is said to be switched off.
j. A high voltage input keeps the transistor in saturation state (working) and the
transistor is said to be switched on. [3]

340
Physics
ii. Einstein’s photoelectric equation is given by,
1 2
a. mv max  h(  0 )
2
b. eV0  h(   0 )
Characteristics of Einstein’s photoelectric equation:
a. The photoelectric work function W0 is constant for a given emitter. Hence if the
frequency ‘’ of the incident radiation is decreased, the maximum kinetic energy of
the emitted photoelectrons decreases, till it becomes zero for a certain frequency 0.
Therefore from Einstein’s equation,
0 = h0  W0
 W0 = h0 ….(1)
b. This shows that the threshold frequency is related to the work function of the metal
and hence it has different values for different metals.
Using equation (1), Einstein’s equation can be written as
1
mv2max = h  h0
2
1
 mv2max = h(  0)
2
This equation shows that:
1. If  < 0, then K.E is negative which is not possible. In this case, photoelectric
emission is not possible.
2. If  > 0, then photoelectrons move with some velocity.
 K.E > 0, which is possible. Hence photoelectrons are emitted.
3. If  = 0, the photoelectrons are just emitted. In this case, K.E = 0. [3]
iii. Solution:
Given: 1.5, v = 4 ×1014 Hz, c = 3 × 108 m/s
ag =
To find: Change in wavelength of light (a  g)
c
Formulae: a. c =  b. =

Calculation: Using formula (a),
3 × 108 = 4 × 1014 × a
3  108
a =
4  1014
 a = 7.5 × 10–7 m
Using formula (b),
a
ag =
g
7.5  107
1.5 =
g
7.5  107 –7
g =  5 × 10 m
1.5
 a – g = 7.5 × 10–7 – 5 × 10–7 = 2.5 × 10–7m
Ans: The change in wavelength of light during its passage from air to glass is
2.5 × 10–7 m. [3]
iv. Solution:
Given: d = 0.5 mm = 0.5103 m = 5104 m, D = 100 cm = 1 m,
x9  x2 = 8.835 mm= 8.835  103 m (Assuming both bright and dark fringes are
on same side of centre of fringe pattern.)
To find: Wavelength of light ()
Xd
Formula: =
D

341
Board Answer Paper : July 2018
Calculation: Since distance of nth bright band from centre is,
nλD
xn =
d
9D
 x9 =
d
Distance of nth dark band from centre is,
D
xn = (2n  1)
2d
λD 3λD
 x 2 = (2  2  1) =
2d 2d
9λD 3λD 15λD
Now, x9  x 2 =  =
d 2d 2d
2d  x 9  x2 
 =
15D
2× 5×104 ×8.835×103
=
15×1

= 5.89  107 m
  = 5890 Å
Ans: The wavelength of light is 5890 Å. [3]
Q.8 A. Definition: A series combination of inductor of inductance L, capacitor of capacitance C
and resistor of resistance R connected with an alternating e.m.f forms a series LCR circuit
Derivation:

L C R


e = e0 sin t
Figure (a)

i. Let ‘e’ be the applied rms value of e.m.f and ‘I’ be the rms current of the circuit.

ii. In series LCR circuit, eR is in phase with current, eL leads the current by radian
2

while eC lags the current by radians.
2
Y

eL

eL  eC e


(0,0) O X
eR
eC
Y Figure (b)

Hence, the resultant of eR, eL and eC gives the r.m.s. value of applied e.m.f.
From figure (b),
e2 = e 2R + (eL  eC)2

342
Physics

 e2 = I2  R 2   X L  X C  
2

 R 2   XL  XC 
2
e =I ….( eR = IR, eL = IXL and eC = IXC)
e
 I=
R   XL  XC 
2 2

iii. The impedance of the circuit is given by,


e
R 2   X L  XC 
2
Z= = ….(1)
I
But XL = L
1
and XC =
C
 Equation (1) can be written as,
2
 1 
Z= R 2   L  
 C 
….(2)
This is the expression for impedance of LCR series circuit.
Condition for series resonance:
If the frequency of the applied e.m.f source is adjusted so that at particular frequency
XL = XC then Z = R. Thus the circuit behaves like a purely resistive circuit. The impedance
of the circuit is minimum and current is maximum. The current and e.m.f of the source are
in phase. Hence, there is resonance in the circuit.
Expression for resonant frequency:
The r.m.s. current of the series combination is given by,
e e
I= =
Z R   XL  XC 
2 2

The inductive reactance, XL and the capacitive reactance, XC are given by,
1
XL = L and XC =
C
In case of series resonance circuit, inductive reactance is equal to capacitive reactance and
hence maximum current flows through the circuit.
 XL = XC
1
 L =
C
1
 2 =
LC
1
 =
LC
1
 2fr =
LC
1
 fr =
2 LC
This is required expression for resonance frequency. [5]
B. Solution:
Given: C = 10 F = 10 × 10–6 F, V = 100 V
To find: Electrostatics energy stored (U)
1
Formula: U= CV2
2
Calculation: Using Formula,
1
U= × 10 × 10–6 × 1002
2
 U = 0.05 J
Ans: The electrostatic energy stored in capacitor is 0.05 J. [2]
343
Board Answer Paper : July 2018
OR
Q.8. A. Expression for energy of electron:
i. Kinetic energy:
Let, m = mass of electron
rn = radius of nth orbit of Bohr’s hydrogen atom
vn = velocity of electron
e= charge of electron
+e= charge on the nucleus

vn

+e e
rn

mv 2n 1 e2
According to Bohr’s first postulate, =  2
rn 40 rn
where, 0 is permittivity of free space.
1 e2
 mv 2n =  ….(1)
40 rn
The revolving electron in the circular orbit has linear speed and hence it possesses kinetic
energy.
1
It is given by, K.E = mv 2n
2
1  1 e2 
 K.E =    ….[From equation (1)]
2  40 rn 
e2
 K.E = .…(2)
8 0 rn
ii. Potential energy:
Potential energy of electron is given by, P.E = V(e)
where, V = electric potential at any point due to charge on nucleus
 e = charge on electron.
1 e
 P.E =   (e)
40 rn
1 e 2
 P.E = 
40 rn
e2
 P.E =  ….(3)
4 0 rn
iii. Total energy:
The total energy of the electron in any orbit is its sum of P.E and K.E.
 T.E = K.E + P.E
 e2   e2 
=  +  ….[From equations (2) and (3)]
 80rn   4 0 rn 
 T.E = 1 e2
 
8 0 rn
e2
 T.E =  ….(4)
80rn
1
 T.E. 
rn
Thus, the total energy of the electron in Bohr orbit is inversely proportional to the radius of
the orbit.
344
Physics
 0 h 2  2
iv. But, rn =  2 
n
 me 
Substitute for rn in equation (4),
1 e2
 T.E =  
80  0 h 2  2
 2 
n
 me 
1 e2me2
= 
80 0h 2n 2
 me4  1
 T.E =   2 2 
 2 ….(5)
 80 h  n
This is required expression for energy of electron in nth orbit of Bohr’s hydrogen atom.
v. The negative sign in equation (5) shows that the electron is bound to the nucleus by an
attractive force and hence energy must be supplied to the electron in order to make it free
from the influence of the nucleus.
1
To show: T.E 
n2
As m, e, 0, and h in equation (5) are constant,
me 4
 = constant
8 0 2 h 2
Using equation (5),
1
T.E = constant 
n2
1
 T.E 
n2
Hence, the total energy of electron in a Bohr’s orbit is inversely proportional to the square
of the principal quantum number.
Binding energy:
B.E + T.En = 0
 B.E = T.En
 me 4
iv. But T.En =
8 02 h 2 n 2
 me 4 
 B.E =    2 2 2 
 8 0 h n 
me 4
 B.E = [5]
8 02 h 2 n 2

B. Solution:
Given: n = 100, l = 1cm = 1 × 10–2 m, b = 0.5 cm = 0.5 × 10–2 m
B = 0.002 T, C = 2 × 10–8 Nm/degree
To find: Current sensitivity of MCG (Si)
nAB
Formula: Si =
C
Calculation: Using formula,
100  1  102  0.5  102   0.002
Si =
2  108
 Si = 500 div/A
Ans: The current sensitivity of moving coil galvanometer is 500 div/A. [2]

345
Board Answer Paper : July 2018

BOARD ANSWER PAPER : JULY 2018


CHEMISTRY

Note: Answer to every question must be written on a new page.

SECTION – I

Q.1. Attempt any SIX:


i. a. Role CaF2 in metallurgy of aluminium:
1. CaF2 makes alumina a good conductor of electricity.
2. It lowers the fusion temperature to around 1140 K.
b. The copper matte containing Cu2S and FeS is put in silica lined converter in order to
remove impurities as fusible slag. Some silica is also added and hot air blast is blown
to convert remaining FeS to FeO, which is removed as slag with silica.
2FeS + 3O2  2FeO + 2SO2
Ferrous (From blast Ferrous Sulphur
sulphide of hot air) oxide dioxide

Ferrous oxide combines with silica (flux) to form fusible slag.


FeO + SiO2  FeSiO3
Ferrous Silica Ferrous silicate
oxide (Flux) (Fusible slag)
[2]
ii.
Molecular solid Type
a. HCl Polar molecular solid
b. CO2 Non-polar molecular solid
c. Solid ice Hydrogen bonded molecular solid
d. SO2 Polar molecular solid
[2]
iii. a. Cryoscopic constant: Molal depression constant or cryoscopic constant is the
depression in the freezing point of a solution containing one mole of the non-volatile
solute in one kilogram of solvent.
b. Resistivity or specific resistance: Resistivity is defined as the resistance of the
conductor that is 1m long and 1m2 in cross-sectional area.
[2]
iv. Decreasing order of thermal stability of oxoacids of chlorine is
HClO4 > HClO3 > HClO2 > HClO
Reason: With the decrease in oxidation state of chlorine, the oxidizing power of its
oxoacids increases and the thermal stability of oxoacids of chlorine decreases.
[2]
v. van’t Hoff equation for osmotic pressure of a dilute solution:
a. According to van’t Hoff–Boyle’s law, osmotic pressure of a dilute solution is
inversely proportional to volume containing 1 mole of solute at constant temperature
and according to van’t HoffCharles’ law, osmotic pressure of a dilute solution is
directly proportional to the absolute temperature, at constant concentration.
b. If  is the osmotic pressure, V is the volume of the solution and T is the absolute
temperature, then
1
 ….(1) ….[van’t Hoff–Boyle’s law at constant temperature]
V
 V = constant

346
Chemistry
T ….(2) ….[van’t Hoff–Charles’ law at constant concentration]

 = constant
T
c. Combining (1) and (2) we get,
T

V
T
  = Constant 
V
 V = RT, where R is a constant.
d. This equation is parallel to the ideal gas equation PV = RT (n = 1).
Since, the calculated value of R is almost same as R, the equation can be written as
V = RT (for 1 mole of solute)
e. This equation was derived for 1 mole of solute dissolved in V dm3. If n moles of
solute are dissolved in V dm3 of solution, the equation becomes
V = nRT
nRT
 =
V
n
f. C=
V
  = CRT
where,  = osmotic pressure, C = concentration of solution in moles/litre
R = gas constant = 0.082 L atm mol1 K1 or 8.314 J mol1 K1, T = absolute
temperature
n = number of moles of solute, V = volume of the solution
Note:If  is expressed in Nm2 and volume in m3, then R = 8.314 J mol1 K1 and
when  is expressed in atmosphere and volume in dm3, then R = 0.082 L atm
mol1 K1.
[2]
vi. The given data indicates that Mg undergoes oxidation and Ag+ undergoes reduction.
Hence, cell reaction is
Mg(s)  Mg 2aq  + 2e (Oxidation half reaction at anode)
2Ag aq  + 2e  2Ag(s) (Reduction half reaction at cathode)

Mg(s) + 2Ag aq   Mg 2aq  + 2Ag(s) (overall cell reaction)


The cell representation is
Mg(s) Mg 2aq  (x M)  Ag aq  (y M)Ag(s)
[2]
vii. a. Order of chemical reaction:
The order of a chemical reaction with respect to each reactant is defined as the
exponent to which the concentration term of that reactant, in the rate law, is raised.
or
The overall order of reaction is defined as the sum of the exponents to which the
concentration terms in rate law are raised.
b. 1. Overall order of reaction for
CHCl3(g) + Cl2(g)  CCl4(g) + HCl(g)
Rate = k [CHCl3][Cl2]
The reaction is first order with respect to CHCl3 and first order with respect to
Cl2
Hence, the overall order of reaction is 1 + 1 = 2.

347
Board Answer Paper : July 2018
2. Overall order of reaction for
2NO(g) + O2(g)  2NO2(g)
Rate = k [NO]2 [O2]
The reaction is second order with respect to NO and first order with respect to
O2. Hence, the overall order of reaction is 2 + 1 = 3.
[2]
viii. a. Hess’s law of constant heat summation states that, “The change in enthalpy for a
reaction is the same whether the reaction takes place in one or a series of steps”.
b. The Hess’s law is a direct consequence of fact that the enthalpy is a state function
and so enthalpy change depends only on the initial and final states of the system and
not on the path by which the reaction takes place.
B
eg. The conversion of A to C can take place directly
in a single step.
A  C, H = H1
H2 H3
c. The reaction can also take place in two steps for which H
values are known.
Step (1) A  B, H = H2
Step (2) B  C, H = H3 A C
Overall A  C, H = H2 + H3 H1
d. According to Hess’s law, H1 = H2 + H3. The sequence of Hess’s law
steps is represented in the figure.
[2]
Q.2. Attempt any THREE:
i. a. Bleaching action of SO2: SO2 is used as a bleaching agent in presence of moisture.
It bleaches due to reduction and bleaching action is temporary.
SO2 + 2H2O  H2SO4 + 2(H)
Sulphur dioxide water Sulphuric
acid

b. Dehydration of formic acid by concentrated H2SO4:


Concentrated H2SO4 acts as a powerful dehydrating agent due to strong affinity for
water.
HCOOH 
Conc.H 2SO 4
 CO + H2O
Formic acid Carbon Water
monoxide
c. Burning of benzene in presence of excess of dioxygen:
Combustion of benzene in excess of dioxygen (air) results in the formation of carbon
dioxide and water.
15
C6H6 + O2  6CO2 + 3H2O
2
Benzene Dioxygen Carbon Water
dioxide [3]
ii. Given: Density (d) = 10.8 g cm–3
Edge length (a) = 4.05  10–8 cm
Molar mass = 107.8 g mol–1
Avogadro’s number (NA) = 6.022  1023
To find: Number of atoms in the unit cell
Atomic mass
Formula: a. Mass of one atom =
Avogadro number
b. Volume of unit cell= a3
Mass of unit cell
c. Density =
Volume of unit cell

348
Chemistry
Atomic mass of Ag
Calculation: Mass of one Ag atom =
Avogadro number
107.8
= = 1.79  10–22 g
6.022 1023
Volume of unit cell = a3
= (4.05  10–8)3
= 6.64  10–23 cm3
Mass of unit cell
Density (d) =
Volume of unit cell
Number of atomsin unit cell×Mass of one atom
=
Volume of unit cell
Number of atomsin unit cell 1.79 1022
10.8 =
6.64 1023
10.8  6.64 1023
Number of atoms in unit cell =
1.79 1022
= 40.06  10–1 = 4.0 ≈ 4
Ans: The number of atoms in the unit cell of silver is 4.
[3]
–3
iii. Given: Mass of Ag deposited = 1.346  10 kg
Time (t) = 3.5 min = 3.5  60 s
Molar mass of Ag = 108  10–3 kg mol–1
To find: Quantity of electricity required in coulomb:
Moles of product formed in half reaction
Formulae: 1. Mole ratio =
Moles of electrons required in half reaction
2. Mass of the substance produced
I  A  t s 
=  Mole ratio  Molar mass of substance
96500(C/mole )
3. Quantity of electricity in coulomb (Q) = I (in amp)  t (in sec)
Calculation: The half reaction for the formation of Ag is,
Ag aq  + e– 
 Ag(s)
From formula (1),
Moles of Ag 1 mol Ag 
Mole ratio =  = 1 mol Ag/mol e
Moles of electrons 1 mol e 
From formula (2),
I  3.5 60
1.346  10–3 =  1  108 10–3
96500
1.346 103  96500 129889 103
∴ I= = = 5.727 A
3.5 60 108 103 22680 103
From formula (3),
Q = It = 5.727  3.5  60 = 1202.67 C
Ans: The quantity of electricity required is 1202.67 C.
[3]
iv. Given: Number of moles of helium gas (n) = 5
Initial pressure (P1) = 40  10–5 Nm–2
Final pressure (P2) = 4  10–5 Nm–2
Temperature (T) = 300 K
R = 8.314 J K–1 mol–1

349
Board Answer Paper : July 2018
To find: a. Work done (W)
b. Change in internal energy (U)
c. Heat absorbed (q)
P1
Formulae: a. Wmax = –2.303 nRT log10
P2
b. U = q + W

Calculation: a. From formula (a),


P1
Wmax = –2.303 nRT log10
P2
40 105
= –2.303  5  8.314  300 log10
4 105
= –2.303  5  8.314  300 log1010
= –28720.71 J = –28.72 kJ
Work done = –28.72 kJ
b. According to first law of thermodynamics, U = q + W [Formula (b)]
In isothermal process, T = 0, hence U = 0
 Change in internal energy (U) = 0
c. From formula (b),
0=q+W
 q = –W
= – (–28.72 kJ)
= 28.72 kJ
Ans: a. work done (W) = –28.72 kJ
b. Change in internal energy (U) = 0
c. Heat absorbed (q) = 28.72 kJ
[3]
Q.3. Answer the following:
i. a. Orthophosphoric acid:
Molecular formula – H3PO4
Structural formula –
O

P
OH OH
OH
b. Sulphurous acid:
Molecular formula  H2SO3
Structural formula –
O
HO – S – OH
ii. a. Ellingham diagram:
The graphical representation showing the variation of Gibbs energy with increase of
temperature for the formation of oxide (oxidation) is known as Ellingham diagram.
or
The Ellingham diagram is the plot of free energy change G against temperature for
the reaction of metal and other elements with one mole of gaseous oxygen at
1 atmosphere.
b. Significance of Ellingham diagram:
1. The positive slope of metal oxides shows that their stabilities decrease with
increase in temperature. The decrease in their stabilities is due to an increase in
G value.
2. The sudden change in the graph shows a phase change, that is, change from
solid to liquid or from liquid to vapour.

350
Chemistry
3. The negative slope of CO shows that it becomes more stable with increase in
temperature (this is the opposite of that taking place in metal oxides).
4. The relative tendency of the metals to undergo oxidation is in the order, as
shown below:
Mg > Al > Cr > Fe > Hg > Ag.
This is due to the increase in the negative free energy change of the formation
of oxide in the order given below:
MgO > Al2O3 > Cr2O3 > FeO > Ag2O.
(Any two points)
iii. Given: Reaction type = first order
The percentage of substance decomposed = 80 %
Time taken to decompose (t) = 40 min
To find: Half life period ( t1 2 )
2.303 a
Formulae: 1. k= log10
t ax
0.693
2. k =
t1 2
Calculation: 80 % decomposition means, x = 80 % of a = 0.80a
As the reaction is 1st order, from formula (1),
2.303 a 2.303 a
k= log10 = log10
t ax 40 a  0.8a
2.303 1
= log10
40 0.2
2.303
=  0.6990
40
= 4.02  10–2 min–1
For first order reaction, from formula (2),
0.693 0.693
t1 2 =  = 17.24 min
k 4.02 102
 
Ans: The half life period t1 2 of the reaction is 17.24 min.
[7]
OR
Answer the following:
i. Features of reversible process:
a. The driving and opposing forces differ from each other, only infinitesimally.
b. During the course of the process, it can be reversed at any point, by making
infinitesimal change in conditions.
c. The process takes place in an infinite number of steps.
d. The system attains mechanical equilibrium at the end of every step of the process.
e. A reversible process results in obtaining maximum work.
f. The process takes place so slowly that, the system is always in temperature-pressure
equilibrium with its surroundings.
ii. Difference between fluorine and other halogens:
Fluorine Other halogens
Reactivity Fluorine is much more reactive due to its Other halogens are less
small atomic size, high nuclear charge and reactive than fluorine.
low F-F bond dissociation energy.
Hydrogen Hydrogen bonding is present in hydrides of Hydrogen bonding is not
bonding fluorine. present in hydrides of other
halogens.

351
Board Answer Paper : July 2018
Oxidation Due to highest electronegativity and absence Other halogens show –1, +1,
state of the vacant d-orbitals in valence shell, +3, +5 and +7 oxidation states.
fluorine shows only –1 oxidation state in all
its compounds.
Polyhalide Due to absence of vacant d-orbitals in the Other halogens form
ions valence shell, fluorine does not form any polyhalide ions such as I3 ,
polyhalide ion.
Br3 , Cl3 .

iii. a. Freezing point: The freezing point of a liquid may be defined as the temperature at
which the vapour pressure of solid is equal to the vapour pressure of liquid. A liquid
freezes at a temperature at which the liquid and its solid coexist in equilibrium.
b. Given: Freezing point of pure solvent (T) = 278.4 K
Mass of solute (W2) = 2 g = 2  10–3 kg
Molar mass of solute (M2) = 100 g mol–1 = 100  10–3 kg mol–1
Mass of solvent (W1) = 100 g = 100  10–3 kg
Molal depression constant (Kf) = 5.12 K kg mol–1
To find: Freezing point of solution (T)
Formulae: 1. Tf = T – T
K f W2
2. Tf =
M 2 W1
Calculation: From formula (2),
5.12  2 103
Tf =
100 103 100 103
5.12  2
=
10
= 1.024 K
From formula (1),
Tf = T – T
 T = T – Tf
T = 278.4 –1.024 = 277.376 K
Ans: Freezing point of the given benzene solution is 277.376 K.
[7]

Q.4. Select and write the most appropriate answer from the given alternatives for each
sub-question:
i. (D) Cl2(g) [1]
Cl2(g) shows the highest entropy among the given substances due to its gaseous state.

ii. (C) Leaching [1]

iii. (A) Zero [1]


The decomposition of N2O on platinum surface follows zero order kinetics. Most of
the N2O molecules (which remain in gaseous phase) are unreactive while those
which occupy all the active sites on metal surface take part in reaction. Thus, the
reaction rate is independent of total concentration of N2O.

iv. (B) CCl2F2 [1]

v. (D) 0.2 M [1]


4 1
Molarity = = = 0.2 M
40  0.5 5

352
Chemistry
vi. (C) 4 [1]
Four types of orthorhombic unit cell are:
1. Primitive or simple orthorhombic
2. Body centred orthorhombic
3. End centred orthorhombic
4. Face centred orthorhombic
vii. (A) m–1 [1]
SECTION – II
Q.5. Attempt any six:
i. a. The oxidation state of Cu in [Cu(NH3)4]2+ is +2. The electronic configuration of Cu2+
is 3d94s0.
b. dsp2 hybridisation of one d, one s and two p orbitals result in square planar geometry.
c. Coordinate bonds are formed by overlap of metal dsp2 hybrid orbitals and ligand orbitals.

Cu+2     
9
3d 4s 4p

Transfer of one electron     


from 3d to 4p and
dsp2 hybridisation 4s 4p
3d

[Cu(NH3)4]2+         

(8 electrons of Cu) dsp2 hybrid orbitals


(8 electrons of NH3
ligands)

d. Due to presence of one unpaired electron, [Cu(NH3)4]2+ is paramagnetic. The


magnetic moment,  is 1.73 B.M.
[2]
ii. a. d-Block elements: The elements in which the last electron enters the d-orbital of the
penultimate shell i.e., (n1)d-orbital, where, ‘n’ is the outermost shell, are called
d-block elements.
Their general valence or outer electronic configuration is (n  1)d1  10ns1  2.
b. f-Block elements: The elements in which the last electron enters into (n2) f-orbital
of the atoms are called f-block elements.
In these elements, the last electron enters in to the prepenultimate (n2) shell also
called antepenultimate shell.
The general valence electronic configuration is: (n–2)f 114(n–1)d01 ns2.
[2]
iii. Sandmeyer reaction: When a primary aromatic amine, dissolved or suspended in cold
aqueous mineral acid, is treated with sodium nitrite, a diazonium salt is formed. Mixing the
solution of freshly prepared diazonium salt with cuprous chloride or cuprous bromide
results in the replacement of the diazonium group by –Cl or –Br. This reaction is known as
Sandmeyer’s reaction.
+ 
NH2 N2X
NaNO 2  HX

273K

Aniline Benzenediazonium halide
+ 
N2X X

Cu 2 X 2
+ N2
Aryl halide
Benzenediazonium halide
[X = Cl, Br]
353
Board Answer Paper : July 2018
+ 
eg. NH2 N2Cl Cl
NaNO 2  HCl

273K
 
Cu 2 Cl2
+ N2
Benzenediazonium Chlorobenzene
Aniline
chloride
[2]
iv. a. Action of cold HI on isopropyl methyl ether:
When isopropyl methyl ether (mixed ether) is reacted with cold and conc. HI, methyl
iodide (lower alkyl halide) and isopropyl alcohol (higher alcohol) is formed.
(CH3)2CH  O  CH3 + HI  Cold / 273 K
(CH3)2CH  OH + CH3  I
Isopropyl methyl ether (conc.) Isopropyl alcohol Methyl iodide

b. Action of hot HI on isopropyl methyl ether:


When isopropyl methyl ether (mixed ether) is reacted with hot and conc. HI at about
373 K, a mixture of isopropyl iodide and methyl iodide is obtained due to cleavage of
both C – O bond.
(CH3)2 – CH  O  CH3 + 2HI  Hot / 373 K
 (CH3)2 – CH  I + CH3  I + H2O
Isopropyl methyl ether (conc.) Isopropyl iodide Methyl iodide
[2]
v. a. Cyclopropane carboxylic acid to cyclopropyl methanol:
Strong reducing agent like lithium aluminium hydride or diborane reduces
cyclopropane carboxylic acid to cyclopropyl methanol.

COOH 
i. LiAlH 4 ,ether
ii. H O
 CH2OH
3
Cyclopropane
carboxylic acid Cyclopropyl methanol

b. Acetyl chloride to banzyl methyl ketone using dialkyl cadmium:


The reaction of acetyl chloride with dibenzyl cadmium gives the benzyl methyl
ketone.
O O

2CH3  C  Cl + (C6H5CH2)2Cd  2CH3 C  CH2C6H5 + CdCl2


Acetyl chloride Dibenzyl cadmium Benzyl methyl ketone [2]
vi. a. Nitroethane is reduced to ethylamine by the action of lithium aluminium hydride in
the presence of ether.
CH3CH2NO2  LiAlH 4
Ether
 CH3CH2NH2
Nitroethane Ethylamine
b. 2-Methyl-1-nitropropane is reduced to 2-methylpropan-1-amine by the action of
lithium aluminium hydride in the presence of ether.
CH3 CH3

CH3 – CH – CH2 – NO2 


LiAlH 4
Ether
 CH3 – CH – CH2 – NH2
2-Methyl-1-nitropropane 2-Methylpropan-1-amine [2]
vii. Preparation of glucose from starch:
a. Commercially, glucose is obtained on a large scale by hydrolysis of starch, by
boiling at 393 K with dilute sulphuric acid under pressure.
(C6H10O5)n + nH2O 
dil.H 2SO4
393K, 2  3atm
 nC6H12O6
Starch Glucose
b. Excess sulphuric acid is neutralized by adding chalk powder.
c. Activated charcoal is used for the removal of coloured impurities.
d. The solution is cooled, resulting in the separation of glucose crystals, which are
filtered and removed.
[2]
354
Chemistry
viii. a. Analgesics: The drugs which relieve pain by acting on the central nervous system
without loss of consciousness or disturbance of nervous system are called analgesics.
eg. Aspirin
b. Antimicrobials:
The drugs used to kill or stop the growth of micro-organisms like fungi, bacteria and
virus are called antimicrobial drugs.
eg. Salvarsan
[2]
Q.6. Attempt any THREE:
i. a. When 1-chloro-2,4-dinitrobenzene is heated with aqueous alkali (aq. NaOH),
2,4-dinitro sodium phenoxide is formed which on hydrolysis with dilute HCl gives
2,4-dinitrophenol.
Cl ONa OH
NO2 NO2 NO2
NaOH aq  H  , 368K

 

Hydrolysis

NO2 NO2 NO2


1-Chloro-2,4 2,4-Dinitro sodium 2,4-Dinitrophenol
-dinitrobenzene phenoxide

b. Two molecules of benzaldehyde react in the presence of 50% KOH, the products
formed are benzyl alcohol and potassium benzoate. The reaction is called Cannizzaro
reaction.
H H O
 
C=O C=O CH2OH C  OK
+ + KOH  +
(50%)
Benzaldehyde Benzyl alcohol Potassium benzoate

c. Acetone reacts with phenyl hydrazine to give addition product which on dehydration
gives acetone phenyl hydrazone. This reaction occurs in the presence of an acid
catalyst.
CH3 CH3
H+
C = O + H2N – NH – C6H5   C = N – NH – C6H5 + H2O
CH3 CH3
Acetone Phenyl hydrazine Acetone phenyl hydrazone [3]
ii. a. Preparation of phenol from cumene:
1. When an alkaline solution of cumene (isopropylbenzene or 2-phenylpropane)
in sodium carbonate is oxidised by passing air or oxygen in the presence of
cobalt naphthenate as a catalyst at 423 K, cumene hydroperoxide is obtained.

CH3 CH3

H3C  C  H H3C  C  O  O H

O2 
Cobalt naphthenate
+ 423K /(Alkaline medium)
(Air)
Cumene or Cumene hydroperoxide
(Isopropylbenzene
or 2-Phenylpropane)

355
Board Answer Paper : July 2018
2. Auto oxidation:
Cumene hydroperoxide on heating with dilute H2SO4 decomposes forming
phenol and acetone.
CH3
H3C  C  O  O  H OH
O


dil.H 2SO 4
 + CH3  C  CH3
Acetone
Cumene Phenol
hydroperoxide
In this method, acetone is obtained as an important byproduct which is
separated by distillation.
b. Heating of phenol with zinc dust:
When phenol is heated with zinc dust, benzene is formed.
OH


+ Zn   + ZnO
Phenol Zinc Benzene Zinc
oxide [3]
iii. a. Racemic mixture or Racemates: A mixture of equimolar amounts of dextro and
laevo rotatory forms of same optically active substance is called racemic mixture or
racemates. It is optically inactive due to external compensation. It is represented as
() or ‘dl’.
eg. dl-lactic acid or ()-lactic acid.
b. 1. Action of bromine water on glucose:
The oxidation of glucose with bromine water (which is a mild oxidizing agent)
gives gluconic acid. This indicates the presence of aldehyde group.
CHO COOH
Bromine water
(CHOH)4 + [O]   (CHOH)4
CH2OH CH2OH
Glucose Gluconic acid
2. Action of hydroxylamine on glucose:
The reaction of glucose with hydroxylamine gives glucoxime. This indicates
the presence of carbonyl group.
CHO CH = N  OH
NH OH
(CHOH)4 
2  (CHOH)4 + H2O
CH2OH CH2OH
Glucose Glucoxime
[3]
iv. a. Elastomers: The polymers that have elastic character like that of rubber are called
elastomers.
eg. Neoprene
b.
Thermosetting Polymers Thermoplastic Polymers
i. They do not soften on heating under pressure. They soften on heating and harden on cooling.
ii. They cannot be remoulded, recycled or reused These can be remoulded or reshaped.
iii. These have crosslinked structures. These have linear structures.
iv. They are formed by condensation polymerization. They are formed by addition polymerization.
eg. Bakelite, urea-formaldehyde resins, etc. PVC, polythene and polystyrene, etc.
[3]

356
Chemistry
Q.7. Answer the following:
i. a. Action of water on lanthanoids: Reaction of water with lanthanoids gives ionic and
basic lanthanide hydroxides (Ln(OH)3) with liberation of H2 gas.
Ln + 3H2O  Ln(OH)3 + 3H2
b. Action of sulphur on lanthanoids: Heating of sulphur with lanthanoids gives
lanthanide sulphide (Ln2S3).

2Ln + 3S   Ln2S3
c. Action of nitrogen on lanthanoids: Reaction of nitrogen with lanthanoids gives
lanthanide nitride (LnN).

2Ln + N2   2LnN
ii. a. Structure of veronal:
H
O N O
N C2H5
H C2H5
O
Veronal
b. Structure of thymine:
O
CH3 C
C NH

HC C
N O
H
Thymine (T)

iii. Alkyl halide reacts with magnesium metal in the presence of dry ether to form Grignard
reagent (A), which on hydrolysis with water gives methyl cyclohexane (B).
dryether
CH2Cl + Mg   CH2MgCl 
H2O
 CH3 + MgCl(OH)

(A) (B)
Cyclohexylmethyl chloride Grignard Methyl
reagent cyclohexane [7]
OR
i. a. Preparation of benzoic acid from styrene:
Acidic potassium dichromate or acidic potassium permanganate oxidizes styrene
(phenylethene) into benzoic acid.
CH = CH2
COOH

KMnO 4 /H 2SO 4
 + CO2 + H2O
Styrene Benzoic acid
(Phenylethene)
b. Preparation of benzoic acid from benzamide:
Benzamide on hydrolysis in dilute acidic or alkaline medium gives benzoic acid.
CO – NH2
COOH
H 2 O, 

H  or OH 
 + NH3
Benzamide Benzoic Ammonia
acid
357
Board Answer Paper : July 2018
c. Preparation of benzoic acid from dry ice:
When the solution of phenyl magnesium iodide in dry ether is added to dry ice (solid
CO2), it gives a complex (magnesium salt of carboxylic acid), which on acid
hydrolysis gives benzoic acid.
O
MgI C  OMgI COOH
O
C=O +  
dry ether
 H 3O


 + Mg(OH)I
Dry ice
(Solid carbon
dioxide) Phenyl Benzoic acid
magnesium Adduct
iodide
ii. a. Resonance structure of aniline:
In aniline and other arylamines, the amino group is directly attached to benzene ring.
Conjugation of lone pair of electrons stabilises aniline. Lone pair is not easily
available for protonation. Five resonance structures of aniline are as follows:
+ + +
NH 2 NH2 NH2 NH2 NH2
 

I  IV V
II III
b. Action of benzene diazonium chloride on ethanol:
Reduction of benzenediazonium chloride in the presence of ethanol gives benzene.
The ethanol gets oxidized to ethanal.
N 2 Cl

+ CH3CH2OH 
 + N2 + CH3CHO + HCl
Ethanol Acetaldehyde
Benzenediazonium Benzene (Ethanal)
chloride
iii.. The formula of pentaamminechlorocobalt (III) sulphate: [Co(NH3)5Cl]SO4
[7]
Q.8. Select and write the most appropriate answer from the given alternatives for each
sub-question:
i. (C) dextron: [1]
Dextron is a copolymer of lactic acid and glycolic acid and it contains an ester linkage.
ii. (C) seldane [1]
Seldane or terfenadine is an antihistamine drug.
iii. (A) 2-Methylpropene, 1-bromo-2-methylpropane
CH3 CH3 CH3

CH3 – C = CH2 


 CH3 – C – CH2Br
HBr / H 2 O 2
CH3 – C – OH 
20% H 2SO 4
363K Antimarkownikoff addition
 H2O
CH3 H
tert-Butyl alcohol 2-Methylpropene 1-Bromo-2-methylpropane
(A) (B)

iv. (D) tetradentate [1]


v. (B) MnO2 [1]
vi. (D) Finkelstein reaction [1]
vii. (C) ester
Triacylglycerol is a triester of glycerol with higher fatty acids. [1]

358
Mathematics and Statistics

BOARD ANSWER PAPER : JULY 2018


MATHEMATICS AND STATISTICS

Note: Answer to every question must be written on a new page.

SECTION – I

Q.1. (A) Select and write the appropriate answer from the given alternatives in each of the
following sub-questions:
i. (D)
Given equation is x2 + kxy  3y2 = 0.
Comparing with ax2 + 2hxy + by2 = 0, we get
a = 1, 2h = k, b = 3
Let m1 and m2 be the slopes of the lines represented by x2 + kxy  3y2 = 0.
2h k a 1
 m1 + m2 = - = and m1m2 = = 
b 3 b 3
According to the given condition,
ml + m2 = 2(ml m2)
k æ 1ö
 = 2 ççç- ÷÷÷
3 è 3ø
 k = 2 [2]
ii. (C)
Since, the given vectors are coplanar.
1 2 1
 a 5 3 = 0
5 9 4
 1(20 + 27) + 2(4a  15) + 1(9a + 25) = 0
 7 + 8a  30  9a + 25 = 0
 a=2 [2]
iii. (A)
aa1  bb1  cc1
sin  =
a  b 2  c2  a12  b12  c12
2

10(2)  2(3)  11(6)


 sin  =
10  22  ( 11) 2  22  32  62
2

40
 sin  =
15  7
8
 sin  =
21
 8
  = sin1   [2]
 21 

(B) Attempt any THREE of the following:


i. a. ~p  (q  t) [1]
b. Shweta is a doctor or Seema is a teacher. [1]
ii. Consider L.H.S. = ac cos B  bc cos A
 a 2  c2  b2   b2  c2  a 2 
= ac    bc   ….[By cosine rule]
 2ac   2bc 

359
Board Answer Paper : July 2018
1 2
= (a + c2  b2  b2  c2 + a2)
2
1
=  2(a2  b2) = (a2  b2)
2
= R.H.S.
 ac cos B  bc cos A = (a2  b2) [2]
iii. Given equation of pair of lines is
2x2 + xy  y2 + x + 4y  3 = 0
Comparing with
ax2 + 2hxy + by2 + 2gx + 2fy + c = 0, we get
1 1
a = 2, h = , b = 1, g = , f = 2, c = 3
2 2
1 1
2
a h g 2 2
1 1 æ 3 ö 1 æ 1 ö÷
Consider, D = h b f = -1 2 = 2(3  4)  çç- -1÷÷÷ + çç1 + ÷
2 2 çè 2 ø 2 çè 2 ø÷
g f c
1
2 -3
2
5 3 8
= 2 +
+ = 2 + = 2 + 2 = 0
4 4 4
Since D = 0, the given equation represents a pair of straight lines. [2]
iv. 3a + 5b = 8c ….(given)
3a + 5b 5b + 3a
 c = =
8 5+3
 C (c) divides AB internally in the ratio 5 : 3 . [2]

v. A  (5, 5, ), B  (–1, 3, 2), C  (–4, 2, –2)


The direction ratios of line AB are
 1  5, 3  5, 2   i.e.,  6,  2, 2  
The direction ratios of line AC are
 4  5, 2  5, 2   i.e.,  9,  3,  2  
Since, the points A, B, C are collinear.
 the direction ratios of AB and AC are proportional.
 6 2 2  
  
 9 3 2  
2 2
 
3 2  
  4  2= 6  3
 3 2= 6 + 4
  = 10 [2]
Q.2. (A) Attempt any TWO of the following:
i.
p q r p  q q  r (p  q)  r p  (q  r) [(pq)r]  [p(q  r)]
T T T T T T T T
T T F T T T T T
T F T T T T T T
T F F T F T T T
F T T T T T T T
F T F T T T T T
F F T F T T T T
F F F F F F F T

360
Mathematics and Statistics
In the above truth table, all the entries in the last column are T.
 [(p  q)  r]  [p  (q  r)] is a tautology. [3]

3 2 6
ii. A = 1 1 2 
 2 2 5 
3 2 6
 |A| = 1 1 2
2 2 5
= 3(5  4)  2(5  4) + 6(2  2)
=32+0
=10
 A1 exists
1 2
A11 = (1)1 + 1 M11 = 1 = 1(5  4) = 1
2 5
1 2
A12 = (1)1 + 2 M12 = 1 = 1(5  4) = 1
2 5
1 1
A13 = (1)1 + 3 M13 = 1 = 1(2  2) = 0
2 2
2 6
A21 = (1)2 + 1 M21 = 1 = 1(10  12) = 2
2 5
3 6
A22 = (1)2 + 2 M22 = 1 = 1(15  12) = 3
2 5
3 2
A23 = (1)2 + 3 M23 = – 1 = 1(6  4) = 2
2 2
2 6
A31 = (1)3 + 1 M31 = 1 = 1(4  6) = 2
1 2
3 6
A32 = (1)3 + 2 M32 = – 1 = 1(6  6) = 0
1 2
3 2
A33 = (1)3 + 3 M33 = 1 = 1(3  2) = 1
1 1
Hence, matrix of the co-factors is
 A11 A12 A13   1 1 0 
A A 22 A 23    2 3 2 
 21
 A 31 A 32 A 33   2 0 1 
1 2 2 
 adj A =  1 3 0 
 0 2 1 
 1 2 2 
1 1
adj (A) =  1 3 0 
1
 A =
A 1
 0 2 1 
 1 2 2 
=  1 3 0  [3]
 0 2 1 
iii. Let  be the acute angle between the pair of lines given by ax2 + 2hxy + by2 = 0
2 h 2  ab
 tan  = ….(i)
a+b

361
Board Answer Paper : July 2018
Comparing the equation 2x2  5xy + 3y2 = 0 with ax2 + 2hxy + by2 = 0, we get
5
a = 2, h =  , b = 3
2
Let  be the acute angle between the lines given by 2x2  5xy + 3y2 = 0.
2
 5
2     (2)(3)
 2
 tan  =
2+3

25 1
2 6 2 
=  
4 2
=
5 5

1
 tan  = ….(ii)
5
But  =  ….(given)
 tan  = tan 
2 h 2  ab 1
 = ….[From (i) and (ii)]
ab 5
Squaring on both sides, we get
4(h 2  ab) 1
=
(a  b) 2 25
 100(h2 – ab) = (a + b)2 [3]
(B) Attempt any TWO of the following:
i. Let a , b and c be co-planar.
Case I:
Suppose that any two of a , b and c are collinear vectors, say a and b .
 There exist scalars x, y at least one of which is non-zero such that x a + y b = 0 .
 x a + y b + z c = 0 is a required non-zero linear combination, where z = 0.
Case II:
None of the two vectors a , b and c are collinear.
As c is coplanar with a and b ,
 scalars x, y are such that c = x a + y b
 x a + y b + z c = 0 , is a required non-zero linear combination, where z = 1
Conversely, suppose x a + y b + z c = 0 , where one of x, y, z is non-zero, say z  0, then
 x  y
c =   a +   b
 z  z
 c is coplanar with a and b .
 a , b and c are coplanar vectors. [4]
ii. To draw the feasible region, construct table as follows:
Inequality x2 x+y3 2x + y  1
Corresponding equation (of line) x=2 x+y=3 2x + y = 1
Intersection of line with X-axis (2, 0) (3, 0)  1 
  ,0 
 2 
Intersection of line with Y-axis  (0, 3) (0, 1)
Region Origin side Origin side Origin side
Shaded portion OABCD is the feasible region, whose vertices are O(0, 0), A(2, 0), B, C and
D(0, 1).
B is the point of intersection of the lines x = 2 and x + y = 3.
362
Mathematics and Statistics
Putting x = 2 in x + y = 3, we get
y=1 Y
 B  (2, 1)
C is the point of intersection of the lines 5
x + y = 3 and 2x + y = 1.
4
Solving the above equations, we get
2 7 2 7 3
x= ,y=  C=  ,  2 7
3 3 3 3 C , 
2 3 3
Here, the objective function is
1 B(2, 1)
z = 6x + 4y.
(0, 1)D
 z at O(0, 0) = 6(0) + 4(0) = 0 A(2, 0)
X O 1 X
z at A(2, 0) = 6(2) + 4(0) = 12 4 3 2 1 2 3 4
z at B(2, 1) = 6(2) + 4(1) = 12 + 4 = 16 1
2 7  2 7
z at C  ,  = 6   + 4   2 x+y=3
3 3 3 3
12 28 40 3
= + = = 13.33 x=2
3 3 3 2x + y = 1
4
z at D(0, 1) = 6(0) + 4(1) = 4
 z has maximum value 16 at B(2, 1). Y
 z is maximum, when x = 2 and y = 1. [4]
iii. sin 2x + sin 4x + sin 6x = 0
 (sin 2x + sin 6x) + sin 4x = 0
 6x  2x   6x  2x 
 2sin   cos   + sin 4x = 0
 2   2 
 2sin 4x cos 2x + sin 4x = 0
 sin 4x (2cos 2x + 1) = 0
1
 sin 4x = 0 or cos 2x = 
2
   2
 sin 4x = 0 or cos 2x =  cos   = cos     = cos  
3  3  3 
Since, sin  = 0 implies  = n and cos  = cos  implies  = 2n ± , n  Z.
2
 4x = n or 2x = 2m 
3
n 
 the required general solution is x = or x = m  , where n, m  Z. [4]
4 3
Q.3. (A) Attempt any TWO of the following:
i. a. Let p: Diagonals of a parallelogram are perpendicular.
q : It is a rhombus.
 The given statement is of the form p  q.
Its negation is p  q.
i.e. Diagonals of a parallelogram are perpendicular but it is not a rhombus. [1]
b. Let p: Mangoes are delicious.
q: Mangoes are expensive.
 The given statement is of the form p  q.
Its negation is p  q.
i.e., Mangoes are not delicious or they are not expensive. [1]
c. Let p: A person is rich.
q: He is a software engineer.
 The given statement is of the form p  q.
Its negation is (p   q)  (q   p)
i.e., A person is rich and he is not a software engineer or a person is a software
engineer and he is not rich. [1]
363
Board Answer Paper : July 2018
ii. Matrix form of the given system of equations is
1 1 1   x   6 
 3 1 3   y    6 
    
5 5 4   z   3 
This is of the form AX = B,
1 1 1   x 6 
where A = 3 1 3  , X =  y , B =
 
6 
 
5 5 4   z   3 
Applying R2  R2  3R1 and R3  R3  5R1,
1 1 1   x   6 
0 4 0   y    12 
    
0 0 9   z   27 
Hence, the original matrix A is reduced to an upper triangular matrix.
 By equality of matrices, we get
x+y+z=6 …(i)
4y = 12 …(ii)
12
 y= =3
4
9z = 27 …(iii)
27
 z= =3
9
Substituting y = 3 and z = 3 in equation (i), we get
x+3+3=6
 x=0
 x = 0, y = 3, z = 3 is the required solution. [3]
iii. Let a be the position vector of the point (–1, –1, 2).
 a = ˆi  ˆj  2kˆ
Equation of given line is 2x  2 = 3y + 1 = 6z  2
Dividing throughout by 6, we get
æ 1ö æ 1ö
3çç y  ÷÷÷ 6ççz - ÷÷÷
2( x -1) çè 3ø çè 3ø
= =
6 6 6
1 1
y+ z-
x -1 3 3
 = =
3 2 1
 Direction ratios of the line are 3, 2, 1
Let b be the vector parallel to this line.
 b = 3iˆ  2ˆj  kˆ
The vector equation of a line passing through a point with position vector a and parallel to
b is r = a + b
   
vector equation of the line is r = ˆi  ˆj  2kˆ +  3iˆ  2ˆj  kˆ  [3]

(B) Attempt any TWO of the following:


i. Let A  (a, 0, 0), B  (0, b, 0), C  (0, 0, c)
x y z
 Equation of plane in intercept form is + + =1
a b c
But, G(p, q, r) is the centroid of ABC.
a 00 0b0 00c
 (p, q, r)  , , 
 3 3 3 
a b c
 p= ,q= ,r=
3 3 3

364
Mathematics and Statistics
 a = 3p, b = 3q, c = 3r
x y z
 Equation of plane becomes + + =1
3p 3q 3r
x y z
i.e., + + =3 [4]
p q r
Z
ii.

 l
P(x, y, z)

Y
O

X
Let P(x, y, z) be a point on the line l and l(OP) = r.
The position vector of point P is
OP = x î + y ĵ + z k̂
Since, ,  and  are the direction angles of the line l.
 cos , cos  and cos  are the direction cosines.
Consider, OP  î = (x î + y ĵ + z k̂ )  î
=x ….(i)
Also, OP  î = OP  î  cos 
= r cos  ….(ii)
 x = r cos  ….[From (i) and (ii)]
Similarly, we have
y = r cos , z = r cos 
 OP = r cos  î + r cos  ĵ + r cos  k̂
2
Now, OP  OP = OP
 (r cos )2 + (r cos )2 + (r cos )2 = r2
 r2 (cos2  + cos2  + cos2 ) = r2
 cos2  + cos2  + cos2  = 1 .…(i)
sin2  + sin2  + sin2 
= (1  cos2 ) + (1  cos2 ) + (1  cos2 )
= 3  (cos2  + cos2  + cos2 )
=31 … [From (i)]
=2
 sin2  + sin2  + sin2  = 2 [4]
iii. In ABC, sin2A = sin2 [ – (B + C)] …[ A + B + C = ]
2
= [sin (B + C)]
= (sin B cos C + cos B sin C)2
= sin2 B cos2 C + 2 sin B cos C cos B sin C + cos2 B sin2 C
= sin2 B (1 – sin2C) + 2 sin B cos C cos B sin C + (1 – sin2 B) sin2C
= sin2 B – sin2 B sin2C + 2 sin B sin C cos B cos C + sin2C – sin2 B sin2 C
= sin2 B + sin2 C + 2 sin B sin C (cos B cos C  sin B sin C)
= sin2 B + sin2 C + 2 sin B sin C cos (B + C)
= sin2 B + sin2 C + 2 sin B sin C cos ( – A) …[ A + B + C = ]
 sin2A = sin2 B + sin2 C – 2sin B sin C cos A …(i)
365
Board Answer Paper : July 2018
In ABC by sine rule, we have
sinA sin B sin C
  k
a b c
 sin A = ka, sin B = kb, sin C = kc
 (ka)2 = (kb)2 + (kc)2  2(kb) (kc) cos A …[From (i)]
2 2 2
 a = b + c  2bc cos A
Similarly, we can prove that
b2 = a2 + c2  2ac cos B and c2 = a2 + b2  2ab cos C [4]

SECTION - II

Q.4. (A) Select and write the appropriate answer from the given alternatives in each of the
following sub-questions:
i. (B)
Equation of the curve is 2x2 + 3y2  5 = 0
Differentiating w.r.t. x, we get
dy
4x + 6y  =0
dx
dy  4x
 =
dx 6y
 Slope of tangent at (1, 1) is
 dy   4 1 2
  = =
 dx 1, 1 6 1 3

 dy 
Equation of tangent is y  y1 =   (x  x1)
 dx 
Here, (x1, y1)  (1, 1)
2
 (y  1) = (x  1)
3
 3(y  1) =  2(x  1)
 3y  3 = 2x + 2
 2x + 3y  5 = 0 [2]
ii. (D)
1 1
 d 3 y  6  dy  3
 3     0
 dx   dx 
1 1
 d 3 y  6  dy  3
  3   
 dx   dx 
Taking sixth power on both sides, we get
2
d 3 y  dy 
 
dx 3  dx 
d3 y
Here, the highest order derivative is with power 1.
dx3
 order = 3, degree = 1 [2]
iii. (A)
E(X) = np
 6 = n(0.6)
 n = 10
q = 1  p = 1  0.6 = 0.4
Var(X) = npq = (10) (0.6) (0.4) = 2.4 [2]
366
Mathematics and Statistics
(B) Attempt any THREE of the following:
i. The displacement s of a particle at time t is given by
s = t3  4t2  5t
ds
 Velocity = v = = 3t2  8t  5
dt
 Velocity at t = 2
 ds 
  = 3(2) 2  8(2)  5 = 12  16  5 = 9
 dt ( t  2)
dv d 2s
Acceleration = a = = 2 = 6t – 8
dt dt
 Acceleration at t = 2
 dv 
  = 6(2)  8 = 12 – 8 = 4 [2]
 dt ( t  2)

ii. y = cos1 (1  2sin2 x) = cos1 (cos 2x) = 2x


dy d
 = (2 x) = 2 [2]
dx dx
1
iii. Let I =  dx
sin x  cos 2 x
sin 2 x  cos 2 x
= dx
sin x cos 2 x
sin x 1
= 
cos 2 x
dx + 
sin x
dx

sin x 1
=  
cos x cos x 
dx + cosec x dx

=  tan x  sec x dx +  cosec x dx


 x
 I = sec x + log tan   + c [2]
2

dy
iv. x2 = x2 + xy + y2
dx
2
dy y  y
 =1+ +  ....(i)
dx x x
y
Put =u ....(ii)
x
 y = ux
Differentiating w.r.t. x, we get
dy du
=u+x ....(iii)
dx dx
Substituting (ii) and (iii) in (i), we get
du
u+x = 1 + u + u2
dx
du
 x = 1 + u2
dx
du dx
 =
1  u2 x
Integrating on both sides, we get
du dx
1 u 2
=  x
 tan1 u = log |x| + c
 y
 tan1   = log |x| + c [2]
 x
367
Board Answer Paper : July 2018
v. When two dice are tossed, n(S) = 36
Let X denotes the number of sixes.
Thus X can take values 0, 1, 2.
When X = 0 or no six appears,
X = {(1, 1), (1, 2), (1, 3), (1, 4), (1, 5), (2, 1), (2, 2), (2, 3), (2, 4), (2, 5), (3, 1), (3, 2), (3, 3),
(3, 4), (3, 5), (4, 1), (4, 2), (4, 3), (4, 4), (4, 5), (5, 1), (5, 2), (5, 3), (5, 4), (5, 5)}
 n(X) = 25
25
 P(X = 0) =
36
When X = 1 or only one six appears,
X = {(1, 6), (2, 6), (3, 6), (4, 6), (5, 6), (6, 1), (6, 2), (6, 3), (6, 4), (6, 5)}
 n(X) = 10
10
 P(X = 1) =
36
When X = 2 or both the outcomes are six,
X = {6, 6}
 n(X) = 1
1
 P(X = 2) =
36
Thus, the probability distribution of X is
X 0 1 2
25 10 1
P(X = x)
36 36 36
[2]
Q.5. (A) Attempt any TWO of the following:

i. f is continuous at x = .
6
 1  3 tan x
 f   = lim f(x) = lim
6 x x   6x
6 6

 
Put x = + h, as x  , h  0
6 6
 
1  3 tan   h 
  6 
 f   = lim
 
6 h 0   
  6  h 
6 
    1 
 tan 6  tan h   3  tan h 
1 3  1 3 
 1  tan  tan h   1  1 tan h 
 
= lim  6  = lim  3 
h 0     6h h 0 6h
1  1  4
1 tan h  1  3 tan h   3  tan h  tan h
= lim 3 = lim  3  = lim 3
h 0  1  h 0  1  h 0  1 
 6h 1  tan h   6h 1  tan h   6h 1  tan h 
 3   3   3 
2  tan h  1 2  tan h  1
= lim   =  lim  
h 0 3 3  h  1  1 tan h 3 3  h 0 h   1 
lim 1  tan h 
3 h 0
 3 
 
2  1 
= (1)  
3 3 1  1
 0 
 3 
 2
 f  = [3]
 
6 3 3

368
Mathematics and Statistics
 x y
ii. sec1   =a
2

 x y
x y
 = sec a2 = b (say)
x y
 x + y = bx  by
 (1 + b)y = (b  1) x
 b 1 
 y=  x
 b 1
b 1 y
 y = cx, where c =  …(i)
b 1 x
Differentiating w.r.t. x, we get
dy
c
dx
dy y
  …[From (i)] [3]
dx x

ex
iii. Let I =  1  e  2  e 
x x
dx

Put ex = t
 exdx = dt
dt
 I=  1  t  2  t 
1 A B
Let = 
1  t  2  t  1  t   2  t 
 1 = A(2 + t) + B(1 + t) ….(i)
Putting t = – 1 in (i), we get
1 = A(2  1) + B(0)
 A=1
Putting t =  2 in (i), we get
1 = A(0) + B(1 – 2)
 B=–1
1 1 1
 = 
1  t  2  t  1 t 2  t
 1 1 
 I=     dt
1 t 2  t 
dt dt
= 1 t   2  t
= log | 1 + t |  log | 2 + t | + c
= log | 1 + ex |  log | 2 + ex | + c
1  ex
 I = log +c [3]
2  ex

(B) Attempt any TWO of the following:


i. Let r, A be the radius and area respectively of the outermost circular wave, at time t
seconds.
Then, A = r2
dA dr dr
 = (2r) = 2r ….(i)
dt dt dt
dr
a. = 2 inches/sec, r = 5 inches
dt

369
Board Answer Paper : July 2018
From (i), we get
dA
= 2(5) (2) = 20
dt
Thus, the area is increasing at the rate of 20 sq.inches/sec, when the radius is
5 inches. [2]
b. Now after 5 seconds, the area increases at the rate of 2 inches/sec.
dr
i.e., = 2 inches/sec and t = 5seconds
dt
 Radius of circular wave after 5 seconds
= 2  5 = 10 inches.
dA dr
Now, = 2r ….[From (i)]
dt dt
= 2(10)(2) = 40
Thus, the area is increasing at the rate of 40 sq.inches/sec. after 5 seconds. [2]
a
1
ii. Let I = x
0 a 2  x2
dx

Put x = a sin 
 dx = a cos  d

When x = 0,  = 0 and when x = a,  =
2
π
2
a cosθ dθ
 I=  a sin θ 
0 a 2  a 2 sin 2 θ
π
2
cosθ
=  sin θ 
0 1  sin 2 θ

π
2
cosθ
 I=  sin θ  cosθ dθ
0
.…(i)

π 
π
2 cos   θ   a a

 2  ….   f ( x)dx   f (a  x)dx 
 I= 
 π   π 

 
0 sin
  θ   cos   θ  0 0
2  2 
π
2
sin θ
 I=  cosθ  sin θ dθ ….(ii)
0
Adding (i) and (ii), we get
π π
2 2
cosθ sin θ
2I =
0

sin θ  cosθ
dθ 
0
cosθ  sin θ
dθ 
π
2
cosθ  sin θ
=  sin θ  cosθ dθ
0
π
2
π π
=  dθ  [θ]02  0
0
2
1 π
 I= 
2 2

 I= [4]
4

iii. Let ‘x’ be the number of bacteria present at time ‘t’.


dx
 x
dt
370
Mathematics and Statistics
dx
 = kx
dt
dx
 = kdt
x
Integrating on both sides, we get
dx
x 
= k dt

 log x = kt + c
When t = 0, x = 1000
 log (1000) = k(0) + c
 c = log (1000)
 log x = kt + log (1000)
When t = 1, x = 2000
 log (2000) = k(1) + log (1000)
 log (2000)  log (1000) = k
 2000 
 k = log   = log 2 ....(i)
 1000 
1
When t = 2 , we get
2
5
log x = k + log(1000)
2
5
 log x =   log 2 + log (1000) ....[From (i)]
2
 5

= log  2 2  + log (1000)
 
 
= log 4 2 + log (1000)

= log (4000 2 )
= log (4000  1.414)
 log x = log (5656)
 x = 5656
1
Thus, there will be 5656 bacteria after 2 hours. [4]
2

Q.6. (A) Attempt any TWO of the following:


i. The domain of f is [0, 6].
For 0  x  2, f(x) = x2  4.
Since f is a polynomial function it is continuous in [0, 2].
For 2 < x  4, f(x) = 2x + 3.
Since f is a polynomial function it is continuous in (2, 4].
For 4 < x  6, f(x) = x2  5
Since f is a polynomial function it is continuous in (4, 6].
 f is continuous at every point in [0, 2]  (2, 4]  (4, 6]
For x = 2, f(x) = x2  4
f(2) = (2)2  4 = 0
lim f ( x)  lim  x 2  4    2   4  0 lim f ( x )  lim  2 x  3  2  2   3  7
2

x  2 x  2 x  2 x  2

 lim f  x   lim f  x 
x  2 x  2

 f is discontinuous at x = 2.
For x = 4, f(x) = 2x + 3
f(4) = 2(4) + 3 = 11
371
Board Answer Paper : July 2018
lim f  x   lim 2 x  3   2  4   3  11 lim f ( x)  lim  x 2  5    4   5  11
2

x  4 x  4 x  4 x  4

 lim f  x   lim f  x   f  4 
x  4 x  4

 f is continuous at x = 4.
Hence, f is continuous at x = 4 and discontinuous at x = 2. [3]
ii. Let x be a small increment in the value of x.
Since u is a function of x, there should be a corresponding increment u in the value of u.
Also y is a function of u.
 there should be a corresponding increment y in the value of y.
y u
Consider, y = 
x u x
Taking lim on both sides, we get
x  0

y y u
lim = lim  lim ….[ x  0, u  0]
x  0x x  0 u  x  0 x
y y u
 lim = lim  lim ….(i)
x  0 x u  0 u x  0 x

u du
But, lim = exists and is finite.
x  0 x dx
y dy
Also, lim = exists and is finite.
u  0 u du
 limits on R.H.S. of (i) exist and are finite.
Hence, limits on L.H.S. should also exist and be finite.
y dy
 lim = exists and is finite.
x  0 x dx
dy dy du
 =  [3]
dx du dx
iii. Let X be the number of families who own a television set.
P(family who own a television set) = p = 80% = 0.8, q = 1  0.8 = 0.2
Given n = 10
 X ~ B (10, 0.8)
The p.m.f. of X is given by
P(X = x) = p(x) = 10Cx (0.8)x (0.2)10  x, x = 0, 1, 2, ….,10
 P(at most 3 families own a television set) = P(X  3)
= P(X = 0) + P(X = 1) + P(X = 2) + P(X = 3)
= 10C0 (0.8)0 (0.2)10 + 10C1 (0.8)1 (0.2)9 + 10C2 (0.8)2 (0.2)8 + 10C3 (0.8)3 (0.2)7
= 1(1) 
1024  + 10  8 512  + 45   64  256  + 120   512 128
10  10  10  10 
10 10 10 10

1024  40960  737280  7864320 8643584


= =
10  10 
10 10

= 0.00086
Hence, the probability that at most three families own a television set is 0.00086. [3]
(B) Attempt any TWO of the following:
i. Let  vdx  w
dw
 =v
dx
d dw du
Consider, (uw) = u +w
dx dx dx
du
= uv +  vdx 
dx
du
dx 
= uv + vdx

372
Mathematics and Statistics
Integrating on both sides w.r.t. ‘x’, we get
 du 
uw =  u vdx     vdx  dx
 dx 
 du 
 u  v dx=  uvdx     vdx  dx
 dx 
 d 
  u  v dx = u  vdx    dx (u)   vdx  dx

Let I =  log x dx
=  1  log x dx

d 
= log x 1  dx     dx  log x  1 dx  dx
1
= x log x   x  x dx
= x log x –  dx
= x log x  x + c
 I = x (log x  1) + c [4]
a a
ii. Let I =  f ( x)dx  f (2a  x)dx
0 0

= I1 + I2
For I2, put 2a – x = t
 dx = dt
 dx = dt
When x = 0, t = 2a and when x = a, t = a
a
 I2 =   f  t  dt
2a

 I = I1 + I2
a a
=  f ( x)dx   f (t)dt
0 2a
a 2a
 b a

=  f ( x)dx  f (t)dt ....   f ( x)dx   f ( x)dx 
0 a  a b 
a 2a
 b b

=  f ( x)dx   f ( x)dx ….   f ( x)dx f (t)dt 
0 a  a a 
2a
 b c b

=  f ( x)dx ….   f  x  dx   f  x  dx   f  x  dx;a  c  b 
0  a a c 
2a a a
  f ( x)dx f ( x)dx   f (2a  x)dx
0 0 0
[4]

iii. Since the function represents a p.d.f.


1

  f ( x)dx  1
0
1

  kx (1  x) dx = 1
0
1

 k  ( x  x ) dx = 1
2

373
Board Answer Paper : July 2018
1
 x 2 x3  1
    =
 2 3 0 k
1 1 1 1 1
  =  =
2 3 k 6 k
 k=6 [2]
1
2
 1
PX   =
 2   f ( x)dx
0
1
2

=  kx (1  x) dx
0
1
2

=  (6 x  6x ) dx ....[ k = 6]
2

0
1
3 1
= 3x 2  2 x3  02 = 2 0
4 8
3 1 1
=  =
4 4 2
 1
 P  X   = 0.5 [2]
 2

374
Biology

BOARD ANSWER PAPER : JULY 2018


BIOLOGY

Note: Answer to every question must be written on a new page.

SECTION – I
[BOTANY]
Q.1. Select and write the most appropriate answer from the given alternatives for each sub-
question:
i. (B) 146 [1]
ii. (A) anemophilous [1]
iii. (D) Golden rice [1]
iv. (D) producers [1]
v. (B) Azotobacter [1]
vi. (A) Agrobacterium tumefaciens [1]
vii. (C) Ultraviolet rays [1]
Q.2. (A) Answer each question in ‘One’ sentence only :
i. The process of induction and utilization of mutation for development of new crop varieties
having desirable traits is known as mutational breeding. [1]
ii. The octamer of nucleosome is formed of eight molecules of histones, two each of H2A,
H2B, H3 and H4. [1]
iii. The therapeutic products of blood proteins made by using r-DNA technology are
Erythropoetin; Factors VII, VIII, IX, Tissue plasminogen activator; Urokinase.
(Any one example) [1]
iv. Saccharomyces cerevisiae var. ellipsoideus is the strain of yeast used in industrial
production of wine. [1]
v. (C) Tt  tt represents test cross. [1]
vi. The part of the rooted plant on which grafting is done is called stock.
The part which is inserted on stock is called scion. [1]
(B)
Standard

Diadelphous
stamens [(9) + 1]
Wings Carpel

Keel
Structure of pea flower
[2]

(C) Attempt any Two of the following:


i. a. When a single gene controls two (or more) different traits, it is called pleiotropic gene and
this phenomenon is called pleiotropy or pleiotropism. The ratio is 2 : 1 instead of 3 : 1.
375
Board Answer Paper : July 2018
b. According to Mendel’s principle of unit character, one gene (factor) controls one character
(trait), but sometimes single gene produces two related or unrelated phenotypic expressions.
For example, the disease, sickle cell anaemia is caused by a gene HbS. Normal or healthy
gene is HbA and is dominant.
c. The carriers (heterozygotes – HbA/HbS) show signs of mild anaemia as their RBCs become
sickle shaped (half-moon shaped) in oxygen deficiency. They are said to have sickle-cell
trait and are normal in normal conditions.
d. The homozygotes with recessive gene Hbs however, die of fatal anaemia.
Thus, the gene for sickle-cell anaemia is lethal in homozygous condition and produces
sickle cell trait in heterozygous carrier.

Phenotype of parents Sickle-cell carrier  Sickle-cell carrier


Genotype HbA HbS HbA HbS
Gametes HbA HbS HbA HbS

Genotype of offsprings HbAHbA HbAHbS HbSHbA HbSHbS


Phenotype of offsprings Normal Sickle-cell carriers Sickle-cell Anaemic
1 2 1(dies)
Graphical representation of pleiotropy

[2]
ii. The applications of tissue culture technique are:
a. Micropropagation b. Production of disease free plants
c. Production of secondary metabolites d. Somatic hybridization
[2]
iii. Following factors affect the rate of photosynthesis:
a. Internal factors:
Internal factors are mostly genetic. They include factors such as the size, age, number
and orientation of leaves, the amount of chlorophyll, rate of translocation of food, etc.
b. External factors:
1. Light: Light is essential for photosynthesis as it provides energy for photosynthesis.
The quality, intensity and duration of light influence the rate of photosynthesis
significantly.
i. Quality: Light between the wavelength 400 nm and 700 nm is most effective for
photosynthesis.
Maximum photosynthesis takes place in red light, followed by blue light and
minimum in green light.
ii. Intensity: The rate of photosynthesis steadily increases with the light intensity.
However, if light intensity increases beyond a certain point, the chlorophyll
undergoes photo-oxidation and is destroyed.
iii. Duration: Longer duration of light favours photosynthesis. Generally if the plants
get 10 to 12 hours of light per day, it favours good photosynthesis.
2. CO2: The low concentration of CO2 (0.03% – 0.04%) in the atmosphere acts as a
limiting factor in natural photosynthesis, when other environmental factors (light,
temperature, water, etc.) are optimum. If the CO2 supply is increased up to a certain
limit (0.05%), the rate of photosynthesis increases. However, very high concentration
of CO2 becomes toxic to the plants, over long periods of exposure.
3. Water: Water deficiency may decrease the rate of photosynthesis, as it is one of the
raw materials for the process. Less availability of water may further check the rate by
closing the stomata, thereby affecting the entry of CO2. It results in loss of turgidity
in the leaves and wilting.
4. Temperature: The optimum range of temperature for photosynthesis is from 10C to
35C. The optimum temperature depends on the habitat that the plants are adapted to. e.g.
376
Biology
Tropical plants will have a higher optimum temperature for photosynthesis as compared
to temperate plants. The rate of photosynthesis increases by increase in temperature upto
40C. Thus, dark reaction is more sensitive to temperatures than the light reaction, since
it is enzymatically controlled. If temperature is increased too high, the rate of
photosynthesis is also reduced due to denaturation of enzymes involved in the process.
(Any two factors) [2]
iv. a. Codon: It is the smallest possible sequence (triplet) of nucleotides present on m-RNA
strand which can specify one particular amino acid.
b. Anticodon: It is a triplet of nucleotides present on the loop of middle arm of t-RNA and
is complementary with the codon.
[2]
Q.3. (A) Attempt any Two of the following:
i. Following reactions take place during the process of production of biogas:
a. Hydrolysis or anaerobic digestion:
In this step, cellulose is hydrolyzed to simple compounds. Complex insoluble polymers
are converted to simple soluble monomers with the help of bacterial hydrolytic enzymes.
This process occurs by anaerobic bacteria like species of Clostridium, Pseudomonas, etc.
Polymers Anaerobic bacteria
 Monomers
b. Acetogenesis or acidogenesis:
In this step, monomers are converted into organic acids. Acid-producing bacteria,
involved in the second step, convert the intermediates of fermenting bacteria into
acetic acid (CH3COOH), hydrogen (H2) and carbon dioxide (CO2).
Monomers  Acidogenic bacteria
 Organic acids
c. Methanogenesis:
Organic acids such as acetic acid are converted into biogas. Methanogenic bacteria
(species of Methanococcus and Methanobacillus) involved in the third step,
decompose compounds with a low molecular weight. For example, they utilize
hydrogen, carbon dioxide and acetic acid to form methane and carbon dioxide.
Organic acids 
Methanogenic bacteria
 Methane + CO2 + Other gases
Knob
Dome Gas
Inlet tank Water
(Dung, water) Digested
seal sludge
Gas holder

Outlet pipe
Inlet pipe
Digester

Biogas Plant
[3]
ii. Definition: Deforestation is the permanent removal, decrease or deterioration of forests and
woodlands. (1)
The effects of deforestation are:
a. Loss of biodiversity due to extinction of species.
b. Reduced soil fertility and soil erosion.
c. Regional and global climatic changes.
d. Global warming.
e. Shortage of fuel wood and timber.
f. Increased incidents of landslides. (Any four effects) (2)
iii. a. Palindrome in DNA is a sequence of base pairs that reads the same on the two
strands when orientation of reading is kept the same.
377
Board Answer Paper : July 2018
b. The following sequence is an example of palindrome:

5  A C C G A A T T C G C A  3

3  T G G C T T A A G C G T  5

[3]

(B)

Old Old
strand strand

5 3
New New
strand strand

Watson-Crick model for semiconservative


DNA replication [3]

Q.4. Vegetative reproduction is a kind of asexual reproduction which occurs with the help of
vegetative
organs like root, stem, leaf or bud. It is also called as vegetative propagation.
i. Natural methods of vegetative propagation:
a. Tuberous roots: e.g. Sweet Potato (Ipomoea batatas)
1. It is a modification of root for vegetative reproduction.
2. The underground roots in some plants store plenty of reserve food. Due to this, they
become swollen.
3. These roots are called tuberous roots.
4. In addition to the storage of food, these roots also develop adventitious buds on their
surface which sprout under favourable conditions to produce ‘leafy shoots’(slips) and
adventitious roots.
5. Under suitable environmental conditions, these leafy shoots separate and develop into
new plants.
6. The tuberous roots are adventitious and when produced singly are called as simple
tuberous roots. e.g. sweet potato and when produced in groups or clusters are called as
fasciculated tuberous roots. e.g. Asparagus and Dahlia.

Creeping stem
Scar of detached root
Tuberous
root
Adventitious Adventitious
roots roots

Root tuber of sweet potato

378
Biology
b. Stem tuber: e.g. Solanum tuberosum
1. Stem tuber is a modified stem for vegetative reproduction.
2. In case of potato, some lateral branches are produced from the underground part
of stem which grow down in the soil.
3. The tip of these branches store food and hence are swollen.
4. These tubers show nodes, internodes, scale leaves and axillary buds.
5. A stem tuber has many notches on its surface called ‘eye’.
6. The eyes that are seen on the surface of the tuber represent nodes.
7. Each ‘eye’ is actually a node and is made up of one or more axillary buds
subtended by a leaf scar.
8. After termination of dormancy period, under favourable conditions one of the,
axillary buds from an eye sprouts and suppresses the growth of other buds.
Scar of stem

Scale leaf Eye


Axillary bud
Underground
branch

Sprouting bud
Younger
tuber
a. Underground branch with Stem Tuber b. Single Stem Tuber
c. Vegetative propagation by runner: e.g. Cynodon (doob grass), Oxalis
1. Runner is a slender, prostrate, sub-aerial branch with short or long internode and
creeps horizontally on the soil.
2. Runner develops from the axillary bud in the axil of the lowest leaf.
3. After creeping some distance, away from the parent plant, it produces shoots from
upper side and roots from lower side of the nodes.
4. Many runners are produced by the parent plant which may get detached from the
parent plant to develop into new plants.

Leaf

Node Runner
Internode

Roots
Runner
d. Vegetative propagation by leaf: e.g. Bryophyllum, Kalanchoe, Begonia
1. In some plants, leaves also take part in
vegetative propagation.
2. In Bryophyllum, leaves are succulent with
Foliar buds
notched or crenate margin.
(Epiphyllous)
3. Adventitious buds called epiphyllous buds are
developed on the leaves. These buds start
sprouting on the leaf to form the plantlets.
4. These plantlets fall off from parent plant to
continue their growth in the wet soil.
5. In some species of Bryophyllum, the new
plants are formed from the leaf only when the
leaf is separated from the parent plant and fall Leaf of Bryophyllum
on the wet soil.
6. In Begonia, foliar buds are formed on the leaf surface.
[Any three natural methods] [7]
379
Board Answer Paper : July 2018
OR
Q.4. Glycolysis is defined as the stepwise, enzymatic breakdown of hexose sugar (glucose) into
two molecules of pyruvate in the cytoplasm. It is also called cytoplasmic respiration since it
takes place in the cytoplasm.
Glycolysis consists of two major phases:
i. Preparatory phase and cleavage.
ii. Oxidative and payoff phase.
i. Preparatory phase and cleavage:
In this step, the glucose molecule is activated by phosphorylation and then cleaved into two
molecules of triose phosphate, namely 3-PGAL and DHAP. In the next step, only 3-PGAL
participates and hence DHAP gets converted into 3-PGAL. Thus, two molecules of 3-
PGAL are formed.
It includes the following reactions:
a. Phosphorylation I: A glucose molecule is phosphorylated to glucose 6-phosphate in
presence of enzyme hexokinase and co-factor Mg++. In this reaction, ATP supplies
the phosphate and gets converted into ADP.
b. Isomerisation: Glucose-6-phosphate is isomerised to fructose-6-phosphate in the
presence of the enzyme phosphoglucoisomerase.
c. Phosphorylation II: Fructose-6-phosphate is phosphorylated to fructose-1,6-
diphosphate and ADP. Phosphate is supplied by ATP which gets converted to ADP.
This reaction is catalyzed by the enzyme phosphofructokinase in the presence of
Mg++.
d. Cleavage: Fructose-1,6-diphosphate splits up into two inter-convertible compounds,
i.e. dihydroxy-acetone phosphate (DHAP) and 3-phosphoglyceraldehyde (3-PGAL).
This reaction occurs in the presence of the enzyme aldolase.
e. Isomerization: DHAP is isomerised into 3-PGAL with the help of enzyme phospho-
triose isomerase, thus two molecules of 3-PGAL are obtained.

ii. Oxidative and payoff phase:


In this phase, oxidation by removal of hydrogen takes place followed by generation of
ATP.
a. Oxidative Phosphorylation: 3-PGAL undergoes oxidation and phosphorylation to
form 1,3-diphosphoglyceric acid (1,3-diPGA). Hydrogen released combines with
NAD to form NADH2. Phosphate is supplied for the reaction by phosphoric acid
(H3PO4).
b. ATP generation I / Dephosphorylation I: 1,3-diphosphoglyceric acid is
dephosphorylated to 3-phosphoglyceric acid (3-PGA). Phosphate released combines
with ADP to form ATP.
Reaction takes place in the presence of the enzyme phosphoglycero-kinase.
c. Isomerisation: In presence of the enzyme phosphoglycero-mutase, 3-
Phosphoglyceric acid is isomerised to 2-phosphoglyceric acid.
d. Dehydration: 2-Phosphoglyceric acid loses a water molecule to form phosphoenol
pyruvic acid. The enzyme enolase catalyses the reaction.
e. ATP generation II / Dephosphorylation II: Phosphoenol pyruvic acid is
dephosphorylated to pyruvic acid in the presence of the enzyme pyruvate kinase.
Phosphate released combines with ADP to form ATP.
380
Biology

Glucose (6C) sugar

ATP
Hexokinase [Phosphorylation I] 1
ADP
Glucose-6-Phosphate (6C)

Phosphoglucoisomerase [Isomerization] 2

Fructose-6- Phosphate (6C)

ATP
[Phosphorylation II] 3
Phosphofructokinase
ADP
Fructose-1, 6-diPhosphate (6C) [Cleavage] 4
Fructose diphosphate aldolase

3-PGAL (3C) DHAP (3C)


Isomerization [Isomerization] 5

3-PGAL (2M) (3C)

2NAD (H3PO4 ) Phosphoglyceraldehyde


dehydrogenase [Oxidative Phosphorylation] 6
2 iP
2NADH2

1, 3- diphosphoglyceric acid (2M) (3C)


2ADP
Phosphoglycerokinase
[Dephosphorylation I] 7
2ATP
3-PGA (2M) (3C)

Phosphoglyceromutase
[Isomerization] 8
2-PGA (2M) (3C)

H2O Enolase [Dehydration] 9

Phosphoenolpyruvic acid (2M) (3C)


2ADP
Pyruvate kinase [Dephosphorylation II] 10
2ATP
Pyruvic acid (2M) (3C)

Schematic Representation (Glycolysis or EMP Pathway)

[7]

381
Board Answer Paper : July 2018

SECTION – II
[ZOOLOGY]

Q.5. Select and write the most appropriate answer from the given alternatives for each sub-
question:
i. (C) protobiogenesis [1]
ii. (B) 2n + 1 [1]
iii. (B) insulin [1]
iv. (A) Variable Number of Tandem Repeats [1]
v. (D) Coca [1]
vi. (D) cornea [1]
vii. (A) two organisms are benefitted from each other [1]
Q.6. (A) Answer in One sentence each:
i. Archaeopteryx is an example of the connecting link between reptiles and birds. [1]
ii. On the basis of origin, the various poultry breeds used in poultry are:
a. Indian breeds: Aseel, Kadaknath, Busra and Chittagong.
b. Asiatic breeds: Brahma, Cochin, Langshan
c. Mediterranean breeds: Leghorn, Minorca
d. English breeds: Australorp
e. American breeds: Plymouth rock, New Hampshire, Rhode Island Red [1]
iii. Linkage: It is the tendency of genes to remain together on the same chromosome and pass
on to next generation together. [1]
iv. Benefits of transgenic animals:
a. Used to test vaccine safety
b. Used to test chemical safety
c. Serve as model for study of diseases
d. Act as bioreactors for obtaining biological products
e. Used to understand the normal physiology and development [1]
v. Abducens and Facial nerves are the VI and VII cranial nerves respectively. [1]
vi. Natality is calculated as:
Total number of birthsin a year
Natality =  1000 [1]
Mid year population
(B)
Sino auricular node

Right auricle Left auricle

Auriculo ventricular
node Bundle of His

Purkinje fibres
Left ventricle

Right ventricle

Auriculoventricular septum
Conducting System of Human Heart [2]

382
Biology
(C) Attempt any TWO of the following:
i. Structure of human sperm:
a. The human sperm is a haploid, elongated and motile male gamete.
b. It is microscopic, measuring about 0.055 mm in length.
c. Sperms are produced by the process of spermatogenesis in testes.
d. The body of sperm is divided into four parts, viz. head, neck, middle piece and tail.
1. Head: It is flat and oval consisting of large haploid nucleus and an acrosome formed
from Golgi complex.
The acrosome secretes an enzyme hyaluronidase which helps in penetration of egg during
fertilization.
The anterior half of the nucleus is covered by a fibrillar sheath.
2. Neck: It is a narrow constricted region consisting of two centrioles (proximal and distal).
The proximal centriole plays an important role in first cleavage of the zygote. The
distal centriole forms the axial filament present in the tail of the sperm.
3. Middle piece: Middle piece has many mitochondria spirally coiled (Nebenkern)
around the axial filament.
The mitochondria provides energy for the sperm during its movement, hence it is
called the powerhouse of the sperm. Posterior half of nucleus, neck and middle piece
of sperm are covered by a sheath.
4. Tail: The tail is long, slender and tapering.
It is made up of cytoplasm.
The axial filament arises from the distal centriole and traverses the middle piece and tail. [2]

ii. Struggle for existence: According to Darwin, individuals multiply in geometric ratio, however
space and food remain constant. This leads to competition amongst the individuals for getting
requirements of life. This competition is called struggle for existence. The struggle may be intra-
specific, inter-specific or environmental.
a. Intra-specific struggle:
It is the competition among the individuals of the same species. Since the need and
approach of all competing organisms is precisely same, this type of struggle is very
severe. e.g. struggle between cow and cow, horse and horse, deer and deer, etc.
b. Inter-specific struggle:
It is the struggle between the organisms of different species living together.
Individuals of one species compete with other species for food, shelter and breeding
place. e.g. Struggle between cow, horse and deer for getting grass.
c. Environmental struggle:
It is the struggle of all living forms against adverse environmental conditions, i.e.
against natural calamities like extreme heat or cold, drought, storms, earthquakes,
volcanic eruptions, etc. [2]
iii. Economic importance of fishery:
a. Fishery includes catching, processing, fish farming and marketing of fish and other
aquatic animals. It provides good job opportunity and self employment.
b. Culturing of fishes on a large scale in ponds, lakes and reservoirs boost the
productivity and economy of the nation.
c. Fishery byproducts include fish oil, fish meal, fertilizers, fish guano, fish glue and
Isinglass, which are widely used in paints, soaps, oils and medicines.
d. Prawns and lobsters have market value all over the world. [2]

iv. The two types of maps generated by HGP are:


a. Genetic linkage maps b. Physical maps
a. Genetic linkage maps: They determine the relative arrangement and approximate
distances between genes and markers on the chromosomes.
b. Physical maps: They specify the physical location (in base pairs) and distance
between genes or DNA fragments with unknown function. [2]
383
Board Answer Paper : July 2018
Q.7. (A) Attempt any TWO of the following:
i. When a carrier haemophilic female (XhXH) marries normal male (XHY), then all the
daughters will have normal clotting of blood but half of them will be carriers for the disease.
Half the sons will be haemophilic while the remaining will have normal clotting of blood.

Parents: XhXH
 X HY
(Carrier female) (Normal Male)

Gametes: Xh XH XH Y

XhXH XhY X HX H X HY
F1:
Carrier Haemophilic Normal Normal
daughter son daughter son
[3]

ii. Major causes of water pollution are as follows:


a. Domestic wastes: Waste water from homes and commercial establishments are
discharged into water bodies.
b. Industrial water and effluents: Pollutants that come along waste water of various
industries such as petro-chemical complexes, fertilizer factories, oil refineries, etc.,
pollute most of the Indian rivers and fresh-water streams.
c. Agricultural wastes: Sediments, fertilizers and farm wastes, pesticides, etc. enter the
water bodies from agricultural lands and cause water pollution.
d. Major oil spills from ships, small spills, offshore drilling rigs, cleaning operations
cause water pollution.
e. Solid wastes: some amount of water pollution is caused by deposition of solid wastes
in surface and ground water.
f. Air pollutants: Setting of air pollutants also causes water pollution.
[3]
iii. Mechanism of antibody – antigen complex formation:

Antigenic determinants

Antibodies bound to
antigenic determinants

Antigen
Antigen

Globular
proteins
Virus

Formation of antigen-antibody complex

a. Study of antigen-antibody interactions is called Serology.


b. Antibodies are always antigen specific. Combining sites of antigen, called antigenic
determinants or epitopes react with the corresponding antigen binding sites of antibodies
called paratopes.
c. Specificity of an antibody for an antigen lies in its sequence of amino acids within the
variable regions of the two chains.
d. These are active sites of an antibody where the molecules bind to specific antigens.
384
Biology
e. Both the arms have the same amino acid sequence and therefore bind to the same kind of
antigen.
f. Thus, each antibody can join two antigens in a Lock and Key manner forming an
antigen-antibody complex.
[3]
(B) Frontal lobe
Central sulcus
Cerebrum Parietal lobe
Lateral sulcus
Parieto occipital sulcus

Occipital lobe
Temporal lobe
Pons varolii
Medulla oblongata
Cerebellum

Spinal cord
Lateral view of human brain
[3]

Q.8.
Primordial follicle
Germinal epithelium

Graafian follicle

Antral fluid
Corpus Medulla
albicans

Corpus luteum Ovum (secondary oocyte)


Ruptured follicle
T.S of Ovary

The menstrual cycle is divided into four phases:


i. Menstrual phase
ii. Proliferative phase
iii. Ovulatory phase
iv. Post ovulatory or secretory phase or luteal phase.
i. Menstrual phase (bleeding or destructive phase):
This phase extends from 1st to 4th day of menstrual cycle.
During this phase, bleeding occurs as the endometrium of uterus is sloughed off.
Menstrual flow consists of the secretion of endometrial glands, cell debris, unfertilized
ovum along with 35 to 45 ml of blood. It is also called as weeping of ‘uterus’ for lost ovum.
After about 4th day of bleeding, when progesterone level decreases, once again the FSH
secretion from the anterior pituitary is resumed and the new follicle starts developing.
ii. Proliferative phase:
This phase extends from 5th to 13th day of menstrual cycle.
Changes in ovary:
During this phase, the primordial follicle of the ovary develops into Graafian follicle. The
ovum becomes eccentric and is connected by follicular cells called germ hill or cumulus
oophorus or discus proligerous.
385
Board Answer Paper : July 2018
Theca interna cells of the follicle secretes female sex hormone oestrogen.
Only one follicle develops in one cycle.
Changes in uterus:
Oestrogen secreted by follicular cells of ovary stimulate endometrial glands.
This causes repair of endometrium.
The endometrial cells proliferate and thickness of endometrium grows to about 3 mm to 5
mm.
iii. Ovulatory phase:
During this phase, ovulation takes place. It usually occurs on the 14th day of the cycle.
Due to LH secreted by the pituitary, the Graafian follicle bursts and releases the ovum in
abdominal cavity. The ovum is viable for fertilization within 24 to 48 hours after ovulation.
Ovum enters the fallopian tube through ostium assisted by fimbriae. On its way, if the ovum
happens to meet the sperms, it is fertilized and the fertilized egg reaches the uterus on third day
after ovulation.
If unfertilized, the ovum degenerates.
iv. Secretory phase/Luteal phase:
This phase extends from 15th to 28th day of the menstrual cycle.
Changes in ovary:
After ovulation, the ruptured follicle develops into a yellow body called corpus luteum.
It acts as temporary endocrine gland which secretes progesterone, which helps in
maintaining the thickness of endometrium. So, it is also called pregnancy hormone.
Corpus luteum remains active till placenta starts secretion of HCG (Human chorionic
gonadotropin). HCG is similar to LH.
If corpus luteum becomes inactive before placenta formation, it leads to abortion.
If the ovum is not fertilized, the corpus luteum degenerates and transforms into a whitish
scar called corpus albicans.
Changes in Uterus:
Corpus luteum formed in ovary secretes progesterone.
It causes further growth of endometrial glands.
If fertilization occurs, embryo is implanted in thickened endometrium. [7]
OR
Human excretory system of consists of:
i. Kidneys (a pair) ii. Ureters (a pair)
iii. Urinary Bladder iv. Urethra or vestibule in females

Supra renal gland

Hilum Left Kidney


Right Kidney

Renal Vein Renal artery

Inferior vena cava


Dorsal aorta

Ureter

Urinary
bladder

Urethra

Human excretory system


386
Biology
i. Kidneys:
Kidneys are a pair of dark red, bean-shaped organs about 10 cm long, 5cm wide and 4 cm
thick.
They are attached to the dorsal body wall at the level of 12th thoracic to 3rd lumbar vertebra
in the abdominal cavity.
They are retroperitoneal as they have peritoneal covering only on the anterior surface.
They are situated on either side of the vertebral column.
The right kidney is slightly lower than the left kidney due to presence of liver.
Left kidney is slightly back than right kidney due to the position of stomach in front of it.
On the concave surface, there is a notch called hilus or hilum.
Through the hilum, blood vessels, nerves, ureter, lymphatic vessels enter and leave the
kidney.
Renal arteries enter both the kidneys, while renal veins leave the kidneys.
ii. Ureters:
From the hilum of each kidney, a thin, narrow and muscular duct emerges out which is called as
ureter.
It measures about 40 cm in length.
The ureters open into urinary bladder.
iii. Urinary bladder:
It is single, large and pear shaped structure.
It is located in the pelvic cavity just behind the pubis bone.
It is thick walled and highly muscular.
It has a layer of detrusor muscles.
The oblique course of ureters through the wall of bladder prevent the backward flow of
urine into ureters.
iv. Urethra:
It starts from the lower part of the urinary bladder and opens to the exterior.
Its opening is guarded by sphincter muscle called urethral sphincter.
Opening of urethral sphincter is under voluntary control and results in micturition and
emptying of the bladder.
In males, urethra is long (20 cm) and has to pass through the penis.
In females, it is short (4 cm) and opens in vestibule.
The process of urine formation occurs in 3 steps, namely:
a. Ultrafiltration
b. Selective Reabsorption
c. Tubular Secretion
a. Ultrafiltration:
Filtration of blood under pressure is called Ultrafiltration.
It takes place in Malpighian body i.e. glomerulus and Bowman’s capsule.
It is a physical process and occurs through the glomerulus under high osmotic pressure.
Capillaries of glomerulus are extremely porous or perforated and are more permeable to
water and small molecules.
Ultra pressure is developed in glomerulus due to difference in diameters of afferent and
efferent renal arterioles.
Blood enters the glomerulus at a fast rate through afferent arteriole with greater diameter
and drains away by efferent arteriole with smaller diameter at slower rate. This creates
hydrostatic pressure within glomerulus. This is called effective filtration pressure (EFP).
The filtration takes place through semipermeable membrane or filtration membrane under
pressure.
The ultrafiltrate enters the urinary space of Bowman’s capsule. This filtrate is blood plasma
without proteins (deproteinized plasma).
About 125 ml of ultrafiltrate is produced per minute.
387
Board Answer Paper : July 2018
b. Selective reabsorption:
In this process, 99% of the filtrate is reabsorbed.
There are two processes involved in this step  Passive transport or osmosis and Active
transport.
Active transport is transport against the concentration gradient by using ATP molecules.
When the filtrate passes through the renal tubule there is exchange of substances between
the blood in peritubular capillary network and the filtrate.
High threshold substances, e.g. glucose, amino acids, potassium and calcium ions, etc. are
completely reabsorbed.
Urea and uric acid are low threshold substances. They are reabsorbed negligibly.
Water is reabsorbed in PCT, DCT and descending limb of loop of Henle, except in the
ascending loop of Henle. This absorption occurs by osmosis and is known as obligatory
absorption of water.
Amino acids and ions are pumped out through PCT. Potassium, calcium, chloride ions are
absorbed by diffusion. Glucose is maximally reabsorbed in healthy persons so that no
glucose molecules remain in the urine.
Urea is reabsorbed because urea molecule is very small and tubules are partially permeable to it.
c. Tubular Secretion (Renal Secretion):
The separation and secretion of unwanted substances from peritubular capillary network
into tubular fluid is called tubular secretion.
In this process certain substances pass from the blood to the lumen of the tubule.
These substances are actively separated and secreted inside renal tubule e.g. K+ ion, H+ ion,
creatinine, uric acid, penicillin, urea, ammonia, etc. This process is important for
homeostasis.
Finally, fluid becomes hypertonic which is excreted as urine.
Absorption of water is controlled by ADH and absorption of salt is controlled by
aldosterone. [7]

388

Vous aimerez peut-être aussi